Download as docx, pdf, or txt
Download as docx, pdf, or txt
You are on page 1of 341

1. Marco approached Nurse Trish asking for advice on how to deal with his alcohol addiction.

Nurse Trish should tell the client that the only effective treatment for alcoholism is:
A. Psychotherapy
B. Alcoholics anonymous (A.A.)
C. Total abstinence
D. Aversion Therapy

2.Nurse Hazel is caring for a male client who experience false sensory perceptions with no basis
in reality. This perception is known as:
A. Hallucinations
B. Delusions
C. Loose associations
D. Neologisms

3. Nurse Monet is caring for a female client who has suicidal tendency. When accompanying the
client to the restroom, Nurse Monet should...
A. Give her privacy
B. Allow her to urinate
C. Open the window and allow her to get some fresh air
D. Observe her

4. Nurse Maureen is developing a plan of care for a female client with anorexia nervosa. Which
action should the nurse include in the plan?
A. Provide privacy during meals
B. Set-up a strict eating plan for the client
C. Encourage client to exercise to reduce anxiety
D. Restrict visits with the family

5. A client is experiencing anxiety attack. The most appropriate nursing intervention should
include?
A. Turning on the television
B. Leaving the client alone
C. Staying with the client and speaking in short sentences
D. Ask the client to play with other clients

6. A female client is admitted with a diagnosis of delusions of GRANDEUR. This diagnosis


reflects a belief that one is:
A. Being Killed
B. Highly famous and important
C. Responsible for evil world
D. Connected to client unrelated to oneself

7.A 20 year old client was diagnosed with dependent personality disorder. Which behavior is
most likely to be evidence of ineffective individual coping?
A. Recurrent self-destructive behavior
B. Avoiding relationship
C. Showing interest in solitary activities
D. Inability to make choices and decision without advise

8. A male client is diagnosed with schizotypal personality disorder. Which signs would this
client exhibit during social situation?
A. Paranoid thoughts
B. Emotional affect
C. Independence need
D. Aggressive behavior

9. Nurse Claire is caring for a client diagnosed with bulimia. The most appropriate initial goal for
a client diagnosed with bulimia is?
A. Encourage to avoid foods
B. Identify anxiety causing situations
C. Eat only three meals a day
D. Avoid shopping plenty of groceries

10. Nurse Tony was caring for a 41 year old female client. Which behavior by the client
indicates adult cognitive development?
A. Generates new levels of awareness
B. Assumes responsibility for her actions
C. Has maximum ability to solve problems and learn new skills
D. Her perception are based on reality

11. A neuromuscular blocking agent is administered to a client before ECT therapy. The Nurse
should carefully observe the client for?
A. Respiratory difficulties
B. Nausea and vomiting
C. Dizziness
D. Seizures

12. A 75 year old client is admitted to the hospital with the diagnosis of dementia of the
Alzheimer's type and depression. The symptom that is unrelated to depression would be?
A. Apathetic response to the environment
B. "I don't know" answer to questions
C. Shallow of labile effect
D. Neglect of personal hygiene
13. Nurse Trish is working in a mental health facility; the nurse priority nursing intervention for
a newly admitted client with bulimia nervosa would be to?
A. Teach client to measure I & O
B. Involve client in planning daily meal
C. Observe client during meals
D. Monitor client continuously

14. Nurse Patricia is aware that the major health complication associated with intractable
anorexia nervosa would be?
A. Cardiac dysrhythmias resulting to cardiac arrest
B. Glucose intolerance resulting in protracted hypoglycemia
C. Endocrine imbalance causing cold amenorrhea
D. Decreased metabolism causing cold intolerance

15. Nurse Anna can minimize agitation in a disturbed client by?


A. Increasing stimulation
B. limiting unnecessary interaction
C. increasing appropriate sensory perception
D. ensuring constant client and staff contact

16. A 39 year old mother with obsessive-compulsive disorder has become immobilized by her
elaborate hand washing and walking rituals. Nurse Trish recognizes that the basis of O.C.
disorder is often:
A. Problems with being too conscientious
B. Problems with anger and remorse
C. Feelings of guilt and inadequacy
D. Feeling of unworthiness and hopelessness

17. Mario is complaining to other clients about not being allowed by staff to keep food in his
room. Which of the following interventions would be most appropriate?
A. Allowing a snack to be kept in his room
B. Reprimanding the client
C. Ignoring the clients behavior
D. Setting limits on the behavior

18. Conney with borderline personality disorder who is to be discharge soon threatens to "do
something" to herself if discharged. Which of the following actions by the nurse would be most
important?
A. Ask a family member to stay with the client at home temporarily
B. Discuss the meaning of the client's statement with her
C. Request an immediate extension for the client
D. Ignore the clients statement because it's a sign of manipulation

19. Joey a client with antisocial personality disorder belches loudly. A staff member asks Joey,
"Do you know why people find you repulsive?" this statement most likely would elicit which of
the following client reaction?
A. Defensiveness
B. Embarrassment
C. Shame
D. Remorseful

20. Which of the following approaches would be most appropriate to use with a client suffering
from narcissistic personality disorder when discrepancies exist between what the client states and
what actually exist?
A. Rationalization
B. Supportive confrontation
C. Limit setting
D. Consistency

21. Cely is experiencing alcohol withdrawal exhibits tremors, diaphoresis and hyperactivity.
Blood pressure is 190/87 mmhg and pulse is 92 bpm. Which of the medications would the nurse
expect to administer?
A. Naloxone (Narcan)
B. Benzlropine (Cogentin)
C. Lorazepam (Ativan)
D. Haloperidol (Haldol)

22. Which of the following foods would the nurse Trish eliminate from the diet of a client in
alcohol withdrawal?
A. Milk
B. Orange Juice
C. Tea
D. Regular Coffee

23. Which of the following would Nurse Hazel expect to assess for a client who is exhibiting late
signs of heroin withdrawal?
A. Yawning & diaphoresis
B. Restlessness & Irritability
C. Constipation & steatorrhea
D. Vomiting and Diarrhea

24. To establish open and trusting relationship with a female client who has been hospitalized
with severe anxiety, the nurse in charge should?
A. Encourage the staff to have frequent interaction with the client
B. Share an activity with the client
C. Give client feedback about behavior
D. Respect client's need for personal space

25. Nurse Monette recognizes that the focus of environmental (MILIEU) therapy is to:
A. Manipulate the environment to bring about positive changes in behavior
B. Allow the client's freedom to determine whether or not they will be involved in activities
C. Role play life events to meet individual needs
D. Use natural remedies rather than drugs to control behavior

26. Nurse Trish would expect a child with a diagnosis of reactive attachment disorder to:
A. Have more positive relation with the father than the mother
B. Cling to mother & cry on separation
C. Be able to develop only superficial relation with the others
D. Have been physically abuse

27. When teaching parents about childhood depression Nurse Trina should say?
A. It may appear acting out behavior
B. Does not respond to conventional treatment
C. Is short in duration & resolves easily
D. Looks almost identical to adult depression

28. Nurse Perry is aware that language development in autistic child resembles:
A. Scanning speech
B. Speech lag
C. Shuttering
D. Echolalia

29. A 60 year old female client who lives alone tells the nurse at the community health center "I
really don't need anyone to talk to". The TV is my best friend.
The nurse recognizes that the client is using the defense mechanism known as?
A. Displacement
B. Projection
C. Sublimation
D. Denial

30. When working with a male client suffering phobia about black cats, Nurse Trish should
anticipate that a problem for this client would be?
A. Anxiety when discussing phobia
B. Anger toward the feared object
C. Denying that the phobia exist
D. Distortion of reality when completing daily routines
A. Discussion of the feared object triggers an emotional response to the object.

31. Linda is pacing the floor and appears extremely anxious. The duty nurse approaches in an
attempt to alleviate Linda's anxiety. The most therapeutic question by the nurse would be?
A. Would you like to watch TV?
B. Would you like me to talk with you?
C. Are you feeling upset now?
D. Ignore the client
32. Nurse Penny is aware that the symptoms that distinguish post traumatic stress disorder from
other anxiety disorder would be:
A. Avoidance of situation & certain activities that resemble the stress
B. Depression and a blunted affect when discussing the traumatic situation
C. Lack of interest in family & others
D. Re-experiencing the trauma in dreams or flashback

33. Nurse Benjie is communicating with a male client with substance-induced persisting
dementia; the client cannot remember facts and fills in the gaps with imaginary information.
Nurse Benjie is aware that this is typical of?
A. Flight of ideas
B. Associative looseness
C. Confabulation
D. Concretism

34. Nurse Joey is aware that the signs & symptoms that would be most specific for diagnosis
anorexia are?
A. Excessive weight loss, amenorrhea & abdominal distension
B. Slow pulse, 10% weight loss & alopecia
C. Compulsive behavior, excessive fears & nausea
D. Excessive activity, memory lapses & an increased pulse

35. A characteristic that would suggest to Nurse Anne that an adolescent may have bulimia
would be:
A. Frequent regurgitation & re-swallowing of food
B. Previous history of gastritis
C. Badly stained teeth
D. Positive body image

36. Nurse Monette is aware that extremely depressed clients seem to do best in settings where
they have:
A. Multiple stimuli
B. Routine Activities
C. Minimal decision making
D. Varied Activities

37. To further assess a client's suicidal potential. Nurse Katrina should be especially alert to the
client expression of:
A. Frustration & fear of death
B. Anger & resentment
C. Anxiety & loneliness
D. Helplessness & hopelessness

38. A nursing care plan for a male client with bipolar I disorder should include:
A. Providing a structured environment
B. Designing activities that will require the client to maintain contact with reality
C. Engaging the client in conversing about current affairs
D. Touching the client provide assurance

39. When planning care for a female client using ritualistic behavior, Nurse Gina must recognize
that the ritual:
A. Helps the client focus on the inability to deal with reality
B. Helps the client control the anxiety
C. Is under the client's conscious control
D. Is used by the client primarily for secondary gains

40. A 32 year old male graduate student, who has become increasingly withdrawn and neglectful
of his work and personal hygiene, is brought to the psychiatric hospital by his parents. After
detailed assessment, a diagnosis of schizophrenia is made. It is unlikely that the client will
demonstrate:
A. Low self esteem
B. Concrete thinking
C. Effective self boundaries
D. Weak ego

41. A 23 year old client has been admitted with a diagnosis of schizophrenia says to the nurse
"Yes, its march, March is little woman". That's literal you know". These statement illustrate:
A. Neologisms
B. Echolalia
C. Flight of ideas
D. Loosening of association

42. A long term goal for a paranoid male client who has unjustifiably accused his wife of having
many extramarital affairs would be to help the client develop:
A. Insight into his behavior
B. Better self control
C. Feeling of self worth
D. Faith in his wife

43. A male client who is experiencing disordered thinking about food being poisoned is admitted
to the mental health unit. The nurse uses which communication technique to encourage the client
to eat dinner?
A. Focusing on self-disclosure of own food preference
B. Using open ended question and silence
C. Offering opinion about the need to eat
D. Verbalizing reasons that the client may not choose to eat
44. Nurse Nina is assigned to care for a client diagnosed with Catatonic Stupor. When Nurse
Nina enters the client's room, the client is found lying on the bed with a body pulled into a fetal
position. Nurse Nina should?
A. Ask the client direct questions to encourage talking
B. Rake the client into the dayroom to be with other clients
C. Sit beside the client in silence and occasionally ask open-ended question
D. Leave the client alone and continue with providing care to the other clients

45. Nurse Tina is caring for a client with delirium and states that "look at the spiders on the
wall". What should the nurse respond to the client?
A. "You're having hallucination, there are no spiders in this room at all"
B. "I can see the spiders on the wall, but they are not going to hurt you"
C. "Would you like me to kill the spiders"
D. "I know you are frightened, but I do not see spiders on the wall"

46. Nurse Jonel is providing information to a community group about violence in the family.
Which statement by a group member would indicate a need to provide additional information?
A. "Abuse occurs more in low-income families"
B. "Abuser Are often jealous or self-centered"
C. "Abuser use fear and intimidation"
D. "Abuser usually have poor self-esteem"

47. During electroconvulsive therapy (ECT) the client receives oxygen by mask via positive
pressure ventilation. The nurse assisting with this procedure knows that positive pressure
ventilation is necessary because?
A. Anesthesia is administered during the procedure
B. Decrease oxygen to the brain increases confusion and disorientation
C. Grand mal seizure activity depresses respirations
D. Muscle relaxations given to prevent injury during seizure activity depress respirations.

48. When planning the discharge of a client with chronic anxiety, Nurse Chris evaluates
achievement of the discharge maintenance goals. Which goal would be most appropriately
having been included in the plan of care requiring evaluation?
A. The client eliminates all anxiety from daily situations
B. The client ignores feelings of anxiety
C. The client identifies anxiety producing situations
D. The client maintains contact with a crisis counselor

49. Nurse Tina is caring for a client with depression who has not responded to antidepressant
medication. The nurse anticipates that what treatment procedure may be prescribed?
A. Neuroleptic medication
B. Short term seclusion
C. Psychosurgery
D. Electroconvulsive therapy
50. Mario is admitted to the emergency room with drug-included anxiety related to over
ingestion of prescribed antipsychotic medication. The most important piece of information the
nurse in charge should obtain initially is the:
A. Length of time on the med.
B. Name of the ingested medication & the amount ingested
C. Reason for the suicide attempt
D. Name of the nearest relative & their phone number

51.The client with Alzheimer’s disease is being assisted with activities of daily living when the
nurse notes that the client uses her toothbrush to brush her hair. The nurse is aware that the client
is exhibiting:
 Agnosia
 Apraxia
 Anomia
 Aphasia
2) A client with Alzheimer’s disease is awaiting placement in a skilled nursing facility. Which
long-term plans would be most therapeutic for the client?
 Placing mirrors in several locations in the home
 Placing a picture of herself in her bedroom
 Placing simple signs to indicate the location of the bedroom, bathroom, and so on
 Alternating healthcare workers to prevent boredom
3) The client with dementia is experiencing confusion late in the afternoon and before bedtime.
The nurse is aware that the client is experiencing what is known as:
 Chronic fatigue syndrome
 Normal aging
 Sundowning
 Delusions
4) Which age group has the highest rate of Alzheimer’s cases reported?
 85 and older
 74 to 84
 65 to 74
 55 to 65
5) A 75 year old client is admitted to the hospital with the diagnosis of dementia of the
Alzheimer’s type and depression. The symptom that is unrelated to depression would be?
 Apathetic response to the environment
 “I don’t know” answer to questions
 Shallow of labile effect
 Neglect of personal hygiene
6) The client with confusion says to the nurse, “I haven’t had anything to eat all day long. When
are they going to bring breakfast?” The nurse saw the client in the day room eating breakfast
with other clients 30 minutes before this conversation. Which response would be best for the
nurse to make?
 “You know you had breakfast 30 minutes ago.”
 “I am so sorry that they didn’t get you breakfast. I’ll report it to the charge nurse.”
 “I’ll get you some juice and toast. Would you like something else?”
 “You will have to wait a while; lunch will be here in a little while.”
7) The nurse is caring for a client with stage III Alzheimer’s disease. A characteristic of this
stage is:
 Memory loss
 Failing to recognize familiar objects
 Wandering at night
 Failing to communicate
8) The primary nursing intervention in working with a client with moderate stage dementia is
ensuring that the client:
 receives adequate nutrition and hydration
 will reminisce to decrease isolation
 remains in a safe and secure environment
 independently performs self care
9) During the evaluation of the quality of home care for a client with Alzheimer’s disease, the
priority for the nurse is to reinforce which statement by a family member?
 At least 2 full meals a day is eaten.
 We go to a group discussion every week at our community center.
 We have safety bars installed in the bathroom and have 24 hour alarms on the doors.
 The medication is not a problem to have it taken 3 times a day.
10) Signs of Alzheimer’s include which of these symptoms?
 Loss of memory
 Increase in irritability
 Restlessness
 All of the above
11) Which neurotransmitter has been implicated in the development of Alzheimer’s disease?
 Acetylcholine
 Dopamine
 Epinephrine
 Serotonin
12) Alzheimer’s is an INSIDIOUS disease. This means:
 that it is terminal
 that is can be cured
 that it sneaks up on a person over time
 that it only affects the elderly
 none of the above
13) Edward, a 66 year old client with slight memory impairment and poor concentration is
diagnosed with primary degenerative dementia of the Alzheimer’s type. Early signs of this
dementia include subtle personality changes and withdrawal from social interactions. To assess
for progression to the middle stage of Alzheimer’s disease, the nurse should observe the client
for:
 Occasional irritable outbursts.
 Impaired communication.
 Lack of spontaneity.
 Inability to perform self-care activities.
14) Which of the following is not directly related with Alzheimer’s disease?
 Senile plaques
 Diabetes mellitus
 Tangles
 Dementia
15) Alzheimer’s is the most common form of which of these?
 Malnutrition
 Dementia
 Fatigue
 Psychosis
16) Which nursing intervention is most appropriate for a client with Alzheimer’s disease who has
frequent episodes emotional lability?
 Attempt humor to alter the client mood.
 Explore reasons for the client’s altered mood.
 Reduce environmental stimuli to redirect the client’s attention.
 Use logic to point out reality aspects.
17) Which of the following is the most common cause of dementia among elderly persons?
 Parkinson’s disease
 Multiple sclerosis
 Amyotrophic lateral sclerosis (Lou Gerhig’s disease)
 Alzheimer’s disease
18) Rosana is in the second stage of Alzheimer’s disease who appears to be in pain. Which
question by Nurse Jenny would best elicit information about the pain?
 “Where is your pain located?”
 “Do you hurt? (pause) “Do you hurt?”
 “Can you describe your pain?”
 “Where do you hurt?”
19) Rosana is in the second stage of Alzheimer’s disease who appears to be in pain. Which
question by Nurse Jenny would best elicit information about the pain?
 “Where is your pain located?”
 “Do you hurt? (pause) “Do you hurt?”
 “Can you describe your pain?”
 “Where do you hurt?”
20) How is Alzheimer’s diagnosed?
 Mental-status tests
 Blood tests
 Neurological tests
 All of the above
21) The usual span of years that Alzheimer’s may progress in the patient is:
 three to five years
 two to twenty years
 fifty to sixty years
 6 months to one year
 eight to ten years
22) Scientists believe that _________________ develop in the brain of an Alzheimer’s patient,
and may be a cause of the disease.
 cholesterols
 tumors
 ruptured blood vessels
 plaques and tangles
23) To encourage adequate nutritional intake for a female client with Alzheimer’s disease, the
nurse should:
 stay with the client and encourage him to eat.
 help the client fill out his menu.
 give the client privacy during meals.
 fill out the menu for the client.
24) A 93 year-old female with a history of Alzheimer’s Disease gets admitted to an Alzheimer’s
unit. The patient has exhibited signs of increased confusion and limited stability with gait.
Moreover, the patient is refusing to use a w/c. Which of the following is the most appropriate
course of action for the nurse?
 Recommend the patient remain in her room at all times.
 Recommend family members bring pictures to the patient’s room.
 Recommend a speech therapy consult to the doctor.
 Recommend the patient attempt to walk pushing the w/c for safety.
25) The doctor has prescribed Exelon (rivastigmine) for the client with Alzheimer’s disease.
Which side effect is most often associated with this drug?
 Urinary incontinence
 Headaches
 Confusion
 Nausea
26) A patient with Stage One Alzheimers might exhibit these behaviors:
 forgetting names
 missing appointments
 getting lost while driving
 all of the above
 none of the above
27) Which of the following diseases has not been directly linked with Bell’s palsy?
 AIDS
 Diabetes
 Lyme disease
 Alzheimer’s disease
28) The symptom of dementia that involved a more confused state after dark is called:
 dark retreat
 sundowning
 agitation
 dark reaction
29) Which of these is the strongest risk factor for developing the Alzheimer’s disease?
 Heredity
 Age
 Exposure to toxins
 None of the above
30) The priority of care for a client with Alzheimer’s disease is
 Help client develop coping mechanism
 Encourage to learn new hobbies and interest
 Provide him stimulating environment
 Simplify the environment to eliminate the need to make chores
31) An elderly client with Alzheimer’s disease becomes agitated and combative when a nurse
approaches to help with morning care. The most appropriate nursing intervention in this situation
would be to:
 Tell the client family that it is time to get dressed.
 Obtain assistance to restrain the client for safety.
 Remain calm and talk quietly to the client.
 Call the doctor and request an order for sedation.
32) Thomas Elison is a 79 year old man who is admitted with diagnosis of dementia. The doctor
orders a series of laboratory tests to determine whether Mr. Elison’s dementia is treatable. The
nurse understands that the most common cause of dementia in this population is:
 AIDS
 Alzheimer’s disease
 Brain tumors
 Vascular disease
33) A patient who has been admitted to the medical unit with new-onset angina also has a
diagnosis of Alzheimer’s disease. Her husband tells you that he rarely gets a good night’s sleep
because he needs to be sure she does not wander during the night. He insists on checking each of
the medications you give her to be sure they are the same as the ones she takes at home. Based
on this information, which nursing diagnosis is most appropriate for this patient?
 Decreased Cardiac Output related to poor myocardial contractility
 Caregiver Role Strain related to continuous need for providing care
 Ineffective Therapeutic Regimen Management related to poor patient memory
 Risk for Falls related to patient wandering behavior during the night
34) Physiologically, what happens to the brain as Alzheimer’s progresses?
 Tissue swells
 Fluid collects
 Many cells die
 Brain-stem atrophies
35) The nurse is aware that the following ways in vascular dementia different from Alzheimer’s
disease is:
 Vascular dementia has more abrupt onset
 The duration of vascular dementia is usually brief
 Personality change is common in vascular dementia
 The inability to perform motor activities occurs in vascular dementia
36) A 65 years old client is in the first stage of Alzheimer’s disease. Nurse Patricia should plan
to focus this client’s care on:
 Offering nourishing finger foods to help maintain the client’s nutritional status.
 Providing emotional support and individual counseling.
 Monitoring the client to prevent minor illnesses from turning into major problems.
 Suggesting new activities for the client and family to do together.
37) A nurse caring to a client with Alzheimer’s disease overheard a family member say to the
client, “if you pee one more time, I won’t give you any more food and drinks”. What initial
action is best for the nurse to take?
 Take no action because it is the family member saying that to the client
 Talk to the family member and explain that what she/he has said is not appropriate for the
client
 Give the family member the number for an Elder Abuse Hot line
 Document what the family member has said
38) Alzheimer’s disease is the secondary diagnosis of a client admitted with myocardial
infarction. Which nursing intervention should appear on this client’s plan of care?
 Perform activities of daily living for the client to decease frustration.
 Provide a stimulating environment.
 Establish and maintain a routine.
 Try to reason with the client as much as possible.
39) As the manager in a long-term-care (LTC) facility, you are in charge of developing a
standard plan of care for residents with Alzheimer’s disease. Which of these nursing tasks is best
to delegate to the LPN team leaders working in the facility?
 Check for improvement in resident memory after medication therapy is initiated.
 Use the Mini-Mental State Examination to assess residents every 6 months.
 Assist residents to toilet every 2 hours to decrease risk for urinary intolerance.
 Develop individualized activity plans after consulting with residents and family.
40) The nurse would expect a client with early Alzheimer’s disease to have problems with:
 Balancing a checkbook.
 Self-care measures.
 Relating to family members.
 Remembering his own name

1. Question
Nurse Rob has observed a co-worker arriving to work drunk at least three times in the past
month. Which action by Nurse Rob would best ensure client safety and obtain necessary
assistance for the co-worker?
A. Ignore the co worker's behavior, and frequently assess the clients assigned to the co-worker.
B. Make general statements about safety issues at the next staff meeting.
C. Report the coworker's behavior to the appropriate supervisor.
D. Warn the co-worker that this practice is unsafe.

2. Question
Elsa is being treated in a chemical dependency unit. She tells the nurse that she only uses drugs
when under stress and therefore does not have a substance problem. Which defense mechanism
is the client using?
A. Compensation
B. Denial
C. Suppression
D. Undoing

3. Question
Nurse Tara is teaching a community group about substance abuse. She explains that a genetic
component has been implicated in which of the following commonly abused substances?
A. Alcohol
B. Barbiturates
C. Heroin
D. Marijuana

4. Question
Nurse Julie recommends that the family of a client with substance-related disorder attend a
support group, such as Al-Anon and Alateen. The purpose of these groups is to help family
members understand the problem and to:
A. Change the problem behaviors of the abuser.
B. Learn how to assist the abuser in getting help.
C. Maintain focus on changing their own behaviors.
D. Prevent substance problems in vulnerable family members.

5. Question
Ryan who is a chronic alcohol abuser is being assessed by Nurse Gina. Which problems are
related to thiamine deficiency?
A. Cardiovascular symptoms, such as decreased hemoglobin and hematocrit levels.
B. CNS symptoms, such as ataxia and peripheral neuropathy.
C. Gastrointestinal symptoms, such as nausea and vomiting.
D. Respiratory symptoms, such as cough and sore throat.

6. Question
Nurse Wilma is teaching a client about disulfiram (Antabuse), which the client is taking to deter
his use of alcohol. She explains that using alcohol when taking this medication can result in:
A. Abdominal cramps and diarrhea
B. Drowsiness and decreased respiration
C. Flushing, vomiting, and dizziness
D. Increased pulse and blood pressure

7. Question
The nurse administers bromocriptine (Parlodel) to Bryan who is undergoing detoxification for
amphetamine abuse. The rationale for this medication is to:
A. Aid in GABA inhibition
B. Prevent norepinephrine excess
C. Restore depleted dopamine levels
D. Treat psychotic symptoms

8. Question
Which medication is commonly used in treatment programs for heroin abusers to produce a non-
euphoric state and to replace heroin use?
A. Diazepam
B. Carbamazepine
C. Clonidine
D. Methadone

9. Question
Nurse Christine is teaching an adolescent health class about the dangers of inhalant abuse; the
nurse warns about the possibility of:
A. Contracting an infectious disease, such as hepatitis or AIDS.
B. Recurrent flashback events.
C. Psychological dependence after initial use.
D. Sudden death from cardiac or respiratory depression

10. Question
The newly hired nurse at Nurseslabs Medical Center is assessing a client who abuses barbiturates
and benzodiazepine. The nurse would observe for evidence of which withdrawal symptoms?
A. Respiratory depression, stupor, and bradycardia
B. Anxiety, tremors, and tachycardia
C. Muscle aches, cramps, and lacrimation
D. Paranoia, depression, and agitation

11. Question
The community nurse practicing primary prevention of alcohol abuse would target which groups
for educational efforts?
A. Adolescents in their late teens and young adults in their early twenties.
B. Elderly men who live in retirement communities.
C. Women working in careers outside the home.
D. Women working in the hom
12. Question
Johnette is reviewing her lessons in Pharmacology. She is aware that the general classification of
drugs belonging to the opioid category is analgesic and:
A. Tranquilizing
B. Hallucinogenic
C. Stimulant
D. Depressant

13. Question
When a client abuses a CNS depressant, withdrawal symptoms will be caused by which of the
following?
A. Acetylcholine excess
B. Dopamine depletion
C. Serotonin inhibition
D. Norepinephrine rebound
14. Question
Kendall, the sister of a client with a substance-related disorder, tells the nurse she calls out sick
for her sister Kylie occasionally when the latter has too much to drink and cannot work. This
behavior can be described as:
A. Caretaking
B. Codependent
C. Helpful
D. Supportive

15. Question
During an initial assessment of a client admitted to a substance abuse unit for detoxification and
treatment, the nurse asks questions to determine patterns of use of substances. Which of the
following questions are most appropriate at this time? Select all that apply.
A. How long have you used substances?
B. How often do you use substances?
C. How do you get substances into your body?
D. Do you feel bad or guilty about your use of substances?
E. How much of each substance do you use?
F. Have you ever felt you should cut down substance use?
G. What substances do you use?

16. Question
The nurse is planning activities for a client who has bipolar disorder with aggressive social
behavior. Which of the following activities would be most appropriate for this client?
A. Ping pong
B. Writing
C. Chess
D. Basketball

17. Question
A client is admitted to the hospital with a diagnosis of major depression, severe, single episode.
The nurse assesses the client and identifies a nursing diagnosis of imbalanced nutrition related to
poor nutritional intake. The most appropriate nursing intervention related to this diagnosis is:
A. Explain to the client the importance of a good nutritional intake.
B. Weight the client 3 times per week before breakfast.
C. Report the nutritional concern to the psychiatrist and obtain a nutritional consultation as soon
as possible.
D. Consult with the nutritionist, offer the client several small meals per day, and schedule brief
nursing interactions with the client during these times.

18. Question
In planning activities for the depressed client, especially during the early stages of
hospitalization, which of the following plans is best?
A. Provide an activity that is quiet and solitary to avoid increased fatigue, such as working on a
puzzle or reading a book.
B. Plan nothing until the client asks to participate in milieu.
C. Offer the client a menu of daily activities and insist the client participate in all of them
D. Provide a structured daily program of activities and encourage the client to participate.

19. Question
The depressed client verbalizes feelings of low self-esteem and self-worth typified by statements
such as "I'm such a failure... I can't do anything right!" The best nursing response would be:
A. To tell the client this is not true; that we all have a purpose in life.
B. To remain with the client and sit in silence; this will encourage the client to verbalize feelings.
C. To reassure the client that you know how the client is feeling and that things will get better.
D. To identify recent behaviors or accomplishments that demonstrate skill ability.

A client with a diagnosis of major depression, recurrent with psychotic features is admitted to the
mental health unit. To create a safe environment for the client, the nurse most importantly
devises a plan of care that deals specifically with the client's:
A. Disturbed thought processes
B. Imbalanced nutrition
C. Self-care deficit
D. Deficient knowledge

21. Question
A depressed client is ready for discharge. The nurse feels comfortable that the client has a good
understanding of the disease process when the client states:
A. "I'll never let this happen to me again. I won't let my boss or my job or my family get to me!"
B. "It's important for me to eat well, exercise, and to take my medication. If I begin to lose my
appetite or not sleep well, I've got to get in to see my doctor."
C. "I've learned that I'm a good person and that I am worthy of giving and receiving love. I don't
need anyone; I have myself to rely on!"
D. "I don't know what happened to me. I've always been able to make decisions for myself and
for my business. I don't ever want to feel so weak or vulnerable again!"

22. Question
The nurse assesses a client with the admitting diagnosis of bipolar affective disorder, mania. The
symptom presented by the client that requires the nurse's immediate intervention is the client's:
A. Outlandish behaviors and inappropriate dress.
B. Grandiose delusions of being a royal descendant of King Arthur.
C. Nonstop physical activity and poor nutritional intake.
D. Constant, incessant talking that includes sexual innuendos and teasing the staff.

23. Question
The nurse reviews the activity schedule for the day and plans which activity for the manic client?
A. Brown-bag luncheon and book review
B. Tetherball
C. Paint-by-number activity
D. Deep breathing and progressive relaxation group

24. Question
A hospitalized client is being considered for ECT. The client appears calm, but the family is
anxious. The client's mother begins to cry and states "My son's brain will be destroyed. How can
the doctor do this to him?" The nurse's best response is:
A. "It sounds as though you need to speak with the psychiatrist"
B. "Your son has decided to have this treatment. You should be supportive of him."
C. "Perhaps you'd like to see the ECT room and speak to the staff."
D. "It sounds as though you have some concerns about the ECT procedure. Why don't we sit
down together and discuss any concerns you may have."

25. Question
The manic client announces to everyone in the dayroom that a stripper is coming to perform this
evening. When the nurse firmly states that this will not happen, the manic client becomes
verbally abusive and threatens physical violence to the nurse. Based on the analysis of this
situation, the nurse determines that the most appropriate action would be to:
A. With assistance, escort the manic client to her room and administer Haldol as prescribed if
needed.
B. Tell the client that smoking privileges are revoked for 24 hours.
C. Orient the client to time, person, and place
D. Tell the client that the behavior is not appropriate.

26. Question
Which of the following nursing interventions is applicable for a hospitalized client with mania
who is exhibiting manipulative behavior. Select all that apply.
A. Communicate expected behaviors to the client.
B. Enforce rules and inform the client that he or she will not be allowed to attend group therapy
sessions.
C. Ensure that the client knows that he or she is not in charge of the nursing unit.
D. Be clear with the client regarding the consequences of exceeding limits set regarding
behavior.
E. Assist the client in testing out alternative behaviors for obtaining needs.
27. Question
A woman comes into the ER in a severe state of anxiety following a car accident. The most
appropriate nursing intervention is to:
A. Remain with the client.
B. Put the client in a quiet room.
C. Teach the client deep breathing.
D. Encourage the client to talk about their feelings and concerns.

28. Question
When planning the discharge of a client with chronic anxiety, the nurse directs the goals at
promoting a safe environment at home. The most appropriate maintenance goal should focus on
which of the following?
A. Continued contact with a crisis counselor.
B. Identifying anxiety-producing situations.
C. Ignoring feelings of anxiety.
D. Eliminating all anxiety from daily situations.

29. Question
The nurse is monitoring a client who abuses alcohol for signs of alcohol withdrawal. Which of
the following would alert the nurse to the potential for delirium tremens?
A. Hypertension, changes in LOC, hallucinations
B. Hypotension, ataxia, hunger
C. Stupor, agitation, muscular rigidity
D. Hypotension, coarse hand tremors, agitation

30. Question
The spouse of a client admitted to the mental health unit for alcohol withdrawal says to the nurse
"I should get out of this bad situation." The most helpful response by the nurse would be:
A. "I agree with you. You should get out of this situation."
B. "What do you find difficult about this situation?"
C. "Why don't you tell your husband about this?"
D. "This is not the best time to make that decision."

31. Question
The nurse determines that the wife of an alcoholic client is benefiting from attending Al-Anon
group when she hears the wife say:
A. "My attendance at the meetings has helped me to see that I provoke my husband's violence."
B. "I no longer feel that I deserve the beatings my husband inflicts on me."
C. "I can tolerate my husband's destructive behavior now that I know they are common with
alcoholics."
D. "I enjoy attending the meetings because they get me out of the house and away from my
husband."

32. Question
The client has been hospitalized and is participating in a substance abuse therapy group sessions.
On discharge, the client has consented to participate in AA community groups. The nurse is
monitoring the client's response to the substance abuse sessions. Which statement by the client
best indicates that the client has developed effective coping response styles and has processed
information effectively for self use?
A. "I know I'm ready to be discharged. I feel I can say 'no' and leave a group of friends if they are
drinking... 'No Problem.'"
B. "This group has really helped a lot. I know it will be different when I go home. But I'm sure
that my family and friends will all help me like the people in this group have... They'll all help
me... I know they will... They won't let me go back to my old ways."
C. "I'm looking forward to leaving here. I know that I will miss all of you. So, I'm happy and I'm
sad, I'm excited and I'm scared. I know that I have to work hard to be strong and that everyone
isn't going to be as helpful as you people."
D. "I'll keep all my appointments; go to all my AA groups; I'll do everything I'm supposed to...
Nothing will go wrong that way."

33. Question
A hospitalized client with a history of alcohol abuse tells the nurse, "I am leaving now. I have to
go. I don't want any more treatment. I have things that I have to do right away." The client has
not been discharged. In fact, the client is scheduled for an important diagnostic test to be
performed in 1 hour. After the nurse discusses the client's concerns with the client, the client
dresses and begins to walk out of the hospital room. The most important nursing action is to:
A. Restrain the client until the physician can be reached.
B. Call security to block all areas.
C. Tell the client that the client cannot return to this hospital again if the client leaves now.
D. Call the nursing supervisor.

34. Question
What are the appropriate interventions for caring for the client in alcohol withdrawal. Select all
that apply.
A. Monitor vital signs.
B. Provide stimulation in the environment.
C. Maintain NPO status.
D. Provide reality orientation as appropriate.
E. Address hallucinations therapeutically.

35. Question
Which of the following nursing actions would be included in a care plan for a client with PTSD
who states the experience was "bad luck"?
A. Encourage the client to verbalize the experience.
B. Assist the client in defining the experience.
C. Work with the client to take steps to move on with his life.
D. Help the client accept positive and negative feelings.

36. Question
Which of the following psychological symptoms would the nurse expect to find in a hospitalized
client who is the only survivor of a train accident?
A. Denial
B. Indifference
C. Perfectionism
D. Trust

37. Question
Which of the following communication guidelines should the nurse use when talking with a
client experiencing mania?
A. Address the client in a light and joking manner.
B. Focus and redirect the conversation as necessary.
C. Allow the client to talk about several different topics.
D. Ask only open ended questions to facilitate conversations.

38. Question
What information is important to include in the nutritional counseling of a family with a member
who has bipolar disorder?
A. If sufficient roughage isn't eaten while taking lithium, bowel problems will occur.
B. If the intake of carbohydrates increases, the lithium level increases.
C. If the intake of calories is reduced, the lithium level will increase.
D. If the intake of sodium increases, the lithium level will decrease.

39. Question
In conferring with the treatment team, the nurse should make which of the following
recommendations for a client who tells the nurse that everyday thoughts of suicide are present?
A. A no-suicide contract
B. Weekly outpatient therapy
C. A second psychiatric opinion
D. Intensive inpatient treatment

40. Question
Which of the following short-term goals is most appropriate for a client with bipolar disorder
who is having difficulty sleeping?
A. Obtain medication for sleep
B. Work on solving a problem
C. Exercise before bedtime
D. Develop a sleep ritual

41. Question
Nurse Sharie is assessing a parent who abused her child. Which of the following risk factors
would the nurse expect to find in this case?
A. Flexible role functioning between parents.
B. History of the parent having been abused as a child.
C. Single-parent home situation.
D. Presence of parental mental illness.

42. Question
A group of nursing students at Nurseslabs University is currently learning about family violence.
Which of the following is true about the topic mentioned?
A. Family violence affects every socioeconomic level.
B. Family violence is caused by drugs and alcohol abuse.
C. Family violence predominantly occurs in lower socioeconomic levels.
D. Family violence rarely occurs during pregnancy.

43. Question
During a well-child checkup, a mother tells Nurse Rio about a recent situation in which her child
needed to be disciplined by her husband. The child was slapped in the face for not getting her
husband breakfast on Saturday, despite being told on Thursday never to prepare food for him.
Nurse Rio analyzes the family system and concludes it is dysfunctional. All of the following
factors contribute to this dysfunction except:
A. Conflictual relationships of parents
B. Inconsistent communication patterns
C. Rigid, authoritarian roles
D. Use of violence to establish control

45. Question
Joseph, a 12-year-old child, complains to the school nurse about nausea and dizziness. While
assessing the child, the nurse notices a black eye that looks like an injury. This is the third time
in 1 month that the child has visited the nurse. Each time, the child provides vague explanations
for various injuries. Which of the following is the school nurse's priority intervention?
A. Contact the child's parents and ask about the child's injury.
B. Encourage the child to be truthful with her.
C. Question the teacher about the parent's behavior.
D. Report suspicion of abuse to the proper authorities.

Nurse Meredith is observing 8-year-old Anna during a community visit. Which of the following
findings would lead the nurse to suspect that Anna is a victim of sexual abuse?
A. The child is fearful of the caregiver and other adults.
B. The child has a lack of peer relationships.
C. The child has self-injurious behavior.
D. The child has an interest in things of a sexual nature.

47. Question
Nurse Angela is working in the emergency department of Nurseslabs Medical Center. She is
conducting an interview with a victim of spousal abuse. Which step should the nurse take first?
A. Contact the appropriate legal services.
B. Ensure privacy for interviewing the victim away from the abuser.
C. Establish a rapport with the victim and the abuser.
D. Request the presence of a security guard.

48. Question
Mariefer is studying about abuse for the upcoming exam. For her to fully instill the topic, she
should know that the priority nursing intervention for a child or elder victim of abuse is:
A. Assess the scope of the abuse problem.
B. Analyze family dynamics.
C. Implement measures to ensure the victim's safety.
D. Teach appropriate coping skills.

49. Question
A community nurse conducts a primary prevention, home-visit assessment for a newborn and
mother. Mrs. Smith has three other children, the oldest of whom is age 12. She tells the nurse
that her 12-year-old daughter is expected to prepare family meals, to look after the young
children, and to clean the house once a week. Which of the following is the most appropriate
nursing diagnosis for this family situation?
A. Delayed growth and development, related to performance expectations of the child.
B. Anxiety (moderate), related to difficulty managing the home situation.
C. Impaired parenting, related to the role reversal of mother and child.
D. Social isolation, related to lack of extended family assistance.
50. Question
Mrs. Smith was admitted to the emergency department of Nurseslabs Medical Center with a
fractured arm. She explains to the nurse that her injury resulted when she provoked her drunken
husband, Mr. Smith, who then pushed her. Which of the following best describes the nurse's
understanding of the wife's explanation?
A. Mrs. Smith's explanation is appropriate acceptance of her responsibility.
B. Mrs. Smith's explanation is an atypical reaction of an abused woman.
C. Mrs. Smith's explanation is evidence that the woman may be an abuser as well as a victim.
D. Mrs. Smith's explanation is a typical response of a victim accepting blame for the abuser.

51. Question
Sheila tells the community nurse that her boyfriend has been abusive and she is afraid of him, but
she doesn't want to leave. The client asks the nurse for assistance. Which nursing interventions
are appropriate in this situation? Select all that apply.
A. Help Sheila to develop a plan to ensure safety, including phone numbers for emergency help.
B. Help Sheila to get her boyfriend into an appropriate treatment program.
C. Communicate acceptance, avoiding any implication that Sheila is at fault for not leaving.
D. Help Sheila to explore available options, including shelters and legal protection.
E. Tell Sheila that she should leave because things will not improve.
F. Reinforce concern for Sheila's safety and her right to be free of abuse.

52. Question
Which nursing assessment findings are physical signs of sexual abuse of a female child? Select
all that apply.
A. Enuresis
B. Red and swollen labia and rectum
C. Vaginal tears
D. Injuries in different stages of healing
E. Cigarette burns
F. Lice infestation

53. Question
During a prenatal assessment, the clinic nurse suspects that her client was abused. Which of the
following questions would be most appropriate?
A. "Are you being threatened or hurt by your partner?"
B. "Are you frightened of your partner?"
C. "Is something bothering you?"
D. "What happens when you and your partner argue?"

54. Question
Which situation would Nurse Sally identify as placing a client at high risk for caregiver abuse?
A. Antonia, an adult child, quits her job to move in and care for a parent with severe dementia.
B. Mr. Wright, an elderly man with severe heart disease, resides in a personal care home and is
frequently visited by his adult child.
C. Mrs. Hale, an elderly parent with limited mobility, lives alone and receives help from several
adult children.
D. Antoinette cares for her husband who is in early stages of Alzheimer's disease and has a
network of available support persons.

55. Question
The interventions common to treatment plans for survivors include which of the following?
Select all that apply.
A. Establish trust and rapport.
B. Identify areas of control.
C. Remove the client from home.
D. Support the client in the decisions he/she makes.
E. Encourage the client to pursue legal action.

56. Question
A man is admitted to the nursing care unit with a diagnosis of cirrhosis. He has a long history of
alcohol dependence. During the late evening following his admission, he becomes increasingly
disoriented and agitated. Which of the following would the client be least likely to experience?
A. Diaphoresis and tremors
B. Increased blood pressure and heart rate
C. Illusions
D. Delusions of grandeur

7. Question
Mr. Peterson, 35, is admitted for bipolar illness, manic phase, after assaulting his landlord in an
argument over Mr. Peterson is staying up all night playing loud music. Mr. Peterson is
hyperactive, intrusive, and has rapid, pressured speech. He has not slept in three days and
appears thin and disheveled. Which of the following is the most essential nursing action at this
time?
A. Providing a meal and beverage for Mr. Peterson to eat in the dining room.
B. Providing linens and toiletries for Mr. Peterson to attend to his hygiene.
C. Consulting with the psychiatrist to order a hypnotic to promote sleep.
D. Providing for client safety by limiting his privileges.

8. Question
Which of the following would best indicate to the nurse that a depressed client is improving?
A. Reduced levels of anxiety
B. Changes in vegetative signs
C. Compliance with medications
D. Requests to talk to the nurse

59. Question
An elderly man is admitted to the hospital. He was alert and oriented during the admission
interview. However, his family states that he becomes disruptive and disoriented around
dinnertime. One night he was shouting furiously and didn't know where he was. He was sedated
and the next morning he was fine. At dinnertime, the disruptive behavior returned. The client is
diagnosed as having sundown syndrome. The client's son asks the nurse what causes sundown
syndrome. The nurse's best response is that it is attributed to
A. An underlying depression
B. Inadequate cerebral flow
C. Changes in the sensory environment
D. Fluctuating levels of oxygen exchange

60. Question
The nurse is discussing electroconvulsive therapy (ECT) with a client who asks how long it will
be before she feels better. The nurse explains that the beneficial effects of ECT usually occur
within:
A. One week
B. Three weeks
C. Four weeks
D. Six week

61. Question
The nurse is assessing a 17-year-old female who is admitted to the eating disorders unit with a
history of weight fluctuation, abdominal pain, teeth erosion, receding gums, and bad breath. She
states that her health has been a problem but there are no other concerns in her life. Which of the
following assessments will be the least useful as the nurse develops the care plan?
A. Information regarding recent mood changes
B. Family functioning using a genogram
C. Ability to socialize with peers
D. Whether she has a sexual relationship with a boyfriend

A 34-year-old woman is admitted for treatment of depression. Which of these symptoms would
the nurse be least likely to find in the initial assessment?
A. Inability to make decisions
B. Feelings of hopelessness
C. Family history of depression
D. Increased interest in sex

63. Question
The nurse is planning care for a client who has a phobic disorder manifested by a fear of
elevators. Which goal would need to be accomplished first?
A. The client will demonstrate the relaxation response when asked.
B. The client will verbalize the underlying cause of the disorder.
C. The client will ride the elevator in the company of the nurse.

A. The client will demonstrate the relaxation response when asked.

A teenage female is admitted with the diagnosis of anorexia nervosa. Upon admission, the nurse
finds a bottle of assorted pills in the client's drawer. The client tells the nurse that they are
antacids for stomach pains. The best response by the nurse would be:
A. "These pills aren't antacids since they are all different."
B. "Some teenagers use pills to lose weight."
C. "Tell me about your week prior to being admitted."
D. "Are you taking pills to change your weight?"

65. Question
A mother with a Roman Catholic belief has given birth in an ambulance on the way to the
hospital. The neonate is in very critical condition with little expectation of surviving the trip to
the hospital. Which of these requests should the nurse in the ambulance anticipate and be
prepared to do?
A. The refusal of any treatment for self and the neonate until she talks to a reader.
B. The placement of a rosary necklace around the neonate's neck and not to remove it unless
absolutely necessary.
C. Arrange for a church elder to be at the emergency department when the ambulance arrives so
a "laying on hands" can be done.
D. Pour fluid over the forehead backward towards the back of the head and say "I baptize you in
the name of the father, the son and the holy spirit. Amen."

66. Question
Which statement by the client during the initial assessment in the emergency department is most
indicative of suspected domestic violence?
A. "I am determined to leave my house in a week."
B. "No one else in the family has been treated like this."
C. "I have only been married for two (2) months."
D. "I have tried leaving, but have always gone back."

67. Question
Which of these statements by the nurse reflects the best use of therapeutic interaction
techniques?
A. "You look upset. Would you like to talk about it?"
B. "I'd like to know more about your family. Tell me about them."
C. "I understand that you lost your partner. I don't think I could go on if that happened to me."
D. "You look very sad. How long have you been this way?"

8. Question
When planning the therapeutic milieu, it is most important to select group activities which:
A. Match the clients' preferences.
B. Are consistent with clients' skills.
C. Achieve clients' therapeutic goals.
D. Build skills of group participation.

69. Question
A client was admitted to the psychiatric unit for severe depression. After several days, the client
continues to withdraw from other clients. Which of the following would be the most appropriate
statement by the nurse to promote interaction with other clients?
A. "Your doctor thinks it's good for you to spend time with others."
B. "It is important for you to participate in group activities."
C. "Painting this picture will help you feel better."
D. "Come play Chinese Checkers with Gerry and me."

70. Question
The nurse can best ensure the safety of a demented client who wanders from the room by:
A. Repeatedly reminding the client of time and place.
B. Explaining the risks of becoming lost.
C. Using soft restraints.
D. Attaching a wander guard sensor band to the client's wrist.

71. Question
A client with paranoid thoughts refuses to eat because he believes the food has poisoned. The
most appropriate initial action is to
A. Taste the food in the client's presence.
B. Suggest that food be brought from home.
C. Simply state the food is not poisoned.
D. Inform the client he will be tube fed if he does not eat.

72. Question
The nurse is caring for a severely depressed client who has just been admitted to the in-client
psychiatric unit. Which of the following is a priority of care?
A. Nutrition
B. Elimination
C. Rest
D. Safety
A nurse is teaching a stress-management program for a client. Which of the following beliefs
will the nurse advocate as a method of coping with stressful life events?
A. Avoidance of stress is an important goal for living.
B. Control over one's response to stress is possible.
C. Most people have no control over their level of stress.
D. Significant others are important to provide care and concern.

74. Question
A student nurse is caring for a 75-year-old client who is very confused. The student's
communication tools should include:
A. Written directions for bathing.
B. Speaking very loudly.
C. Gentle touch while guiding ADLs (activities of daily living).
D. Flat facial expression.

When a husband takes out his work frustrations and anger by abusing his wife at home, the nurse
will identify this crisis as which type?
A. Psychiatric emergency crisis
B. Developmental crisis
C. Anticipated life transition
D. Dispositional crisis

A male client is diagnoses with schizotypal personality disorder. Which signs would this client
exhibit during social situation?
A. Paranoid thoughts
B. Emotional affect
C. Independence need
D. Aggressive behavior
A

Nurse Claire is caring for a client diagnosed with bulimia. The most appropriate initial goal for a
client diagnosed with bulimia is?
A. Encourage to avoid foods
B. Identify anxiety causing situations
C. Eat only three meals a day
D. Avoid shopping plenty of groceries.
B

Nurse Tony was caring for a 41 year old female client. Which behavior by the client indicates
adult cognitive development?
A. Generates new levels of awareness
B. Assumes responsibility for her actions
C. Has maximum ability to solve problems and learn new skills
D. Her perceptions are based on reality
A

A neuromuscular blocking agent is administered to a client before ECT therapy. The Nurse
should carefully observe the client for?
A. Respiratory difficulties
B. Nausea and vomiting
C. Dizziness
D. Seizures
A

A 75 year old client is admitted to the hospital with the diagnosis of dementia of the Alzheimer's
type and depression. The symptom that is unrelated to depression would be?
A. Apathetic response to the environment
B. "I don't know" answer to questions
C. Shallow of labile effect
D. Neglect of personal hygiene
C

Nurse Trish is working in a mental health facility; the nurse priority nursing intervention for a
newly admitted client with bulimia nervosa would be to?
A. Teach clients to measure I&O
B. Involve client in planning daily meal
C. Observe client during meals
D. Monitor client continuously
B

Nurse Patricia is aware that the major health complication associated with intractable anorexia
nervosa would be?
A. Cardiac dysrhythmias resulting to cardiac arrest
B. Glucose intolerance resulting in protracted hypoglycemia
C. Endocrine imbalance causing cold amenorrhea
D. Decreased metabolism causing cold intolerance
A

Nurse Anna can minimize agitation in a disturbed client by?


A. Increasing stimulation
B. Limiting unnecessary interaction
C. Increasing appropriate sensory perception
D. Ensuring constant client and staff contact
B

A 39 year old mother with obsessive-compulsive disorder has become immobilized by her
elaborate hand washing and walking rituals. Nurse Trish recognizes that the basis of O.C.
disorder is often:
A. Problems with being too conscientious
B. Problems with anger and remorse
C. Feelings of guilt and inadequacy
D. Feeling of unworthiness and hopelessness
C

Mario is complaining to other clients about not being allowed by staff to keep food in his room.
Which of the following interventions would be most appropriate?
A. Allowing a snack to be kept in his room
B. Reprimanding the client
C. Ignoring the clients behavior
D. Setting limits on the behavior
D

Conney with borderline personality disorder who is to be discharged soon threatens to "do
something" to herself if discharged. Which of the following actions by the nurse would be most
important?
A. Ask a family member to stay with the client at home temporarily.
B. Discuss the meaning of the client's statement with her
C. Request an immediate extension for the client
D. Ignore the clients statement because it's a sign of manipulation.
B

Joey, a client with antisocial personality disorder, belches loudly. A staff member asks Joey, "Do
you know why people find you repulsive?" This statement most likely would elicit which of the
following client reactions?
A. Defensiveness
B. Embarassment
C. Shame
D. Remorsefulness
A
Which of the following approaches would be most appropriate to use with a client suffering from
narcissistic personality disorder when discrepancies exist between what the client states and what
actually exist?
A. Rationalization
B. Supportive confrontation
C. Limit setting
D. Consistency
B

Cely is experiencing alcohol withdrawal exhibits tremors, diaphoresis, and hyperactivity. Blood
pressure is 190/87 mmhg and pulse is 92 bpm. Which of the medications would the nurse expect
to administer?
A. Naloxone (Narcan)
B. Benzlropine (Cogentin)
C. Lorazepam (Ativan)
D. Haloperidol (Haldol)
C

Which of the following foods would the nurse Trish eliminate from the diet of a client in alcohol
withdrawal?
A. Milk
B. Orange Juice
C. Soda
D. Regular Coffee
D

Which of the following would Nurse Hazel to assess for a client who is exhibiting late signs of
heroin withdrawal?
A. Yawning and diaphoresis
B. Restlessness and irritability
C. Constipation and steatorrhea
D. Vomiting and Diarrhea
D

To establish open and trusting relationship with a female client who has been hospitalized with
severe anxiety, the nurse in charge should?
A. Encourage the staff to have frequent interaction with the client
B. Share an activity with the client
C. Give client feedback about behavior
D. Respect client's need for personal space
D
Nurse Monette recognizes that the focus of environmental (MILIEU) therapy is to:
A. Manipulate the environment to bring about positive changes in behavior
B. Allow the client's freedom to determine whether or not they will be involved in activities
C. Role play life events to meet individual needs
D. Use natural remedies rather than drugs to control behavior
A

Nurse Trish would expect a child with a diagnosis of reactive attachment disorder to:
A. Have more positive relation with the father than the mother
B. Cling to mother and cry on separation
C. Be able to develop only superficial relation with the others
D. Have been physically abuse
C

When teaching parents about childhood depression Nurse Trina should say?
A. It may appear acting out behavior
B. Does not respond to conventional treatment
C. Is short in duration and resolves easily
D. Looks almost identical to adult depression
A

Nurse Perry is aware that language development in autistic child resembles:


A. Scanning Speech
B. Speech lag
C. Shuttering
D. Echolalia
D

A 60 year old female client who lives alone tells the nurse at the community health center "I
really don't need anyone to talk to". The TV is my best friend. The nurse recognizes that the
client is using the defense mechanism known as?
A. Displacement
B. Projection
C. Sublimation
D. Denial
D

When working with a male client suffering phobia about black cats, Nurse Trish should
anticipate that a problem for this client would be?
A. Anxiety when discussing phobia
B. Anger toward the feared object
C. Denying that the phobia exist
D. Distortion of reality when completing daily routines
A

Linda is pacing the floor and appears extremely anxious. The duty nurse approaches in an
attempt to alleviate Linda's anxiety. The most therapeutic question by the nurse would be?
A. Would you like to watch TV?
B. Would you like me to talk to you?
C. Are you feeling upset now?
D. Ignores the client.
B

Nurse Penny is aware that the symptoms that distinguish post traumatic stress disorder from
other anxiety disorder would be:
A. Avoidance of situation and certain activities that resemble the stress.
B. Depression and a blunted affect when discussing the traumatic situation
C. Lack of interest in family and others
D. Re-experiencing the trauma in dreams or flashback
D

Nurse Benjie is communicating with a male client with substance-induced persisting dementia;
the client cannot remember facts and fills in the gaps with imaginary information. Nurse Benjie
is aware that this is typical of?
A. Flight of ideas
B. Associative Looseness
C. Confabulation
D. Concretism
C

Nurse Joey is aware that the signs and symptoms that would be most specific for diagnosis
anorexia are?
A. Excessive weight loss, amenorrhea & abdominal distension
B. Slow pulse, 10% weight loss and alopecia
C. Compulsive behavior, excessive fears and nausea
D. Excessive activity, memory lapses and an increased pulse
A

A characteristic that would suggest to Nurse Anne that an adolescent may have bulimia would
be:
A. Frequent regurgitation and re-swallowing food
B. Previous history of gastritis
C. Badly stained teeth
D. Positive body image
C

Nurse Monette is aware that extremely depressed cleints seem to do best in settings where they
have:
A. Multiple stimuli
B. Routine Activities
C. Minimum decision making
D. Varied Activities
B

To further assess a client's suicidal potential. Nurse Katrina should be especially alert to the
client expression of:
A. Frustration and fear of death
B. Anger and resentment
C. Anxiety and loneliness
D. Helplessness and hopelessness
D

A nursing care plan for a male client with bipolar 1 disorder should include:
A. Providing a structured environment
B. Designing activities that will require the client to maintain contact with reality
C. Engaging the client in conversing about current affairs
D. Touching the client provide assuranve
A

When planning care for a female client using ritualistic behavior, Nurse Gina must recognize that
the ritual:
A. Helps the client focus on the inability to deal with reality
B. Helps the client control the anxiety
C. Is under the client's conscious control
D. Is used by the client primarily for secondary gains
B

A 32 year old male graduate student, who has become increasingly withdrawn and neglectful of
his work and personal hygiene, is brought to the psychiatric hospital by his parents. After
detailed assessment, a diagnosis of schizophrenia is made. It is unlikely that the client will
demonstrate:
A. Low self esteem
B. Concrete thinking
C. Effective self boundaries
D. Weak ego
C

A 23 year old client has been admitted with a diagnosis of schizophrenia says to the nurse "Yes,
it's march, March is little woman". That's literal you know". These statement illustrate:
A. Neologisms
B. Echolalia
C. Flight of ideas
D. Loosening of association
D

A long term goal for a paranoid male client who has unjustifiably accused his wife of having
many extramarital affairs would be to help the client develop:
A. Insight into his behavior
B. Better self control
C. Feeling of self worth
D. Faith in his wife
C

A male client who is experiencing disordered thinking about food being poisoned is admitted to
the mental health unit. The nurse uses which communication technique to encourage the client to
eat dinner?
A. Focusing on self-disclosure of own food preference
B. Using open ended questions and silence
C. Offering opinion about the need to eat
D. Verbalizing reasons that the client may not choose to eat
B

Nurse Nina is assigned to care for a client diagnosed with Catatonic Stupor. When nurse Nina
enters the client's room, the client is found lying on the bed with a body pulled into a fetal
position. Nurse Nina should?
A. Ask the client direct questions to encourage talking
B. Rake the client into the dayroom to be with other clients
C. Sit beside the client in silence and occasionally ask open-ended questions.
D. Leave the client alone and continue with providing care to the other clients
C

Nurse Tina is caring for client with delirium and states that "look at the spiders on the wall".
What should the nurse respond to the client?
A. "You're having hallucination, there are no spiders in this room at all."
B. "I can see the spiders on the wall, but they are not going to hurt you."
C. "Would you like me to kill the spiders."
D. "I know you are frightened, but I do not see spiders on the wall."
D

Nurse Jonel is providing information to a community group about violence in the family. Which
statement by a group member would indicate a need to provide additional information?
A. "Abuse occurs more in low-income families"
B. "Abuser are often jealous or self-centered."
C. "Abuser use fear and intimidation."
D. "Abuser usually have poor self-esteem."
A

During electroconvulsive therapy (ECT) the client recieves oxygen by mask via positive pressure
ventilation. The nurse assisting with this procedure knows that positive pressure ventilation is
necessary because?
A. anesthesia is administered during the procedure
B. Decrease oxygen to the brain increases confusion and disorientation
C. Grand mal seizure activity depresses respirations
D. Muscle relaxations given to prevent injury during seizure activity depress respirations.
D

When planning the discharge of a client with chronic anxiety, Nurse Chris evaluates achievement
of the discharge maintenance goals. Which goal would be most appropriately having been
included in the plan of care requiring evaluation?
A. The client eliminates all anxiety from daily situations
B. The client ignores feelings of anxiety
C. The client identifies anxiety producing situations
D. The client maintains contact with a crisis counselor
C

Nurse Tina is caring for a client with depression who has not responded to antidepressant
medication. The nurse anticipates that what treatment procedure may be prescribed?
A. Neuroleptic Medication
B. Short term seclusion
C. Psychotherapy
D. Electroconvulsive therapy
D
Mario is admitted to the emergency room with drug-included anxiety related to over ingestion of
prescribed antipsychotic medication . The most important piece of information the nurse in
charge should obtain initially is the:
A. Length of time on the med
B. Name of the ingested medication and the amount ingested
C. Reason for the suicide attempt
D. Name of the nearest relative and their phone number
B

Terms in this set (40)


Original

The most appropriate outocome of care for a male cleint who has experienced a dissociative
fugue is that the client will do which of the following?
A. Remember what occured during his fugue state
B. Gain additonal coping skills to deal with his current problems
C. Report no feelings of being detached from his body
D. State three positive aspects about himself
B. The client who gains coping skills reduces anxiety to a levle at which dissociation is unlikely
to occure. The client does not remember what occured during th fugue state, nor does he
experience depersonalization.

A client with dissociatve identity disorder (DID) who has been admitted with several fresh burns
on her ankles and wrists is refusing to attend group therapy. What is the priority nursing
diagnosis?
A. Self-care deficit
B. Impaired sensory perception
C. Risk for self-mutilation
D. Noncompliance
C. Self-mutilation, not common with DID clients, is identified in the assessment. The client
remains at risk for injuring herself, producing tissue damage that provides tension relief. There is
not intent to kill; however, the client will need to learn less damaging ways to obtain relief.

A client reports depersonalization experiences that have been frightening to him. Which of the
following is the most therapeutic response by the nurse?
A. "It must be very scary for you. Tell me more about how they occur."
B. "Don't worry, you will always come back together."
C. "Being in the hospital must be very frightening."
D. "Let's focus on the stresses in your life."
A. The response demonstrates empathy and encourages the client to elaborate further about his
experience. Options 2 and 3 dismss the affective component of miss the point of the client's
statement. Option 4 is helpful is making connections between events but is not the best response
to the client's original comment.

Which of the followng behaviors would indicate that care for a client who dissociates has been
effective?
A. Client reports dissociative episodes to the nurse
B. Cleint seeks out social relationships
C. Client demonstrates three stress management techniques
D. Ckeint is free from injury
C. The goal of care is to eliminate or reduce dissociative experiences, which can be
accomplished in part by anxiety-produced stress-management techniques.

A client with amnesia is hospitalized. What might the nurse expect to find during the initial
assessment?
A. Confabulation of historical information
B. Gradual loss of memory over months
C. Disheveled appearance
D. History of severe stress
D. Amnesia is precipitated by stress related to trauma or conflict. The amnesia occurs abruptly
and there is no attempt to cover the memory loss. Confabulation, gradual loss of memory, and
disheveled apperance are common in clients experiencing dementia.

A cleint continues to have pain despite negative neurological findings. The nurse concludes that
such pain is likely to continue because of which of the following?
A. Secondary gain
B. High endophine levels
C. Structural changes of tissue
D. Derealization
A. The continuance of pain is related to reinforcement of the symptoms, such as the caring
responses of others, which give the client benefits that otherwise might not occur. There is no
organic basis for the pain. High endorphin levels are associated with feelins of euphoria.

A female client with illness anxiety disorder discloses that she may decide to leave the
psychiatric facility without completing her course of treatment and seek exploratory surgery. The
nurse's best response is which of the following?
A. "If you decide to leave now, you will be committed against your will."
B. "You should not go until your doctor releases you. She knows what you need."
C."Tell me more about your decision."
D. "Your surgery will just prove useless. Please stay."
C. Exploration of the client's decision is nonjudgmental and affirms the client's personal power.
The response also helps the client understand connections in her own decision-making process.

The nurse evaluates that the plan of care for a client who suddenly lost her hearing (diagnosed as
a conversion disorder) was effective if the client:
A. resumed normal hearing
B. began learning sign language
C. was fitted for a hearing aid
D. agreed to have a stapedectomy
A. When stressors and anxiety are decreased, there remains no need for conversion symptoms,
and normal function resumes.

Which of the following approaches would be best for the nurse who is communicating with the
cognitively impaired client?
A. Loud and precise
B. Simple and direct
C. As nonberbal as possible
D. Sign language
B. Verbal communication should be clear, concise, and unhurried. Shouting may be interpreted
as anger; therefore, a pleasant, calm, supportive tone of voice should be used. The use of sign
language or mostly nonverbal gestures would be frustrating to the client who may not understand
what is being said.

Which of the following evaluation criteria should the nurse give first priority to when planning
the care of a client with dementia?
A. Preventing further deterioration
B. Finding suitable nursing home placement
C. Supporting family caregivers
D. Preventing injury
D. The most important area of concern indentify by both familyand staff is the safety of clients
with dementia. The risk for injuryis always present in clients with dementia, and as the disease
progresses, the need for a safe and secure environment increases. The other options are
appropriate for dementia but are not the first priority.

A client with suspected Alzheimer's disease is undergoing diagnostic workup. Whe the family
asks the nuse the reasons for the "tests", the nurse responds that the diagnosis of Alzheimer's is
usually based on which of the following?
A. Abnormal laboratory findings
B. A definitive CT scan
C. Physiological findings
D. Ruling out other causes for symptoms
D. Alzheimer''s disease is diagnosed by ruling out causes for the client's symptoms. The only
definitive method of diagnosis is post mortem examination of brain tissue.

A client rates anxiety at 8 out of 10 on a scale of 1 to 10, is restless, and has narrowed
perceptions. Which of the following medications would appropriately be prescribed to address
these symptoms? Select all that apply.
A. Chlordiazepoxide (Librium).
B. Clonazepam (Klonopin).
C. Lithium carbonate (Lithium).
D. Clozapine (Clozaril).
E. Oxazepam (Serax)
A. Chlordiazepoxide (Librium) is a benzodiazepine. Benzodiazepines are classified as
antianxiety medications and would be appropriately prescribed to
address signs and symptoms of anxiety.
B. Clonazepam (Klonopin) is a benzodiazepine. Benzodiazepines are classified as antianxiety
medications and would be appropriately prescribed to
address signs and symptoms of anxiety.
E. Oxazepam (Serax) is a benzodiazepine. Benzodiazepines are classified as antianxiety
medications and would be appropriately prescribed to address signs and symptoms of anxiety.

TEST-TAKING HINT: The test taker first must recognize the signs and symptoms presented in
the question as an indication of increased levels of anxiety. Next, the test taker must recognize
the medications that address these symptoms. Also, it is common to confuse lithium carbonate
(lithium) and Librium and clozapine and clonazepam. To answer this question correctly, the test
taker needs to distinguish between medications that are similar in spelling.

A client diagnosed with generalized anxiety disorder is placed on clonazepam (Klonopin) and
buspirone (BuSpar). Which client statement indicates teaching has been effective?
A. The client verbalizes that the clonazepam (Klonopin) is to be used for long-term
therapy in conjunction with buspirone (BuSpar).
B. The client verbalizes that buspirone (BuSpar) can cause sedation and should be
taken at night.
C. The client verbalizes that clonazepam (Klonopin) is to be used short-term until the
buspirone (BuSpar) takes full effect.
D. The client verbalizes that tolerance can result with long-term use of buspirone
(BuSpar).
C. Clonazepam would be used for shortterm treatment while waiting for the buspirone to take
full effect, which can take 4 to 6 weeks.

TEST-TAKING HINT: To answer this question correctly, the test taker must note appropriate
teaching needs for clients prescribed different classifications of antianxiety medications.
A client newly admitted to an in-patient psychiatric unit is diagnosed with obsessive compulsive
disorder. Which behavioral symptom would the nurse expect to assess?
A. The client uses excessive hand washing to relieve anxiety.
B. The client rates anxiety at 8/10.
C. The client uses breathing techniques to decrease anxiety.
D. The client exhibits diaphoresis and tachycardia.
A. Using excessive hand washing to relieve
anxiety is a behavioral symptom exhibited
by clients diagnosed with obsessivecompulsive disorder (OCD).

TEST-TAKING HINT: To answer this question correctly, the test taker must be able to
differentiate various classes of symptoms exhibited by clients diagnosed with OCD. The
keyword "behavioral"
determines the correct answer to this question.

Anxiety is a symptom that can result from which of the following physiological conditions?
Select all that apply.
A. Chronic obstructive pulmonary disease.
B. Hyperthyroidism.
C. Hypertension.
D. Diverticulosis.
E. Hypoglycemia.
A. Chronic obstructive pulmonary disease
causes shortness of breath. Air deprivation
causes anxiety, sometimes to the point of
panic.
B. Hyperthyroidism (Graves's disease)
involves excess stimulation of the sympathetic nervous system and excessive levels of thyroxine.
Anxiety is one of several
symptoms brought on by these increases.
E. Marked irritability and anxiety are some
of the many symptoms associated with
hypoglycemia.

TEST-TAKING HINT: To answer this question correctly, the test taker needs to understand that
anxiety is manifested by physiological responses.

The nurse has received evening report. Which client would the nurse need to assess first?
A. A newly admitted client with a history of panic attacks.
B. A client who slept 2 to 3 hours last night because of flashbacks.
C. A client pacing the halls and stating that his anxiety is an 8/10.
D. A client diagnosed with generalized anxiety disorder awaiting discharge.
C. A client pacing the halls and experiencing an increase in anxiety commands immediate
assessment. If the nurse does not take action on this assessment, there is a potential for client
injury to self or others.

TEST-TAKING HINT: When the nurse is prioritizing


client assessments, it is important to note which client might be a safety risk. When asked to
prioritize, the test taker must review all the situations presented before deciding which one to
address first.

The elderly client diagnosed with a panic attack disorder is in the busy day room of a long-term
care facility and appears anxious, is starting to hyperventilate, is trembling, and is sweating.
Which action should the nurse implement first?
A. Administer the benzodiazepine alprazolam (Xanax)
B. Assess the client's vital signs
C. Remove the client from the day room
D. Administer the selective serotonin reuptake inhibitor (SSRI) sertraline (Zoloft)
C. Remove the client from the day room
This is the most appropriate intervention; the nurse should remove the client from the busy day
room to help decrease the anxiety attack

A client presents with symptoms of paralysis but no organic cause can be found. The client has
recently undergone an emotionally traumatic event. What is a possible diagnosis:
A. Epiglottitis
B. Esophageal atresia
C. Somatoform disorder
D. Garvin's syndrome
A. Somatoform disorders are when psychological issues translate into physical symptoms. This
can be as severe as somatoform paralysis or even pregnancy.

A soldier returning from deployment is diagnosed with post-traumatic stress disorder or PTSD.
What is not a symptom to be expected?
A. Anxiety
B. Flashbacks
C. Dementia
D. Worsened concentration
C. Dementia is not associated with PTSD while the other symptoms listed are.

An old woman was brought for evaluation due to the hospital for evaluation due to increasing
forgetfulness and limitations in daily function.She tearfully tells the nurse "I can't take it when
she accuses me of stealing her things." Which response by the nurse will be most therapeutic?
A. "Don't take it personally. Your mother does not mean it."
B. "Have you tried discussing this with your mother?"
C. "This must be difficult for you and your mother."
D. "Next time ask your mother where her things were last seen."
C

An old woman was brought for evaluation due to the hospital for evaluation due to increasing
forgetfulness and limitations in daily function.Dementia unlike delirium is characterized by:
A. slurred speech
B. insidious onset
C. clouding of consciousness
D. sensory perceptual change
D.

Which is the desired outcome in conducting desensitization:


A. The client verbalize his fears about the situation
B. The client will voluntarily attend group therapy in the social hall.
C. The client will socialize with others willingly
D. The client will be able to overcome his disabling fear.
D.

An old woman was brought for evaluation due to the hospital for evaluation due to increasing
forgetfulness and limitations in daily function. The daughter revealed that the client used her
toothbrush to comb her hair. She is manifesting:
A. apraxia
B. aphasia
C. agnosia
D. amnesia
C.

A female victim of a sexual assault is being seen in the crisis center. The patient states that she
still feels "as though the rape just happened yesterday," even though it has been a few months
since the incident. What is the most appropriate nursing response?
A. "You need to try to be realistic. The rape did not just occur."
B. "It will take some time to get over these feelings about your rape."
C. "Tell me more about the incident that causes you to feel like the rape just occured."
D. "What do you think that you can do to alleviate some of your fears about being raped again?"
C. The correct option allows the pateint to express her ideas and feelings more fully and portrays
a nonhurried, nonjudgemental, supportive attitude on the part of the nurse. Clients need to be
reassured that their feelings are normal and tthat they may express their concerns freely in a safe,
caring environment.
The nurse performing an assessment on a client with dementia. Which data gathered during the
assessment indicates a manifestation associated with dementia?
A. Uses confabulation
B. Improvement in sleeping
C. Abscence of sundown syndrome
D. Presence of personal hygienic care
A. The clinical picture of dementia ranges from mild cognitive deficits to severe, life threatening
alterations in neurological functioning. For th client to use confabulation or the fabrication of
events or experiences to fill in memory gaps is not usual.

The emergency department nurse is caring for a client who has been identified as a victim of
physical abuse. In planning care for the client, which is the priority nursing action?
A. Adhering to the mandatory abuse-reporting laws.
B. Notifying the case worker of the family situation
C. Removing the client from any immediate danger.
D. Obtaining treatment for the abusing family member.
C. Whenever an abused client remains in the abusive environment, priority must be placed on
ascertaining whether the client is in any immediate danger. If so, emergency action must be
taken to remove the client from the abusing situation.

A 69-year-old client is admitted and diagnosed with delirium. Later in the day, he tries to get out
of the locked unit. He yells, "Unlock this door. I've got to go see my doctor. I just can't miss my
monthly Friday appointment." Which of the following responses by the nurse is most
appropriate?
A.. "Please come away from the door. I'll show you your room."
B. "It's Tuesday and you are in the hospital. I'm Anne, a nurse."
C. "The door is locked to keep you from getting lost."
D. "I want you to come eat your lunch before you go the doctor."
B. Loss of orientation, especially for time and place, is common in delirium. The nurse should
orient the client by telling him the time, date, place, and who the client is with. Taking the client
to his room and telling him why the door is locked does not address his disorientation. Telling
the client to eat before going to the doctor reinforces his disorientation.

When caring for the client diagnosed with delirium, which condition is the most important for
the nurse to investigate?
A. Cancer of any kind.
B. Impaired hearing.
C. Prescription drug intoxication.
D. Heart failure.
C. Polypharmacy is much more common in the elderly. Drug interactions increase the incidence
of intoxication from prescribed medications, especially with combinations of analgesics, digoxin,
diuretics, and anticholinergics. With drug intoxication, the onset of the delirium typically is
quick. Although cancer, impaired hearing, and heart failure could lead to delirium in the elderly,
the onset would be more gradual.

A nurse on the Geropsychiatric unit receives a call from the son of a recently discharged client.
He reports that his father just got a prescription for memantine (Namenda) to take "on top of his
donepezil (Aricept)." The son then asks, "Why does he have to take extra medicines?" The nurse
should tell the son:
A. "Maybe the Aricept alone isn't improving his dementia fast enough or well enough."
B. "Namenda and Aricept are commonly used together to slow the progression of dementia."
C. "Namenda is more effective than Aricept. Your father will be tapered off the Aricept."
D. "Aricept has a short half-life and Namenda has a long half-life. They work well together."
B. The two medicines are commonly given together. Neither medicine will improve dementia,
but may slow the progression. Neither medicine is more effective than the other; they act
differently in the brain. Both medicines have a half-life of 60 or more hours.

A client is experiencing agnosia as a result of vascular dementia. She is staring at dinner and
utensils without trying to eat. Which intervention should the nurse attempt first?
A. Pick up the fork and feed the client slowly.
B. Say, "It's time for you to start eating your dinner."
C. Hand the fork to the client and say, "Use this fork to eat your green beans."
D. Save the client's dinner until her family comes in to feed her.
C. Agnosia is the lack of recognition of objects and their purpose. The nurse should inform the
client about the fork and what to do with it. Feeding the client does not address the agnosia or
give the client specific directions. It should only be attempted if identifying the fork and
explaining what to do with it is ineffective. Waiting for the family to care for the client is not
appropriate unless identifying the fork and explaining or feeding the client are not successful.

The client in the early stage of Alzheimer's disease and his adult son attend an appointment at the
community mental health center. While conversing with the nurse, the son states, "I'm tired of
hearing about how things were 30 years ago. Why does Dad always talk about the past?" The
nurse should tell the son:
A. "Your dad lost his short-term memory, but he still has his long-term memory."
B. "You need to be more accepting of your dad's behavior."
C. "I want you to understand your dad's level of anxiety."
D. "Telling your dad that you are tired of hearing about the past will help him stop."
A. The son's statements regarding his father's recalling past events is typical for family members
of clients in the early stage of Alzheimer's disease, when recent memory is impaired. Telling the
son to be more accepting is critical and not an attempt to educate. Understanding the client's
level of anxiety is unrelated to the memory loss of Alzheimer's disease. The client cannot stop
reminiscing at will.
An older client with chronic low back pain receives cooking and cleaning help from her
extended family. The mental health nurse anticipates that this client benefits from which of the
following in this situation?
A. Primary gain
B. Secondary gain
C. Attention-seeking
D. Malingering
B.

What would the nurse expect a client who has a somatization disorder to reveal in the nursing
history?
A. Abrupt onset of physical symptoms at menopause
B. Episodes of personality dissociation
C. Ignoring physical symptoms until role performance
was altered
D. Numerous physical symptoms in many organ areas
D.

The spouse of a client who is experiencing a fugue state asks the nurse if the spouse will be able
to remember what happened during the time of fugue. What is the nurse's best response?
A. "Your spouse will probably have no memory for events during the fugue."
B. "Your spouse will be able to tell you - if you can gently encourage talking."
C. "It is not possible to predict whether your spouse will remember the fugue state."
D. "Avoid mentioning it, or your spouse may start alternating old and new identities."
A.

Lorraine, a client diagnosed with Somatization Disorder, states, "My doctor thinks I should see a
psychiatrist. I can't imagine why he would make such a suggestion!" What is the basis for
Lorraine's statement?
A. She thinks her doctor wants to get rid of her as a client.
B. She does not understand the correlation of symptoms an stress.
C. She thinks psychiatrists are only for "crazy" people.
D. She thinks her doctor has made an error in diagnosis.
B. She does not understand the correlation of symptoms an stress.

Ellen has a history of childhood physical and sexual abuse. She was diagnosis with Dissociative
Identity Disorder (DID) 6 years ago. She has been admitted to the psychiatric unit following a
suicide attempt. The primary nursing diagnosis for Ellen would be:
A. Disturbed personal identity r/t childhood abuse.
B. Disturbed sensory perception r/t repressed anxiety.
C. Disturbed thought process related to memory deficit.
D. Risk for suicide related to unresolved grief.
D. Risk for suicide related to unresolved grief.

A nurse on the psychiatric unit has the understanding that which of the following would describe
a client with "la belle indifference"?
a. A client with somatization disorder who is demanding narcotics.
B. A client with depersonalization disorder who believes he has died.
C. A client with DID who is threatening suicide.
D. A client with conversion disorder showing lack of concern for an altered body function.
D. A client with conversion disorder showing lack of concern for an altered body function.

A client is brought to an emergency department after being violently raped. Which nursing
action is appropriate?
A. Discourage the client from discussing the event as this may lead to further emotional trauma.
B. Remain nonjudgmental and actively listen to the client's description of the event.
C. Meet the client's self-care needs by assisting with showering and perineal care.
D. Provide cues, based on police information, to encourage further description of the event.
B. The most appropriate nursing action is to remain nonjudgmental and actively listen to the
client's description of the event. It is important to also communicate to the victim that he or she is
safe and that it is not his or her fault. Nonjudgmental listening provides an avenue for client
catharsis needed in order to begin the process of healing.

A college student was sexually assaulted when out on a date. After several weeks of crisis
intervention therapy, which client statement should indicate to a nurse that the student is
handling this situation in a healthy manner?
A. "I know that it was not my fault."
B. "My boyfriend has trouble controlling his sexual urges."
C. "If I don't put myself in a dating situation, I won't be at risk."
D. "Next time I will think twice about wearing a sexy dress."
A.The client who realizes that sexual assault was not her fault is handling the situation in a
healthy manner. The nurse should provide nonjudgmental listening and communicate statements
that instill trust and validate self-worth.

A kindergarten student is frequently violent toward other children. A school nurse notices bruises
and burns on the child's face and arms. What other symptom should indicate to the nurse that the
child might have been physically abused?
A. The child shrinks at the approach of adults.
B. The child begs or steals food or money.
C. The child is frequently absent from school.
D. The child is delayed in physical and emotional development.
A. The nurse should determine that a child who shrinks at the approach of adults in addition to
having bruises and burns might be a victim of abuse. Whether or not the adult intended to harm
the child, maltreatment should be considered.
er 11
Study

Chapter 11

12 studiers today

Leave the first rating


Flashcards

Learn

Test

Match

Created by
joehan84
psych cert

Share

NURSING CARE OF CLIENTS WITH DISORDERS RELATED


TO ALTERATIONS IN COGNITION AND PERCEPTION
Study

NURSING CARE OF CLIENTS WITH DISORDERS RELATED TO ALTERATIONS IN


COGNITION AND PERCEPTION

11 studiers recently

5.0 (2 reviews)

Flashcards
Learn

Test

Match

Created by
Dixie_Nora1994

Share

Terms in this set (49)


Original

When conducting a mini-mental status exam on an older client, the nurse should test specifically
for short-term memory by asking the client to:

A. Subtract serial 7s from 100


B. Copy a simple geometric figure
C. State three random words mentioned earlier in the exam
D. Name two common objects when the nurse points to them
C. State three random words mentioned earlier in the exam

This technique tests the client's ability to recall from short-term memory

The nurse recognizes that dementia of the Alzheimer's type is characterized by:

A. Aggressive acting-out behavior


B. Periodic remissions and exacerbations
C. Hypoxia of selected areas of brain tissue
D. Areas of brain destruction called senile plaques
D. Areas of brain destruction called senile plaques

When an older person's brain atrophies, some unusual deposits of iron are scattered on nerve
cells. Throughout the brain, areas of deeply straining amyloid, called senile plaques, can be
found; these plaques are end stages in the destruction of brain tissue.
A 75-year-old man with the diagnosis of dementia has been cared for by his wife for 5 years. For
the past 2 years he has not spoken and s incontinent of urine and feces. During the last month he
has changed form being placid and easygoing to agitated and aggressive. He is admitted to a
psychiatric hospital for treatment with psychopharmacology. Which is the priority nursing care
while this client is in the psychiatric facility?

A. Managing his behavior


B. Preventing further deterioration
C. Focusing on the n needs of the wife
D. Establishing an elimination retraining program
A. Managing his behavior

The client must be kept from harming himself or others; he needs a calm, supportive
environment that meets his needs and maintains his dignity.

Which nursing intervention is most helpful in meeting the needs of an older adult hospitalized
with the diagnosis of dementia of the Alzheimer's type?

A. Providing a nutritious diet high in carbohydrates and protein


B. Simplifying the environment as much as possible while eliminating the need for choices
C. Developing a consistent nursing plan with fixed time schedules to provide for emotional
needs
D. Providing an opportunity for many alternative choices in the daily schedule to stimulate
interest
B. Simplifying the environment as much as possible while eliminating the need for choices

Clients with this disorder need a simple environment. Because of brain cell destruction, that are
unable to make choices.

When attempting to understand the behavior of an older adult diagnosed with vascular dementia,
the nurse recognizes the the client is probably:

A.Not capable of using any defense mechanisms


B. Using one method of defense for every situation
C. Making exaggerated use of old, familiar mechanisms
D. Attempting to develop new defense mechanisms to meet the current situation
C. Making exaggerated use of old, familiar mechanisms

These clients attempt to utilize defense mechanisms that have worked in the past but use them in
an exaggerated manner. Because of brain cell destruction, these clients are unable to develop
new defense mechanisms.
Which should the nurse include in the plan of care for the client with vascular dementia:

A. A reeducation program
B. Details for supportive care
C. An introduction of new leisure-time activities
D. Plans to involve the client in group therapy sessions
B. Details for supportive care

Damaged brain cells do not regenerate. Care is therefore directed towards preventing further
damage and providing protection and support.

The nurse is assessing a client and attempting to distinguish between dementia and delirium.
Which factors are unique to delirium? Select all that apply.

A. Slurred speech
B. Lability of mood
C. Long-term memory loss
D. Visual or tactile hallucinations
E. Insidious deterioration in cognition
F. Fluctuating levels of consciousness
A. Slurred Speech

Delirium, a transient cognitive disorder caused by global dysfunction in cerebral metabolism,


causes sparse or rapid speech that may be slurred and incoherent.

D. Visual or tactile hallucinations

Visual or tactile hallucinations and illusions may occur with delirium because of altered cerebral
functioning; hallucinations are not prominent with dementia

F. Fluctuating levels of consciousness

Clients with delirium fluctuate from hyper alert to difficult to arouse; they may lose orientation
to time and place; clients with dementia do not have fluctuating levels of consciousness, but they
may be confused and disoriented.

A delirious client sees a design on the wallpaper and perceives it as an animal. The nurse should
report this as an example of:

A. A delusion
B. An illusion
C. A hallucination
D. An idea of reference
B. An illusion

An illusion is a misperception of misinterpretation of an actual external stimulus.

The nurse's best approach when caring for a confused, older client is to provide an environment
with:

A. Space for privacy


B. Group involvement
C. Trusting relationships
D. Activities that are varied
C. Trusting relationships

A one-to-one trusting relationship is essential to help the client become more involved and
interested in interpersonal relationships.

An older male client on the psychiatric unit becomes upset while in the day room. When
attempting to help the client, what should be the nurse's initial intervention?

1. Instruct the client to be quiet


2. Allow the client to act-out until he tires
3. Give directions in the a firm, low-pitched voice
4. Lead the client from the room by taking him by his arm
3. Give directions a firm, low-pitched voice

Clients who are out of control are seeking control and frequently respond to simple directions
stated in a firm voice.

An older adult is admitted to a psychiatric hospital with the diagnosis of dementia. The nurse
recognizes that it would be most unusual for this client to demonstrate:

1. Resistance to change
2. Preoccupation with personal appearance
3. A tendency to dwell on the past and ignore the present
4. The inability to concentrate on new activities or interests
2. Preoccupation with personal appearance

The client with delirium, dementia, or another cognitive disorder rarely expresses any concern
about personal appearance. The staff must meet most of the client's needs in the area.

When answering questions from the family of a client with Alzheimer's disease, the nurse
explains, "This disease is:
1. A slow and relentless deterioration of the mind."
2. A functional disorder that occurs in the later years."
3. A disease that first emerges in the fourth decade of line."
4. Easily diagnosed through laboratory and psychologic tests."
1. A slow and relentless deterioration of the mind."

This is a true statement; clients become progressively worse over time

It is important for nurses working with clients who have a diagnosis of dementia to adopt a
common approach of care because these clients need to:

1. Relate in a consistent manner to staff


2. Learn that the staff cannot be manipulated
3. Accept controls that are concrete and fairly applied
4. Have sameness and consistency in their environment
4. Have sameness and consistency in their environment

A consistent approach and consistent communication from all members of the health team help
the client who has dementia to remain a bit more reality-oriented.

A nurse is assessing client with dementia. Which client assessment is unexpected?

1. Acts pessimistic
2. Appears agitated
3. Has a short attention span
4. Exhibits disordered reasoning
1. Acts pessimistic

A client who acts apathetic and pessimistic is demonstrating characteristics of depression, not
dementia.

The nursing goal of the therapeutic psychiatric environment for the confused client is to:

1. Assist the client to relate to others


2. Make the hospital atmosphere more homelike
3. Help the client become popular in a controlled setting
4. Maintain the highest level of safe, independent functioning
4. Maintain the highest level of safe, independent functioning

The therapeutic milieu is directed toward helping the client develop effective ways of
functioning safely independently.
What is the most appropriate nursing intervention when working with clients who exhibit mild
cognitive impairment?

1. Reality orientation
2. Behavioral confrontation
3. Reflective communication
4. Reminiscence group therapy
1. Reality orientation

Reality orientation is generally helpful to clients exhibiting mild cognitive impairment; these
clients are aware of their impairment, and orientation then reduces anxiety.

Geriatric clients with behavioral changes are often admitted to psychiatric unit for screening and
evaluation. As part of the nursing assessment, it is important to observe for signs of dementia.
The four "As" of Alzheimer's disease are:

1. Amnesia, apraxia, agnosia, aphasia


2. Avoidance, aloofness, asocial, asexual
3. Autism, loose association, apathy, affect
4. Aggressive, amoral, ambivalent, attractive
1. Amnesia, apraxia, agnosia, aphasia

Neurofibrillary tangles in the hippocampus cause recent memory loss (amnesia); temporoparietal
deterioration causes cognitive deficiencies in speech (aphasia), purposeful movement (apraxia),
and comprehension of visual, auditory, and other sensations (agnosia).

A 78-year-old male has been brought to the clinic by his family because they believe he has
become increasing confused over the past week. What can the nurse ask the client to assess his
orientation?

1. Explain a proverb
2. State where he was born
3. Identify the name of the hospital
4. Recall what he had eaten for breakfast
3. Identify the name of the hospital

Orientation to place refers to an individual's awareness of the objective world in its relation to the
self; orientation to time, place, and person are part of the assessment of cerebral functioning.

A nurse is assigned to care for a regressed 19-year-old college student recently admitted to the
psychiatric unit with a 1-month history of taking to unseen people and refusing to get out of bed,
go to class, or get involved in daily grooming activities. The nurse's initial efforts should be
directed toward helping the client by:

1. Providing frequent rest periods and avoiding exhaustion


2. Facilitating the client's social relationships with a peer group
3. Reducing environmental stimuli and maintaining dietary intake
4. Attempting to establish a meaningful relationship with the client
4. Attempting to establish a meaningful relationship with the client

The first step in a plan of care should be the establishment of a meaningful relationship because
it is through this relationship that the client can be helped.

A client diagnosed with schizophrenia is experiencing auditory hallucinations. The nurse makes
the following statements when interacting with this client. Place these statements in the order in
which they should occur.

1. "I do not hear any voices"


2. "Come with me for a walk"
3. "Hearing voices must be frightening."
4. "The voices you hear are part of your illness."
3, 4, 1, 2

The nurse should first identify the client's feelings (3); after identifying the client's feelings, the
nurse should then simply explain why the voices occur (4); the nurse should then point out
reality (1); finally, the nurse should attempt to distract the client from the hallucination (2)

Th nurse understands that projection, rationalization, denial, and distortion by hallucinations and
delusions are examples of disturbances in:

1. Logic
2. Association
3. Reality testing
4. Thought processes
3. Reality testing

When individuals use these defense mechanisms, they are unable to test out their feelings or
differentiate the real world from their personal intrapsychic perceptions.

A male client with schizophrenia has a history of hearing voices that tell him he is a bad person.
While having a conversation with a nurse with whom he has been working, the client states that
he is starting to hear the voices again. What is the best response by the nurse?

1. "Try to ignore the voices."


2. "What are the voices saying to you?"
3. "Don't believe what the voices are saying."
4. "Try not to be afraid because they are only voices."
1. "Try to ignore the voices."

Clients can sometimes learn to push voices aside, particularly within the framework of a trusting
relationship; it may provide the client wth a sense of power to manage the hallucinatory voices.

When caring for a client whose behavior is characterized by pathologic suspicion, the nurse
should:

1. Remove the client fro environmental stress


2. Help the client realize the suspicions are unrealistic
3. Ask the client to explain the reasons of the feelings
4. Help the client to feel accepted by the staff on the unit
4. Help the client to feel accepted by the staff on the unit

Delusions are protective and can be abandoned only when the individual feels secure and
adequate. This response is the only one directed at building the client's security and reducing
anxiety.

One evening the nurse finds a client, who has been experiencing persecutory delusions, trying to
get out the door. The client states, "Please let me go. I trust you. The mafia is going to kill me
tonight." Which response is most therapeutic?

1. "You are frightened. Come with me to your room and we can talk about it."
2. "Nobody here wants to harm you, you know that. I'll come with you to your room."
3. "Come with me to your room. I'll lock the door and no one will get in to harm you."
4. "Thank you for trusting me. Maybe you can trust me when I tell you no one can kill you while
you're here."
1. "You are frightened. Come with me to your room and we can talk about it."

This response recognizes the client's feelings and provides assurance that the staff member will
be present.

A delusional client refuses to eat because of a belief that the food is poisoned. One of the most
appropriate ways for the nurse to initially intervene is to:

1. Taste the food in the client's presence


2. Simply state the food is not poisoned
3. Show the client that other people are eating without being harmed
4. Tell the client that tube feedings will be started if eating does not begin
2. Simply state the food is not poisoned
Clients cannot be argued out of delusions, so the best approach is a simple statement of reality.

A client with schizophrenia is admitted to an acute care psychiatric unit. Which positive signs
and symptoms exhibited by this client should the nurse document?

1. Withdrawal, poverty of speech, inattentiveness


2. Flat affect, decreased spontaneity, asocial behavior
3. Hypomania, labile mood swings, episodes of euphoria
4. Hyperactivity, auditory hallucinations, loose associations
4. Hyperactivity, auditory hallucinations, loose associations

These are positive symptoms associated with schizophrenia; positive symptoms reflect a
distortion or excess of normal functions.

An acutely ill client with the diagnosis of schizophrenia has just been admitted to the mental
health unit. When working with this client initially, the nurse's most therapeutic action should be
to:

1. Spend time with the client to build trust and demonstrate acceptance
2. Involve the client in occupational therapy and use diversional activity
3. Delay one-to-one interactions until medications reduce psychiatric symptoms
4. Involve the client in multiple small-group discussions to distract attention fro the fantasy
world
1. Spend time with the client to build trust and demonstrate acceptance

The initial goal should be to demonstrate acceptance and work toward developing trust; spending
time with the client best meets this initial goal.

A client with the diagnosis of schizophrenia plans an activity schedule with the help of the
treatment team. After agreement by all, a written copy is posted in the client's room. When it is
time of the client to go for a walk, what should the nurse say when approaching the client?

1. "It's time for you to go for a walk now."


2. "Do you want to take your scheduled walk now?"
3. "When would you like to go for your walk today?"
4. "You are supposed to be going for your walk now."
1. "It's time for you to go for a walk now."

This message is concise and does not require decision making; it is less likely to increase
anxiety.
During the admission procedure, a client appears to be responding to voices. The client crime out
at intervals, "No, no, I didn't kill him. You know the truth; tell that policeman. Please help me!"
The nurse should:

1. Listen attentively and assume a facial expression of disbelief


2. Sit there quietly and not respond to the client's statements
3. Respond by saying, "I want to help you. I realize you must be very frightened."
4. Say, "Do not become upset. No one is talking to you; the accusing voices are part of your
illness."
3. Respond by saying, "I want to help you. I realize you must be very frightened."

This response demonstrates an understanding of the clients's feelings and encourages the client to
share feelings, which is an immediate need.

The nurse has been observing a client for some time. The client is delusional, talking about
people who are plotting to do harm. The staff notices that the client is pacing more than usual.
The nurse decides that the client is beginning to lose control. What is the best nursing
intervention?

1. Encourage the client to use a punching bag


2. Move the client to a quiet place on the unit
3. Suggest that the client sit down for a while
4. Allow the client to continue pacing with supervision
2. Move the client to a quiet place on the unit

Clients losing control feel frightened and threatened. They need external controls and a reduction
in external stimuli.

A male client with a history of schizophrenia comes to a mental health clinic for a regularly
scheduled group therapy session. When the client enters the office, he is agitated and exhibits
behaviors that indicate he is hearing voices. When the nurse begins to walk toward the client
from across the room, the client pulls out a large knife. What is the best approach to use with this
client?

1. Firm
2. Passive
3. Empathetic
4. Confrontational
1. Firm

A firm approach prevent anxiety transference and provides structure and control for a client who
is out of control.
The nurse knows that prominent symptoms of lasting at least 1 month that are diagnostic for
paranoid schizophrenia are:

1. Delusions and hallucinations


2. Poverty of speech with apathy
3. Bizarre behaviors associated with drug use
4. Disturbed relationships and poor grooming
1. Delusions and hallucinations

Diagnostic criteria for paranoid schizophrenia include two or more symptoms such as delusions
and hallucinations; other less prominent criteria are disorganized behavior and negative
symptoms.

While the nurse is talking with a client, a female client comes up and yells, "I hate you. You're
talking about me again," and throws a glass of juice at the nurse. What is the best nursing
approach to respond to this situation?

1. Repeat the client's words and ask for clarification


2. Remove the client to her room because she needs limits placed on her behavior
3. Ignore both the behavior and the client, clean up the juice, and talk to her when she is better
4. Verbalize feelings of annoyance as an example to the client that it is more acceptable to
verbalize feelings than to act-out
2. Remove the client to her room because she needs limits placed on her behavior

The client's behavior is escalating and unsafe. She needs to be brought to her room, where there
is decreased environmental stimulation and less chance for her to act-out against others.

As the nurse enters a room and approaches a male client who has been diagnosed with
schizophrenia, the client states, "Get out of here before I hit you! Go away!" The nurse
recognizes that this client's aggressive behavior was probably related to the fact that he:

1. Felt hallucinating and the voices were directing his response


2. Was afraid that he might harm the nurse is the nurse came nearer
3. Was reminded of someone who was frightening and threatening to him
4. Felt hemmed in and trapped when the nurse came around the bed toward him
4. Felt hemmed in and trapped when the nurse came around the bed toward him

Clients acutely ill with schizophrenia frequently do not trust others; feeling hemmed in may be
frightening, causing them to last out.

A client who experience auditory hallucinations agrees to discuss with the nurse alternative
coping strategies. For the next 3 days when the nurse attempts to focus on alternative strategies,
the client gets up and leaves the interaction. It is most therapeutic for the nurse to state:
1. "Come back; you agreed that you would discuss other ways to cope."
2. "You seem very uncomfortable every time I bring up a new way to cope."
3. "Did you agree to talk about other ways to cope because you thought that was what I wanted?"
4. "You walk out each time I start to discuss the hallucinations; does that mean you've changed
your mind?"
2. "You seem very uncomfortable every time I bring up a new time to cope."

This response focuses on a feeling that the client may be experiencing and provides an
opportunity to validate the nurse's statement.

What is the nurse's most appropriate action when a client is seen openly masturbating in the
recreation room?

1. Restraining the client's hands


2. Putting the client is seclusion
3. Stating that such behavior is unacceptable
4. Demonstrating no reaction to the behavior
3. Stating that such behavior is unacceptable

The nurse should set limits on this behavior when it is not performed in a private area; this
accepts the client but rejects the behavior. Also, limits may need to be set on this behavior when
it is excessive.

To achieve one of the primary objectives of providing a therapeutic daycare environment for a
client who is withdrawn and seclusive, the nurse should:

1. Foster a trusting relationship


2. Administer medications on time
3. Involve the client in a group with peers
4. Remove the client from the family home
1. Foster a trusting relationship

An interpersonal relationship based on trust must be established before clients can be helped
back to reality.

A client experiencing hallucinations tells the nurse, "The voices are telling me I'm no good." The
client asks whether the nurse hears the voices. Which is the most appropriate response by the
nurses?

1. "It is the voice of your conscience, which only you can control."
2. "No, I do not hear voices, but I believe you can hear them."
3. "The voices are coming from within you; only you can hear them."
4. "Hearing the voices are a symptoms of your illness; don't pay attention to them."
2. "No, I do not hear voices, but I believe you can hear them."

The nurse, demonstrating knowledge and understanding, accepts the client's perceptions even
thought they are hallucinatory.

The nurse enters a client's room and notes that the client appears preoccupied. Then, turning to
the nurse, the clients states, "They are saying terrible things about me. Can't you hear them?" The
most therapeutic response by the nurse is:

1. "It seems you heard them before?"


2. "Try to get control of your feelings."
3. "There is no on here but me, and I don't hear anything."
4. "I don't hear what you say you hear, but I can see you are upset."
4. "I don't hear what you say you hear, but I can see you are upset."

This is the most therapeutic option; it interjects reality and focuses on the client's behavior.

The nurse observes a dressed, emotionally disturbed client using the hands to eat soft foods The
nurse can best intervene by:

1. Placing a spoon in the client's hand and suggesting it be used


2. Saying in a joking way, "Well, I guess fingers were made before forks."
3. Ingoring the behavior and observing several additional meals before intervening
4. Removing the food while saying, "You can't have any more until you use your spoon."
1. Placing a spoon in the client's hand and suggesting it be used

The client needs limits to be set. This response sets limits and rejects the behavior but accepts the
client.

The most serious indication of impending assaultive behavior is when the client:

1. Uses profane language


2. Touches people excessively
3. Exhibits a sudden withdrawal
4. Experiences command hallucinations
4. Experiences command hallucinations

Command hallucinations are dangerous because they may influence the client to engage in
behavior dangerous to self or others.
While watching TV in the day room, a female client who has demonstrated withdrawn, regressed
behavior suddenly screams, bursts into tears, and runs out of the room to the far end of the
hallway. What is the most therapeutic action for the nurse to take?

1. Walk to the end of the hallway where the client is standing


2. Accept the action as just being impulsive behavior of a sick person
3. Document the incident in the client's record while the memory is fresh
4. Ask another client who was in the day room what made the client act the way she did
1. Walk to the end of the hallway where the client is standing

This lets the client known the nurse is available. It also demonstrates an acceptance of the client.

When a regressed, emotionally disturbed client voids on the flood in the sitting room of the
psychiatric unit, the nurse should intervene by:

1. Making the client mop the flood


2. Restricting the client's fluids throughout the day
3. Toileting the client more frequently with supervision
4. Withholding privileges each time the client voids on the floor
3. Toileting the client more frequently with supervision

The client is voiding on the flood not to express hostility, but because of confusion. Taking the
client to the toilet frequently limits voiding in inappropriate places.

A regressed, emotionally disturbed client who has been watching the nurse for a few days
suddenly walks up and shouts, "You thing you're so damned perfect and good. I think you stink!"
Which is the most appropriate response by the nurse?

1. "Do you mean I smell?"


2. "You seem angry with me."
3. "Boy, you're in a bad mood."
4. "I can't be all that bad, can I?"
2. "You seem angry with me."

This response reflects on the client's feelings rather than focusing on the verbalization.

A client is experiencing auditory hallucinations. The client tells the nurse, "I am a terrible, evil
person; the voices are telling me that God needs to punish me." The most therapeutic initial
response by the nurse is:

1. "God is loving and will not punish you."


2. "The voices you are hearing are just a fantasy."
3. "Tell me what you are thinking about yourself."
4. "You aren't a wicked person; God and I both love you."
3. "Tell me what you are thinking about yourself."

Encouraging the client to focus on the self will facilitate communication and foster self-
perception.

The most appropriate way for the nurse to help a withdrawn, emotionally disturbed adolescent
client to accept the realities of daily living is to:

1. Assist the client to care for personal hygiene needs


2. Encourage the client to keep up with school studies
3. Encourage the client to join the other clients in group singing
4. Leave the client alone when there appears to be disinterest int he activities at hand
1. Assist the client to care for personal hygiene needs

Assisting clients with grooming helps keep them in contact with reality and allows them to see
that staff members care enough to help. It also places value on appearance.

Which is the best nursing intervention to encourage a withdrawn, non communicative client to
talk?

1. Focus on nonthreatening subjects


2. Try to get the client to discuss feelings
3. Sit and look through magazines with the client
4. Ask questions that require "yes" or "no" answers
1. Focus on nonthreatening subjects

Nursing care involves a steady attempt to draw the client into some response. This can best be
accomplished by focusing on nonthreatening subjects that do not demand a specific response.

When caring for client with exhibiting psychotic patterns of thinking and behavior, an important
aspect of nursing care is to:

1. Help keep the client oriented to reality


2. Involve the client in activities throughout the day
3. Help the client understand that it is harmful to withdraw fro situations
4. Encourage the client to discuss why mixing with other people is being avoided
1. Help keep the client oriented to reality

Keeping the withdrawn client oriented to reality prevents the client form withdrawing even
further into a private world.
Observation is an important aspect of nursing care. It is especially important in the care of the
withdrawn client because it:

1. Is useful in making a diagnosis


2. Tells the staff how ill the client is
3. Indicates the degree of depression
4. Helps in understanding the client's behavior
4. Helps in understanding the client's behavior

By observing the client the nurse is better able to understand the clients behavior, which can be
an indication of feelings

chapter 14 Eating disorders


Study

chapter 14 Eating disorders

17 studiers today

5.0 (2 reviews)

Flashcards

Learn

Test

Match

Created by
snlcjz

Share

Chapter 22: Eating Disorders


Study
Chapter 22: Eating Disorders

Leave the first rating


Flashcards

Learn

Test

Match

Created by
singer_becca

Share

NCLEX Questions Eating Disorders (Pearson)


Study

NCLEX Questions Eating Disorders (Pearson)

Leave the first rating


Flashcards

Learn

Test

Match
Created by
ejorgensen

Share

SESSION 43
Study

SESSION 43

Leave the first rating


Flashcards

Learn

Test

Match

KIMYE0NCHI
Top creator on Quizlet
Created 2 years ago
Nursing Care of a Family When a Child Has Cognitive Alterations: Eating Disorders

Share

Health Nursing Review 1


Study

Mental Health Nursing Review 1

32 studiers today

4.3 (6 reviews)

Flashcards

Learn

Test

Match

Created by
Supin_Kim

Share

NCLEX questions from 306 group projects


Study

NCLEX questions from 306 group projects

Leave the first rating


Flashcards

Learn

Test

Match

Created by
daniellepettus2

Share
Terms in this set (31)
Original

A nurse is completing an admission assessment for a client who has schizophrenia. Which of the
following findings should the nurse document as positive symptoms? Select all that apply.

A. auditory hallucination
B. lack of motivation
C. use of clang associations
D. delusion of persecution
E. constantly waving arms
F. flat affect
A, C, D, E

A nurse is speaking with a client who has schizophrenia when suddenly seems to stop focusing
on he nurses questions and begins looking at the ceiling and talking to themselves. Which of the
following actions should the nurse take?

A.Stop the interview at this point and resume later when the client is better able to concentrate.
B. Ask the client are you seeing something on the ceiling?
C. Tell the client you seem to be looking at something on the ceiling, I see something there too
D. Continue the interview without comment on the clientsbehavior
B

Which of the following would be a major risk factor for developing Dissociative Identity
Disorder? (Select all that apply)

A. Suicide
B. Experiencing childhood trauma
C. Alcohol or substance abuse
D. Poor Coping Skills
B & D.

Experiencing a trauma and poor coping skills are major risk factors for developing DID. All the
other answers are risk factors associated with the disorder

What information should the nurse give to the family of a client who has had a dissociative
episode?

A. Dissociation is a method for coping with severe stress.


B. Dissociation suggests the possibility of early dementia.
C. Alert family that brief periods of psychotic behavior may occur.
D. How to intervene to prevent self-mutilation and suicide attempts.
A. Dissociation is a method for coping with severe stress.

In Dissociative Identity Disorder, the host's identity refers to

A. The one that was the first to emerge as a new identity.


B. The one that is the most dominant.
C. The one that existed before the onset of the disorder.
D. The one that is the most aggressive.
C. The one that existed before the onset of the disorder.

A nurse's priority related to clients who are experiencing clinical manifestations of borderline
personality disorder should be to:
A.Engage in one-to-one discussions about childhood experiences.
B. Set limits when the client exhibits threats of self-damaging behaviors
C. Employ behavior modification using covert techniques.
D. Increase therapeutic communication when the client exhibits intrusiveness.
B. Set limits when the client exhibits threats of self-damaging behaviors

A 20-year-old college student has been brought to the psychiatric hospital by her parents. Her
admitting diagnosis is borderline personality disorder. When talking with the parents, which
information would the nurse expect to be included in the client's history? Select all that apply.

A.Impulsiveness
B.Lability of mood
C.Ritualistic behavior
D.Self-destructive behaviors
E.Eccentric appearance
A, B, D Impulsiveness, Lability of mood, Self-destructive behaviors

A 71-year-old patient with Alzheimer's disease who is being admitted to a long-term care facility
has had several episodes of wandering away from home. Which action will the nurse include in
the plan of care?

A.Reorient the patient several times daily.


B.Have the family bring in familiar items.
C.Place the patient in a room close to the nurses' station.
D.Ask the patient why the wandering episodes have occurred.
C. Place the patient in a room close to the nurses' station.
A patient who has severe Alzheimer's disease is being admitted to the hospital for surgery.
Which intervention will the nurse include in the plan of care?

A.Encourage the patient to discuss events from the past.


B.Maintain a consistent daily routine for the patient's care.
C.Reorient the patient to the date and time every 2 to 3 hours.
D.Provide the patient with current newspapers and magazines.
B. Maintain a consistent daily routine for the patient's care.

A nurse is planning care for a client who has bipolar disorder and is experiencing a manic
episode. Which of the following interventions should the nurse include in the plan of care?
(Select all that apply.)

A. Provide flexible client behavior expectations


B. Offer concise explanations
C. Establish consistent limits
D. Disregard client concerns
E. Use a firm approach with communication
B, C, E

A client is taking lithium, a mood stabilizer. The patient reports having abdominal pain,
vomiting, blurred vision. The serum lithium level is 2.0. What does this level represent?
a. Toxic
b. Below Normal
c. Normal
d. Above Normal
A Toxic

the normal lithium level is 0.6-1.2

A nurse working on an acute mental health unit is admitting a client who has major depressive
disorder and comorbid anxiety disorder. Which of the following actions is the nurse's priority?

A.Placing the client on one-to-one observation


B.Assisting the client to perform ADLs
C.Encouraging the client to participate in counseling
D.Teaching the client about medication adverse effects
A

the greatest risk for a client who has MDD and comorbid anxiety is injury due to self harm
A nurse is caring for a client who is on suicide precautions. Which of the following interventions
should the nurse include in the plan of care?

A.Assign the client to a private room.


B.Ensure that the client swallows the medication given.
C.Allow the client to keep perfume in their room.
D.Document the client's behavior every hour.
B

Which of the following accurately describes obsession and compulsions? SATA

a.Obsessions are recurrent and intrusive thoughts that occurs despite the client's attempt at
ignoring them.
b.Compulsions are persistent ideas that go away when the client utilizes grounding techniques.
c.Compulsions are seen in behaviors.
d.Compulsions are performed in effort to reduce anxiety.
A, C, D

A nursing professor is describing obsessive compulsive PD and obsessive compulsive anxiety


disorders. Which of following is the most accurate statement?

a.The personality disorder always has obsessive thoughts and compulsive behaviors.
b.There is no difference between the OC anxiety disorder and the OC personality disorder. The
terms are interchangeable.
c.The personality disorder usually does not have obsessive thoughts and compulsive behaviors.
d.The obsessive compulsive personality disorder is usually aware of their behaviors and see them
as abnormal, whereas the anxiety disorder sees their actions as morally correct.
C

The nurse is teaching about autism spectrum disorder to a group of community members. Which
risk factor should the nurse include?
A.Maternal age less than 30
B.Female gender
C.Paternal age greater than 50
D.Paternal age less than 30
C

Which assessment finding should the nurse expect in a child with autism spectrum disorder
(ASD)? Select all that apply

A.Reiteration of question as opposed to answering them


B.Use of the word "you" to represent "I"
C.Stuttering
D.Echolalia
E.Enchantment with rhythmic repetition of verse or song
A, B, D, E

The nurse is conducting a group therapy session. During the session, a client diagnosed with
mania consistently disrupts the group's interactions. Which intervention should the nurse initially
implement?

a.Setting limits on the client's behavior


b.Asking the client to leave the group setting
c.Asking another nurse to escort the client out
d.Telling the client that they will not be able to attend any furturegroup settings
A.

manic clients may be talkative and can dominate group meetings or therapy sessions. If this
occurs, the nurse would initiallyset limits on the client's behavior.

The nurse observes that a client is pacing, agitated, and presenting aggressive gestures. The
client's speech pattern is rapid, and affect is belligerent. Based on these observations, which is
the nurse's immediate priority of care?

a.Provide safety for the client and other clients on the unit
b.Provide the clients on the unit with a sense of comfort and safety
c.Assist the staff in caring for the client in a controlled environment
d.Offer the client a less stimulating area in which to calm down and gain control
A.
Safety of the client and other clients is the immediate priority. The correct option is the only one
that addresses the safety needs of the client as well as those of the other clients.

Nurse Arya assesses for evidence of positive symptoms of schizophrenia in a newly admitted
client. Which of the following symptoms are considered positive evidence? Select all that apply.
A)Anhedonia
B)Delusions
C)Flat Affect
D)Halluications
E)Loose Associations
F)Social Withdrawal
B,D,E
Rationale: These are positive symptoms of schizophrenia . Options A,C,F are negative symptoms
of schizophrenia.
A client on the Mental Health Unit states that the FBI is monitoring and recording his every
move and there are microphones and cameras in the unit walls. Which of the following would be
the most therapeutic response?

A)Confront the delusion directly by telling the client that this simply is not the case.
B)Tell the client that this must seem frightening, but you believe that he is safe here.
C)Tell the client to wait and talk about these beliefs in his one-on-one counseling sessions.
D)Isolate the client when he begins talking about these beliefs.
B.
Rationale: The nurse must realize that these perceptions are very real to the client.
Acknowledging his feelings provides support and assists in building rapport.

1)In the film, David presents with clear signs of schizophrenia. He displays signs of manic and
depression as well. What disorder does this correlate with?
a)Delusional Disorder
b)Schizophreniform Disorder
c)Schizoaffective Disorder
d)Bipolar Disorder
C

DuringDavid's college years, he experienced hallucinations of himself performing in front of


crowds where he either fainted or nearly died. If David were to be given antipsychotics to
address the hallucinations, what would the nurse report immediately to the psychiatrist?

a)Sweet smelling breath


b)Respirations of 21 breaths/min
c)Temperature of 103.5 F
d)Excessive salivation
C

The client has been prescribed an antipsychotic, which of the following should the nurse monitor
for parkinsonism, acute dystonia and akathisia? (Select all that apply)
A.Haloperidol
B.Risperidone
C.Clozapine
D.Fluephenazine
E.Paroxetine
A and D; these are both first generation antipsychotics that must be monitored for these
symptoms

-B and C are second generation antipsychotics that don't require monitoring for these symptoms
-E is an antidepressant(SSRI)
The client has begun to experience an auditory hallucination. Which of the following is the
nurse's priority?
A.Take the client to a private room
B.Ask the client to describe what he's hearing
C.Offer a PRN medication
D.Notify the client's provider immediately
-B
-According to the nursing framework, the first step in nursing care is assessment. So the nurse
must first assess the client's hallucinations and then intervene based on the client's answer.
- Also the nurse must determine if the client is experiencing command hallucinations as he is at
risk to harm himself or others

Which medications would you expect to administer to a client with bipolar disorder? Select all
the apply.

A.Lithium
B.ValproicAcid
C.Isocarboxizid
D.Atomoxetine
E.Risperidone
A, B, E

Lithium and valproic acid are both mood stabilizers and risperidone is an antipsychotic

Mr. Peterson, 35, is admitted for bipolar illness, manic phase, after assaulting his landlord in an
argument over Mr. Peterson is staying up all night playing loud music. Mr. Peterson is
hyperactive, intrusive, and has rapid, pressured speech. He has not slept in three days and
appears thin and disheveled. Which of the following is the most essential nursing action at this
time?

A.Providing a meal and beverage for Mr. Peterson to eat in the dining room.
B.Providing linens and toiletries for Mr. Peterson to attend to his hygiene.
C.Consulting with the psychiatrist to order a hypnotic to promote sleep.
D.Providing for client safety by limiting his privileges.
D.

Patient safety is always the nursespriority action.

A patient is admitted to the ED with the diagnosis of bipolar disorder, with a single manic
episode. Which of the following behaviors would the nurse expect to assess?
A.Apathy, poor insight, and poverty of ideas
B.Anxiety, somatic complaints, and insomnia
C.Elation, hyperactivity, and impaired judgement
D.Social isolation, delusional thinking, and clang association
C. Elation, hyperactivity, and impaired judgement

The symptoms in option A would be more characteristic of an individual with long-term


schizophrenia. The symptoms in option B would be more characteristic of someone with an
anxiety disorder, although a manic individual may also not sleep because of excessive energy.
The symptoms in option D are more characteristic of schizophrenia.

In a day treatment program, a manic client is creating considerable chaos, behaving in a


dominating and manipulative way. Which nursing intervention is most appropriate?
A.Allow the peer group to intervene
B.Describe acceptable behavior and set realistic limits with the client
C.Recommend that the client be hospitalized for treatment
D.Tell the client that their behavior is inappropriate
B. Describe acceptable behavior and set realistic limits with the client

The peer group is not responsible for monitoring the client's behavior. The client's behavior does
not warrant hospitalization. Option D is inappropriate because the client is told only what is
unacceptable and is not given any alternatives.

Which one of these is a major adverse effect the nurse would need to monitor for a pttaking
Chlorpromazine?
A. Pruritis
B. Bradycardia
C. Extrapyramidal symptoms
D. Edema
C. Chlorpromazine is a first-generation antipsychotic that has high risk for EPS

You are talking with a new nurse about Delusional disorders. Which of the following statements
would you reply when asked what is a delusion?
A. No longer finding pleasure in things used to enjoy
B. Disorder that prevents client from being independent
C. Disorder characterized by mania and major depression
D. Unshakeable belief in something untrue
D. Delusional disorder is characterized by not able to differentiate reality from fantasy despite
evidence

Terms in this set (50)


Original
1. Marco approached Nurse Trish asking for advice on how to deal with his alcohol addiction.
Nurse Trish should tell the client that the only effective treatment for alcoholism is:
A. Psychotherapy
B. Alcoholics anonymous (A.A.)
C. Total abstinence
D. Aversion Therapy
C. Total abstinence is the only effective treatment for alcoholism.

2.Nurse Hazel is caring for a male client who experience false sensory perceptions with no basis
in reality. This perception is known as:
A. Hallucinations
B. Delusions
C. Loose associations
D. Neologisms
A. Hallucinations are visual, auditory, gustatory, tactile or olfactory perceptions that have no
basis in reality.

3. Nurse Monet is caring for a female client who has suicidal tendency. When accompanying the
client to the restroom, Nurse Monet should...
A. Give her privacy
B. Allow her to urinate
C. Open the window and allow her to get some fresh air
D. Observe her
D. The Nurse has a responsibility to observe continuously the acutely suicidal client. The Nurse
should watch for clues, such as communicating suicidal thoughts, and messages; hoarding
medications and talking about death.

4. Nurse Maureen is developing a plan of care for a female client with anorexia nervosa. Which
action should the nurse include in the plan?
A. Provide privacy during meals
B. Set-up a strict eating plan for the client
C. Encourage client to exercise to reduce anxiety
D. Restrict visits with the family
B. Establishing a consistent eating plan and monitoring client's weight are important to this
disorder.

5. A client is experiencing anxiety attack. The most appropriate nursing intervention should
include?
A. Turning on the television
B. Leaving the client alone
C. Staying with the client and speaking in short sentences
D. Ask the client to play with other clients
C. Appropriate nursing interventions for an anxiety attack include using short sentences, staying
with the client, decreasing stimuli, remaining calm and medicating as needed.

6. A female client is admitted with a diagnosis of delusions of GRANDEUR. This diagnosis


reflects a belief that one is:
A. Being Killed
B. Highly famous and important
C. Responsible for evil world
D. Connected to client unrelated to oneself
B. Delusion of grandeur is a false belief that one is highly famous and important.

7.A 20 year old client was diagnosed with dependent personality disorder. Which behavior is
most likely to be evidence of ineffective individual coping?
A. Recurrent self-destructive behavior
B. Avoiding relationship
C. Showing interest in solitary activities
D. Inability to make choices and decision without advise
D. Individual with dependent personality disorder typically shows indecisiveness submissiveness
and clinging behavior so that others will make decisions with them.

8. A male client is diagnosed with schizotypal personality disorder. Which signs would this
client exhibit during social situation?
A. Paranoid thoughts
B. Emotional affect
C. Independence need
D. Aggressive behavior
A. Clients with schizotypal personality disorder experience excessive social anxiety that can lead
to paranoid thoughts.

9. Nurse Claire is caring for a client diagnosed with bulimia. The most appropriate initial goal for
a client diagnosed with bulimia is?
A. Encourage to avoid foods
B. Identify anxiety causing situations
C. Eat only three meals a day
D. Avoid shopping plenty of groceries
B. Bulimia disorder generally is a maladaptive coping response to stress and underlying issues.
The client should identify anxiety causing situation that stimulate the bulimic behavior and then
learn new ways of coping with the anxiety.

10. Nurse Tony was caring for a 41 year old female client. Which behavior by the client
indicates adult cognitive development?
A. Generates new levels of awareness
B. Assumes responsibility for her actions
C. Has maximum ability to solve problems and learn new skills
D. Her perception are based on reality
A. An adult age 31 to 45 generates new level of awareness.

11. A neuromuscular blocking agent is administered to a client before ECT therapy. The Nurse
should carefully observe the client for?
A. Respiratory difficulties
B. Nausea and vomiting
C. Dizziness
D. Seizures
A. Neuromuscular Blocker, such as SUCCINYLCHOLINE (Anectine) produces respiratory
depression because it inhibits contractions of respiratory muscles.

12. A 75 year old client is admitted to the hospital with the diagnosis of dementia of the
Alzheimer's type and depression. The symptom that is unrelated to depression would be?
A. Apathetic response to the environment
B. "I don't know" answer to questions
C. Shallow of labile effect
D. Neglect of personal hygiene
C. With depression, there is little or no emotional involvement therefore little alteration in affect.

13. Nurse Trish is working in a mental health facility; the nurse priority nursing intervention for
a newly admitted client with bulimia nervosa would be to?
A. Teach client to measure I & O
B. Involve client in planning daily meal
C. Observe client during meals
D. Monitor client continuously
D. These clients often hide food or force vomiting; therefore they must be carefully monitored.

14. Nurse Patricia is aware that the major health complication associated with intractable
anorexia nervosa would be?
A. Cardiac dysrhythmias resulting to cardiac arrest
B. Glucose intolerance resulting in protracted hypoglycemia
C. Endocrine imbalance causing cold amenorrhea
D. Decreased metabolism causing cold intolerance
A. These clients have severely depleted levels of sodium and potassium because of their
starvation diet and energy expenditure, these electrolytes are necessary for cardiac functioning.

15. Nurse Anna can minimize agitation in a disturbed client by?


A. Increasing stimulation
B. limiting unnecessary interaction
C. increasing appropriate sensory perception
D. ensuring constant client and staff contact
B. Limiting unnecessary interaction will decrease stimulation and agitation.

16. A 39 year old mother with obsessive-compulsive disorder has become immobilized by her
elaborate hand washing and walking rituals. Nurse Trish recognizes that the basis of O.C.
disorder is often:
A. Problems with being too conscientious
B. Problems with anger and remorse
C. Feelings of guilt and inadequacy
D. Feeling of unworthiness and hopelessness
C. Ritualistic behavior seen in this disorder is aimed at controlling guilt and inadequacy by
maintaining an absolute set pattern of behavior.

17. Mario is complaining to other clients about not being allowed by staff to keep food in his
room. Which of the following interventions would be most appropriate?
A. Allowing a snack to be kept in his room
B. Reprimanding the client
C. Ignoring the clients behavior
D. Setting limits on the behavior
D. The nurse needs to set limits in the client's manipulative behavior to help the client control
dysfunctional behavior. A consistent approach by the staff is necessary to decrease manipulation.

18. Conney with borderline personality disorder who is to be discharge soon threatens to "do
something" to herself if discharged. Which of the following actions by the nurse would be most
important?
A. Ask a family member to stay with the client at home temporarily
B. Discuss the meaning of the client's statement with her
C. Request an immediate extension for the client
D. Ignore the clients statement because it's a sign of manipulation
B. Any suicidal statement must be assessed by the nurse. The nurse should discuss the client's
statement with her to determine its meaning in terms of suicide.

19. Joey a client with antisocial personality disorder belches loudly. A staff member asks Joey,
"Do you know why people find you repulsive?" this statement most likely would elicit which of
the following client reaction?
A. Defensiveness
B. Embarrassment
C. Shame
D. Remorseful
A. When the staff member ask the client if he wonders why others find him repulsive, the client
is likely to feel defensive because the question is belittling. The natural tendency is to
counterattack the threat to self image.

20. Which of the following approaches would be most appropriate to use with a client suffering
from narcissistic personality disorder when discrepancies exist between what the client states and
what actually exist?
A. Rationalization
B. Supportive confrontation
C. Limit setting
D. Consistency
B. The nurse would specifically use supportive confrontation with the client to point out
discrepancies between what the client states and what actually exists to increase responsibility
for self.

21. Cely is experiencing alcohol withdrawal exhibits tremors, diaphoresis and hyperactivity.
Blood pressure is 190/87 mmhg and pulse is 92 bpm. Which of the medications would the nurse
expect to administer?
A. Naloxone (Narcan)
B. Benzlropine (Cogentin)
C. Lorazepam (Ativan)
D. Haloperidol (Haldol)
C. The nurse would most likely administer benzodiazepine, such as lorazepan (ativan) to the
client who is experiencing symptom: The client's experiences symptoms of withdrawal because
of the rebound phenomenon when the sedation of the CNS from alcohol begins to decrease.

22. Which of the following foods would the nurse Trish eliminate from the diet of a client in
alcohol withdrawal?
A. Milk
B. Orange Juice
C. Tea
D. Regular Coffee
D. Regular coffee contains caffeine which acts as psychomotor stimulants and leads to feelings
of anxiety and agitation. Serving coffee top the client may add to tremors or wakefulness.

23. Which of the following would Nurse Hazel expect to assess for a client who is exhibiting late
signs of heroin withdrawal?
A. Yawning & diaphoresis
B. Restlessness & Irritability
C. Constipation & steatorrhea
D. Vomiting and Diarrhea
D. Vomiting and diarrhea are usually the late signs of heroin withdrawal, along with muscle
spasm, fever, nausea, repetitive, abdominal cramps and backache.

24. To establish open and trusting relationship with a female client who has been hospitalized
with severe anxiety, the nurse in charge should?
A. Encourage the staff to have frequent interaction with the client
B. Share an activity with the client
C. Give client feedback about behavior
D. Respect client's need for personal space
D. Moving to a client's personal space increases the feeling of threat, which increases anxiety.

25. Nurse Monette recognizes that the focus of environmental (MILIEU) therapy is to:
A. Manipulate the environment to bring about positive changes in behavior
B. Allow the client's freedom to determine whether or not they will be involved in activities
C. Role play life events to meet individual needs
D. Use natural remedies rather than drugs to control behavior
A. Environmental (MILIEU) therapy aims at having everything in the client's surrounding area
toward helping the client.

26. Nurse Trish would expect a child with a diagnosis of reactive attachment disorder to:
A. Have more positive relation with the father than the mother
B. Cling to mother & cry on separation
C. Be able to develop only superficial relation with the others
D. Have been physically abuse
C. Children who have experienced attachment difficulties with primary caregiver are not able to
trust others and therefore relate superficially

27. When teaching parents about childhood depression Nurse Trina should say?
A. It may appear acting out behavior
B. Does not respond to conventional treatment
C. Is short in duration & resolves easily
D. Looks almost identical to adult depression
A. Children have difficulty verbally expressing their feelings, acting out behavior, such as
temper tantrums, may indicate underlying depression.

28. Nurse Perry is aware that language development in autistic child resembles:
A. Scanning speech
B. Speech lag
C. Shuttering
D. Echolalia
D. The autistic child repeat sounds or words spoken by others.
29. A 60 year old female client who lives alone tells the nurse at the community health center "I
really don't need anyone to talk to". The TV is my best friend.
The nurse recognizes that the client is using the defense mechanism known as?
A. Displacement
B. Projection
C. Sublimation
D. Denial
D. The client statement is an example of the use of denial, a defense that blocks problem by
unconscious refusing to admit they exist.

30. When working with a male client suffering phobia about black cats, Nurse Trish should
anticipate that a problem for this client would be?
A. Anxiety when discussing phobia
B. Anger toward the feared object
C. Denying that the phobia exist
D. Distortion of reality when completing daily routines
A. Discussion of the feared object triggers an emotional response to the object.

31. Linda is pacing the floor and appears extremely anxious. The duty nurse approaches in an
attempt to alleviate Linda's anxiety. The most therapeutic question by the nurse would be?
A. Would you like to watch TV?
B. Would you like me to talk with you?
C. Are you feeling upset now?
D. Ignore the client
B. The nurse presence may provide the client with support & feeling of control.

32. Nurse Penny is aware that the symptoms that distinguish post traumatic stress disorder from
other anxiety disorder would be:
A. Avoidance of situation & certain activities that resemble the stress
B. Depression and a blunted affect when discussing the traumatic situation
C. Lack of interest in family & others
D. Re-experiencing the trauma in dreams or flashback
D. Experiencing the actual trauma in dreams or flashback is the major symptom that
distinguishes post traumatic stress disorder from other anxiety disorder.

Nurse Benjie is communicating with a male client with substance-induced persisting dementia;
the client cannot remember facts and fills in the gaps with imaginary information. Nurse Benjie
is aware that this is typical of?
A. Flight of ideas
B. Associative looseness
C. Confabulation
D. Concretism
C. Confabulation or the filling in of memory gaps with imaginary facts is a defense mechanism
used by people experiencing memory deficits.

34. Nurse Joey is aware that the signs & symptoms that would be most specific for diagnosis
anorexia are?
A. Excessive weight loss, amenorrhea & abdominal distension
B. Slow pulse, 10% weight loss & alopecia
C. Compulsive behavior, excessive fears & nausea
D. Excessive activity, memory lapses & an increased pulse
A. These are the major signs of anorexia nervosa. Weight loss is excessive (15% of expected
weight).

35. A characteristic that would suggest to Nurse Anne that an adolescent may have bulimia
would be:
A. Frequent regurgitation & re-swallowing of food
B. Previous history of gastritis
C. Badly stained teeth
D. Positive body image
C. Dental enamel erosion occurs from repeated self-induced vomiting.

36. Nurse Monette is aware that extremely depressed clients seem to do best in settings where
they have:
A. Multiple stimuli
B. Routine Activities
C. Minimal decision making
D. Varied Activities
B. Depression usually is both emotional & physical. A simple daily routine is the best, least
stressful and least anxiety producing.

37. To further assess a client's suicidal potential. Nurse Katrina should be especially alert to the
client expression of:
A. Frustration & fear of death
B. Anger & resentment
C. Anxiety & loneliness
D. Helplessness & hopelessness
D. The expression of these feeling may indicate that this client is unable to continue the struggle
of life.

38. A nursing care plan for a male client with bipolar I disorder should include:
A. Providing a structured environment
B. Designing activities that will require the client to maintain contact with reality
C. Engaging the client in conversing about current affairs
D. Touching the client provide assurance
A. Structure tends to decrease agitation and anxiety and to increase the client's feeling of
security.

39. When planning care for a female client using ritualistic behavior, Nurse Gina must recognize
that the ritual:
A. Helps the client focus on the inability to deal with reality
B. Helps the client control the anxiety
C. Is under the client's conscious control
D. Is used by the client primarily for secondary gains
B. The rituals used by a client with obsessive compulsive disorder help control the anxiety level
by maintaining a set pattern of action.

40. A 32 year old male graduate student, who has become increasingly withdrawn and neglectful
of his work and personal hygiene, is brought to the psychiatric hospital by his parents. After
detailed assessment, a diagnosis of schizophrenia is made. It is unlikely that the client will
demonstrate:
A. Low self esteem
B. Concrete thinking
C. Effective self boundaries
D. Weak ego
C. A person with this disorder would not have adequate self-boundaries.

41. A 23 year old client has been admitted with a diagnosis of schizophrenia says to the nurse
"Yes, its march, March is little woman". That's literal you know". These statement illustrate:
A. Neologisms
B. Echolalia
C. Flight of ideas
D. Loosening of association
D. Loose associations are thoughts that are presented without the logical connections usually
necessary for the listening to interpret the message.

42. A long term goal for a paranoid male client who has unjustifiably accused his wife of having
many extramarital affairs would be to help the client develop:
A. Insight into his behavior
B. Better self control
C. Feeling of self worth
D. Faith in his wife
C. Helping the client to develop feeling of self worth would reduce the client's need to use
pathologic defenses.

43. A male client who is experiencing disordered thinking about food being poisoned is admitted
to the mental health unit. The nurse uses which communication technique to encourage the client
to eat dinner?
A. Focusing on self-disclosure of own food preference
B. Using open ended question and silence
C. Offering opinion about the need to eat
D. Verbalizing reasons that the client may not choose to eat
B. Open ended questions and silence are strategies used to encourage clients to discuss their
problem in descriptive manner.

44. Nurse Nina is assigned to care for a client diagnosed with Catatonic Stupor. When Nurse
Nina enters the client's room, the client is found lying on the bed with a body pulled into a fetal
position. Nurse Nina should?
A. Ask the client direct questions to encourage talking
B. Rake the client into the dayroom to be with other clients
C. Sit beside the client in silence and occasionally ask open-ended question
D. Leave the client alone and continue with providing care to the other clients
C. Clients who are withdrawn may be immobile and mute, and require consistent, repeated
interventions. Communication with withdrawn clients requires much patience from the nurse.
The nurse facilitates communication with the client by sitting in silence, asking open-ended
question and pausing to provide opportunities for the client to respond.

45. Nurse Tina is caring for a client with delirium and states that "look at the spiders on the
wall". What should the nurse respond to the client?
A. "You're having hallucination, there are no spiders in this room at all"
B. "I can see the spiders on the wall, but they are not going to hurt you"
C. "Would you like me to kill the spiders"
D. "I know you are frightened, but I do not see spiders on the wall"
D. When hallucination is present, the nurse should reinforce reality with the client.

46. Nurse Jonel is providing information to a community group about violence in the family.
Which statement by a group member would indicate a need to provide additional information?
A. "Abuse occurs more in low-income families"
B. "Abuser Are often jealous or self-centered"
C. "Abuser use fear and intimidation"
D. "Abuser usually have poor self-esteem"
A. Personal characteristics of abuser include low self-esteem, immaturity, dependence, insecurity
and jealousy.
47. During electroconvulsive therapy (ECT) the client receives oxygen by mask via positive
pressure ventilation. The nurse assisting with this procedure knows that positive pressure
ventilation is necessary because?
A. Anesthesia is administered during the procedure
B. Decrease oxygen to the brain increases confusion and disorientation
C. Grand mal seizure activity depresses respirations
D. Muscle relaxations given to prevent injury during seizure activity depress respirations.
D. A short acting skeletal muscle relaxant such as succinylcholine (Anectine) is administered
during this procedure to prevent injuries during seizure.

48. When planning the discharge of a client with chronic anxiety, Nurse Chris evaluates
achievement of the discharge maintenance goals. Which goal would be most appropriately
having been included in the plan of care requiring evaluation?
A. The client eliminates all anxiety from daily situations
B. The client ignores feelings of anxiety
C. The client identifies anxiety producing situations
D. The client maintains contact with a crisis counselor
C. Recognizing situations that produce anxiety allows the client to prepare to cope with anxiety
or avoid specific stimulus.

49. Nurse Tina is caring for a client with depression who has not responded to antidepressant
medication. The nurse anticipates that what treatment procedure may be prescribed?
A. Neuroleptic medication
B. Short term seclusion
C. Psychosurgery
D. Electroconvulsive therapy
D. Electroconvulsive therapy is an effective treatment for depression that has not responded to
medication.

50. Mario is admitted to the emergency room with drug-included anxiety related to over
ingestion of prescribed antipsychotic medication. The most important piece of information the
nurse in charge should obtain initially is the:
A. Length of time on the med.
B. Name of the ingested medication & the amount ingested
C. Reason for the suicide attempt
D. Name of the nearest relative & their phone number
B. In an emergency, lives saving facts are obtained first. The name and the amount of medication
ingested are of outmost important in treating this potentially life threatening situation.
Terms in this set (47)
Original

A client with a diagnosis of depression who has attempted suicide says to the nurse, "I should
have died. I've always been a failure. Nothing ever goes right for me." Which response by the
nurse demonstrates therapeutic communication?

1. "You have everything to live for."

2. "Why do you see yourself as a failure?"

3. "Feeling like this is all part of being depressed."

4. "You've been feeling like a failure for a while?"


4

The nurse visits a client at home. The client states, "I haven't slept at all the last couple of
nights." Which response by the nurse demonstrates therapeutic communication?

1. "I see."

2. "Really?"

3. "You're having difficulty sleeping?"

4. "Sometimes I have trouble sleeping too."


3

A client experiencing disturbed thought processes believes that his food is being poisoned.
Which communication technique should the nurse use to encourage the client to eat?

1. Using open-ended questions and silence

2. Sharing personal preference regarding food choices

3. Documenting reasons why the client does not want to eat

4. Offering opinions about the necessity of adequate nutrition


1
A client admitted voluntarily for treatment of an anxiety disorder demands to be released from
the hospital. Which action should the nurse take initially?

1. Contact the client's health care provider (HCP).

2. Call the client's family to arrange for transportation.

3. Attempt to persuade the client to stay "for only a few more days."

4. Tell the client that leaving would likely result in an involuntary commitment.
1

When reviewing the admission assessment, the nurse notes that a client was admitted to the
mental health unit involuntarily. Based on this type of admission, the nurse should provide which
intervention for this client?

1. Monitor closely for harm to self or others.

2. Assist in completing an application for admission.

3. Supply the client with written information about his or her mental illness.

4. Provide an opportunity for the family to discuss why they felt the admission was needed.
1

When a client is admitted to an inpatient mental health unit with the diagnosis of anorexia
nervosa, a cognitive behavioral approach is used as part of the treatment plan. The nurse plans
care based on which purpose of this approach?

1. Providing a supportive environment

2. Examining intrapsychic conflicts and past issues

3. Emphasizing social interaction with clients who withdraw

4. Helping the client to examine dysfunctional thoughts and beliefs


4

A client is preparing to attend a Gamblers Anonymous meeting for the first time. The nurse
should tell the client that which is the first step in this 12-step program?

1. Admitting to having a problem


2. Substituting other activities for gambling

3. Stating that the gambling will be stopped

4. Discontinuing relationships with people who gamble


1

The nurse employed in a mental health clinic is greeted by a neighbor in a local grocery store.
The neighbor says to the nurse, "How is Carol doing? She is my best friend and is seen at your
clinic every week." Which is the most appropriate nursing response?

1. "I cannot discuss any client situation with you."

2. "If you want to know about Carol, you need to ask her yourself."

3. "Only because you're worried about a friend, I'll tell you that she is improving."

4. "Being her friend, you know she is having a difficult time and deserves her privacy."
1

The nurse calls security and has physical restraints applied to a client who was admitted
voluntarily when the client becomes verbally abusive, demanding to be discharged from the
hospital. Which represents the possible legal ramifications for the nurse associated with these
interventions? Select all that apply.

1. Libel

2. Battery

3. Assault

4. Slander

5. False imprisonment
2,3,5

The nurse in the mental health unit plans to use which therapeutic communication techniques
when communicating with a client? Select all that apply.

1. Restating

2. Listening
3. Asking the client "Why?"

4. Maintaining neutral responses

5. Providing acknowledgment and feedback

6. Giving advice and approval or disapproval


1, 2, 4, 5

What is the most appropriate nursing action to help manage a manic client who is monopolizing
a group therapy session?

1. Ask the client to leave the group for this session only.

2. Refer the client to another group that includes other manic clients.

3. Tell the client to stop monopolizing in a firm but compassionate manner.

4. Thank the client for the input, but inform the client that others now need a chance to
contribute.
4

The nurse is working with a client who despite making a heroic effort was unable to rescue a
neighbor trapped in a house fire. Which client-focused action should the nurse engage in during
the working phase of the nurse-client relationship?

1. Exploring the client's ability to function

2. Exploring the client's potential for self-harm

3. Inquiring about the client's perception or appraisal of why the rescue was unsuccessful

4. Inquiring about and examining the client's feelings for any that may block adaptive coping
4

The nurse provides an educational session on client rights. Which statement by a member of the
session demonstrates the best understanding of the nurse's role regarding ensuring that each
client's rights are respected?

1. "Autonomy is the fundamental right of each and every client."

2. "A client's rights are guaranteed by both state and federal laws."
3. "Being respectful and concerned will ensure that I'm attentive to my clients' rights."

4. "Regardless of the client's condition, all nurses have the duty to value client rights."
3

A client says to the nurse, "The federal guards were sent to kill me." Which is the best response
by the nurse to the client's concern?

1. "I don't believe this is true."

2. "The guards are not out to kill you."

3. "Do you feel afraid that people are trying to hurt you?"

4. "What makes you think the guards were sent to hurt you?"
3

A client diagnosed with delirium becomes disoriented and confused at night. Which intervention
should the nurse implement initially?

1. Move the client next to the nurses' station.

2. Use an indirect light source and turn off the television.

3. Keep the television and a soft light on during the night.

4. Play soft music during the night, and maintain a well-lit room.
2

A client is admitted to the mental health unit with a diagnosis of depression. The nurse should
develop a plan of care for the client that includes which intervention?

1. Encouraging quiet reading and writing for the first few days

2. Identification of physical activities that will provide exercise

3. No socializing activities, until the client asks to participate in milieu

4. A structured program of activities in which the client can participate


4
A client diagnosed with terminal cancer says to the nurse, "I'm going to die, and I wish my
family would stop hoping for a cure! I get so angry when they carry on like this. After all, I'm the
one who's dying." Which response by the nurse is therapeutic?

1. "Have you shared your feelings with your family?"

2. "I think we should talk more about your anger with your family."

3. "You're feeling angry that your family continues to hope for you to be cured?"

4. "You are probably very depressed, which is understandable with such a diagnosis."
3

On review of the client's record, the nurse notes that the admission was voluntary. Based on this
information, the nurse plans care anticipating which client behavior?

1. Fearfulness regarding treatment measures

2. Anger and aggressiveness directed toward others

3. An understanding of the pathology and symptoms of the diagnosis

4. A willingness to participate in the planning of the care and treatment plan


4

Recognizing the frequency of depression among the American population, the nurse should
advocate for which mental health promotion intervention?

a. Including discussions on depression as part of school health classes

b. Providing regular depression screening for adolescent and teenage students

c. Increasing the number of community-based depression hotlines available to the public

d. Encouraging senior centers to provide information on accessing community depression


resources
b

Which statement made by a patient demonstrates a healthy degree of resilience? Select all that
apply.

a. "I try to remember not to take other people's bad moods personally."
b. "I know that if I get really mad I'll end up being depressed."

c. "I really feel that sometimes bad things are meant to happen."

d. "I've learned to calm down before trying to defend my opinions."

e. "I know that discussing issues with my boss would help me get my point across."
a, d, e

Epidemiological studies contribute to improvements in care for individuals with mental disorders
by:

a. Providing information about effective nursing techniques.

b. Identifying risk factors that contribute to the development of a disorder.

c. Identifying individuals in the general population who will develop a specific disorder.

d. Identifying which individuals will respond favorably to a specific treatment.


b, d

Which statement demonstrates the nurse's understanding of the effect of environmental factors
on a patient's mental health?

a. "I'll need to assess how the patient's family views mental illness."

b. "There is a history of depression in the patient's extended family."

c. "I'm not familiar with the patient's Japanese's cultural view on suicide."

d. "The patient's ability to pay for mental health services needs to be assessed."
c

Which statement about mental illness is true?

a. Mental illness is a matter of individual nonconformity with societal norms.

b. Mental illness is present when irrational and illogical behavior occurs.

c. Mental illness changes with culture, time in history, political systems, and the groups defining
it.
d. Mental illness is evaluated solely by considering individual control over behavior and
appraisal of reality.
b

When considering stigmatization, which statement made by the nurse demonstrates a need for
immediate intervention by the nurse manager?

a. "Depression seems to be a real problem among the teenage population."

b. "My experience has been that the Irish have a problem with alcohol use."

c. "Women are at greater risk for developing suicidal thoughts then acting on them."

d. "We've admitted several military veterans with posttraumatic stress disorder this month."
b

When discussing therapy options, the nurse should provide information about interpersonal
therapy to which patient? Select all that apply.

a. The teenager who is the focus of bullying at school

b. The older woman who has just lost her life partner to cancer

c. The young adult who has begun demonstrating hoarding tendencies

d. The adolescent demonstrating aggressive verbal and physical tendencies

e. The middle-aged adult who recently discovered her partner has been unfaithful
a, b, e

When considering the suggestions of Hildegard Peplau, which activity should the nurse regularly
engage in to ensure that the patient stays the focus of all therapeutic conversations?

a. Assessing the patient for unexpressed concerns and fears

b. Evaluating the possible need for additional training and education

c. Reflecting on personal behaviors and personal needs

d. Avoiding power struggles with the manipulative patient


c
Which action reflects therapeutic practices associated with operant conditioning?

a. Encouraging a parent to read to their children to foster a love for learning

b. Encouraging a patient to make daily journal entries describing their feelings

c. Suggesting to a new mother that she spend time cuddling her newborn often during the day

d. Acknowledging a patient who is often verbally aggressive for complimenting a picture another
patient drew
d

A male patient reports to the nurse, "I'm told I have memories of childhood abuse stored in my
unconscious mind. I want to work on this." Based on this statement, what information should the
nurse provide the patient?

a. To seek the help of a trained therapist to help uncover and deal with the trauma associated
with those memories.

b. How to use a defense mechanism such as suppression so that the memories will be less
threatening.

c. Psychodynamic therapy will allow the surfacing of those unconscious memories to occur in
just a few sessions.

d. Group sessions are valuable to identify underlying themes of the memories being suppressed.
a

A nurse is assessing a patient who graduated at the top of his class but now obsesses about being
incompetent in his new job. The nurse recognizes that this patient may benefit from the
following type of psychotherapy:

a. Interpersonal

b. Operant conditioning

c. Behavioral

d. Cognitive-behavioral
d

Which question should the nurse ask when assessing for what Sullivan's Interpersonal Theory
identifies as the most painful human condition?
a. "Is self-esteem important to you?"

b. "Do you think of yourself as being lonely?"

c. "What do you do to manage your anxiety?"

d. "Have you ever been diagnosed with depression?"


b

Linda is terrified of spiders and cannot explain why. Because she lives in a wooded area, she
would like to overcome this overwhelming fear. Her nurse practitioner suggests which therapy?

a. Behavioral

b. Biofeedback

c. Aversion

d. Systematic desensitization
d

Besides antianxiety agents, which classification of drugs is also commonly given to treat anxiety
and anxiety disorders?

a. Antipsychotics

b. Mood stabilizers

c. Antidepressants

d. Cholinesterase inhibitors
c

Which drug group calls for nursing assessment for development of abnormal movement
disorders among individuals who take therapeutic dosages?

a. SSRIs

b. antipsychotics

c. benzodiazepines
d. tricyclic antidepressants
b

The nurse administers each of the following drugs to various patients. The patient who should be
most carefully assessed for fluid and electrolyte imbalance is the one receiving:

a. lithium (Eskalith)

b. clozapine (Clozaril)

c. diazepam (Valium)

d. amitriptyline
a

Which nursing intervention demonstrates the ethical principle of beneficence?

a. Refusing to administer a placebo to a patient.

b. Attending an in-service on the operation of the new IV infusion pumps

c. Providing frequent updates to the family of a patient currently in surgery

d. Respecting the right of the patient to make decisions about whether or not to have
electroconvulsive therapy
c

According to Maslow's hierarchy of needs, the most basic needs category for nurses to address
is:

a. physiological

b. safety

c. love and belonging

d. self-actualization
a

When considering facility admissions for mental healthcare, what characteristic is unique to a
voluntary admission?
a. The patient poses no substantial threat to themselves or to others

b. The patient has the right to seek legal counsel

c. A request in writing is required before admission

d. A mental illness has been previously diagnosed


c

In an outpatient psychiatric clinic, a nurse notices that a newly admitted young male patient
smiles when he sees her. One day the young man tells the nurse, "You are pretty like my
mother." The nurse recognizes that the male is exhibiting:

a. Transference

b. Id expression

c. Countertransference

d. A cognitive distortion
a

Which drug group calls for nursing assessment for development of abnormal movement
disorders among individuals who take therapeutic dosages?

a. SSRIs

b. antipsychotics

c. benzodiazepines

d. tricyclic antidepressants
b

A patient is telling a tearful story. The nurse listens empathically and responds therapeutically
with:

a. "The next time you find yourself in a similar situation, please call me."

b. "I am sorry this situation made you feel so badly. Would you like some tea?"

c. "Let's devise a plan on how you will react next time in a similar situation."
d. "I am sorry that your friend was so thoughtless. You should be treated better."
c

You realize that your patient who is being treated for a major depressive disorder requires more
teaching when she makes the following statement:

a. "I have been on this antidepressant for 3 days. I realize that the full effect may not happen for
a period of weeks."

b. "I am going to ask my nurse practitioner to discontinue my Prozac today and let me start
taking a monoamine oxidase inhibitor tomorrow."

c. "I may ask to have my medication changed to Wellbutrin due to the problems I am having
being romantic with my wife."

d. "I realize that there are many antidepressants and it might take a while until we find the one
that works best for me."
b

A nurse makes a post on a social media page about his peer taking care of a patient with a crime-
related gunshot wound in the emergency department. He does not use the name of the patient.
The nurse:

a. Has not violated confidentiality laws because he did not use the patient's name.

b. Cannot be held liable for violating confidentiality laws because he was not the primary nurse
for the patient.

c. Has violated confidentiality laws and can be held liable.

d. Cannot be held liable because postings on a social media site are excluded from confidentiality
laws.
c

Implied consent occurs when no verbal or written agreement takes place prior to a caregiver
delivering treatment. Which of the following examples represents implied consent?

a. The mother of an unconscious patient saying okay to surgery

b. Care given to a heroin overdose victim

c. Immobilizing a patient who has refused to take medication


d. Signing general intake paperwork with specific parameters
b

Based on Maslow's hierarchy of needs, physiological needs for a restrained patient include:

Select all that apply.

a. Private toileting, oral hydration

b. Checking the tightness of the restraints

c. Therapeutic communication

d. Maintaining a patent airway


a, b, d

What assessment question is focused on identifying a long-term consequence of chronic stress on


physical health?

a. "Do you have any problems with sleeping well?"

b. "How many infections have you experienced in the past 6 months?"

c. "How much moderate exercise do you engage in on a regular basis?"

d. "What management techniques to you regularly use to manage your stress?"


b

Which nursing assessments are directed at monitoring a patient's fight-or-flight response? Select
all that apply.

a. Blood pressure

b. Heart rate

c. Respiratory rate

d. Abdominal pain

e. Dilated pupils
a, b, c, e
When considering stress, what is the primary goal of making daily entries into a personal
journal?

a. Providing a distraction from the daily stress

b. Expressing emotions to manage stress

c. Identifying stress triggers

d. Focusing on one's stress


c

Terms in this set (10)


Original

Marcus is experiencing a severe panic attack. Which nursing intervention would meet this
client's immediate need?

A. Teach deep breathing relaxation exercises


B. Place the client in a Trendelenburg position
C. Stay with the client and offer reassurance of safety
D. Administer the ordered PRN medication
C

Leila has been caring for a client diagnosed with post-traumatic stress disorder. What short-term,
realistic, correctly written outcome should be included in this client's plan of care?

A. The client will have no flashbacks.


B. The client will be able to feel a full range of emotions by discharge.
C. The client will not require medication to obtain adequate sleep by discharge.
D. The client will refrain from discussing the traumatic event.
C

Which nursing diagnosis would best describe the problems evidenced by the following client
symptoms: avoidance, poor concentration, nightmares, hypervigilance, exaggerated startle
response, detachment, emotional numbing, and flashbacks?

A. Ineffective coping
B. Post-trauma syndrome
C. Complicated grieving
D. Panic anxiety
B

Barbara presents at the urgent care clinic and states, "My heart feels like it's skipping beats." The
client also reports always feeling cold, and has a BMI of 18. The nurse suspects anorexia. Which
other clinical manifestation should the nurse assess? SELECT ALL THAT APPLY

A. Strenuous exercise
B. Feelings of euphoria
C. Extreme perfectionism
D. Obsession over body shape
E. Rigidity and the need to control situations
ACDE

The parents of a teenage girl bring their daughter to the healthcare provider, citing their
increasing concern about the teen's weight and their suspicion that their daughter has anorexia
nervosa (AN). During assessment, the nurse notes a BMI of 16.75 kg/m2. In which category
does the client fall, according to DSM-5 criteria and
considering the severity of anorexia nervosa?

A. Extreme
B. Severe
C. Mild
D. Moderate
D

The client with anorexia nervosa is improving if:

A. She eats meals in the dining room.


B. Weight gain
C. She attends ward activities.
D. She has a more realistic self-concept.
B

The characteristic manifestation that will differentiate bulimia nervosa from anorexia nervosa is
that bulimic individuals

A. have episodic binge eating and purging


B. have repeated attempts to stabilize their weight
C. have peculiar food handling patterns
D. have threatened self-esteem
A
A nursing diagnosis for bulimia nervosa is powerlessness related to feeling not in control of
eating habits. The goal for this problem is:

A. Patient will learn problem solving skills


B. Patient will have decreased symptoms of anxiety.
C. Patient will perform self-care activities daily.
D. Patient will verbalize how to set limits on others.
A

In the management of bulimic patients, the following nursing interventions will promote a
therapeutic relationship EXCEPT:

A. Establish an atmosphere of trust


B. Discuss their eating behavior.
C. Help patients identify feelings associated with binge-purge behavior
D. Teach patient about bulimia nervosa
B

Lana a nurse at Sky Castle Medical Center is developing a care plan for a female client with
post-traumatic stress disorder. Which of the following would she do initially?

A. Instruct the client to use distraction techniques to cope with flashbacks.


B. Encourage the client to put the past in proper perspective.
C. Encourage the client to verbalize thoughts and feelings about the trauma.
D. Avoid discussing the traumatic event with client.
C

Terms in this set (10)


Original

How do cultural stereotypes contribute to the development of eating disorders?


A Eating disorders result from biological and genetic factors.
B There is a strong emphasis on low body weight justifying high self-esteem.
C Cultural stereotypes increase an individual's insight regarding his or her own personal weight
issues.
D The stereotypes identify the population at risk for developing eating disorders.
B
According to the family systems theory, family behavior characteristics associated with anorexia
include (select all that apply):
A Isolation between family members.
B Successful conflict resolution
C Unclear boundaries between family members.
D Family members' preoccupation with food and eating.
E Individual autonomy.
C, D

The most common coexisting mental health issue associated with anorexia and bulimia is:
A Anxiety.
B Agoraphobia.
C Depression.
D Panic attacks.
C

A nurse is teaching a group of adolescents about the risk factors and complications of anorexia
nervosa. Which of the following complications should the nurse stress as the most serious?
A Increased risk of mortality
B Depression
C Ineffective family relationships
D Ineffective coping skills
B

Which of the following objective data would the nurse expect to find in the client with anorexia
nervosa?
A A score of 13 on the Mini-Mental State Exam
B Feeling isolated and lonely
C Osteoporosis
D Preoccupation with food
C

The nursing diagnosis for a client with bulimia is Fluid Volume Deficit. Nursing interventions
specific to the fluid volume deficit include:
A Weighing the client after each meal.
B Monitoring the client for at least 1 hour after meals.
C Monitoring body temperature every 4 hours.
D Ensuring daily consumption of 1000 to 2000 mL of liquid.
B

Educational guidelines for family members of clients with eating disorders include:
A Expecting a full recovery within 6 months of starting treatment.
B Scheduling family activities that include food.
C Expressing love and affection both verbally and physically.
D Recognizing the client's need to have his or her behaviors controlled by family members.
C

The nurse observes a client admitted with anorexia nervosa doing repeated, vigorous sit-ups. The
most appropriate action from the nurse is to:
A Tell the client she cannot do exercises.
B Allow the client to complete the exercise routine.
C Take away the client's visitor privileges.
D Interrupt the behavior and offer to walk with the client.
D

The nurse is assessing a client with severe anorexia nervosa. Which of the following physical
findings should be immediately reported to the physician?
A Pulse rate of 102
B Blood pressure of 80/40mm Hg
C Amenorrhea
D Urine output of 50cc/hour
B

An effective intervention to facilitate individual coping for clients with eating disorders is to:
A Provide the client with limited information on a need-to-know basis.
B Provide flexibility in activities of daily living.
C Prohibit the client from making decisions regarding care.
D Have the treatment team determine the client's plan of care.
B

Terms in this set (13)


Original

1. Family dynamics are thought to be a major influence in the development of anorexia nervosa.
Which statement regarding a client's home environment should a nurse associate with the
development of anorexia nervosa?
1. The home environment maintains loose personal boundaries.
2. The home environment places an overemphasis on food.
3. The home environment is overprotective and demands perfection.
4. The home environment condones corporal punishment.
ANS: 3
Rationale: The nurse should assess that a home environment that is overprotective and demands
perfection may be a major influence in the development of anorexia nervosa. In adolescence,
distorted eating patterns may represent a rebellion against the parents viewed by the child as a
means of gaining and remaining in control.

Cognitive Level: Application


Integrated Process: Assessment

2. A client's altered body image is evidenced by claims of "feeling fat," even though the client is
emaciated. Which is the appropriate outcome criterion for this client's problem?
1. The client will consume adequate calories to sustain normal weight.
2. The client will cease strenuous exercise programs.
3. The client will perceive personal ideal body weight and shape as normal.
4. The client will not express a preoccupation with food.
ANS: 3
Rationale: The nurse should identify that the appropriate outcome for this client is to perceive
personal ideal body weight and shape as normal. Additional goals include accepting self based
on self-attributes instead of appearance and to realize that perfection is unrealistic.

Cognitive Level: Application


Integrated Process: Planning

3. A nurse is counseling a client diagnosed with bulimia nervosa about the symptom of tooth
enamel deterioration. Which explanation for this complication of bulimia nervosa, should the
nurse provide?
1. The emesis produced during purging is acidic and corrodes the tooth enamel.
2. Purging causes the depletion of dietary calcium.
3. Food is rapidly ingested without proper mastication.
4. Poor dental and oral hygiene leads to dental caries.
ANS: 1
Rationale: The nurse should explain to the client diagnosed with bulimia nervosa that his or her
teeth will eventually deteriorate, because the emesis produced during purging is acidic and
corrodes the tooth enamel. Excessive vomiting may also lead to dehydration and electrolyte
imbalance.

Cognitive Level: Application


Integrated Process: Implementation

13. Which of the following would contribute to a client's excessive weight gain? (Select all that
apply.)
1. A hypothalamus lesion
2. Hyperthyroidism
3. Diabetes mellitus
4. Cushing's disease
5. Low levels of serotonin
ANS: 1, 3, 4
Rationale: Lesions in the appetite and satiety centers in the hypothalamus may contribute to
overeating and lead to obesity. Hypothyroidism, not hyperthyroidism, is a problem that interferes
with basal metabolism and may lead to weight gain. Weight gain can also occur in response to
the decreased insulin production of diabetes mellitus and the increased cortisone production of
Cushing's disease. New evidence also exists to indicate that low levels of the neurotransmitter
serotonin may play a role in compulsive eating.

12. A nursing instructor is teaching about the DSM-5 criteria for the diagnosis of binge-eating
disorder. Which of the following student statements indicates that further instruction is needed?
(Select all that apply.)
1. "In this disorder, binge eating occurs exclusively during the course of bulimia nervosa."
2. "In this disorder, binge eating occurs, on average, at least once a week for three months."
3. "In this disorder, binge eating occurs, on average, at least two days a week for six months."
4. "In this disorder, distress regarding binge eating is present."
5. "In this disorder, distress regarding binge eating is absent."
ANS: 1, 3, 5
Rationale: According to the DSM-5 criteria for the diagnosis of binge-eating disorder, binge
eating should not occur exclusively during the course of anorexia nervosa or bulimia nervosa.
The new time frame criteria in the DSM-5 states that binge eating must occur, on average, at
least once a week for three months not two days a week for six months. The DSM-5 criteria
states that distress regarding binge eating would be present.

Cognitive Level: Application


Integrated Process: Assessment

A nurse should identify topiramate (Topamax) as the drug of choice for which of the following
conditions? (Select all that apply.)
1. Binge eating with a diagnosis of obesity
2. Bingeing and purging with a diagnosis of bulimia nervosa
3. Weight loss with a diagnosis of anorexia nervosa
4. Amenorrhea with a diagnosis of anorexia nervosa
5. Emaciation with a diagnosis of bulimia nervosa
ANS: 1, 2
Rationale: The nurse should identify that topiramate is the drug of choice when treating binge
eating with a diagnosis of obesity or bingeing and purging with a diagnosis of bulimia nervosa.
Topiramate is an anticonvulsant that produces a significant decline in binge frequency and
reduction in body weight.

Cognitive Level: Application


Integrated Process: Assessment
10. A client diagnosed with a history of anorexia nervosa comes to an outpatient clinic after
being medically cleared. The client states, "My parents watch me like a hawk and never let me
out of their sight." Which nursing diagnosis would take priority at this time?
1. Altered nutrition less than body requirements
2. Altered social interaction
3. Impaired verbal communication
4. Altered family processes
ANS: 4
Rationale: The nurse should determine that once the client has been medically cleared, the
diagnosis of altered family process should take priority. Clients diagnosed with anorexia nervosa
have a need to control and feel in charge of their own treatment choices. Behavioral-modification
therapy allows the client to maintain control of eating.

Cognitive Level: Analysis


Integrated Process: Diagnosis

9. A nurse is attempting to differentiate between the symptoms of anorexia nervosa and the
symptoms of bulimia. Which statement delineates the difference between these two disorders?
1. Clients diagnosed with anorexia nervosa experience extreme nutritional deficits, whereas
clients diagnosed with bulimia nervosa do not.
2. Clients diagnosed with bulimia nervosa experience amenorrhea, whereas clients diagnosed
with anorexia nervosa do not.
3. Clients diagnosed with bulimia nervosa experience hypotension, edema, and lanugo, whereas
clients diagnosed with anorexia nervosa do not.
4. Clients diagnosed with anorexia nervosa have eroded tooth enamel, whereas clients diagnosed
with bulimia nervosa do not.
ANS: 1
Rationale: The nurse should understand that clients diagnosed with anorexia nervosa experience
nutritional deficits, whereas clients diagnosed with bulimia do not. Anorexia is characterized by
a morbid fear of obesity and often results in low caloric and nutritional intake. Bulimia is
characterized by episodic, rapid consumption of large quantities of food followed by purging.

Cognitive Level: Application


Integrated Process: Assessment

8. A morbidly obese client is prescribed an anorexiant medication. The nurse should expect to
teach the client about which medication?
1. Phentermine (Mirapront)
2. Dexfenfluramine (Redux)
3. Sibutramine (Meridia)
4. Pemoline (Cylert)
ANS: 1
Rationale: The nurse should teach the client that phentermine is an anorexiant medication
prescribed for morbidly obese clients. Phentermine works on the hypothalamus to stimulate the
adrenal glands to release norepinephrine, a neurotransmitter that signals a fight-or-flight
response, reducing hunger. Dexfenfluramine has been removed from the market because of its
association with serious heart and lung disease. Several deaths have been associated with the use
of sibutramine by high-risk clients. Based on pressure from the FDA, the manufacturer issued a
recall of the drug in October 2010. Withdrawal from anorexiants can result in rebound weight
gain, lethargy, and depression.

Cognitive Level: Application


Integrated Process: Implementation

7. A client diagnosed with bulimia nervosa has been attending a mental health clinic for several
months. Which factor should a nurse identify as an appropriate indicator of a positive client
behavioral change?
1. The client gained two pounds in one week.
2. The client focused conversations on nutritious food.
3. The client demonstrated healthy coping mechanisms that decreased anxiety.
4. The client verbalized an understanding of the etiology of the disorder.
ANS: 3
Rationale: The nurse should identify that a client who demonstrates healthy coping mechanisms
to decrease anxiety indicates a positive behavioral change. Stress and anxiety can increase
bingeing, which is followed by inappropriate compensatory behavior.

Cognitive Level: Application


Integrated Process: Evaluation

6. The family of a client diagnosed with anorexia nervosa becomes defensive when the treatment
team calls for a family meeting. Which is the appropriate nursing response?
1. "Tell me why this family meeting is causing you to be defensive. All clients are required to
participate in two family sessions."
2. "Eating disorders have been correlated to certain familial patterns; without addressing these,
your child's condition will not improve."
3. "Family dynamics are not linked to eating disorders. The meeting is to provide your child with
family support."
4. "Clients diagnosed with anorexia nervosa are part of the family system, and any alteration in
family processes needs to be addressed."
ANS: 2
Rationale: The nurse should educate the family on the correlation between certain familial
patterns and anorexia nervosa. Families engaging in conflict avoidance and struggling with
issues of power and control may contribute to the development of anorexia nervosa.

Cognitive Level: Application


Integrated Process: Implementation
5. A potential Olympic figure skater collapses during practice and is hospitalized for severe
malnutrition. Anorexia nervosa is diagnosed. Which client statement best reflects insight related
to this disorder?
1. "Skaters need to be thin to improve their daily performance."
2. "All the skaters on the team are following an approved 1200-calorie diet."
3. "The exercise of skating reduces my appetite but improves my energy level."
4. "I am angry at my mother. I can only get her approval when I win competitions."
ANS: 4
Rationale: The client reflects insight when referring to feelings toward family dynamics that may
have influenced the development of the disease. Families who are overprotective and
perfectionistic can contribute to the development of anorexia nervosa.

Cognitive Level: Application


Integrated Process: Evaluation

4. A nurse is teaching a client diagnosed with an eating disorder about behavior-modification


programs. Why is this intervention the treatment of choice?
1. It helps the client correct a distorted body image.
2. It addresses the underlying client anger.
3. It manages the client's uncontrollable behaviors.
4. It allows clients to maintain control.
ANS: 4
Rationale: Behavior-modification programs are the treatment of choice for clients diagnosed
with eating disorders, because these programs allow clients to maintain control. Issues of control
are central to the etiology of these disorders. Behavior modification techniques function to
restore healthy weight.

Cognitive Level: Application


Integrated Process: Implementation

Terms in this set (29)


Original

1. Over the past year, a woman has cooked gourmet meals for her family but eats only tiny
servings. She wears layered, loose clothing and now has amenorrhea. Her current weight is 95
pounds, a loss of 35 pounds. Which medical diagnosis is most likely?
a. Binge eating disorder
b. Anorexia nervosa
c. Bulimia nervosa
d. Pica
ANS: B
Overly controlled eating behaviors, extreme weight loss, amenorrhea, preoccupation with food,
and wearing several layers of loose clothing to appear larger are part of the clinical picture of an
individual with anorexia nervosa. The individual with bulimia usually is near normal weight. The
binge eater is often overweight. Pica refers to eating nonfood items.

2. Disturbed body image is the nursing diagnosis for a patient diagnosed with an eating disorder.
Which outcome indicator is most appropriate to monitor?
a. Weight, muscle, and fat are congruent with height, frame, age, and sex.
b. Calorie intake is within the required parameters of the treatment plan.
c. Weight reaches the established normal range for the patient.
d. Patient expresses satisfaction with body appearance.
ANS: D
Body image disturbances are considered improved or resolved when the patient is consistently
satisfied with his or her own appearance and body function. This consideration is subjective. The
other indicators are more objective but less related to the nursing diagnosis.

3. A patient who is referred to the eating disorders clinic has lost 35 pounds in the past 3 months.
To assess the patients oral intake, the nurse should ask:
a. Do you often feel fat?
b. Who plans the family meals?
c. What do you eat in a typical day?
d. What do you think about your present weight?
ANS: C
Although all the questions might be appropriate to ask, only What do you eat in a typical day?
focuses on the eating patterns. Asking if the patient often feels fat focuses on distortions in body
image. Questions about family meal planning are unrelated to eating patterns. Asking for the
patients thoughts on present weight explores the patients feelings about weight.

4. A patient diagnosed with anorexia nervosa virtually stopped eating 5 months ago and has lost
25% of body weight. A nurse asks, Describe what you think about your present weight and how
you look. Which response by the patient is most consistent with the diagnosis?
a. I am fat and ugly.
b. What I think about myself is my business.
c. I am grossly underweight, but thats what I want.
d. I am a few pounds overweight, but I can live with it.
ANS: A
Patients diagnosed with anorexia nervosa do not recognize their thinness. They perceive
themselves to be overweight and unattractive. The patient with anorexia will usually disclose
perceptions about self to others. The patient with anorexia will persist in trying to lose more
weight.
5. A patient was diagnosed with anorexia nervosa. The history shows the patient virtually
stopped eating 5 months ago and has lost 25% of body weight. The patients current serum
potassium is 2.7 mg/dl. Which nursing diagnosis applies?
a. Adult failure to thrive, related to abuse of laxatives as evidenced by electrolyte imbalances and
weight loss
b. Disturbed energy field, related to physical exertion in excess of energy produced through
caloric intake as evidenced by weight loss and hyperkalemia
c. Ineffective health maintenance, related to self-induced vomiting as evidenced by swollen
parotid glands and hyperkalemia
d. Imbalanced nutrition: less than body requirements, related to malnutrition as evidenced by loss
of 25% of body weight and hypokalemia
ANS: D
The patients history and laboratory results support the correct nursing diagnosis. Available data
do not confirm that the patient uses laxatives, induces vomiting, or exercises excessively. The
patient has hypokalemia rather than hyperkalemia.

6. Outpatient treatment is planned for a patient diagnosed with anorexia nervosa. Select the most
important outcome related to the nursing diagnosis: Imbalanced nutrition: less than body
requirements. Within 1 week, the patient will:
a. weigh self accurately using balanced scales.
b. limit exercise to less than 2 hours daily.
c. select clothing that fits properly.
d. gain 1 to 2 pounds.
ANS: D
Only the outcome of a gain of 1 to 2 pounds can be accomplished within 1 week when the
patient is an outpatient. The focus of an outcome is not on the patient weighing self. Limiting
exercise and selecting proper clothing are important, but weight gain takes priority.

7. Which nursing intervention has priority as a patient diagnosed with anorexia nervosa begins to
gain weight?
a. Assess for depression and anxiety.
b. Observe for adverse effects of re-feeding.
c. Communicate empathy for the patients feelings.
d. Help the patient balance energy expenditure and caloric intake.
ANS: B
The nursing intervention of observing for adverse effects of re-feeding most directly relates to
weight gain and is a priority. Assessing for depression and anxiety and communicating empathy
relate to coping. Helping the patient balance energy expenditure and caloric intake is an
inappropriate intervention.
8. A patient diagnosed with anorexia nervosa is resistant to weight gain. What is the rationale for
establishing a contract with the patient to participate in measures designed to produce a specified
weekly weight gain?
a. Because severe anxiety concerning eating is expected, objective and subjective data must be
routinely collected.
b. Patient involvement in decision-making increases a sense of control and promotes compliance
with the treatment.
c. A team approach to planning the diet ensures that physical and emotional needs of the patient
are met.
d. Because of increased risk for physical problems with re-feeding, obtaining patient permission
is required.
ANS: B
A sense of control for the patient is vital to the success of therapy. A diet that controls weight
gain can allay patient fears of a too-rapid weight gain. Data collection is not the reason for
contracting. A team approach is wise but is not a guarantee that the patients needs will be met.
Permission for treatment is a separate issue. The contract for weight gain is an additional aspect
of treatment.

9. The nursing care plan for a patient diagnosed with anorexia nervosa includes the intervention
Monitor for complications of re-feeding. Which body system should a nurse closely monitor for
dysfunction?
a. Renal
b. Endocrine
c. Central nervous
d. Cardiovascular
ANS: D
Re-feeding resulting in a too-rapid weight gain can overwhelm the heart, resulting in
cardiovascular collapse. Focused assessment becomes a necessity to ensure patient physiologic
integrity. The other body systems are not initially involved in the re-feeding syndrome.

10. A psychiatric clinical nurse specialist uses cognitive therapy techniques with a patient
diagnosed with anorexia nervosa. Which statement by the staff nurse supports this type of
therapy?
a. What are your feelings about not eating the food that you prepare?
b. You seem to feel much better about yourself when you eat something.
c. It must be difficult to talk about private matters to someone you just met.
d. Being thin does not seem to solve your problems. You are thin now but still unhappy.
ANS: D
The correct response is the only strategy that attempts to question the patients distorted thinking.

11. An appropriate intervention for a patient diagnosed with bulimia nervosa who binges and
purges is to teach the patient to:
a. eat a small meal after purging.
b. avoid skipping meals or restricting food.
c. concentrate oral intake after 4 PM daily.
d. understand the value of reading journal entries aloud to others.
ANS: B
One goal of health teaching is the normalization of eating habits. Food restriction and skipping
meals lead to rebound bingeing. Teaching the patient to eat a small meal after purging will
probably perpetuate the need to induce vomiting. Teaching the patient to concentrate intake after
4 PM will lead to late-day bingeing. Journal entries are private.

12. What behavior by a nurse caring for a patient diagnosed with an eating disorder indicates the
nurse needs supervision?
a. The nurses comments are nonjudgmental.
b. The nurse uses an authoritarian manner when interacting with the patient.
c. The nurse teaches the patient to recognize signs of increasing anxiety and ways to intervene.
d. The nurse refers the patient to a self-help group for individuals with eating disorders.
ANS: B
In the effort to motivate the patient and take advantage of the decision to seek help and be
healthier, the nurse must take care not to cross the line toward authoritarianism and assume the
role of a parent. The helpful nurse uses a problem-solving approach and focuses on the patients
feelings of shame and low self-esteem. Referral to a self-help group is an appropriate
intervention.

13. A nursing diagnosis for a patient diagnosed with bulimia nervosa is: Ineffective coping,
related to feelings of loneliness as evidenced by overeating to comfort self, followed by self-
induced vomiting. The best outcome related to this diagnosis is, Within 2 weeks the patient will:
a. appropriately express angry feelings.
b. verbalize two positive things about self.
c. verbalize the importance of eating a balanced diet.
d. identify two alternative methods of coping with loneliness.
ANS: D
The outcome of identifying alternative coping strategies is most directly related to the diagnosis
of Ineffective coping. Verbalizing positive characteristics of self and verbalizing the importance
of eating a balanced diet are outcomes that might be used for other nursing diagnoses.
Appropriately expressing angry feelings is not measurable.

14. Which nursing intervention has the highest priority for a patient diagnosed with bulimia
nervosa?
a. Assist the patient to identify triggers to binge eating.
b. Provide corrective consequences for weight loss.
c. Explore patient needs for health teaching.
d. Assess for signs of impulsive eating.
ANS: A
For most patients with bulimia nervosa, certain situations trigger the urge to binge; purging then
follows. The triggers are often anxiety-producing situations. Identifying these triggers makes it
possible to break the binge-purge cycle. Because binge eating and purging directly affect
physical status, the need to promote physical safety assumes the highest priority. The question
calls for an intervention rather than an assessment.

15. One bed is available on the inpatient eating disorders unit. Which patient should be admitted?
The patient whose weight dropped from:
a. 150 to 100 pounds over a 4-month period. Vital signs: temperature, 35.9 C; pulse, 38
beats/min; blood pressure, 60/40 mm Hg
b. 120 to 90 pounds over a 3-month period. Vital signs: temperature, 36 C; pulse, 50 beats/min;
blood pressure, 70/50 mm Hg
c. 110 to 70 pounds over a 4-month period. Vital signs: temperature, 36.5 C; pulse, 60 beats/min;
blood pressure, 80/66 mm Hg
d. 90 to 78 pounds over a 5-month period. Vital signs: temperature, 36.7 C; pulse, 62 beats/min;
blood pressure, 74/48 mm Hg
ANS: A
Physical criteria for hospitalization include weight loss of more than 30% of body weight within
6 months, temperature below 36 C (hypothermia), heart rate less than 40 beats/min, and systolic
blood pressure less than 70 mm Hg.

16. While providing health teaching for a patient diagnosed with bulimia nervosa, a nurse should
emphasize information about:
a. self-monitoring of daily food and fluid intake.
b. establishing the desired daily weight gain.
c. recognizing symptoms of hypokalemia.
d. self-esteem maintenance.
ANS: C
Hypokalemia results from potassium loss associated with vomiting. Physiologic integrity can be
maintained if the patient can self-diagnose potassium deficiency and adjust the diet or seek
medical assistance. Self-monitoring of daily food and fluid intake is not useful if the patient
purges. Daily weight gain may not be desirable for a patient with bulimia nervosa. Self-esteem is
an identifiable problem but is of lesser priority than the risk for hypokalemia.

17. As a patient admitted to the eating disorders unit undresses, a nurse observes that the patients
body is covered by fine, downy hair. The patient weighs 70 pounds and is 5 feet, 4 inches tall.
Which condition should be documented?
a. Amenorrhea
b. Alopecia
c. Lanugo
d. Stupor
ANS: C
The fine, downy hair noted by the nurse is called lanugo. It is frequently seen in patients with
anorexia nervosa. None of the other conditions can be supported by the data the nurse has
gathered.

18. A patient being admitted to the eating disorders unit has a yellow cast to the skin and fine,
downy hair covering the body. The patient weighs 70 pounds; height is 5 feet, 4 inches. The
patient is quiet and says only, I wont eat until I look thin. What is the priority initial nursing
diagnosis?
a. Anxiety, related to fear of weight gain
b. Disturbed body image, related to weight loss
c. Ineffective coping, related to lack of conflict resolution skills
d. Imbalanced nutrition: less than body requirements, related to self-starvation
ANS: D
The physical assessment shows cachexia, which indicates imbalanced nutrition. Addressing the
patients self-starvation is the priority above the incorrect responses.

19. A nurse conducting group therapy on the eating disorders unit schedules the sessions
immediately after meals for the primary purpose of:
a. maintaining patients concentration and attention.
b. shifting the patients focus from food to psychotherapy.
c. focusing on weight control mechanisms and food preparation.
d. processing the heightened anxiety associated with eating.
ANS: D
Eating produces high anxiety for patients with eating disorders. Anxiety levels must be lowered
if the patient is to be successful in attaining therapeutic goals. Shifting the patients focus from
food to psychotherapy and focusing on weight control mechanisms and food preparation are not
desirable. Maintaining patients concentration and attention is important, but not the primary
purpose of the schedule.

20. Physical assessment of a patient diagnosed with bulimia nervosa often reveals:
a. prominent parotid glands.
b. peripheral edema.
c. thin, brittle hair.
d. amenorrhea.
ANS: A
Prominent parotid glands are associated with repeated vomiting. The other options are signs of
anorexia nervosa and are not usually observed in bulimia.

21. Which personality characteristic is a nurse most likely to assess in a patient diagnosed with
anorexia nervosa?
a. Carefree flexibility
b. Rigidity, perfectionism
c. Open displays of emotion
d. High spirits and optimism
ANS: B
Rigid thinking, inability to demonstrate flexibility, and difficulty changing cognitions are
characteristic of patients diagnosed with eating disorders. The incorrect options are rare in a
patient with anorexia nervosa. Inflexibility, controlled emotions, and pessimism are more the
norm.

22. Which assessment finding for a patient diagnosed with an eating disorder meets a criterion
for hospitalization?
a. Urine output: 40 ml/hr
b. Pulse rate: 58 beats/min
c. Serum potassium: 3.4 mEq/L
d. Systolic blood pressure: 62 mm Hg
ANS: D
Systolic blood pressure less than 70 mm Hg is an indicator for inpatient care. Many people
without eating disorders have bradycardia (pulse less than 60 beats/min). Urine output should be
more than 30 ml/hr. A potassium level of 3.4 mEq/L is within the normal range.

23. Which statement is a nurse most likely to hear from a patient diagnosed with anorexia
nervosa?
a. I would be happy if I could lose 20 more pounds.
b. My parents dont pay much attention to me.
c. Im thin for my height.
d. I have nice eyes.
ANS: A
Patients with eating disorders have distorted body images and cognitive distortions. They see
themselves as overweight even when their weight is subnormal. Im thin for my height is
therefore unlikely to be heard from a patient with anorexia nervosa. Poor self-image precludes
making positive statements about self, such as I have nice eyes. Many patients with eating
disorders see supportive others as intrusive and out of tune with their needs.

24. Which nursing diagnosis is more applicable for a patient diagnosed with anorexia nervosa
who restricts intake and is 20% below normal weight than for a 130-pound patient diagnosed
with bulimia nervosa who purges?
a. Powerlessness
b. Ineffective coping
c. Disturbed body image
d. Imbalanced nutrition: less than body requirements
ANS: D
The patient with bulimia nervosa usually maintains a close to normal weight, whereas the patient
with anorexia nervosa may approach starvation. The incorrect options may be appropriate for
patients with either anorexia nervosa or bulimia nervosa.
25. An outpatient diagnosed with anorexia nervosa has begun re-feeding. Between the first and
second appointments, the patient gained 8 pounds. The nurse should:
a. assess lung sounds and extremities.
b. suggest the use of an aerobic exercise program.
c. positively reinforce the patient for the weight gain.
d. establish a higher goal for weight gain the next week.
ANS: A
Weight gain of more than 2 to 5 pounds weekly may overwhelm the hearts capacity to pump,
leading to cardiac failure. The nurse must assess for signs of pulmonary edema and congestive
heart failure. The incorrect options are undesirable because they increase the risk for cardiac
complications.

26. When a nurse finds a patient diagnosed with anorexia nervosa vigorously exercising before
gaining the agreed-upon weekly weight, the nurse should state:
a. You and I will have to sit down and discuss this problem.
b. It bothers me to see you exercising. Youll lose more weight.
c. Lets discuss the relationship between exercise and weight loss and how that affects your body.
d. According to our agreement, no exercising is permitted until you have gained a specific
amount of weight.
ANS: D
A matter-of-fact statement that the nurses perceptions are different helps avoid a power struggle.
Treatment plans have specific goals for weight restoration. Exercise is limited to promote weight
gain. Patients must be held accountable for required behaviors.

27. A patient diagnosed with anorexia nervosa has a body mass index (BMI) of 14.8 kg/m2.
Which assessment finding is most likely to accompany this value?
a. Cachexia
b. Leukocytosis
c. Hyperthermia
d. Hypertension
ANS: A
The BMI value indicates extreme malnutrition. Cachexia is a hallmark of this problem. The
patient would be expected to have leukopenia rather than leukocytosis. Hypothermia and
hypotension are likely assessment findings.

1. A patient referred to the eating disorders clinic has lost 35 pounds in 3 months and has
developed amenorrhea. For which physical manifestations of anorexia nervosa should a nurse
assess? Select all that apply.
a. Peripheral edema
b. Parotid swelling
c. Constipation
d. Hypotension
e. Dental caries
f. Lanugo
ANS: A, C, D, F
Peripheral edema is often present because of hypoalbuminemia. Constipation related to
starvation is often present. Hypotension is often present because of dehydration. Lanugo is often
present and is related to starvation. Parotid swelling is associated with bulimia. Dental caries are
associated with bulimia.

2. A patient diagnosed with anorexia nervosa is hospitalized for treatment. What features should
the milieu provide? Select all that apply.
a. Flexible mealtimes
b. Unscheduled weight checks
c. Adherence to a selected menu
d. Observation during and after meals
e. Monitoring during bathroom trips
f. Privileges correlated with emotional expression
ANS: C, D, E
Priority milieu interventions support the restoration of weight and a normalization of eating
patterns. These goals require close supervision of the patients eating habits and the prevention of
exercise, purging, and other activities. Menus are strictly adhered to. Patients are observed
during and after meals to prevent them from throwing away food or purging. All trips to the
bathroom are monitored. Mealtimes are structured, not flexible. Weighing is performed on a
regular schedule. Privileges are correlated with weight gain and treatment plan compliance.

Terms in this set (15)


Original

11
Cognitive Impairment Disorders

1. The nurse enters the room of a client with a cognitive impairment disorder and asks what day
of the week it is; what the date, month, and year are; and where the client is. The nurse is
attempting to assess:
1. confabulation.
2. delirium.
3. orientation.
4. perseveration.
Hide Answer
1. The answer is 3.
The initial, most basic assessment of a client with cognitive impairment involves determining his
level of orientation (awareness of time, place, and person). The nurse may also assess for
confabulation and perseveration in a client with cognitive impairment; but the questions in this
situation would not elicit the symptom response. Delirium is a type of cognitive impairment;
however, other symptoms are necessary to establish this diagnosis.

2. Which of the following best describes dementia?


1. Memory loss occurring as part of the natural consequence of aging
2. Difficulty coping with physical and psychological change
3. Severe cognitive impairment that occurs rapidly
4. Loss of cognitive abilities, impairing ability to perform activities of daily living
Hide Answer
2. The answer is 4.
The impaired ability to perform self-care is an important measure of a client's dementia
progression and loss of cognitive abilities. Difficulty or impaired ability to perform normal
activities of daily living, such as maintaining hygiene and grooming, toileting, making meals,
and maintaining a household, are significant indications of dementia. Slowing of processes
necessary for information retrieval is a normal consequence of aging. However, the global
statement that memory loss occurs as part of natural aging is not true. Dementia is not normal; it
is a disease. Difficulty coping with changes can be experienced by any client, not just one with
dementia. The rapid occurrence of cognitive impairment refers to delirium.

3. Which of the following will the nurse use when communicating with a client who has a
cognitive impairment?
1. Complete explanations with multiple details
2. Pictures or gestures instead of words
3. Stimulating words and phrases to capture the client's attention
4. Short words and simple sentences
Hide Answer
3. The answer is 4.
Short words and simple sentences minimize client confusion and enhance communication.
Complete explanations with multiple details and stimulating words and phrases would increase
confusion in a client with short attention span and difficulty with comprehension. Although
pictures and gestures may be helpful, they would not substitute for verbal communication.

4. A 75-year-old client has dementia of the Alzheimer's type and confabulates. The nurse
understands that this client:
1. denies confusion by being jovial.
2. pretends to be someone else.
3. rationalizes various behaviors.
4. fills in memory gaps with fantasy.
Hide Answer
4. The answer is 4.
Confabulation is a communication device used by patients with dementia to compensate for
memory gaps. The remaining answer choices are incorrect.
5. Which ability should a nurse expect from a client in the mild stage of dementia of the
Alzheimer's type?
1. Remembering the daily schedule
2. Recalling past events
3. Coping with anxiety
4. Solving problems of daily living
Hide Answer
5. The answer is 2.
Recent memory loss is the characteristic sign of cognitive difficulty in early Alz-heimer's
disease. The ability to recall past events is usually retained until the later stages of this disorder.
Remembering daily schedules, coping with anxiety, and solving problems of daily living are
areas that would pose difficulty in the early phase of Alzheimer's disease.

6. An 82-year-old man is admitted to the medical-surgical unit for diagnostic confirmation and
management of probable delirium. Which statement by the client's daughter best supports the
diagnosis?
1. "Maybe it's just caused by aging. This usually happens by age 82."
2. "The changes in his behavior came on so quickly! I wasn't sure what was happening."
3. "Dad just didn't seem to know what he was doing. He would forget what he had for breakfast."
4. "Dad has always been so independent. He's lived alone for years since Mom died."
Hide Answer
6. The answer is 2.
Delirium is an acute process characterized by abrupt, spontaneous cognitive dysfunction.
Cognitive impairment disorders (dementia or delirium) are not normal consequences of aging.
Option 3 would be characteristic of someone with dementia. Although option 4 provides
background data about the client, it is unrelated to the current problem of delirium.

7. An elderly client with Alzheimer's disease becomes agitated and combative when a nurse
approaches to help with morning care. The most appropriate nursing intervention in this situation
would be to:
1. tell the client firmly that it is time to get dressed.
2. obtain assistance to restrain the client for safety.
3. remain calm and talk quietly to the client.
4. call the doctor and request an order for sedation.
Hide Answer
7. The answer is 3.
Maintaining a calm approach when intervening with an agitated client is extremely important.
Telling the client firmly that it is time to get dressed may increase his agitation, especially if the
nurse touches him. Restraints are a last resort to ensure client safety and are inappropriate in this
situation. Sedation should be avoided, if possible, because it will interfere with CNS functioning
and may contribute to the client's confusion.
8. Which goal is a priority for a client with a DSM-IV TR diagnosis of delirium and the nursing
diagnosis Acute confusion related to recent surgery secondary to traumatic hip fracture ?
1. The client will complete activities of daily living.
2. The client will maintain safety.
3. The client will remain oriented.
4. The client will understand communication.
Hide Answer
8. The answer is 2.
Maintaining safety is the priority goal for an acutely confused client who recently had surgery.
All measures to promote physiologic safety and psychosocial well-being would be implemented.
This client would not be capable of completing activities of daily living, and safety is a priority
over these tasks. The goals of remaining oriented and understanding communication would be
appropriate only after the client's acute confusion has resolved.

9. Which of the following is not included in the care plan of a client with a moderate cognitive
impairment involving dementia of the Alzheimer's type?
1. Daily structured schedule
2. Positive reinforcement for performing activities of daily living
3. Stimulating environment
4. Use of validation techniques
Hide Answer
9. The answer is 3.
A stimulating environment is a source of confusion and anxiety for a client with a moderate level
of impairment and, therefore, would not be included in the plan of care. The remaining options
are all appropriate interventions for this client.

10. In clients with a cognitive impairment disorder, the phenomenon of increased confusion in
the early evening hours is called:
1. aphasia.
2. agnosia.
3. sundowning.
4. confabulation.
Hide Answer
10. The answer is 3.
Sundowning is a common phenomenon that occurs after daylight hours in a client with a
cognitive impairment disorder. The other options are incorrect responses, although all may be
seen in this client.

11. An 80-year-old man is accompanied to the clinic by his son, who tells the nurse that the
client's constant confusion, incontinence, and tendency to wander are intolerable. The client was
diagnosed with chronic cognitive impairment disorder. Which nursing diagnosis is most
appropriate for the client's son?
1. Risk for other-directed violence
2. Disturbed sleep pattern
3. Caregiver role strain
4. Social isolation
Hide Answer
11. The answer is 3.
The son's description exemplifies some of the problems commonly encountered by a primary
caregiver who is caring for someone with a cognitive impairment disorder. Although the other
nursing diagnoses are possibilities, the scenario does not provide enough information to validate
any of these.

12. Which of the following outcome criteria is appropriate for the client with dementia?
1. The client will return to an adequate level of self-functioning.
2. The client will learn new coping mechanisms to handle anxiety.
3. The client will seek out resources in the community for support.
4. The client will follow an established schedule for activities of daily living.
Hide Answer
12. The answer is 4.
Following established activity schedules is a realistic expectation for clients with dementia. All
of the remaining outcome statements require a higher level of cognitive ability than can be
realistically expected of clients with this disorder.

13. A family member expresses concern to a nurse about behavioral changes in an elderly aunt.
Which would cause the nurse to suspect a cognitive impairment disorder?
1. Decreased interest in activities that she once enjoyed
2. Fearfulness of being alone at night
3. Increased complaints of physical ailments
4. Problems with preparing a meal or balancing her checkbook
Hide Answer
13. The answer is 4.
Making a meal and balancing a checkbook are higher level cognitive functions that, when unable
to be performed, may signal onset of a cognitive disorder. Although the remaining behaviors
may occur, they are not associated only with cognitive impairment and may indicate depression
or other problems.

14. During the home visit of a client with dementia, the nurse notes that an adult daughter
persistently corrects her father's misperceptions of reality, even when her father becomes upset
and anxious. Which intervention should the nurse teach the caregiver?
1. Anxiety-reducing measures
2. Positive reinforcement
3. Reality orientation techniques
4. Validation techniques
Hide Answer
14. The answer is 4.
Validation techniques are useful measures for making emotional connections with a client who
can no longer maintain reality orientation. These measures are also helpful in decreasing anxiety.
Anxiety-reducing measures and positive reinforcements will also be appropriate, but validation
techniques will provide both anxiety reduction and positive reinforcement for the client. Reality
orientation techniques are not useful when the client can no longer maintain reality contact and
becomes upset when misperceptions are corrected.

15. A client with moderate dementia has frequent catastrophic reactions during shower time.
Which of the following interventions should be implemented in the plan of care? Select all that
apply.
1. Assign consistent staff members to assist the client.
2. Accomplish the task quickly, with several staff members assisting.
3. Schedule the client's shower at the same time of day.
4. Sedate the client 30 minutes prior to showering.
5. Tell the client to remain calm while showering.
6. Use a calm, supportive, quiet manner when assisting the client.
Hide Answer
15. The answer is 1, 3, 6.
Maintaining a consistent routine with the same staff members will help decrease the client's
anxiety that occurs whenever changes are made. A calm, quiet manner will be reassuring to the
client, also helping to minimize anxiety. Moving quickly with several staff will increase the
client's anxiety and may precipitate a catastrophic reaction. The use of sedation is not indicated
and may increase the risk of client injury from side effect of drowsiness. Telling the client to
remain calm is inappropriate because a client with dementia cannot respond to such a direction.

Incorrect
75% got this correct

1) The primary nursing intervention in working with a client with moderate


stage dementia is ensuring that the client:
Receives adequate nutrition and hydration
Will reminisce to decrease isolation
Remains in a safe and secure environment
Independently performs self-care
Incorrect
50% got this correct

2) Dementia. unlike delirium. is characterized by:


Slurred speech
Insidious onset
Clouding of consciousness
Sensory perceptual change
Incorrect
53% got this correct

3) Situation: A 17-year-old gymnast is admitted to the hospital due to weight


loss and dehydration secondary to starvation. Which of the following
nursing diagnoses will be given priority for the client?
Altered self-image
Fluid volume deficit
Altered nutrition less than body requirements
Altered family process
Incorrect
67% got this correct

4) What is the best intervention to teach the client when she feels the need to
starve?
Allow her to starve to relieve her anxiety
Do a short term exercise until the urge passes
Approach the nurse and talk out her feelings
Call her mother on the phone and tell her how she feels
Incorrect
58% got this correct

5) She tearfully tells the nurse "I can't take it when she accuses me of stealing
her things." Which response by the nurse will be most therapeutic?
”Don’t take it personally. Your mother does not mean it.”
“Have you tried discussing this with your mother?”
“This must be difficult for you and your mother.”
“Next time ask your mother where her things were last seen.”

More Psychiatric Nursing Nclex Quizzes

Psychiatric Nursing | NCLEX Quiz 190

NCLEX Practice Questions On Psychiatric Nursing

Psychiatric Nursing | NCLEX Quiz 188


Featured Quizzes
Do I Have A Crush On Him? Quiz

Data Privacy Quiz Questions And Answers

The Ultimate Environment Quiz To Test Your Knowledge


Related Topics
 NCLEX RN NCLEX Pn Newborn Nursing Obstetrical Nursing Pediatric Nursing

Advertisement

Florida Nature Lover Home SPONSORED BY 12721 20TH ST E


LEARN MORE

 1. Question 1 point(s)

A psychotic client reports to the evening nurse that the day nurse put something
suspicious in his water with his medication. The nurse replies, “You’re worried about
your medication?” The nurse’s communication is:

 A. An example of presenting reality


B. Reinforcing the client’s delusions
C. Focusing on emotional content
D. A non-therapeutic technique called mind-reading

Incorrect Correct Answer: C. Focusing on emotional content
The nurse should help the client focus on the emotional content rather than delusional
material. Sometimes during a conversation, patients mention something particularly
important. When this happens, nurses can focus on their statement, prompting patients to
discuss it further. Patients don’t always have an objective perspective on what is relevant
to their case; as impartial observers, nurses can more easily pick out the topics to focus
on.
 Option A: Presenting reality isn’t helpful because it can lead to confrontation and
disengagement. It’s frequently useful for nurses to summarize what patients have
said after the fact. This demonstrates to patients that the nurse was listening and
allows the nurse to document conversations. Ending a summary with a phrase like
“Does that sound correct?” gives patients explicit permission to make corrections
if they’re necessary.
 Option B: Agreeing with the client and supporting his beliefs are reinforcing
delusions. Patients often ask nurses for advice about what they should do about
particular problems or in specific situations. Nurses can ask patients what they
think they should do, which encourages patients to be accountable for their own
actions and helps them come up with solutions themselves.
 Option D: Mind reading isn’t therapeutic. Similar to active listening, asking
patients for clarification when they say something confusing or ambiguous is
important. Saying something like “I’m not sure I understand. Can you explain it
to me?” helps nurses ensure they understand what’s actually being said and can
help patients process their ideas more thoroughly

 2. Question 1 point(s)

A client is admitted to the inpatient unit of the mental health center with a diagnosis of
paranoid schizophrenia. He’s shouting that the government of France is trying to
assassinate him. Which of the following responses is most appropriate?

 A. “I think you’re wrong. France is a friendly country and an ally of the United
States. Their government wouldn’t try to kill you.”
B. “I find it hard to believe that a foreign government or anyone else is trying to
hurt you. You must feel frightened by this.”
C. “You’re wrong. Nobody is trying to kill you.”
D. “A foreign government is trying to kill you? Please tell me more about it.”

Incorrect Correct Answer: B. “I find it hard to believe that a foreign government or
anyone else is trying to hurt you. You must feel frightened by this.”
Responses should focus on reality while acknowledging the client’s feelings. Sometimes
during a conversation, patients mention something particularly important. When this
happens, nurses can focus on their statement, prompting patients to discuss it further.
Patients don’t always have an objective perspective on what is relevant to their case; as
impartial observers, nurses can more easily pick out the topics to focus on.
 Option A: Arguing with the client or denying his belief isn’t therapeutic. By
using nonverbal and verbal cues such as nodding and saying “I see,” nurses can
encourage patients to continue talking. Active listening involves showing interest
in what patients have to say, acknowledging that you’re listening and
understanding, and engaging with them throughout the conversation. Nurses can
offer general leads such as “What happened next?” to guide the conversation or
propel it forward.
 Option C: Arguing can also inhibit development of a trusting relationship.
Continuing to talk about delusions may aggravate the psychosis. It’s frequently
useful for nurses to summarize what patients have said after the fact. This
demonstrates to patients that the nurse was listening and allows the nurse to
document conversations. Ending a summary with a phrase like “Does that sound
correct?” gives patients explicit permission to make corrections if they’re
necessary.
 Option D: Asking the client if a foreign government is trying to kill him may
increase his anxiety level and can reinforce his delusions. Voicing doubt can be a
gentler way to call attention to the incorrect or delusional

 3. Question 1 point(s)

A client receiving haloperidol (Haldol) complains of a stiff jaw and difficulty


swallowing. The nurse’s first action is to:

 A. Reassure the client and administer as needed lorazepam (Ativan) I.M.


B. Administer as needed dose of benztropine (Cogentin) I.M. as ordered.
C. Administer as needed dose of benztropine (Cogentin) by mouth as ordered.
D. Administer as needed dose of haloperidol (Haldol) by mouth.

Incorrect Correct Answer: B. Administer as needed dose of benztropine (Cogentin)
I.M. as ordered.
The client is most likely suffering from muscle rigidity due to haloperidol. I.M.
benztropine should be administered to prevent asphyxia or aspiration. The extrapyramidal
symptoms are muscular weakness or rigidity, a generalized or localized tremor that may
be characterized by the akinetic or agitation types of movements, respectively.
Haloperidol overdose is also associated with ECG changes known as torsade de pointes,
which may cause arrhythmia or cardiac arrest.
 Option A: Lorazepam treats anxiety, not extrapyramidal effects. Lorazepam is a
benzodiazepine medication developed by DJ Richards. It went on the market in
the United States in 1977. Lorazepam has common use as the sedative and
anxiolytic of choice in the inpatient setting owing to its fast (1 to 3 minute) onset
of action when administered intravenously. Lorazepam is also one of the few
sedative-hypnotics with a relatively clean side effect profile. Lorazepam is FDA
approved for short-term (4 months) relief of anxiety symptoms related to anxiety
disorders, anxiety-associated insomnia, anesthesia premedication in adults to
relieve anxiety, or to produce sedation/amnesia, and treatment of status
epilepticus.
 Option C: Benztropine belongs to the synthetic class of muscarinic receptor
antagonists (anticholinergic drugs). Thus, it has a structure similar to that of
diphenhydramine and atropine. However, it is long-acting so that its
administration can be with less frequency than diphenhydramine. It also induces
less CNS stimulation effect compared to that of trihexyphenidyl, making it a
preferable drug of choice for geriatric patients.
 Option D: Another dose of haloperidol would increase the severity of the
reaction. Since there is no specific antidote, supportive treatment is the mainstay
of haloperidol toxicity. If a patient develops signs and symptoms of toxicities, the
clinician should consider gastric lavage or induction of emesis as soon as
possible, followed by the administration of activated charcoal. Maintenance of
Airway, Breathing, and circulation are the most important factors for survival.

 4. Question 1 point(s)

The nurse is caring for a client with schizophrenia who experiences auditory
hallucinations. The client appears to be listening to someone who isn’t visible. He
gestures, shouts angrily, and stops shouting in mid-sentence. Which nursing intervention
is the most appropriate?

 A. Approach the client and touch him to get his attention.


B. Encourage the client to go to his room where he’ll experience fewer
distractions.
C. Acknowledge that the client is hearing voices but make it clear that the nurse
doesn’t hear these voices.
D. Ask the client to describe what the voices are saying.

Incorrect Correct Answer: C. Acknowledge that the client is hearing voices but
make it clear that the nurse doesn’t hear these voices.
By acknowledging that the client hears voices, the nurse conveys acceptance of the client.
By letting the client know that the nurse doesn’t hear the voices, the nurse avoids
reinforcing the hallucination. Auditory hallucinations are the sensory perceptions of
hearing voices without an external stimulus. This symptom is particularly associated with
schizophrenia and related psychotic disorders but is not specific to it. Auditory
hallucinations are one of the major symptoms of psychosis.
 Option A: The nurse shouldn’t touch the client with schizophrenia without
advance warning. The hallucinating client may believe that the touch is a threat or
act of aggression and respond violently.
 Option B: Being alone in his room encourages the client to withdraw and may
promote more hallucinations. The nurse should provide an activity to distract the
client.
 Option D: By asking the client what the voices are saying, the nurse is
reinforcing the hallucination. The nurse should focus on the client’s feelings,
rather than the content of the hallucination.

 5. Question 1 point(s)

A client with paranoid schizophrenia has been experiencing auditory hallucinations for
many years. One approach that has proven to be effective for hallucinating clients is to:

 A. Take an as-needed dose of psychotropic medication whenever they hear


voices.
B. Practice saying “Go away” or “Stop” when they hear voices.
C. Sing loudly to drown out the voices and provide a distraction.
D. Go to their room until the voices go away.

Incorrect Correct Answer: B. Practice saying “Go away” or “Stop” when they hear
voices.
Researchers have found that some clients can learn to control bothersome hallucinations
by telling the voices to go away or stop. The estimated prevalence of auditory
hallucinations in the general population ranges from 5 to 28%. Auditory hallucinations
are the most commonly reported in psychotic patients. They are prevalent in 75% of
individuals suffering from schizophrenia, 20-50% of individuals with bipolar disorder,
10% of individuals with major psychotic depression, and 40% of individuals with PTSD.
 Option A: Taking an as-needed dose of psychotropic medication whenever the
voices arise may lead to overmedication and put the client at risk for adverse
effects. Because the voices aren’t likely to go away permanently, the client must
learn to deal with the hallucinations without relying on drugs.
 Option C: Although distraction is helpful, singing loudly may upset other clients
and would be socially unacceptable after the client is discharged. In children and
adolescents, the prevalence has been noted to be 9% and ranging between 5 to
16%, respectively. In children, it is mostly seen in conjunction with conduct
disorder, migraine, and anxiety. The discontinuation rate of auditory
hallucinations in adolescence ranges from 3 to 40% each year.
 Option D: Hallucinations are most bothersome in a quiet environment when the
client is alone, so sending the client to his room would increase, rather than
decrease, the hallucinations. Auditory hallucinatory experiences are
psychopathological end-points. Disturbances in consciousness may occur earlier
in the course that includes thought blocking, thought pressure, obsessive
perseveration, and failure to discriminate between thought and perception.

 6. Question 1 point(s)
A client with catatonic schizophrenia is mute, can’t perform activities of daily living, and
stares out the window for hours. What is the nurse’s first priority?

 A. Assist the client with feeding


B. Assist the client with showering
C. Reassure the client about safety
D. Encourage socialization with peers

Incorrect Correct Answer: A. Assist the client with feeding
According to Maslow’s hierarchy of needs, the need for food is among the most
important. The initial management includes supportive measures such as IV fluids and
even nasogastric tubes given that patients with catatonia are susceptible to malnutrition,
dehydration, pneumonia, etc. The key is early identification of catatonia in a patient with
schizophrenia and initiation of treatment.
 Option B: Catatonia again is a complex combination of psychomotor
abnormalities and mood and thought processes. There are at least forty different
signs and symptoms that have been associated with catatonia. The Diagnostic and
Statistical Manual V has criteria for catatonia with specifiers, including that for
schizophrenia.
 Option C: Features of catatonia had been described since the 1800s with
prominent physicians such as Kahlbaum and even Kraepelin, who defined
catatonia within the larger definition of dementia praecox.[2] There are several
theories behind the same as catatonia can be part of a larger psychiatric or
neurological illness. Kahlbaum has ultimately been credited with the
understanding that symptoms such as stupor and catalepsy were part of a larger
syndrome of psychomotor abnormalities, which he termed as “catatonia.” This
can be a part of a larger schizophrenic illness or even a bipolar affective illness or
medical illness.
 Option D: Other needs, in order of decreasing importance, include hygiene,
safety, and a sense of belonging. The epidemiology of catatonic schizophrenia can
be multivariate. It is said that about 10% of patients in psychiatric inpatient
services have catatonic features.[7] On the one hand, the older school of
psychiatry associated schizophrenia with catatonia, while newer epidemiological
studies show that 20% of patients with catatonia have schizophrenia, and about
45% have symptoms of mood disorders and medical illness.

 7. Question 1 point(s)

A client tells the nurse that the television newscaster is sending a secret message to her.
The nurse suspects the client is experiencing:

 A. A delusion
B. Flight of ideas
C. Ideas of reference
D. Hallucination

Incorrect Correct Answer: C. Ideas of reference
Ideas of reference refers to the mistaken belief that neutral stimuli have special meaning
to the individual such as the television newscaster sending a message directly to the
individual. In people with bipolar disorder, mania and hypomania can comprise various
symptoms, from reckless spending to sexual promiscuity. In addition, some more subtle
symptoms may also occur, such as the belief held by some patients that everything
occurring around them is related somehow to them when in fact it isn’t. This symptom is
known as ideas of reference.
 Option A: A delusion is a false belief. Delusions are defined as fixed, false
beliefs that conflict with reality. Despite contrary evidence, a person in a
delusional state can’t let go of their convictions. Delusions are often reinforced by
the misinterpretation of events. Many delusions also involve some level of
paranoia. For example, someone might contend that the government is controlling
our every move via radio waves despite evidence to the contrary.
 Option B: Flight of ideas is a speech pattern in which the client skips from one
unrelated subject to another. A nearly continuous flow of accelerated speech with
abrupt changes from topic to topic that are usually based on understandable
associations, distracting stimuli, or plays on words. When severe, speech may be
disorganized and incoherent. It is part of the DSM -5 criteria for Manic episodes.
 Option D: A hallucination is a sensory perception, such as hearing voices and
seeing objects, that only the client experiences. Hallucinations involve sensing
things such as visions, sounds, or smells that seem real but are not. These things
are created by the mind. Common hallucinations can include feeling sensations in
the body, such as a crawling feeling on the skin or the movement of internal
organs; hearing sounds, such as music, footsteps, windows or doors banging;
hearing voices when no one has spoken (the most common type of hallucination).
These voices may be positive, negative, or neutral. They may command someone
to do something that may cause harm to themselves or others.

 8. Question 1 point(s)

The nurse knows that the physician has ordered the liquid form of the drug
chlorpromazine (Thorazine) rather than the tablet form because the liquid:

 A. Has a more predictable onset of action.


B. Produces fewer anticholinergic effects.
C. Produces fewer drug interactions.
D. Has a longer duration of action.

Incorrect Correct Answer: A. Has a more predictable onset of action.
A liquid phenothiazine preparation will produce effects in 2 to 4 hours. The onset of
tablets is unpredictable. If your medicine comes in a dropper bottle, measure each dose
with the special dropper provided with your prescription and dilute it in a small glass (4
ounces) of orange or grapefruit juice or water just before taking it. The dose medicines in
this class will be different for different patients. Follow your doctor’s orders or the
directions on the label. The following information includes only the average doses of
these medicines. If your dose is different, do not change it unless your doctor tells you to
do so.
 Option B: Before using any prescription or over-the-counter (OTC) medicine for
colds or allergies, check with your doctor. These medicines may increase the
chance of developing heatstroke or other unwanted effects, such as dizziness, dry
mouth, blurred vision, and constipation, while you are taking a phenothiazine.
 Option C: Certain medicines should not be used at or around the time of eating
food or eating certain types of food since interactions may occur. Using alcohol or
tobacco with certain medicines may also cause interactions to occur. Discuss with
your healthcare professional the use of your medicine with food, alcohol, or
tobacco.
 Option D: The amount of medicine that you take depends on the strength of the
medicine. Also, the number of doses you take each day, the time allowed between
doses, and the length of time you take the medicine depend on the medical
problem for which you are using the medicine.

 9. Question 1 point(s)

A client who has been hospitalized with disorganized type schizophrenia for 8 years can’t
complete activities of daily living (ADLs) without staff direction and assistance. The
nurse formulates a nursing diagnosis of Self-care deficit: Dressing/grooming related to
inability to function without assistance. What is an appropriate goal for this client?

 A. “Client will be able to complete ADLs independently within 1 month.”


B. “Client will be able to complete ADLs with only verbal encouragement within
1 month.”
C. “Client will be able to complete ADLs with assistance in organizing grooming
items and clothing within 1 month.”
D. “Client will be able to complete ADLs with complete assistance within 1
month.”

Incorrect Correct Answer: C. “Client will be able to complete ADLs with assistance
in organizing grooming items and clothing within 1 month.”
The client’s disorganized personality and history of hospitalization have affected the
ability to perform self-care activities. Disorganized schizophrenia is one of the five
subtypes of schizophrenia. It is characterized by disorganized behavior and speech and
includes disturbance in emotional expression. Hallucinations and delusions are less
pronounced with disorganized schizophrenia, though there is evidence of these symptoms
occurring.
 Option A: Interventions should be directed at helping the client complete ADLs
with the assistance of staff members, who can provide needed structure by
helping the client select grooming items and clothing. This goal promotes realistic
independence. Assistance with life skills can help individuals with disorganized
schizophrenia improve social interactions and increase daily living skills, with a
goal of increasing independence.
 Option B: As the client improves and achieves the established goal, the nurse can
set new goals that focus on the client completing ADLs with only verbal
encouragement and, ultimately, completing them independently. Individuals with
disorganized schizophrenia benefit from ongoing contact with the family. Support
to educate the family about treatment and how to support a family member with
schizophrenia is crucial. Family support increases family member’s understanding
of the disorder and helps family members develop coping strategies.
 Option D: The client’s condition doesn’t indicate a need for complete assistance,
which would only foster dependence. Assistance with daily living skills,
educational attainment, employment services, and family support plays a key role
in improving the course of the disease for individuals diagnosed with
disorganized schizophrenia.

 10. Question 1 point(s)

The nurse is planning care for a client admitted to the psychiatric unit with a diagnosis of
paranoid schizophrenia. Which nursing diagnosis should receive the highest priority?

 A. Risk for violence toward self or others


B. Imbalanced nutrition: Less than body requirements
C. Ineffective family coping
D. Impaired verbal communication

Incorrect Correct Answer: A. Risk for violence toward self or others
Because of such factors as suspiciousness, anxiety, and hallucinations, the client with
paranoid schizophrenia is at risk for violence toward himself or others. Paranoid
schizophrenia is characterized by predominantly positive symptoms of schizophrenia,
including delusions and hallucinations. These debilitating symptoms blur the line
between what is real and what isn’t, making it difficult for the person to lead a typical
life.
 Option B: The positive symptoms of schizophrenia—things like hallucinations
and delusions—are less likely to go unnoticed. After the prodromal phase, the
patient enters the active phase of schizophrenia, during which they experience
debilitating thoughts and perceptual distortions. They may experience impaired
motor or cognitive functions, including disorganized speech and disorganized or
catatonic behavior.
 Option C: Early symptoms of schizophrenia may seem rather ordinary and could
be explained by a number of other factors. This includes socializing less often
with friends, trouble sleeping, irritability, or a drop in grades. During the onset of
schizophrenia — otherwise known as the prodromal phase — negative symptoms
mount. These negative symptoms might include an increasing lack of motivation,
decreasing inability to pay attention or social isolation.
 Option D: The paranoia in paranoid schizophrenia stems from delusions—firmly
held beliefs that persist despite evidence to the contrary — and hallucinations —
seeing or hearing things that others do not. Both of these experiences can be
persecutory or threatening in nature. A patient may hear a voice or voices in their
head that they do not recognize as their own thoughts or internal voice. These
voices can be demeaning or hostile, driving a person to do things they would not
do otherwise.

 11. Question 1 point(s)

The nurse is preparing for the discharge of a client who has been hospitalized for
paranoid schizophrenia. The client’s husband expresses concern over whether his wife
will continue to take her daily prescribed medication. The nurse should inform him that:

 A. His concern is valid but his wife is an adult and has the right to make her own
decisions.
B. He can easily mix the medication in his wife’s food if she stops taking it.
C. His wife can be given a long-acting medication that is administered every 1 to
4 weeks.
D. His wife knows she must take her medication as prescribed to avoid future
hospitalizations.

Incorrect Correct Answer: C. His wife can be given a long-acting medication that is
administered every 1 to 4 weeks.
Long-acting psychotropic drugs can be administered by depot injection every 1 to 4
weeks. These agents are useful for noncompliant clients because the client receives the
injection at the outpatient clinic. When schizophrenia is diagnosed, antipsychotic
medication is most typically prescribed. This can be given as a pill, a patch, or an
injection. There are long-term injections that have been developed which could eliminate
the problems of a patient not regularly taking their medication (called “medication
noncompliance”).
 Option A: A client has the right to refuse medication, but this issue isn’t the focus
of discussion at this time. Someone with schizophrenia may not recognize that
their behavior, hallucinations, or delusions are unusual or unfounded. This can
cause a person to stop taking antipsychotic medication, stop participating in
therapy, or both, which can result in a relapse into active phase psychosis.
 Option B: Medication should never be hidden in food or drink to trick the client
into taking it; besides destroying the client’s trust, doing so would place the client
at risk for overmedication or under medication because the amount administered
is hard to determine. While antipsychotic medication is effective in treating the
positive symptoms of schizophrenia, it does not address negative symptoms.8 In
addition, these drugs can have unwanted side effects including weight gain,
drowsiness, restlessness, nausea, vomiting, low blood pressure, dry mouth, and
lowered white blood cell count.
 Option D: Assuming the client knows she must take the medication to avoid
future hospitalizations would be unrealistic. Psychotherapy also plays an
important role in the treatment of schizophrenia. Cognitive-behavioral therapy has
been shown to help patients develop and retain social skills, alleviate comorbid
anxiety and depression symptoms, cope with trauma in their past, improve
relationships with family and friends, and support occupational recovery.

 12. Question 1 point(s)

Benztropine (Cogentin) is used to treat the extrapyramidal effects induced by


antipsychotics. This drug exerts its effect by:

 A. Decreasing the anxiety causing muscle rigidity.


B. Blocking the cholinergic activity in the central nervous system (CNS).
C. Increasing the level of acetylcholine in the CNS.
D. Increasing norepinephrine in the CNS.

Incorrect Correct Answer: B. Blocking the cholinergic activity in the central
nervous system (CNS).
This is the action of Cogentin. Benztropine antagonizes acetylcholine and histamine
receptors. In the CNS and smooth muscles, benztropine exerts its action through
competing with acetylcholine at muscarinic receptors. Consequently, it reduces central
cholinergic effects by blocking muscarinic receptors that appear to improve the
symptoms of Parkinson disease. Thus, benztropine blocks the cholinergic muscarinic
receptor in the central nervous system. Therefore, it reduces the cholinergic effects
significantly during Parkinson disease which becomes more pronounced in the
nigrostriatal tract because of reduced dopamine concentrations.
 Option A: Anxiety doesn’t cause extrapyramidal effects. Benztropine belongs to
the synthetic class of muscarinic receptor antagonists (anticholinergic drug).
Thus, it has a structure similar to that of diphenhydramine and atropine. However,
it is long-acting so that its administration can be with less frequency than
diphenhydramine. It also induces less CNS stimulation effect compared to that of
trihexyphenidyl, making it a preferable drug of choice for geriatric patients.
 Option C: Overactivity of acetylcholine and lower levels of dopamine are the
causes of extrapyramidal effects. It is also useful for drug-induced extrapyramidal
symptoms and the prevention of dystonic reactions and acute treatment of
dystonic reactions. Furthermore, benztropine has further off-label use as it can
treat chronic sialorrhea occurring in developmentally-disabled patients. Also,
several clinical studies worked on using benztropine in managing intractable
hiccups.
 Option D: Benztropine doesn’t increase norepinephrine in the CNS. Benztropine
overdose can cause an anticholinergic toxidrome, which, in its role, may require
supportive care. Commonly, the risk assessment for benztropine overdose can
take place as soon as 6 hours after overdose ingestion, and toxicity effects may
last variably between 12 hours to 5 days at most. The most crucial step of proper
detection of benztropine overdose starts from carrying out intensive and inclusive
investigations. For example, ECG can be an essential assessment tool using 12
leads during testing. Also, monitoring the acetaminophen concentrations as well
as blood glucose concentrations can become a useful method for toxicity
investigations if the toxicant is unknown.

 13. Question 1 point(s)

A dopamine receptor agonist such as bromocriptine (Parlodel) relieves muscle rigidity


caused by antipsychotic medication by:

 A. Blocking dopamine receptors in the central nervous system (CNS)


B. Blocking acetylcholine in the CNS
C. Activating norepinephrine in the CNS
D. Activating dopamine receptors in the CNS

Correct Correct Answer: D. Activating dopamine receptors in the CNS
Extrapyramidal effects and the muscle rigidity induced by antipsychotic medications are
caused by a low level of dopamine. In groups of patients with Parkinson’s disease where
levodopa is no longer as effective, co-administration with dopamine agonists such as
bromocriptine historically was a successful option. Further, bromocriptine is also used as
an early treatment for PD to delay the onset of the use of levodopa, ultimately delaying
the likely dyskinesia and motor fluctuations that occur with chronic use.
 Option A: Dopamine receptor agonists stimulate dopamine receptors and thereby
reduce rigidity. Bromocriptine is a dopamine receptor agonist that has selective
agonist activity on D2 dopamine receptors while simultaneously acting as a
partial antagonist for D1 dopamine receptors.[12] Dopamine agonism has variable
effects depending on the target tissue. In Parkinson disease, bromocriptine binds
directly to striatal dopamine D2 receptors, stimulating locomotion and attenuating
the bradykinetic symptoms caused by the degeneration of dopaminergic
nigrostriatal neurons.
 Option B: Parkinson disease (PD) is a progressive neurological disorder
characterized by resting tremor, rigidity, akinesia or bradykinesia, and postural
instability due to the loss of dopaminergic neurons in the substantia nigra.
Although levodopa is an effective treatment of PD, with chronic use, there is a
decline in efficacy and motor complications.
 Option C: They don’t affect norepinephrine or acetylcholine. Bromocriptine is a
medication currently used in the management and treatment of Type II diabetes
mellitus. It is an ergot alkaloid derivative in the dopamine D2 agonist class of
drugs. This discussion reviews the indications, contraindications, and mechanism
of action for bromocriptine as a valuable agent in the management for Type II
diabetes mellitus, as well as its more traditional uses in Parkinson’s disease,
acromegaly, and pituitary prolactinomas.

 14. Question 1 point(s)

Most antipsychotic medications exert the following effects on the central nervous system
(CNS)?

 A. Stimulates the CNS by blocking postsynaptic dopamine, norepinephrine, and


serotonin receptors.
B. Sedate the CNS by stimulating serotonin at the synaptic cleft.
C. Depress the CNS by blocking the postsynaptic transmission of dopamine,
serotonin, and norepinephrine.
D. Depress the CNS by stimulating the release of acetylcholine.

Incorrect Correct Answer: C. Depress the CNS by blocking the postsynaptic
transmission of dopamine, serotonin, and norepinephrine.
The exact mechanism of antipsychotic medication action is unknown, but appears to
depress the CNS by blocking the transmission of three neurotransmitters: dopamine,
serotonin, and norepinephrine. The first-generation antipsychotics work by inhibiting
dopaminergic neurotransmission. Their effectiveness is best when they block about 72%
of the D2 dopamine receptors in the brain. They also have noradrenergic, cholinergic, and
histaminergic blocking action.
 Option A: Second-generation antipsychotics work by blocking D2 dopamine
receptors as well as serotonin receptor antagonist action. the 5-HT2A subtype of
serotonin receptor is most commonly involved. Second-generation antipsychotics
are serotonin-dopamine antagonists and are also known as atypical antipsychotics.
The Food and Drug Administration (FDA) has approved 12 atypical
antipsychotics as of the year 2016. They are risperidone, olanzapine, quetiapine,
ziprasidone, aripiprazole, paliperidone, asenapine, lurasidone, iloperidone,
cariprazine, brexpiprazole, and clozapine.
 Option B: First and second-generation antipsychotics (except clozapine) are
indicated for the treatment of an acute episode of psychoses as well as
maintenance therapy of schizophrenia and schizoaffective disorders. First-
generation antipsychotics are better for treating positive symptoms of
schizophrenia, e.g., hallucinations, delusions, among others. They also decrease
the risk of a repeat episode of psychosis. Second-generation antipsychotics treat
both positive symptoms and negative symptoms of schizophrenia, e.g.,
withdrawal, ambivalence, among others, and are known to reduce relapse rates.
 Option D: They don’t sedate the CNS by stimulating serotonin, and they don’t
stimulate neurotransmitter action or acetylcholine release. First-generation
antipsychotics are effective in the treatment of acute mania with psychotic
symptoms. All second-generation antipsychotics except clozapine can also be
used as a treatment of symptoms of acute mania. Antipsychotics are used with
mood stabilizers like lithium, valproic acid, or carbamazepine initially, and then
after symptoms stabilize can be gradually decreased and withdrawn.

 15. Question 1 point(s)

A client is admitted to the psychiatric unit of a local hospital with chronic


undifferentiated schizophrenia. During the next several days, the client is seen laughing,
yelling, and talking to herself. This behavior is characteristic of:

 A. Delusion
B. Looseness of association
C. Illusion
D. Hallucination

Correct Correct Answer: D. Hallucination
Auditory hallucination, in which one hears voices when no external stimuli exist, is
common in schizophrenic clients. Such behaviors as laughing, yelling, and talking to
oneself suggest such a hallucination. Auditory hallucinations are the sensory perceptions
of hearing voices without an external stimulus. This symptom is particularly associated
with schizophrenia and related psychotic disorders but is not specific to it. Auditory
hallucinations are one of the major symptoms of psychosis.
 Option A: Delusions, also common in schizophrenia, are false beliefs or ideas
that arise without external stimuli. Jaspers (1883-1969) was amongst the first to
describe and classify delusions. In his book General Psychopathology (1913), he
suggests that a delusion is a “perverted view of reality, incorrigibly held.” These
perversions are enigmatically derived, held with extraordinary certainty, and
absolutely unamenable. He further emphasized that these false beliefs exist along
a continuum of thought disturbance, increasing in severity of distortion from
normal thinking patterns to ‘true’ delusions. One hundred years later, Jaspers’
postulation remains a leading candidate in the investigation of delusion
morphology.
 Option B: Clients with schizophrenia may exhibit looseness of association, a
pattern of thinking and communicating in which ideas aren’t clearly linked to one
another. A thought disturbance demonstrated by speech that is disconnected and
fragmented, with the individual jumping from one idea to another unrelated or
indirectly related idea. It is essentially equivalent to derailment.
 Option C: Illusion is a less severe perceptual disturbance in which the client
misinterprets actual external stimuli. Illusions are rarely associated with
schizophrenia. Illusion, a misrepresentation of a “real” sensory stimulus—that is,
an interpretation that contradicts objective “reality” as defined by general
agreement. For example, a child who perceives tree branches at night as if they
are goblins may be said to be having an illusion.

 16. Question 1 point(s)

Which of the following medications would the nurse expect the physician to order to
reverse a dystonic reaction?
 A. prochlorperazine (Compazine)
B. diphenhydramine (Benadryl)
C. haloperidol (Haldol)
D. midazolam (Versed)

Incorrect Correct Answer: B. diphenhydramine (Benadryl)
Diphenhydramine, 25 to 50 mg I.M. or I.V., would quickly reverse this condition. An
acute dystonic reaction is characterized by involuntary contractions of muscles of the
extremities, face, neck, abdomen, pelvis, or larynx in either sustained or intermittent
patterns that lead to abnormal movements or postures. The symptoms may be reversible
or irreversible and can occur after taking any dopamine receptor-blocking agents.
Treatment of acute dystonic reaction centers around balancing the disrupted
dopaminergic-cholinergic balance in the basal ganglia and discontinuation of the
offending agent. The most commonly available drugs in the emergency setting for the
treatment of acute dystonic reaction are diphenhydramine and benztropine.
 Option A: Prochlorperazine can be used to treat both acute psychotic episodes
and chronic mental illnesses. As a first-generation antipsychotic, the drug is better
at treating positive symptoms than negative ones, including delusions,
hallucinations, agitation, and disorganized speech and behavior.
 Option C: Haloperidol is capable of causing dystonia, not reversing it. Due to the
blockade of the dopamine pathway in the brain, typical antipsychotic medications
such as haloperidol have correlations with extrapyramidal side effects. The
extrapyramidal symptoms are muscular weakness or rigidity, a generalized or
localized tremor that may be characterized by the akinetic or agitation types of
movements, respectively. Haloperidol overdose is also associated with ECG
changes known as torsade de pointes, which may cause arrhythmia or cardiac
arrest.
 Option D: Midazolam would make this client drowsy. Midazolam can be used
for anxiolysis and hypnosis during the maintenance phase of general anesthesia
and is also superior to thiopental in the maintenance of anesthesia because of the
less need for adjunct medications. Midazolam is used as an adjunct medication to
regional and local anesthesia for a wide range of diagnostic and therapeutic
procedures and has greater patient and physician acceptance.

 17. Question 1 point(s)

A schizophrenic client states, “I hear the voice of King Tut.” Which response by the
nurse would be most therapeutic?

 A. “I don’t hear the voice, but I know you hear what sounds like a voice.”
B. “You shouldn’t focus on that voice.”
C. “Don’t worry about the voice as long as it doesn’t belong to anyone real.”
D. “King Tut has been dead for years.”

Incorrect Correct Answer: A. “I don’t hear the voice, but I know you hear what
sounds like a voice.”
This response states reality about the client’s hallucination. Voicing doubt can be a
gentler way to call attention to the incorrect or delusional ideas and perceptions of
patients. By expressing doubt, nurses can force patients to examine their assumptions.
 Option B: Sometimes during a conversation, patients mention something
particularly important. When this happens, nurses can focus on their statement,
prompting patients to discuss it further. Patients don’t always have an objective
perspective on what is relevant to their case; as impartial observers, nurses can
more easily pick out the topics to focus on.
 Option C: For patients experiencing sensory issues or hallucinations, it can be
helpful to ask about them in an encouraging, non-judgmental way. Phrases like
“What do you hear now?” or “What does that look like to you?” give patients a
prompt to explain what they’re perceiving without casting their perceptions in a
negative light.
 Option D: The other options are judgmental, flippant, or dismissive. Similar to
active listening, asking patients for clarification when they say something
confusing or ambiguous is important. Saying something like “I’m not sure I
understand. Can you explain it to me?” helps nurses ensure they understand
what’s actually being said and can help patients process their ideas more
thoroughly.

 18. Question 1 point(s)

A client has been receiving chlorpromazine (Thorazine), an antipsychotic, to treat his


psychosis. Which findings should alert the nurse that the client is experiencing
pseudoparkinsonism?

 A. Restlessness, difficulty sitting still, and pacing


B. Involuntary rolling of the eyes
C. Tremors, shuffling gait, and masklike face
D. Extremity and neck spasms, facial grimacing, and jerky movements

Incorrect Correct Answer: C. Tremors, shuffling gait, and mask-like face
Pseudoparkinsonism may appear 1 to 5 days after starting an antipsychotic and may also
include drooling, rigidity, and “pill-rolling.” Despite being a low-potency drug,
chlorpromazine can still cause extrapyramidal side effects (EPS) such as acute dystonia,
akathisia, parkinsonism, and tardive dyskinesia (TD). The evolution of EPS side effects
can occur through hours to days. Acute dystonia refers to muscle stiffness or spasm of the
head, neck, and eye muscles that can start hours after starting the medication. Akathisia
includes restlessness and fast pacing. Parkinsonism includes bradykinesia, “cogwheel”
rigidity, and shuffling gait.
 Option A: Akathisia may occur several weeks after starting antipsychotic therapy
and consists of restlessness, difficulty sitting still, and fidgeting. Patients are at
risk of developing neuroleptic malignant syndrome (NMS), which is a life-
threatening manifestation, where the patient presents with “lead-pipe” muscle
rigidity, autonomous instability, hyperpyrexia more than 40 degrees Celsius,
altered mental status, leukocytosis, and elevated serum creatinine kinase.
 Option B: An oculogyric crisis is recognized by uncontrollable rolling back of
the eyes and, along with dystonia, should be considered an emergency.
Chlorpromazine use also requires caution in patients with cerebrovascular and
cardiovascular diseases. Patients should start on a low dose of chlorpromazine as
an initial dosage, and the increase in subsequent dosing should be gradual.
However, treatment should be discontinued if the patient develops
agranulocytosis.
 Option D: Dystonia may occur minutes to hours after receiving an antipsychotic
and may include extremity and neck spasms, jerky muscle movements, and facial
grimacing. Chlorpromazine belongs to the category of typical antipsychotics or
neuroleptics, also known as first-generation antipsychotics (FGAs). It produces its
antipsychotic effect by the post-synaptic blockade at the D2 receptors in the
mesolimbic pathway. However, the blockade of D2 receptors in the nigrostriatal
pathway is responsible for its extrapyramidal side effects.

 19. Question 1 point(s)

For several years, a client with chronic schizophrenia has received 10 mg of fluphenazine
hydrochloride (Prolixin) by mouth four times per day. Now the client has a temperature
of 102° F (38.9° C), a heart rate of 120 beats/minute, a respiratory rate of 20
breaths/minute, and a blood pressure of 210/140 mm Hg. Because the client also is
confused and incontinent, the nurse suspects malignant neuroleptic syndrome. What steps
should the nurse take?

 A. Give the next dose of fluphenazine, call the physician, and monitor vital signs.
B. Withhold the next dose of fluphenazine, call the physician, and monitor vital
signs.
C. Give the next dose of fluphenazine and restrict the client to the room to
decrease stimulation.
D. Withhold the next dose of fluphenazine, administer an antipyretic agent, and
increase the client’s fluid intake.

Incorrect Correct Answer: B. Withhold the next dose of fluphenazine, call the
physician, and monitor vital signs.
Malignant neuroleptic syndrome is a dangerous adverse effect of neuroleptic drugs such
as fluphenazine. Although an antipyretic agent may be used to reduce fever, increased
fluid intake is contraindicated because it may increase the client’s fluid volume further,
raising blood pressure even higher. Rare but serious side effects include neuroleptic
malignant syndrome, liver function abnormalities and jaundice, seizures, and
agranulocytosis. Like other antipsychotic medications, fluphenazine carries a black-box
warning for increased risk of cerebrovascular events and death in elderly patients with
psychosis related to major neurocognitive disorder. There are reports of allergic reactions
with the use of fluphenazine and other phenothiazine typical antipsychotics.
 Option A: Patients taking any antipsychotic medication require close monitoring
for the appearance of side effects. Baseline electrocardiograms should be obtained
in all patients with preexisting cardiac conduction abnormalities starting
fluphenazine; close monitoring for electrocardiogram changes is necessary.
Assessments for the appearance of extrapyramidal side effects should be routine
for patients taking fluphenazine; the Abnormal Involuntary Movement Scale is a
well-studied, easy-to-administer assessment for the emergence of extrapyramidal
effects.
 Option C: Clinicians should routinely obtain complete blood counts and
metabolic panels to monitor for changes in white blood cell counts, liver
transaminases, and blood urea nitrogen and creatinine levels. Patients taking
multiple medications also require monitoring for interactions with fluphenazine;
this medication is a major substrate of the CYP-2D6 system and is a weak
inhibitor of CYP-2C9 and CYP-2E1 systems. Although serum levels of
fluphenazine are not a routine part of therapy and dosing typically depends on
clinical response, the therapeutic reference range is 1 nanogram to 10 nanograms
per milliliter.
 Option D: Fluphenazine has an adverse effect profile similar to other first-
generation or typical antipsychotics, which is due to its dopamine receptor
antagonism as well as its anticholinergic, antihistaminic, and alpha-adrenergic
antagonistic properties. Common side effects include sedation, dry mouth,
constipation, dry eyes, blurred vision, constipation, orthostasis, dizziness,
hypotension, and urinary retention. Other possible side effects include rebound
tachycardia, urinary retention, and weight gain. Due to dopaminergic antagonism,
fluphenazine can cause extrapyramidal symptoms, including akathisia,
parkinsonian features such as resting tremor and shuffling gait, acute dystonic
reactions, oculogyric crises, opisthotonos, and tardive dyskinesia.

 20. Question 1 point(s)

A schizophrenic client with delusions tells the nurse, “There is a man wearing a red coat
who’s out to get me.” The client exhibits increasing anxiety when focusing on the
delusions. Which of the following would be the best response?

 A. “This subject seems to be troubling you. Let’s walk to the activity room.”
B. “Describe the man who’s out to get you. What does he look like?”
C. “There is no reason to be afraid of that man. This hospital is very secure.”
D. “There is no need to be concerned with a man who isn’t even real.”

Incorrect Correct Answer: A. “This subject seems to be troubling you. Let’s walk to
the activity room.”
This remark distracts the client from the delusion by engaging the client in a less
threatening or more comforting activity at the first sign of anxiety. The nurse should
reinforce reality and discourage the false belief. As a nursing diagnosis in the presence of
delusions, the Nursing Interventions Classification (NIC) defines Delusion Control
(6440), which is defined as the provision of a safe and therapeutic environment to the
patient in acute state of confusion.2 Thus, in the presence of delusions, the nurses must
demonstrate to patients that they accept that the patient has this belief, although they do
not share the belief.
 Option B: It is important not to discuss or deny belief so as not to risk
compromising trust. Reasonable doubt must therefore be used as a therapeutic
technique. For example, “I understand that you believe this to be true, but I do not
think the same.”
 Option C: One should also reinforce reality and talk about things and people that
are real, avoiding ruminant thinking in false beliefs. The nurse should also be
attentive during feeding and taking medication, since the delirium of poisoning
may be present and the patient may believe that the food or medication is to
poison him. Thus, it may be necessary to confirm whether the patient has taken
the medication.
 Option D: The other options focus on the content of the delusion rather than the
meaning, feeling, or intent that it provokes. Establishing a therapeutic relationship
is not easy, requiring special attention on the part of the nurse. Active listening
and empathy are especially important and should focus on the patient’s
experiences. Authenticity is necessary, allowing the person to distinguish between
what is part of the disease and what is not part of it, i.e., what is real and what is
not. It is crucial to help the patient find their personal resources and identify
achievable goals in the medium and long term and the means to achieve them.

 21. Question 1 point(s)

Important teaching for women in their childbearing years who are receiving antipsychotic
medications includes which of the following?

 A. Occurrence of increased libido due to medication adverse effects.


B. Increased incidence of dysmenorrhea while taking the drug.
C. Continuing previous use of contraception during periods of amenorrhea.
D. Instruction that amenorrhea is irreversible.

Incorrect Correct Answer: C. Continuing previous use of contraception during
periods of amenorrhea
Women may experience amenorrhea, which is reversible, while taking antipsychotics.
Amenorrhea doesn’t indicate cessation of ovulation; therefore, the client can still become
pregnant. The client should be instructed to continue contraceptive use even when
experiencing amenorrhea.
 Option A: Libido generally decreases because of the depressant effect. Although
women with serious mental illness have high rates of lifetime sexual partners,
they infrequently use contraception. Consequently, the prevalence of sexually
transmitted infections is high in this population. In addition, while the overall rate
of pregnancy in women with schizophrenia of child-bearing age is lower than in
the general population, the percentage of pregnancies that are unwanted is higher
than that in the general population.
 Option B: Dysmenorrhea isn’t an adverse effect of antipsychotics. Contraceptive
counseling to women and their partners is an important part of comprehensive
care for women with serious and persistent mental illness. Women with
schizophrenia who smoke, are overweight, or have diabetes, migraine,
cardiovascular disease, or a family history of breast cancer should be offered non-
hormonal contraception. Women with more than one sexual partner should be
advised on barrier methods in addition to any other contraceptive measures they
are using.
 Option D: Amenorrhea is reversible, so the woman could still become pregnant.
Contraceptive counseling to women and their partners is an important part of
comprehensive care for women with serious and persistent mental illness. Women
with schizophrenia who smoke, are overweight, or have diabetes, migraine,
cardiovascular disease, or a family history of breast cancer should be offered non-
hormonal contraception. Women with more than one sexual partner should be
advised on barrier methods in addition to any other contraceptive measures they
are using.

 22. Question 1 point(s)

A client is admitted to a psychiatric facility with a diagnosis of chronic schizophrenia.


The history indicates that the client has been taking neuroleptic medication for many
years. Assessment reveals unusual movements of the tongue, neck, and arms. Which
condition should the nurse suspect?

 A. Tardive dyskinesia
B. Dystonia
C. Neuroleptic malignant syndrome
D. Akathisia

Incorrect Correct Answer: A. Tardive dyskinesia
Unusual movements of the tongue, neck, and arms suggest tardive dyskinesia, an adverse
reaction to neuroleptic medication. Tardive dyskinesia (TD) is a syndrome which
includes a group of iatrogenic movement disorders caused due to a blockade of dopamine
receptors. The movement disorders include akathisia, dystonia, buccolingual stereotypy,
myoclonus, chorea, tics and other abnormal involuntary movements which are commonly
caused by the long-term use of typical antipsychotics. TD is most common in
schizophrenics and bipolar patients treated with antipsychotic medications, but they can
occur in any patients. In addition, these medications can also induce parkinsonian
syndromes. Research reveals that there is a dysfunction of the dopamine transporter that
leads to TD.
 Option B: Dystonia is characterized by cramps and rigidity of the tongue, face,
neck, and back muscles. Dystonia is defined by involuntary maintained
contraction of agonist and antagonist muscles yielding abnormal posturing,
twisting and repetitive movements, or tremulous and can be initiated or worsened
by attempted movement. Dystonia is a dynamic disorder that changes in severity
based on the activity and posture. Dystonia may assume a pattern of
overextension or over-flexion of the hand, inversion of the foot, lateral flexion or
retroflection of the head, torsion of the spine with arching and twisting of the
back, forceful closure of the eyes, or a fixed grimace. It may come to an end when
the body is in action and during sleep.
 Option C: Neuroleptic malignant syndrome causes rigidity, fever, hypertension,
and diaphoresis. Neuroleptic malignant syndrome (NMS) is a life-threatening
syndrome associated with the use of dopamine-receptor antagonist medications or
with rapid withdrawal of dopaminergic medications. NMS has been associated
with virtually every neuroleptic agent but is more commonly reported with the
typical antipsychotics like haloperidol and fluphenazine. Classic clinical
characteristics include mental status changes, fever, muscle rigidity, and
autonomic instability.
 Option D: Akathisia causes restlessness, anxiety, and jitteriness. Akathisia is
defined as an inability to remain still. It is a neuropsychiatric syndrome that is
associated with psychomotor restlessness. The individual with akathisia will
generally experience an intense sensation of unease or an inner restlessness that
usually involves the lower extremities. This results in a compulsion to move. In
most cases the movement is repetitive. The individual may cross, uncross, swing,
or shift from one foot to the other. To the observer, this may appear as a persistent
fidget.

 23. Question 1 point(s)

What medication would probably be ordered for the acutely aggressive schizophrenic
client?

 A. chlorpromazine (Thorazine)
B. haloperidol (Haldol)
C. lithium carbonate (Lithonate)
D. amitriptyline (Elavil)

Incorrect Correct Answer: B. haloperidol (Haldol)
Haloperidol administered I.M. or I.V. is the drug of choice for acute aggressive psychotic
behavior. Haloperidol is a first-generation (typical) antipsychotic medication that is used
widely around the world. Food and Drug Administration (FDA) approved the use of
haloperidol is for schizophrenia, Tourette syndrome (control of tics and vocal utterances
in adults and children), hyperactivity (which may present as impulsivity, difficulty
maintaining attention, severe aggressivity, mood instability, and frustration intolerance),
severe childhood behavioral problems (such as combative, explosive hyperexcitability),
intractable hiccups. It is a typical antipsychotic because it works on positive symptoms of
schizophrenia, such as hallucinations and delusions.
 Option A: Chlorpromazine is also an antipsychotic drug; however, it causes more
pronounced sedation than haloperidol. Chlorpromazine belongs to the category of
typical antipsychotics or neuroleptics, also known as first-generation
antipsychotics (FGAs). It produces its antipsychotic effect by the post-synaptic
blockade at the D2 receptors in the mesolimbic pathway. However, the blockade
of D2 receptors in the nigrostriatal pathway is responsible for its extrapyramidal
side effects.
 Option C: Lithium was the first mood stabilizer and is still the first-line treatment
option, but is underutilized because it is an older drug. Lithium is a commonly
prescribed drug for a manic episode in bipolar disorder as well as maintenance
therapy of bipolar disorder in a patient with a history of a manic episode. The
primary target symptoms of lithium are mania and unstable mood.
 Option D: Amitriptyline is used for depression. Amitriptyline is in the tricyclic
antidepressant (TCA) drug classification and acts by blocking the reuptake of
both serotonin and norepinephrine neurotransmitters. The three-ring central
structure, along with a side chain, is the basic structure of tricyclic
antidepressants. Amitriptyline is a tertiary amine and has strong binding affinities
for alpha-adrenergic, histamine (H1), and muscarinic (M1) receptors. It is more
sedating and has increased anticholinergic properties compared to other TCAs.

 24. Question 1 point(s)

A client is admitted with a diagnosis of schizotypal personality disorder. Which signs


would this client exhibit during social situations?

 A. Aggressive behavior
B. Paranoid thoughts
C. Emotional affect
D. Independence needs

Incorrect Correct Answer: B. Paranoid thoughts
Clients with schizotypal personality disorder experience excessive social anxiety that can
lead to paranoid thoughts. Isolation is a salient feature in the history of a schizoid patient.
Rarely do they have close relationships, and often they will choose to participate in
occupations that are solitary in nature. They infrequently experience strong emotion,
express little to no desire for sexual activity with a partner, and tend to be ambivalent to
criticism or praise.
 Option A: Aggressive behavior is uncommon, although these clients may
experience agitation with anxiety. Schizotypal can be differentiated with its more
pronounced “magical” and eccentric thought processes. Paranoid, avoidant, and
obsessive-compulsive personality disorders are also often on the clinician’s list of
differential diagnoses. Unlike the aloofness observed in schizoid, however,
patients with paranoid personality disorder are often overly resentful and can
demonstrate explosive anger.
 Option C: Their behavior is emotionally cold with a flattened affect, regardless
of the situation. Individuals afflicted with personality disorders tend to externalize
their problems, viewing others as the etiology of any conflict. If, by chance, a
person with schizoid personality disorder presents in the clinical setting, DSM V
has outlined specific diagnostic criteria for the clinician to use for evaluation. A
pronounced blunted affect will immediately be observable on presentation. The
patient will be disengaged, aloof, and will most likely diminish symptomatology.
 Option D: These clients demonstrate a reduced capacity for close or dependent
relationships. It is unlikely that a person with a schizoid personality disorder will
present in the clinical setting of his own volition unless prompted by family, or as
a result of a co-occurring disorder, such as depression. As with most personality
disorders, the behavior is in synchrony with the ego, and thus the patient does not
acknowledge the need to adapt his or her behavior.

 25. Question 1 point(s)

During the initial interview, a client with schizophrenia suddenly turns to the empty chair
beside him and whispers, “Now just leave. I told you to stay home. There isn’t enough
work here for both of us!” What is the nurse’s best initial response?

 A. “When people are under stress, they may see things or hear things that others
don’t. Is that what just happened?”
B. “I’m having a difficult time hearing you. Please look at me when you talk.”
C. “There is no one else in the room. What are you doing?”
D. “Who are you talking to? Are you hallucinating?”

Incorrect Correct Answer: A. “When people are under stress, they may see things
or hear things that others don’t. Is that what just happened?”
This response makes the client feel that experiencing hallucinations is acceptable and
promotes an open, therapeutic relationship. Authenticity, empathy, understanding of
illness and the person, non-stigmatization, and the ability to work as a team are essential
characteristics that the nurse must have for the therapeutic relationship to be effective. It
is crucial to work with insights into the disease, the importance of adherence, and the
reduction of self-stigma. Establishing a therapeutic relationship with the person with
schizophrenia is therefore a constant challenge that must accompany the various stages of
the disease in cooperation with the family and the community.
 Option B: Directing the client to look at the nurse wouldn’t address the obvious
issue of the hallucination. As for the presence of hallucinations, the NIC defines
the nursing diagnosis of Hallucination Control(6510) as the promotion of safety,
comfort, and the hallucinating patient’s orientation towards reality.2 Auditory
hallucinations are the most frequent, so the nurse must observe certain signs, such
as taking a listening posture, unmotivated laughter, talking to oneself, and blocks
in thinking, lack of attention, and distraction. In the presence of these signs, the
nurse should avoid touching the patient without warning, as the touch may be
understood as a threat.
 Option C: It is important that the patient understands that the voices are unreal
and are part of the disease, and distraction techniques can be used to direct the
patient towards reality. Listening to music or watching television may be a good
technique to distract the patient from the attention given to auditory
hallucinations. These interventions are intended to establish a relationship of
empathy and trust with the patient, causing the patient to begin to be critical
towards the disease so that new intervention strategies can be implemented.
 Option D: Confrontational approaches are likely to elicit an uninformative or
negative response. Nurses must display an attitude of acceptance to help the
patient share the content of the hallucination. This sharing is important to avoid
unwanted reactions towards the self or others, if command hallucinations are
present. The hallucination should not be reinforced, and the word “voices” should
be used to refer to it, avoiding the word “they” which may indicate validation. It
is also essential to make the patient realize that the nurse does not share the
perception by saying, “I know the voices for you are real, but I do not hear any
voices.”

 26. Question 1 point(s)

The definition of nihilistic delusions is:

 A. A false belief about the functioning of the body.


B. Belief that the body is deformed or defective in a specific way.
C. False ideas about the self, others, or the world
D. The inability to carry out motor activities.

Incorrect Correct Answer: C. False ideas about the self, others, or the world.
Nihilistic delusions are false ideas about the self, others, or the world. Nihilistic
delusions, also known as délires de négation, are specific psychopathological entities
characterized by the delusional belief of being dead, decomposed or annihilated, having
lost one’s own internal organs or even not existing entirely as a human being.
 Option A: Somatic delusions involve a false belief about the functioning of the
body. Of the delusional symptoms, somatic delusions-those that pertain to the
body-are rather rare. Somatic delusions are defined as fixed false beliefs that
one’s bodily function or appearance is grossly abnormal. They are a poorly
understood psychiatric symptom and pose a significant clinical challenge to
clinicians.
 Option B: Body dysmorphic disorder is characterized by a belief that the body is
deformed or defective in a specific way. People who have body dysmorphic
disorder (BDD) think about their real or perceived flaws for hours each day. They
can’t control their negative thoughts and don’t believe people who tell them that
they look fine. Their thoughts may cause severe emotional distress and interfere
with their daily functioning. They may miss work or school, avoid social
situations and isolate themselves, even from family and friends, because they fear
others will notice their flaws.
 Option D: Apraxia is the inability to carry out motor activities. Apraxia is a
motor disorder caused by damage to the brain (specifically the posterior parietal
cortex or corpus callosum) in which the individual has difficulty with the motor
planning to perform tasks or movements when asked, provided that the request or
command is understood and the individual is willing to perform the task. The
nature of the brain damage determines the severity, and the absence of sensory
loss or paralysis helps to explain the level of difficulty.

 27. Question 1 point(s)

A client who’s taking antipsychotic medication develops a very high temperature, severe
muscle rigidity, tachycardia, and rapid deterioration in mental status. The nurse suspects
what complication of antipsychotic therapy?

 A. Agranulocytosis
B. Extrapyramidal effects
C. Anticholinergic effects
D. Neuroleptic malignant syndrome (NMS)

Correct Correct Answer: D. Neuroleptic malignant syndrome (NMS)
A rare but potentially fatal condition of antipsychotic medication is called NMS. It
generally starts with an elevated temperature and severe extrapyramidal effects.
Neuroleptic malignant syndrome (NMS) is a life-threatening syndrome associated with
the use of dopamine-receptor antagonist medications or with rapid withdrawal of
dopaminergic medications. NMS has been associated with virtually every neuroleptic
agent but is more commonly reported with the typical antipsychotics like haloperidol and
fluphenazine. Classic clinical characteristics include mental status changes, fever, muscle
rigidity, and autonomic instability.
 Option A: Agranulocytosis is a blood disorder. Agranulocytosis is a condition in
which the absolute neutrophil count (ANC) is less than 100 neutrophils per
microlitre of blood. People with this condition are at a very high risk of severe
infection. Broadly, it can be due to hereditary disease due to genetic mutation or
acquired disease. Agranulocytosis can have various presentations, including fever,
chills, sore throat, etc. It can be a life-threatening condition that requires prompt
diagnosis and treatment.
 Option B: Symptoms of extrapyramidal effects include tremors, restlessness,
muscle spasms, and pseudoparkinsonism. Extrapyramidal side effects (EPS),
commonly referred to as drug-induced movement disorders are among the most
common adverse drug effects patients experience from dopamine-receptor
blocking agents. It was first described in 1952 after chlorpromazine-induced
symptoms resembling Parkinson disease. A variety of movement phenotypes has
since been described along the EPS spectrum, including dystonia, akathisia, and
parkinsonism, which occur more acutely, as well as more chronic manifestations
of tardive akathisia and tardive dyskinesia.
 Option C: Anticholinergic effects include blurred vision, drowsiness, and dry
mouth. Anticholinergics are drugs that block the action of acetylcholine.
Acetylcholine is a neurotransmitter, or a chemical messenger. It transfers signals
between certain cells to affect how your body functions. They also help block
involuntary muscle movements associated with certain diseases such as
Parkinson’s disease. Sometimes, they’re used before surgery to help maintain
body functions while a person is treated with anesthesia.

 28. Question 1 point(s)

The nurse formulates a nursing diagnosis of Impaired social interaction related to


disorganized thinking for a client with schizotypal personality disorder. Based on this
nursing diagnosis, which nursing intervention takes the highest priority?

 A. Helping the client to participate in social interactions.


B. Establishing a one-on-one relationship with the client.
C. Exploring the effects of the client’s behavior on social interactions.
D. Developing a schedule for the client’s participation in social interactions.

Incorrect Correct Answer: B. Establishing a one-on-one relationship with the client
By establishing a one-on-one relationship, the nurse helps the client learn how to interact
with people in new situations. Regardless of the clinical setting, the nurse must provide
structure and limit setting in the therapeutic relationship; in a clinic setting, this may
mean seeing the client for scheduled appointments of a predetermined length rather than
whenever the client appears and demands the nurse’s immediate attention.
 Option A: It is important to teach basic communication skills such as eye contact,
active listening, taking turns talking, validating the meaning of another’s
communication, and using “I” statements. The nurse must be quite clear about
establishing the boundaries of the therapeutic relationship to ensure that neither
the client’s nor the nurse’s boundaries are violated.
 Option C: Cognitive restructuring is a technique useful in changing patterns of
thinking by helping clients to recognize negative thoughts and feelings and to
replace them with positive patterns of thinking; thought stopping is a technique to
alter the process of negative or self-critical thought patterns.
 Option D: The other options are appropriate but should take place only after the
nurse-client relationship is established. Minimizing unstructured time by planning
activities can help clients to manage time alone; clients can make a written
schedule that includes appointments, shopping, reading the paper, and going for a
walk. The nurse can help the clients to identify their feelings and learn to tolerate
them without exaggerated responses such as destruction of property or self-harm;
keeping a journal often helps clients gain awareness of feelings.

 29. Question 1 point(s)

A client with schizophrenia hears a voice telling him he is evil and must die. The nurse
understands that the client is experiencing:

 A. A delusion
B. Flight of ideas
C. Ideas of reference
D. Hallucination

Correct Correct Answer: D. Hallucination
A hallucination is a sensory perception, such as hearing voices and seeing objects, that
only the client experiences. The word “hallucination” comes from Latin and means “to
wander mentally.” Hallucinations are defined as the “perception of a nonexistent object
or event” and “sensory experiences that are not caused by stimulation of the relevant
sensory organs.” Hallucinations occur frequently in people with psychiatric conditions,
including schizophrenia and bipolar disorder, however, you don’t necessarily need to
have a mental illness to experience hallucinations.
 Option A: A delusion is a false belief. Delusions are defined as fixed, false
beliefs that conflict with reality. Despite contrary evidence, a person in a
delusional state can’t let go of their convictions. Delusions are often reinforced by
the misinterpretation of events. Many delusions also involve some level of
paranoia. For example, someone might contend that the government is controlling
our every move via radio waves despite evidence to the contrary. Delusions are
often part of psychotic disorders. They may occur along with hallucinations,
which involve perceiving something that isn’t really there, like hearing voices or
feeling bugs crawling on your skin.
 Option B: Flight of ideas refers to a speech pattern in which the client skips from
one unrelated subject to another. A nearly continuous flow of accelerated speech
with abrupt changes from topic to topic that are usually based on understandable
associations, distracting stimuli, or plays on words. When severe, speech may be
disorganized and incoherent. It is part of the DSM -5 criteria for Manic episodes.
 Option C: Ideas of reference refers to the mistaken belief that someone or
something outside the client is controlling the client’s ideas or behavior. In people
with bipolar disorder, mania and hypomania can comprise various symptoms,
from reckless spending to sexual promiscuity. In addition, some more subtle
symptoms may also occur, such as the belief held by some patients that
everything occurring around them is related somehow to them when in fact it
isn’t. This symptom is known as ideas of reference. An extension of those
irrational beliefs, delusions of reference, can cause patients to change their
behavior significantly because of this mistaken belief.

 30. Question 1 point(s)

A client with delusional thinking shows a lack of interest in eating at meal times. She
states that she is unworthy of eating and that her children will die if she eats. Which
nursing action would be most appropriate for this client?

 A. Telling the client that she may become sick and die unless she eats.
B. Paying special attention to the client’s rituals and emotions associated with
meals.
C. Restricting the client’s access to food except at specified meal and snack times.

D. Encouraging the client to express her feelings at meal times.



Incorrect Correct Answer: C. Restricting the client’s access to food except at
specified meal and snack times
Restricting access to food except at specified times prevents the client from eating when
she feels anxious, guilty, or depressed; this, in turn, decreases the association between
these emotions and food. Be consistent in setting expectations, enforcing rules, and so
forth. Clear, consistent limits provide a secure structure for the patient.
 Option A: Telling the client she may become sick or die may reinforce her
behavior because illness or death may be her goal. Present reality concisely and
briefly and do not challenge illogical thinking. Avoid vague or evasive remarks.
Delusional patients are extremely sensitive about others and can recognize
insincerity. Evasive comments or hesitation reinforces mistrust or delusions.
 Option B: Paying special attention to rituals and emotions associated with meals
also would reinforce undesirable behavior. Reduce provocative stimuli, negative
criticism, arguments, and confrontations. This is to avoid triggering fight/flight
responses. Identify specific conflicts that remain unresolved, and assist the patient
to identify possible solutions. Unless these underlying conflicts are resolved, any
improvement in coping behaviors must be viewed as only temporary.
 Option D: Encouraging the client to express feelings at mealtimes would increase
the association between emotions and food; instead, the nurse should encourage
her to express feelings at other times. Encourage the patient to verbalize true
feelings. Avoid becoming defensive when angry feelings are directed at him or
her. Verbalization of feelings in a non-threatening environment may help the
patient come to terms with long-unresolved issues.

 31. Question 1 point(s)

Which of the following groups of characteristics would the nurse expect to see in the
client with schizophrenia?

 A. Loose associations, grandiose delusions, and auditory hallucinations


B. Periods of hyperactivity and irritability alternating with depression
C. Delusions of jealousy and persecution, paranoia, and mistrust
D. Sadness, apathy, feelings of worthlessness, anorexia, and weight loss

Incorrect Correct Answer: A. Loose associations, grandiose delusions, and auditory
hallucinations
Loose associations, grandiose delusions, and auditory hallucinations are all characteristic
of the classic schizophrenic client. These clients aren’t able to care for their physical
appearance. They frequently hear voices telling them to do something either to
themselves or to others. Additionally, they verbally ramble from one topic to the next. In
the Diagnostic and Statistical Manual of Mental Disorders 5 (DSM-5), Two or more of
the following symptoms must be present for a significant portion of time during a one-
month period: delusions, hallucinations, disorganized speech, grossly disorganized or
catatonic behavior, or negative symptoms.
 Option B: Periods of hyperactivity and irritability alternating with depression are
characteristic of bipolar or manic disease. The defining characteristics of mania
are increased talkativeness, rapid speech, decreased the need for sleep (unlike
depression or anxiety in which the need for sleep exists, but there is an inability to
sleep), racing thoughts, distractibility, increase in goal-directed activity, and
psychomotor agitation. Some other hallmarks of mania are an elevated or
expansive mood, mood lability, impulsivity, irritability, and grandiosity.
 Option C: Delusions of jealousy and persecution, paranoia, and mistrust are
characteristics of paranoid disorders. Often, these patients think that others have
greatly and irreversibly injured them. They are hypervigilant for potential insults,
slights, threats, and disloyalty and look for hidden meanings in remarks and
actions. They closely scrutinize others for evidence to support their suspicions.
For example, they may misinterpret an offer of help as implication that they are
unable to do the task on their own. If they think that they have been insulted or
injured in any way, they do not forgive the person who injured them. They tend to
counterattack or to become angry in response to these perceived injuries. Because
they distrust others, they feel a need to be autonomous and in control.
 Option D: Sadness, apathy, feelings of worthlessness, anorexia, and weight loss
are characteristics of depression. Depression is a mood disorder that causes a
persistent feeling of sadness and loss of interest. The common features of all the
depressive disorders are sadness, emptiness, or irritable mood, accompanied by
somatic and cognitive changes that significantly affect the individual’s capacity to
function.

 32. Question 1 point(s)

The nurse must administer a medication to reverse or prevent Parkinson-type symptoms


in a client receiving an antipsychotic. The medication the client will likely receive is:

 A. benztropine (Cogentin)
B. diphenhydramine (Benadryl)
C. propranolol (Inderal)
D. haloperidol (Haldol)

Incorrect Correct Answer: A. benztropine (Cogentin)
Benztropine, trihexyphenidyl, or amantadine are prescribed for a client with Parkinson-
type symptoms. It is also useful for drug-induced extrapyramidal symptoms and the
prevention of dystonic reactions and acute treatment of dystonic reactions. Furthermore,
benztropine has further off-label use as it can treat chronic sialorrhea occurring in
developmentally-disabled patients. Benztropine antagonizes acetylcholine and histamine
receptors. In the CNS and smooth muscles, benztropine exerts its action through
competing with acetylcholine at muscarinic receptors. Consequently, it reduces central
cholinergic effects by blocking muscarinic receptors that appear to improve the
symptoms of Parkinson disease. Thus, benztropine blocks the cholinergic muscarinic
receptor in the central nervous system. Therefore, it reduces the cholinergic effects
significantly during Parkinson disease which becomes more pronounced in the
nigrostriatal tract because of reduced dopamine concentrations.
 Option B: Diphenhydramine provides rapid relief for dystonia.
Diphenhydramine, which is available as an over-the-counter medication, is a first-
generation antihistamine that is used in a variety of conditions to treat and prevent
dystonias, insomnia, pruritus, urticaria, vertigo, and motion sickness. It also
possesses local anesthetic properties for those patients who have allergies to other,
more commonly used local anesthetics; however, this is an off-label use of the
medication.
 Option C: Propranolol relieves akathisia. Beta-blockers such as propranolol and
benzodiazepines have historically been used for the treatment of akathisia
although the amount of high-quality data supporting their use is limited. When
using beta-blockers, clinicians should be aware of the risk of bradycardia and
hypotension.
 Option D: Haloperidol can cause Parkinson-type symptoms. Toxicities are the
exaggerated symptoms of known pharmacologic effects and known adverse
reactions. The most prominent toxicities of haloperidol are 1) severe
extrapyramidal symptoms, hypotension, sedation. The patient may appear
comatose with severe respiratory depression or shock from hypotension. The
extrapyramidal symptoms are muscular weakness or rigidity, a generalized or
localized tremor that may be characterized by the akinetic or agitation types of
movements, respectively.

 33. Question 1 point(s)

A client is receiving haloperidol (Haldol) to reduce psychotic symptoms. As he watches


television with other clients, the nurse notes that he has trouble sitting still. He seems
restless, constantly moving his hands and feet and changing position. When the nurse
asks what is wrong, he says he feels jittery. How should the nurse manage this situation?

 A. Ask the client to sit still or leave the room because he is distracting the other
clients.
B. Ask the client if he is nervous or anxious about something.
C. Give an as needed dose of a prescribed anticholinergic agent to control
akathisia.
D. Administer an as needed dose of haloperidol to decrease agitation.

Incorrect Correct Answer: C. Give an as needed dose of a prescribed
anticholinergic agent to control akathisia.
Akathisia, characterized by restlessness, is a common but often overlooked adverse
reaction to haloperidol and other antipsychotic agents; it may be confused with psychotic
agitation. To control akathisia, the nurse should give an as needed dose of a prescribed
anticholinergic agent. Anticholinergic agents such as benztropine may be utilized if
concomitant pseudoparkinsonism is present.
 Option A: The client can’t control the movements, so asking him to sit still
would be pointless. Patient’s presenting with akathisia typically have recently
started an antipsychotic agent, or their dose has been increased. Akathisia usually
develops within the first 2 weeks of antipsychotic therapy. There are subjective
and objective components to akathisia. Patients will typically describe a feeling of
restlessness with a desire to move. Additionally, patients will be objectively seen
manifesting that restlessness by pacing, rocking, and shifting position. Patients
with akathisia often feel distressed and uncomfortable.
 Option B: Asking him to leave the room wouldn’t address the underlying cause
of the problem. Encouraging him to talk about the symptoms wouldn’t stop them
from occurring. Providers should be aware that the inner restlessness often causes
extreme anxiety and dysphoria in the individual. In chronic cases, akathisia has
also been associated with a high risk of self-harm or suicidal behavior; therefore,
the clinician should obtain a history of depression, anxiety, and suicidal ideations.
 Option D: Giving more antipsychotic medication would worsen akathisia. The
exact etiology of akathisia is unknown, but it is thought to be due to antipsychotic
agents blocking dopamine type-2 receptors in the brain. The general belief is that
there is an imbalance between cholinergic/dopaminergic or
serotonergic/dopaminergic systems. The organ where this imbalance occurs is
most likely the shell of the nucleus accumbens.

 34. Question 1 point(s)

A man is brought to the hospital by his wife, who states that for the past week her
husband has refused all meals and accused her of trying to poison him. During the initial
interview, the client’s speech, only partly comprehensible, reveals that his thoughts are
controlled by delusions that he is possessed by the devil. The physician diagnoses
paranoid schizophrenia. Schizophrenia is best described as a disorder characterized by:

 A. Disturbed relationships related to an inability to communicate and think


clearly.
B. Severe mood swings and periods of low to high activity.
C. Multiple personalities, one of which is more destructive than the others.
D. Auditory and tactile hallucinations.

Incorrect Correct Answer: A. Disturbed relationships related to an inability to
communicate and think clearly.
Schizophrenia is best described as one of a group of psychotic reactions characterized by
disturbed relationships with others and an inability to communicate and think clearly.
Schizophrenic thoughts, feelings, and behavior commonly are evidenced by withdrawal,
fluctuating moods, disordered thinking, and regressive tendencies. Traditionally,
symptoms have divided into two main categories: positive symptoms which include
hallucinations, delusions, and formal thought disorders, and negative symptoms such as
anhedonia, poverty of speech, and lack of motivation. The diagnosis of schizophrenia is
clinical; made exclusively after obtaining a full psychiatric history and excluding other
causes of psychosis.
 Option B: Severe mood swings and periods of low to high activity are typical of
bipolar disorder. The bipolar affective disorder is a chronic and complex disorder
of mood that is characterized by a combination of manic (bipolar mania),
hypomanic and depressive (bipolar depression) episodes, with substantial
subsyndromal symptoms that commonly present between major mood episodes.
 Option C: Multiple personality, sometimes confused with schizophrenia, is a
dissociative personality disorder, not a psychotic illness. Dissociative identity
disorder (DID) was formerly called multiple personality disorder. People with
DID develop one or more alternate personalities that function with or without the
awareness of the person’s usual personality. DID is one of a group of conditions
called dissociative disorders. Dissociative disorders are mental illnesses that
involve disruptions or breakdowns of memory, consciousness or awareness,
identity and/or perception — mental functions that normally work smoothly.
 Option D: Many schizophrenic clients have auditory hallucinations; tactile
hallucinations are more common in organic or toxic disorders. Auditory
hallucinations are the sensory perceptions of hearing voices without an external
stimulus. This symptom is particularly associated with schizophrenia and related
psychotic disorders but is not specific to it. These voices can be stressful when
they are threatening, derogatory, commanding, or haunting, affecting the social
and occupational functioning of an individual. They can be distressing to families
and friends.

 35. Question 1 point(s)

A client has a history of chronic undifferentiated schizophrenia. Because she has a history
of noncompliance with antipsychotic therapy, she’ll receive fluphenazine decanoate
(Prolixin Decanoate) injections every 4 weeks. Before discharge, what should the nurse
include in her teaching plan?

 A. Asking the physician for droperidol (Inapsine) to control any extrapyramidal


symptoms that occur.
B. Sitting up for a few minutes before standing to minimize orthostatic
hypotension.
C. Notifying the physician if her thoughts don’t normalize within 1 week.
D. Expecting symptoms of tardive dyskinesia to occur and to be transient.

Incorrect Correct Answer: B. Sitting up for a few minutes before standing to
minimize orthostatic hypotension
The nurse should teach the client how to manage common adverse reactions, such as
orthostatic hypotension and anticholinergic effects. Fluphenazine has an adverse effect
profile similar to other first-generation or typical antipsychotics, which is due to its
dopamine receptor antagonism as well as its anticholinergic, antihistaminic, and alpha-
adrenergic antagonistic properties. Common side effects include sedation, dry mouth,
constipation, dry eyes, blurred vision, constipation, orthostasis, dizziness, hypotension,
and urinary retention.
 Option A: Droperidol increases the risk of extrapyramidal effects when given in
conjunction with phenothiazines such as fluphenazine. The most common
behavioral adverse effects of INAPSINE (droperidol) include dysphoria,
postoperative drowsiness, restlessness, hyperactivity and anxiety, which can
either be the result of an inadequate dosage (lack of adequate treatment effect) or
of an adverse drug reaction (part of the symptom complex of akathisia). Care
should be taken to search for extrapyramidal signs and symptoms (dystonia,
akathisia, oculogyric crisis) to differentiate these different clinical conditions.
When extrapyramidal symptoms are the cause, they can usually be controlled with
anticholinergic agents.
 Options C: Antipsychotic effects of the drug may take several weeks to appear.
Oral fluphenazine has a half-life of 14 to 16 hours. Intramuscular (IM)
formulation for acute administration is typically a 1.25 mg initial dose with
options ranging from 2.5 mg to 10 mg per day. IM, short-acting formulations can
be administered every 6 to 8 hours as needed for acute agitation in patients with
psychosis. The half-life of the intramuscular formulation of fluphenazine is 6 to
10 days. The long-acting intramuscular or subcutaneous formulation is dosed
initially 12.5 mg to 25 mg, and typical dosing is every 28 days.
 Option D: Tardive dyskinesia is a possible adverse reaction and should be
reported immediately. Tardive dyskinesia is caused due to long-term exposure to
first and second-generation neuroleptics, certain antidepressants, lithium, and
some antiemetic medications. Typically, the first-generation antipsychotics with
increased dopamine D2 receptor affinity are affiliated with a higher risk of
causing permanent abnormal involuntary movements.

 36. Question 1 point(s)

A client with chronic schizophrenia who takes neuroleptic medication is admitted to the
psychiatric unit. Nursing assessment reveals rigidity, fever, hypertension, and
diaphoresis. These findings suggest which life-threatening reaction:

 A. Tardive dyskinesia
B. Dystonia
C. Neuroleptic malignant syndrome
D. Akathisia

Incorrect Correct Answer: C. Neuroleptic malignant syndrome
The client’s signs and symptoms suggest neuroleptic malignant syndrome, a life-
threatening reaction to neuroleptic medication that requires immediate treatment.
Neuroleptic malignant syndrome (NMS) is a life-threatening syndrome associated with
the use of dopamine-receptor antagonist medications or with rapid withdrawal of
dopaminergic medications. NMS has been associated with virtually every neuroleptic
agent but is more commonly reported with the typical antipsychotics like haloperidol and
fluphenazine. Classic clinical characteristics include mental status changes, fever, muscle
rigidity, and autonomic instability. While uncommon, NMS remains an important part of
the differential diagnosis of fever and mental status changes because it requires early
diagnosis and treatment to prevent significant mortality and death.
 Option A: Tardive dyskinesia causes involuntary movements of the tongue,
mouth, facial muscles, and arm and leg muscles. Tardive dyskinesia (TD) is a
syndrome which includes a group of iatrogenic movement disorders caused due to
a blockade of dopamine receptors. The movement disorders include akathisia,
dystonia, buccolingual stereotypy, myoclonus, chorea, tics and other abnormal
involuntary movements which are commonly caused by the long-term use of
typical antipsychotics.
 Option B: Dystonia is characterized by cramps and rigidity of the tongue, face,
neck, and back muscles. Dystonia is defined by involuntary maintained
contraction of agonist and antagonist muscles yielding abnormal posturing,
twisting and repetitive movements, or tremulous and can be initiated or worsened
by attempted movement. Dystonia is a dynamic disorder that changes in severity
based on the activity and posture. Dystonia may assume a pattern of
overextension or over-flexion of the hand, inversion of the foot, lateral flexion or
retroflection of the head, torsion of the spine with arching and twisting of the
back, forceful closure of the eyes, or a fixed grimace. It may come to an end when
the body is in action and during sleep.
 Option D: Akathisia causes restlessness, anxiety, and jitteriness. Akathisia is
defined as an inability to remain still. It is a neuropsychiatric syndrome that is
associated with psychomotor restlessness. The individual with akathisia will
generally experience an intense sensation of unease or an inner restlessness that
usually involves the lower extremities. This results in a compulsion to move. In
most cases the movement is repetitive. The individual may cross, uncross, swing,
or shift from one foot to the other. To the observer, this may appear as a persistent
fidget.

 37. Question 1 point(s)

While looking out the window, a client with schizophrenia remarks, “That school across
the street has creatures in it that are waiting for me.” Which of the following terms best
describes what the creatures represent?
 A. Anxiety attack
B. Projection
C. Hallucination
D. Delusion

Correct Correct Answer: D. Delusion
A delusion is a false belief based on a misrepresentation of a real event or experience.
Delusions are defined as fixed, false beliefs that conflict with reality. Despite contrary
evidence, a person in a delusional state can’t let go of their convictions. Delusions are
often reinforced by the misinterpretation of events. Many delusions also involve some
level of paranoia. For example, someone might contend that the government is
controlling our every move via radio waves despite evidence to the contrary. Delusions
are often part of psychotic disorders. They may occur along with hallucinations, which
involve perceiving something that isn’t really there, like hearing voices or feeling bugs
crawling on your skin.
 Option A: Although anxiety can increase delusional responses, it isn’t considered
the primary symptom. An anxiety attack usually involves a fear of some specific
occurrence or problem that could happen. Symptoms include worry, restlessness,
and possibly physical symptoms, such as changes in heart rate. Anxiety is
different from a panic attack, but it can occur as part of an anxiety or panic
disorder.
 Option B: Projection is falsely attributing to another person one’s own
unacceptable feelings. Projection is a defense mechanism that involves taking our
own unacceptable qualities or feelings and ascribing them to other people. For
example, if you have a strong dislike for someone, you might instead believe that
they do not like you. Projection works by allowing the expression of the desire or
impulse, but in a way that the ego cannot recognize, therefore reducing anxiety.
 Option C: Hallucinations, which characterize most psychoses, are perceptual
disorders of the five senses; the client may see, taste, feel, smell, or hear
something in the absence of external stimulation. The word “hallucination” comes
from Latin and means “to wander mentally.” Hallucinations are defined as the
“perception of a nonexistent object or event” and “sensory experiences that are
not caused by stimulation of the relevant sensory organs.”

 38. Question 1 point(s)

A client with schizophrenia tells the nurse, “My intestines are rotted from the worms
chewing on them.” This statement indicates a:A. Delusion of persecution

 A. Delusion of persecution
B. Delusion of grandeur
C. Somatic delusion
D. Jealous delusion

Incorrect Correct Answer: C. Somatic delusion
Somatic delusions focus on bodily functions or systems and commonly include delusions
about foul odor emissions, insect infestations, internal parasites, and misshapen parts. Of
the delusional symptoms, somatic delusions-those that pertain to the body-are rather rare.
Somatic delusions are defined as fixed false beliefs that one’s bodily function or
appearance is grossly abnormal. They are a poorly understood psychiatric symptom and
pose a significant clinical challenge to clinicians.
 Option A: Delusions of persecution are morbid beliefs that one is being
mistreated and harassed by unidentified enemies. Persecutory delusions occur
when someone believes others are out to harm them despite evidence to the
contrary. It’s a type of paranoid thinking that can be part of several different
mental illnesses. Whether people with this condition think coworkers are
sabotaging their work or they believe the government is trying to kill them,
persecutory delusions vary in severity. Some individuals with persecutory
delusions believe they have to go to great lengths to stay safe—and consequently,
they may struggle to function normally.
 Option B: Delusions of grandeur are gross exaggerations of one’s importance,
wealth, power, or talents. A delusion of grandeur is the false belief in one’s own
superiority, greatness, or intelligence. People experiencing delusions of grandeur
do not just have high self-esteem; instead, they believe in their own greatness and
importance even in the face of overwhelming evidence to the contrary. Someone
might, for example, believe they are destined to be the leader of the world, despite
having no leadership experience and difficulties in interpersonal relationships.
Delusions of grandeur are characterized by their persistence. They are not just
moments of fantasy or hopes for the future.
 Option D: Jealous delusions are delusions that one’s spouse or lover is unfaithful.
Delusional jealousy (also known as morbid jealousy) is one type of delusional
disorder, and as the name implies people with jealous delusions are completely
convinced that their spouses or romantic partners have been unfaithful.

 39. Question 1 point(s)

During the assessment stage, a client with schizophrenia leaves his arm in the air after the
nurse has taken his blood pressure. His action shows evidence of:

 A. Somatic delusions
B. Waxy flexibility
C. Neologisms
D. Nihilistic delusions

Incorrect Correct Answer: B. Waxy flexibility
The correct answer is waxy flexibility, which is defined as retaining any position that the
body has been placed in. Waxy flexibility is a psychomotor symptom of catatonia as
associated with schizophrenia, bipolar disorder, or other mental disorders which leads to
a decreased response to stimuli and a tendency to remain in an immobile posture.
Attempts to reposition the patient are met by “slight, even resistance”, and after being
repositioned, the patient will typically remain in the new position.
 Option A: Somatic delusions involve a false belief about the functioning of the
body. Of the delusional symptoms, somatic delusions-those that pertain to the
body-are rather rare. Somatic delusions are defined as fixed false beliefs that
one’s bodily function or appearance is grossly abnormal. They are a poorly
understood psychiatric symptom and pose a significant clinical challenge to
clinicians.
 Option C: Neologisms are invented meaningless words. In a neurological or
psychopathological context, neologisms, whose origins and meanings are usually
nonsensical and unrecognizable (e.g., klipno for watch), are typically associated
with aphasia or schizophrenia.
 Option D: Nihilistic delusions are false ideas about self, others, or the world.
Nihilistic delusions, also known as délires de négation, are specific
psychopathological entities characterized by the delusional belief of being dead,
decomposed, or annihilated, having lost one’s own internal organs or even not
existing entirely as a human being.

 40. Question 1 point(s)

A client with paranoid-type schizophrenia becomes angry and tells the nurse to leave him
alone. The nurse should

 A. Tell him that she’ll leave for now but will return soon.
B. Ask him if it’s okay if she sits quietly with him.
C. Ask him why he wants to be left alone.
D. Tell him that she won’t let anything happen to him.

Incorrect Correct Answer: A. Tell him that she’ll leave for now but will return soon.
If the client tells the nurse to leave, the nurse should leave but let the client know that
she’ll return so that he doesn’t feel abandoned. If a client is found to be very paranoid,
solitary or one-on-one activities that require concentration are appropriate. Client is free
to choose his level of interaction; however, the concentration can help minimize
distressing paranoid thoughts or voices.
 Option B: Not heeding the client’s request can agitate him further. Teach the
client to remove himself briefly when feeling agitated and work on some anxiety
relief exercises (e.g., meditations, rhythmic exercise, deep breathing exercise).
Teach client skills in dealing with anxiety and increasing a sense of control.
 Option C: Also, challenging the client isn’t therapeutic and may increase his
anger. If the client is very withdrawn, one-on-one activities with a “safe” person
initially should be planned. They learn to feel safe with one person, then gradually
might participate in a structured group activity. Useful coping skills that the client
will need include conversational and assertiveness skills. These are fundamental
skills for dealing with the world, which everyone uses daily with more or less
skill.
 Option D: False reassurance isn’t warranted in this situation. Ensure that the
goals set are realistic; whether in the hospital or community. Avoids pressure on
the client and sense of failure on part of the nurse/family. This sense of failure can
lead to mutual withdrawal. If the client is delusional/hallucinating or is having
trouble concentrating at this time, provide very simple concrete activities with the
client (e.g., looking at a picture or doing a painting). Even simple activities help
draw the client away from delusional thinking into reality in the environment.

 41. Question 1 point(s)

Nursing care for a client with schizophrenia must be based on valid psychiatric and
nursing theories. The nurse’s interpersonal communication with the client and specific
nursing interventions must be:

 A. Clearly identified with boundaries and specifically defined roles.


B. Warm and non-threatening.
C. Centered on clearly defined limits and expression of empathy.
D. Flexible enough for the nurse to adjust the plan of care as the situation
warrants.

Correct Correct Answer: D. Flexible enough for the nurse to adjust the plan of care
as the situation warrants.
A flexible plan of care is needed for any client who behaves in a suspicious, withdrawn,
or regressed manner or who has a thought disorder. Because such a client communicates
at different levels and is in control of himself at various times, the nurse must be able to
adjust nursing care as the situation warrants. Ensure that the goals set are realistic;
whether in the hospital or community. Avoids pressure on the client and sense of failure
on part of the nurse/family. This sense of failure can lead to mutual withdrawal.
 Option A: The nurse’s role should be clear; however, the boundaries or limits of
this role should be flexible enough to meet client needs. Structure activities that
work at the client’s pace and activity. Structure times each day to include planned
times for brief interactions and activities with the client on one-on-one basis.
Helps the client to develop a sense of safety in a non-threatening environment.
 Option B: Because a client with schizophrenia fears closeness and affection, a
warm approach may be too threatening. Avoid touching the client. Touch by an
unknown person can be misinterpreted as a sexual or threatening gesture. This
particularly true for a paranoid client. Keep the client in an environment as free of
stimuli (loud noises, crowding) as possible. The client might respond to noises
and crowding with agitation, anxiety, and increased inability to concentrate on
outside events. The client can lose interest in activities that are too ambitious,
which can increase a sense of failure.
 Option C: Expressing empathy is important, but centering interventions on
clearly defined limits is impossible because the client’s situation may change
without warning. Teach the client to remove himself briefly when feeling agitated
and work on some anxiety relief exercise (e.g., meditations,rhythmic exercise,
deep breathing exercise). Teach client skills in dealing with anxiety and
increasing a sense of control. Useful coping skills that the client will need include
conversational and assertiveness skills. These are fundamental skills for dealing
with the world, which everyone uses daily with more or less skill.

 42. Question 1 point(s)

When discharging a client after treatment for a dystonic reaction, the emergency
department nurse must ensure that the client understands which of the following?

 A. Results of treatment are rapid and dramatic but may not last.
B. Although uncomfortable, this reaction isn’t serious.
C. The client shouldn’t buy drugs on the street.
D. The client must take benztropine (Cogentin) as prescribed to prevent a
return of symptoms.

Correct Correct Answer: D. The client must take benztropine (Cogentin) as
prescribed to prevent a return of symptoms.
An oral anticholinergic agent such as benztropine (Cogentin) is commonly prescribed to
control and prevent the return of symptoms. Benztropine is FDA approved as adjunctive
therapy of all forms of parkinsonism. It is also useful for drug-induced extrapyramidal
symptoms and the prevention of dystonic reactions and acute treatment of dystonic
reactions. Benztropine antagonizes acetylcholine and histamine receptors. In the CNS and
smooth muscles, benztropine exerts its action through competing with acetylcholine at
muscarinic receptors. Consequently, it reduces central cholinergic effects by blocking
muscarinic receptors that appear to improve the symptoms of Parkinson disease.
 Option A: Dystonic reactions are typically acute and reversible. An acute
dystonic reaction is characterized by involuntary contractions of muscles of the
extremities, face, neck, abdomen, pelvis, or larynx in either sustained or
intermittent patterns that lead to abnormal movements or postures. The symptoms
may be reversible or irreversible and can occur after taking any dopamine
receptor-blocking agents.
 Option B: Dystonic reactions can be life-threatening when airway patency is
compromised. Anticholinergic agents and benzodiazepines are the most
commonly used agents to reverse or reduce symptoms in acute dystonic reaction.
Acute dystonic reactions are often transient but can cause significant distress to
the patient. Although rare, laryngeal dystonia can cause life-threatening airway
obstruction.
 Option C: Lecturing the client about buying drugs on the street isn’t appropriate.
Antipsychotic and antiemetic agents are among the most commonly described
causative agents of acute dystonic reactions. Other agents including anti-malarial,
antidepressants, antihistamines, and anticonvulsants have also been implicated in
cases of acute dystonic reaction. Antipsychotic agents with a dopamine-blocking
mechanism are commonly used to treat acute psychosis, acute agitation, bipolar
mania, and many other psychiatric conditions. All currently known antipsychotic
medications carry a risk of causing an acute dystonic reaction.

 43. Question 1 point(s)

Upon evaluation of the patient’s record, the nurse sees the admission was voluntary.
Based on this data, the nurse expects which patient behavior?

 A. Fearfulness regarding treatment measures.


B. Anger and aggressiveness directed toward others.
C. An understanding of the pathology and symptoms of the diagnosis.
D. A willingness to participate in the planning of the care and treatment
plan.

Correct Correct Answer: D. A willingness to participate in the planning of the care
and treatment plan.
In general, patients seek voluntary admission. If a patient seeks voluntary admission, the
most likely expectation is the patient will participate in the treatment program since they
are actively seeking help. There are advantages and disadvantages of being a voluntary
patient. The client has greater control and say over their life. For some people, this is very
important and can improve wellbeing. The client has more freedom. They are able to
leave the ward when they want, within reason. They should also have less restrictions
placed on them, like having their mail checked on the ward or having access to their
possessions.
 Option A: The client can refuse treatment, including medication. This means that
they can have more discussions with their care team, which leads to better
understanding and they are more likely to take the medication. Dispel
misconceptions and myths. These include catastrophic fears—often based on
stereotypes—about coercive treatment and indefinite confinement. Clarifying
what a patient can expect with voluntary admission with regard to the probable
length of stay, participation in the milieu, visitation, and discharge planning is
helpful for allaying such fears.
 Option B: Anger and aggressiveness are more characteristic of involuntary
admission. Maintain an empathic stance. For many patients, psychiatric admission
evokes considerable distress. Remain sensitive to the situational concerns that
typically arise, such as disruption to family and job responsibilities, insurance
coverage, and whether there will be an outpatient plan in place at discharge.
 Option C: The remaining options are not characteristics of this type of admission.
Voluntary admission does not guarantee a patient’s understanding of their illness,
only of their desire for help. Provide a compelling rationale. Stress the need for
immediate, specialized, and intensive services. If the patient is receiving
outpatient mental health care, advise him that these services have been
unsuccessful in achieving safety and clinical stability and that it is not possible to
quickly establish a modified outpatient plan or a day hospital placement that
would meet his needs. For a patient who is not receiving outpatient care, explain
that it is not feasible to implement a workable plan “from the ground up” in a
timely manner.

 44. Question 1 point(s)

A clinical instructor is correcting a nursing student’s worksheet. Which instructor


statement is the best example of effective feedback?

 A. "Why did you use the client's name on your clinical worksheet?"
B. "You were very careless to refer to your client by name on your clinical
worksheet."
C. "Surely you didn't do this deliberately, but you breached confidentiality by
using the client's name."
D. "It is disappointing that after being told, you're still using client names on your
worksheet."

Incorrect Correct Answer: C. “Surely you didn’t do this deliberately, but you
breached confidentiality by using the client’s name.”
The instructor’s statement, “Surely you didn’t do this deliberately, but you breached
confidentiality by using the client’s name.” is an example of effective feedback.
Feedback is a method of communication to help others consider a modification of
behavior. Feedback should be descriptive, specific, and directed toward behavior that the
person has the capacity to modify and should impart information rather than offer advice
or criticize the individual.
 Option A: Positive feedback is communication that recognizes another’s
strengths, achievements or successes. Giving and receiving positive feedback is
beneficial for everyone While providing feedback may seem like a managerial
function, individual contributors should also make it a priority to recognize the
strengths of their colleagues. Using positive feedback helps individuals recognize
and hone their skills, develop their areas of improvement and create a general
sense of positivity in the workplace.
 Option B: For positive (or negative) feedback to be impactful, it should be well-
thought-out before presented to the individual. If you’re in a position to give
positive feedback, think about how you can include specific details so your
audience understands exactly why what they did was good. This way, they can
continue to do these things and improve upon them.
 Option D: Feedback about what people are doing right can result in increased
engagement both with their work and their colleagues. Some of the benefits of
increased engagement include comfort with sharing new ideas and pointing out
problems that need to be addressed. Giving and receiving positive feedback that
aligns with your organization’s standards can help you and your colleagues
maintain the quality of both behavior and work that your employer expects. This
can help you be successful in your role and clear the way for raises and
promotions.

 45. Question 1 point(s)

Which nursing statement is a good example of the therapeutic communication technique


of focusing?

 A. "Describe one of the best things that happened to you this week."
B. "I'm having a difficult time understanding what you mean."
C. "Your counseling session is in 30 minutes. I'll stay with you until then."
D. "You mentioned your relationship with your father. Let's discuss that
further."

Correct Correct Answer: D. “You mentioned your relationship with your father.
Let’s discuss that further.”
This is an example of the therapeutic communication technique of focusing. Focusing
takes notice of a single idea or even a single word and works especially well with a client
who is moving rapidly from one thought to another. Sometimes during a conversation,
patients mention something particularly important. When this happens, nurses can focus
on their statement, prompting patients to discuss it further. Patients don’t always have an
objective perspective on what is relevant to their case; as impartial observers, nurses can
more easily pick out the topics to focus on.
 Option A: Therapeutic communication is often most effective when patients
direct the flow of conversation and decide what to talk about. To that end, giving
patients a broad opening such as “What’s on your mind today?” or “What would
you like to talk about?” can be a good way to allow patients an opportunity to
discuss what’s on their mind.
 Option B: Similar to active listening, asking patients for clarification when they
say something confusing or ambiguous is important. Saying something like “I’m
not sure I understand. Can you explain it to me?” helps nurses ensure they
understand what’s actually being said and can help patients process their ideas
more thoroughly.
 Option C: Hospital stays can be lonely, stressful times; when nurses offer their
time, it shows they value patients and that someone is willing to give them time
and attention. Offering to stay for lunch, watch a TV show, or simply sit with
patients for a while can help boost their mood.

 46. Question 1 point(s)

A person with antisocial personality disorder has toughness relating to others because of
never having learned to:

 A. Count on others
B. Empathize with others
C. Be dependent on others
D. Communicate with others socially

Incorrect Correct Answer: B. Empathize with others
The lack of superego control allows the ego and the id to control the behavior. Self-
motivation and self-satisfaction are of paramount concern. Antisocial personality disorder
(ASPD) is a deeply ingrained and rigid dysfunctional thought process that focuses on
social irresponsibility with exploitive, delinquent, and criminal behavior with no remorse.
Disregard for and the violation of others’ rights are common manifestations of this
personality disorder, which displays symptoms that include failure to conform to the law,
inability to sustain consistent employment, deception, manipulation for personal gain,
and incapacity to form stable relationships.
 Option A: The Diagnostic and Statistical Manual of Mental Disorders (DSM 5)
classifies all ten personality disorders into three clusters (A, B, and C). Antisocial
personality disorder falls into 1 of 4 cluster-B disorders, which also includes
borderline, narcissistic, and histrionic. All of these disorders characteristically
present with dramatic, emotional, and unpredictable interactions with others.
Antisocial personality disorder is the only personality disorder that is not
diagnosable in childhood. Before the age of 18, the patient must have been
previously diagnosed with conduct disorder (CD) by the age of 15 years old to
justify diagnostic criteria for ASPD.
 Option C: Many individuals diagnosed with antisocial personality disorder
remain a burden to their families, coworkers, and closely associated peers, such as
neighbors, despite becoming less troublesome with age. Mental health
comorbidities and associated addictive disorders, as well as higher mortality rates
due to suicides and homicides, only add to this burden.
 Option D: Most of those who improve with age remain unable to re-claim their
lost prospects, including education, domestication, and employment. Those
patients who did show remission were more likely to have spousal or family ties,
with better social support.

 47. Question 1 point(s)

Which of the following behaviors by a client with dependent personality disorder shows
the client has made progress toward the goal of increasing problem-solving skills?

 A. The client is courteous.


B. The client asks questions.
C. The client stops acting out.
D. The client controls emotions.

Incorrect Correct Answer: B. The client asks questions.
The client with a dependent personality disorder is passive and tries to please others. By
asking questions, the client is beginning to gather information, the first step of decision
making. People with DPD have an overwhelming need to have others take care of them.
Often, a person with DPD relies on people close to them for their emotional or physical
needs. Others may describe them as needy or clingy. People with DPD may believe they
can’t take care of themselves. They may have trouble making everyday decisions, such as
what to wear, without others’ reassurance.
 Option A: Dependent personality disorder (DPD) is a type of anxious personality
disorder. People with DPD often feel helpless, submissive, or incapable of taking
care of themselves. They may have trouble making simple decisions. But, with
help, someone with a dependent personality can learn self-confidence and self-
reliance.
 Option C: Mental health experts describe personality as a person’s way of
thinking, feeling, and behaving. A personality disorder affects the way people
think or act, making them behave differently over time. Dependent personality
disorder (DPD) is one of 10 types of personality disorders. Other types include
antisocial personality disorder, narcissistic personality disorder, and paranoid
personality disorder. Dependent personality disorder usually starts during
childhood or by the age of 29.
 Option D: With psychotherapy and CBT, the provider guides the client to
improve their self-confidence. They’ll work to become more active and self-
reliant. The provider will also talk to the client about finding more positive
relationships. A positive, meaningful relationship can build self-confidence and
help them overcome some of the symptoms of DPD.

 48. Question 1 point(s)

Which is the best indicator of success in the long-term management of the client?

 A. His symptoms are replaced by indifference to his feelings.


B. He participates in diversionary activities.
C. He learns to verbalize his feelings and concerns.
D. He states that his behavior is irrational.

Incorrect Correct Answer: C. He learns to verbalize his feelings and concerns.
The client is encouraged to talk about his feelings and concerns instead of using body
symptoms to manage his stressors. Accurate measurement and improvement of
population mental health require the recording of indicators that capture the full spectrum
of disease severity.
 Option A: The client is encouraged to acknowledge feelings rather than being
indifferent to her feelings. Mental health surveillance will require a conglomerate
of indicators, and would best be served by including upstream determinants of
mental health in addition to downstream symptomatic outcomes.
 Option B: Participation in activities diverts the client’s attention away from his
bodily concerns but this is not the best indicator of success. The stigma associated
with mental health disorders may lead to underreporting of symptoms by
participants in the CCHS survey data, as well as by physicians in their billing
practices, and thus create problems in predicting the need for mental health
services.
 Option D: Help the client recognize that his physical symptoms occur because of
or are exacerbated by a specific stressor, not as irrational. The use of a
standardized set of indicators that takes into account health determinants, the
severity of symptoms, and the use of healthcare services would permit more
useful international comparisons.

 49. Question 1 point(s)

The nurse asks a client to roll up his sleeves so she can take his blood pressure. The client
replies “If you want I can go naked for you.” The most therapeutic response by the nurse
is:

 A. “You’re attractive, but I’m not interested.”


B. “You wouldn’t be the first that I will see naked.”
C. “I will report you to the guard if you don’t control yourself.”
D. “I only need access to your arm. Putting up your sleeve is fine.”

Correct Correct Answer: D. “I only need access to your arm. Putting up your sleeve
is fine.”
The nurse needs to deal with the client with sexually connotative behavior in a casual,
matter of fact way. Sometimes during a conversation, patients mention something
particularly important. When this happens, nurses can focus on their statement,
prompting patients to discuss it further. Patients don’t always have an objective
perspective on what is relevant to their case; as impartial observers, nurses can more
easily pick out the topics to focus on.
 Option A: Observations about the appearance, demeanor, or behavior of patients
can help draw attention to areas that might pose a problem for them. Observing
that they look tired may prompt patients to explain why they haven’t been getting
much sleep lately; making an observation that they haven’t been eating much may
lead to the discovery of a new symptom.
 Option B: These responses are not therapeutic because they are challenging and
rejecting. It’s frequently useful for nurses to summarize what patients have said
after the fact. This demonstrates to patients that the nurse was listening and allows
the nurse to document conversations. Ending a summary with a phrase like “Does
that sound correct?” gives patients explicit permission to make corrections if
they’re necessary.
 Option C: Threatening the client is not therapeutic. Nurses should only apply this
technique after they have established trust. It can be vital to the care of patients to
disagree with them, present them with reality, or challenge their assumptions.
Confrontation, when used correctly, can help patients break destructive routines
or understand the state of their situation.

 50. Question 1 point(s)

Which goal is a priority for a client with a DSM-IV-TR diagnosis of delirium and the
nursing diagnosis Acute confusion related to recent surgery secondary to traumatic hip
fracture?

 A. The client will complete activities of daily living.


B. The client will maintain safety.
C. The client will remain oriented.
D. The client will understand communication.

Incorrect Correct Answer: B. The client will maintain safety.
Maintaining safety is the priority goal for an acutely confused client who recently had
surgery. All measures to promote physiologic safety and psychosocial wellbeing would
be implemented. Remove all potentially dangerous objects from the client’s environment;
in a disoriented, confused state, clients may use objects to harm self or others.
 Option A: This client would not be able to complete activities of daily living, and
safety is a priority over these tasks. Provide an appropriate environment. Maintain
a low level of stimuli in the client’s environment (low lighting, few people,
simple decor, low noise level) because anxiety increases in a highly stimulating
environment.
 Option C: Interrupt periods of unreality and reorient; client safety is jeopardized
during periods of disorientation; correcting misinterpretations of reality enhances
client’s feelings of self-worth and personal dignity. Stay calm and reassure the
patient. Maintain a calm manner with the client; attempt to prevent frightening the
client unnecessarily; Provide continual reassurance and support.
 Option D: The goals of remaining oriented and understanding communication
would be appropriate only after the client’s acute confusion has resolved. Assess
the level of anxiety. Assess the client’s level of anxiety and behaviors that
indicate the anxiety is increasing; recognizing these behaviors, the nurse may be
able to intervene before violence occurs.

 ABOUT
 PRIVACY
 DISCLAIMER
 CONTACT
© 2024 Nurseslabs | Ut in Omnibus Glorificetur Deus!
 1. Question 1 point(s)

Which nursing intervention is best for facilitating communication with a psychiatric


client who speaks a foreign language?

 A. Rely on nonverbal communication


B. Select symbolic pictures as aids
C. Speak in universal phrases
D. Use the services of an interpreter

Correct Correct Answer: D. Use the services of an interpreter
An interpreter will enable the nurse to better assess the client’s problems and concerns.
Language barriers pose challenges in terms of achieving high levels of satisfaction among
medical professionals and patients, providing high- quality healthcare and maintaining
patient safety. To address these challenges, many larger healthcare institutions offer
interpreter services to improve healthcare access, patient satisfaction, and
communication.
 Option A: Nonverbal communication is important; however for the nurse to fully
determine the client’s problems and concerns, the assistance of an interpreter is
essential. Language barriers have negative implications for the delivery of
healthcare and patient satisfaction. One study showed that among patients who
received treatment from nurses who did not speak the local language, 30% had
difficulty understanding medical instructions, 30% had a problem with the
reliability of information, and 50% believed that the language barrier contributed
to errors.
 Option B: Online translation tools such as Google Translate and MediBabble
present possible solutions for overcoming these challenges. Further studies on the
implications of language barriers and the effectiveness of online translation tools
are recommended. Furthermore, new updates with more medical phrases for
Google Translate and with more languages included for MediBabble application
are recommended.
 Option C: The use of universal phrases may assist the nurse in understanding the
basic needs of the client; however these are insufficient to assess the client with a
psychiatric problem. Some healthcare organizations use online translation tools
such as Google Translate and MediBabble to address the challenges of language
barriers. These tools are free and easy to access, and they contribute to improving
healthcare delivery, patient safety, and increase (up to 92%) the satisfaction of
both medical professionals and patients.

 2. Question 1 point(s)

The nurse explains to a mental health care technician that a client’s obsessive-compulsive
behaviors are related to an unconscious conflict between id impulses and the superego (or
conscience). On which of the following theories does the nurse base this statement?

 A. Behavioral theory
B. Cognitive theory
C. Interpersonal theory
D. Psychoanalytic theory

Correct Correct Answer: D. Psychoanalytic theory
Psychoanalytic is based on Freud’s beliefs regarding the importance of unconscious
motivation for behavior and the role of the id and superego in opposition to each other.
Psychoanalysis is defined as a set of psychological theories and therapeutic methods
which have their origin in the work and theories of Sigmund Freud . The primary
assumption of psychoanalysis is the belief that all people possess unconscious thoughts,
feelings, desires, and memories. The aim of psychoanalysis therapy is to release
repressed emotions and experiences, i.e., make the unconscious conscious. It is only
having a cathartic (i.e., healing) experience can the person be helped and “cured.”
 Option A: Behaviorism, also known as behavioral psychology, is a theory of
learning based on the idea that all behaviors are acquired through conditioning.
Conditioning occurs through interaction with the environment. Behaviorists
believe that our responses to environmental stimuli shape our actions. According
to this school of thought, behavior can be studied in a systematic and observable
manner regardless of internal mental states. According to this perspective, only
observable behavior should be considered—cognitions, emotions, and moods are
far too subjective.
 Option B: Cognitive theory is an approach to psychology that attempts to explain
human behavior by understanding your thought processes. For example, a
therapist is using principles of cognitive theory when they teach you how to
identify maladaptive thought patterns and transform them into constructive ones.
The assumption of cognitive theory is that thoughts are the primary determinants
of emotions and behavior. Information processing is a common description of this
mental process. Theorists compare the way the human mind functions to a
computer.
 Option C: Interpersonal theory emphasizes the importance of various
developmental stages—infancy, childhood, the juvenile era, preadolescence, early
adolescence, late adolescence, and adulthood. Like Freud and Jung, Sullivan
(1953b) saw personality as an energy system. Energy can exist either as tension
(potentiality for action) or as actions themselves (energy transformations). Energy
transformations transform tensions into either covert or overt behaviors and are
aimed at satisfying needs and reducing anxiety.

 3. Question 1 point(s)

The nurse observes a client pacing in the hall. Which statement by the nurse may help the
client recognize his anxiety?

 A. “I guess you’re worried about something, aren’t you?


B. “Can I get you some medication to help calm you?”
C. “Have you been pacing for a long time?”
D. “I notice that you’re pacing. How are you feeling?”

Correct Correct Answer: D. “I notice that you’re pacing. How are you feeling?”
By acknowledging the observed behavior and asking the client to express his feelings the
nurse can best assist the client to become aware of his anxiety. Recognition
acknowledges a patient’s behavior and highlights it without giving an overt compliment.
A compliment can sometimes be taken as condescending, especially when it concerns a
routine task like making the bed. However, saying something like “I noticed you took all
of your medications” draws attention to the action and encourages it without requiring a
compliment.
 Option A: The nurse is offering an interpretation that may or may not be
accurate; the nurse is also asking a question that may be answered by a “yes” or
“no” response, which is not therapeutic. Therapeutic communication is often most
effective when patients direct the flow of conversation and decide what to talk
about. To that end, giving patients a broad opening such as “What’s on your mind
today?” or “What would you like to talk about?” can be a good way to allow
patients an opportunity to discuss what’s on their mind.
 Option B: The nurse is intervening before accurately assessing the problem. By
using nonverbal and verbal cues such as nodding and saying “I see,” nurses can
encourage patients to continue talking. Active listening involves showing interest
in what patients have to say, acknowledging that you’re listening and
understanding, and engaging with them throughout the conversation. Nurses can
offer general leads such as “What happened next?” to guide the conversation or
propel it forward.
 Option C: This statement encourages a “yes” or “no” response, avoids focusing
on the client’s anxiety, which is the reason for his pacing. Observations about the
appearance, demeanor, or behavior of patients can help draw attention to areas
that might pose a problem for them. Observing that they look tired may prompt
patients to explain why they haven’t been getting much sleep lately; making an
observation that they haven’t been eating much may lead to the discovery of a
new symptom.

 4. Question 1 point(s)

A client with obsessive-compulsive disorder is hospitalized in an inpatient unit. Which


nursing response is most therapeutic?

 A. Accepting the client’s obsessive-compulsive behaviors.


B. Challenging the client’s obsessive-compulsive behaviors.
C. Preventing the client’s obsessive-compulsive behaviors.
D. Rejecting the client’s obsessive-compulsive behaviors.

Incorrect Correct Answer: A. Accepting the client’s obsessive-compulsive behaviors
A client with obsessive-compulsive behavior uses this behavior to decrease anxiety.
Accepting this behavior as the client’s attempt to feel secure is therapeutic. When a
specific treatment plan is developed, other nursing responses may also be acceptable.
Obsessive-compulsive disorder (OCD) is often a disabling condition consisting of
bothersome intrusive thoughts that elicit a feeling of discomfort. To reduce the anxiety
and distress associated with these thoughts, the patient may employ compulsions or
rituals. These rituals may be personal and private, or they may involve others to
participate; the rituals are to compensate for the ego-dystonic feelings of the obsessional
thoughts and can cause a significant decline in function.
 Option B: In The Diagnostic and Statistical Manual of Mental Disorders (DSM)-
5, which was published by the American Psychiatric Association (APA) in 2013,
Obsessive-Compulsive Disorder sits under its own category of Obsessive-
Compulsive and Related Disorders. Obsessions are defined as intrusive thoughts
or urges that cause significant distress; the patient attempts to neutralize this
distress by diverting thoughts or performing rituals. Compulsions are actions the
patient feels pressured to do in response to the anxiety/distress producing
obsessions or to prevent an uncomfortable situation from occurring. These
compulsions may be illogical or excessive.
 Option C: The most common obsessions include fears of contamination, fears of
aggression/harm, sexual fears, religious fears, and need to make things “just
right.” The compensatory compulsions for these obsessions include washing and
cleaning, checking, reassurance-seeking, repeating, and ordering, and arranging.
As OCD has the possibility of hindering one’s social growth and development,
the WHO lists OCD as one of the ten most disabling conditions by financial loss
and a decrease in quality of life.
 Option D: Those who have OCD have a 7% risk of Tourette syndrome and a
20% chance of developing tics. As the treatment for OCD involves selective
serotonin reuptake inhibitors (SSRIs) and possible antipsychotics, adverse effects
of these medications including but not limited to weight gain, tardive dyskinesia,
and dystonia, must also be monitored.

 5. Question 1 point(s)

A 45-year-old woman with a history of depression tells a nurse in her doctor’s office that
she has difficulty with sexual arousal and is fearful that her husband will have an affair.
Which of the following factors would the nurse identify as least significant in
contributing to the client’s sexual difficulty?

 A. Education and work history


B. Medication used
C. Physical health status
D. Quality of spousal relationship

Incorrect Correct Answer: A. Education and work history
Education and work history would have the least significance in relation to the client’s
sexual problem. Depression, performance anxiety, and other sexual disorders can be
strong contributing factors even when organic causes also exist. While having a sexual
dysfunction can feel isolating, it’s actually fairly common. About 40 percent of women
experience some type of sexual dysfunction, such as FSIAD, in their life.
 Option B: Selective serotonin reuptake inhibitors (SSRIs), a type of
antidepressant, may cause FSIAD. Female sexual arousal disorder occurs when
the body doesn’t respond to sexual stimulation. If you’re undergoing
chemotherapy or radiation, you may experience FSIAD. Likewise, a recent
surgery may interfere with arousal and sexual stimulation.
 Option C: While FSIAD can affect any woman, older women seem to experience
it more. Because FSIAD is a newly defined term according to the DSM-5, studies
on its actual occurrence haven’t yet been published. A 2009 study found that 3.3
percent of participants between the ages of 18 and 44 had female sexual arousal
disorder, while 7.5 percent of participants between the ages of 45 and 64
experienced it.
 Option D: You might have trouble getting aroused if the stimulation you receive
from yourself or your partner isn’t sufficient. Arousal sets off a series of events in
the body: Blood flow to the tissues around the vaginal opening and clitoris
increases, causing swelling. The vagina produces natural lubricant. Studies on
female sexual arousal disorder show that low sexual desire and problems with
sexual arousal vary widely by age, cultural setting, duration of symptoms, and
presence of distress.

 6. Question 1 point(s)

Which nursing intervention is most appropriate for a client with anorexia nervosa during
initial hospitalization on a behavioral therapy unit?

 A. Emphasize the importance of good nutrition to establish normal weight.


B. Ignore the client’s mealtime behavior and focus instead on issues of
dependence and independence.
C. Help establish a plan using privileges and restrictions based on compliance
with refeeding.
D. Teach the client information about the long-term physical consequence of
anorexia.

Incorrect Correct Answer: C. Help establish a plan using privileges and restrictions
based on compliance with refeeding.
Inpatient treatment of a client with anorexia usually focuses initially on establishing a
plan for refeeding to combat the effects of self-induced starvation. Refeeding is
accomplished through behavioral therapy, which uses a system of rewards and
reinforcements to assist in establishing weight restoration. Treatment for anorexia
nervosa is centered on nutrition rehabilitation and psychotherapy. Refeeding, nutritional
plans, and weight restoration are crucial parts of the medical stabilization process which
is necessary in order to proceed with treatment and eventually achieve recovery. There
are many serious and deadly complications that arise during the refeeding process which
is why medical supervision is of the utmost importance.
 Option A: Anorexia nervosa is a serious eating disorder which has a very high
morbidity. The disorder is usually managed with an interprofessional team that
consists of a psychiatrist, dietitian, social worker, internist, endocrinologist,
gastroenterologist, and nurses. The disorder cannot be prevented and there is no
cure. Hence patient and family education is key to preventing the high morbidity.
The dietitian should educate the family on the importance of nutrition and limiting
exercise.
 Option B: The nurse needs to assess the client’s mealtime behavior continually to
evaluate treatment effectiveness. Remission in AN varies. Three-fourths of
patients treated in out-patient settings remit within 5 years and the same
percentage experience intermediate-good outcomes (including weight gain).
Relapse is more common in patients who are older with a longer duration of
disease or lower body fat/weight at the end of treatment, have comorbid
psychiatric disorders, or receive therapy outside of a specialized clinic. Patients
who achieve partial remission often develop another form of the eating disorder
(ex. bulimia nervosa or unspecified eating disorder).
 Option D: Emphasizing nutrition and teaching the client about the long-term
physical consequences of anorexia may be appropriate at a later time in the
treatment program. For adults, cognitive behavioral therapy — specifically
enhanced cognitive behavioral therapy — has been shown to help. The main goal
is to normalize eating patterns and behaviors to support weight gain. The second
goal is to help change distorted beliefs and thoughts that maintain restrictive
eating.

 7. Question 1 point(s)

A nurse is evaluating therapy with the family of a client with anorexia nervosa. Which of
the following would indicate that the therapy was successful?
 A. The parents reinforce increased decision making by the client.
B. The parents clearly verbalize their expectations for the client.
C. The client verbalizes that family meals are now enjoyable.
D. The client tells her parents about feelings of low self-esteem.

Incorrect Correct Answer: A. The parents reinforce increased decision making by
the client.
One of the core issues concerning the family of a client with anorexia is control. The
family’s acceptance of the client’s ability to make independent decisions is key to
successful family intervention. Reinforce the importance of parents as a couple who have
rights of their own. The focus on the child with anorexia is very intense and often is the
only area around which the couple interacts. The couple needs to explore their own
relationship and restore the balance within it to prevent its disintegration.
 Option B: Identify patterns of interaction. Encourage each family member to
speak for self. Do not allow two members to discuss a third without that
member’s participation. Helpful information for planning interventions. The
enmeshed, over-involved family members often speak for each other and need to
learn to be responsible for their own words and actions.
 Option C: Make a selective menu available, and allow the patient to control
choices as much as possible. Patient who gains confidence in herself and feels in
control of the environment is more likely to eat preferred foods. Involve patients
in setting up or carrying out a program of behavior modification. Provide a reward
for weight gain as individually determined; ignore the loss. Provides structured
eating situations while allowing the patient some control in choices. Behavior
modification may be effective in mild cases or for short-term weight gain.
 Option D: Encourage the patient to express anger and acknowledge when it is
verbalized. Important to know that anger is part of self and as such is acceptable.
Expressing anger may need to be taught to the patient because anger is generally
considered unacceptable in the family, and therefore the patient does not express
it. Although the remaining options may occur during the process of therapy they
would not necessarily indicate a successful outcome; the central family issues of
dependence and independence are not addressed in these responses.

 8. Question 1 point(s)

The nurse is working with a client with a somatoform disorder. Which client outcome
goal would the nurse most likely establish in this situation?

 A. The client will recognize signs and symptoms of physical illness.


B. The client will cope with physical illness.
C. The client will take prescribed medications.
D. The client will express anxiety verbally rather than through physical
symptoms.

Correct Correct Answer: D. The client will express anxiety verbally rather than
through physical symptoms.
The client with a somatoform disorder displaces anxiety into physical symptoms. The
ability to express anxiety verbally indicates a positive change toward improved health.
These disorders should be considered early in the evaluation of patients with unexplained
symptoms to prevent unnecessary interventions and testing. Up to 50 percent of primary
care patients present with physical symptoms that cannot be explained by a general
medical condition. Some of these patients meet criteria for somatoform disorders.
 Option A: The unexplained symptoms of somatoform disorders often lead to
general health anxiety; frequent or recurrent and excessive preoccupation with
unexplained physical symptoms; inaccurate or exaggerated beliefs about somatic
symptoms; difficult encounters with the health care system; disproportionate
disability; displays of strong, often negative emotions toward the physician or
office staff; unrealistic expectations; and, occasionally, resistance to or
noncompliance with diagnostic or treatment efforts. These behaviors may result in
more frequent office visits, unnecessary laboratory or imaging tests, or costly and
potentially dangerous invasive procedures.
 Option B: The challenge in working with somatoform disorders in the primary
care setting is to simultaneously exclude medical causes for physical symptoms
while considering a mental health diagnosis. The diagnosis of a somatoform
disorder should be considered early in the process of evaluating a patient with
unexplained physical symptoms. Appropriate nonpsychiatric medical conditions
should be considered, but over-evaluation and unnecessary testing should be
avoided.
 Option C: Studies supporting the effectiveness of pharmacologic interventions
targeting specific somatoform disorders are limited. Antidepressants are
commonly used to treat depressive or anxiety disorders and may be part of the
approach to treating the comorbidities of somatoform disorders. Antidepressants
such as fluvoxamine (Luvox, brand not available) for treating body dysmorphic
disorder, and St. John’s wort for treating somatization and undifferentiated
somatoform disorders have been proposed.

 9. Question 1 point(s)

Which method would a nurse use to determine a client’s potential risk for suicide?

 A. Wait for the client to bring up the subject of suicide.


B. Observe the client’s behavior for cues of suicide ideation.
C. Question the client directly about suicidal thoughts.
D. Question the client about future plans.

Incorrect Correct Answer: C. Question the client directly about suicidal thoughts.
Directly questioning a client about suicide is important to determine suicide risk. A host
of thoughts and behaviors are associated with self-destructive acts. Although many
assume that people who talk about suicide will not follow through with it, the opposite is
true; a threat of suicide can lead to the completed act, and suicidal ideation is highly
correlated with suicidal behaviors. A clear and complete evaluation and clinical interview
provide the information upon which to base a suicide intervention. Although risk factors
offer major indications of the suicide danger, nothing can substitute for a focused patient
inquiry. However, although all the answers a patient gives may be inclusive, a therapist
often develops a visceral sense that his or her patient is going to commit suicide. The
clinician’s reaction counts and should be considered in the intervention.
 Option A: The client may not bring up this subject for several reasons, including
guilt regarding suicide, wishing not to be discovered, and his lack of trust in staff.
Determine whether the person has any thoughts of hurting him or herself. Suicidal
ideation is highly linked to completed suicide. Some inexperienced clinicians
have difficulty asking this question. They fear the inquiry may be too intrusive or
that they may provide the person with an idea of suicide. In reality, patients
appreciate the question as evidence of the clinician’s concern. A positive response
requires further inquiry.
 Option B: Behavioral cues are important, but direct questioning is essential to
determine suicide risk. If suicidal ideation is present, the next question must be
about any plans for suicidal acts. The general formula is that more specific plans
indicate greater danger. Although vague threats, such as a threat to commit
suicide sometime in the future, are the reason for concern, responses indicating
that the person has purchased a gun, has ammunition, has made out a will, and
plans to use the gun are more dangerous. The plan demands further questions. If
the person envisions a gun-related death, determine whether he or she has the
weapon or access to it.
 Option D: Indirect questions convey to the client that the nurse is not comfortable
with the subject of suicide and, therefore, the client may be reluctant to discuss
the topic. Determine what the patient believes his or her suicide would achieve.
This suggests how seriously the person has been considering suicide and the
reason for death. For example, some believe that their suicide would provide a
way for family or friends to realize their emotional distress. Others see their death
as a relief from their own psychic pain. Still others believe that their death would
provide a heavenly reunion with a departed loved one. In any scenario, the
clinician has another gauge of the seriousness of the planning.

 10. Question 1 point(s)

A client with a bipolar disorder exhibits manic behavior. The nursing diagnosis is
Disturbed thought processes related to difficulty concentrating, secondary to flight of
ideas. Which of the following outcome criteria would indicate improvement in the client?

 A. The client verbalizes feelings directly during treatment.


B. The client verbalizes a positive “self” statement.
C. The client speaks in coherent sentences.
D. The client reports feelings calmer.

Incorrect Correct Answer: C. The client speaks in coherent sentences
A client exhibiting flight of ideas typically has a continuous speech flow and jumps from
one topic to another. Speaking in coherent sentences is an indicator that the client’s
concentration has improved and his thoughts are no longer racing. The defining
characteristics of mania are increased talkativeness, rapid speech, decreased the need for
sleep (unlike depression or anxiety in which the need for sleep exists, but there is an
inability to sleep), racing thoughts, distractibility, increase in goal-directed activity, and
psychomotor agitation.
 Option A: Some other hallmarks of mania are an elevated or expansive mood,
mood lability, impulsivity, irritability, and grandiosity. If the individual
experiencing these symptoms requires hospitalization, then this period
automatically qualifies as true mania and not hypomania, even if the symptoms
are present for less than one week.
 Option B: Mania must be distinguished from heightened energy and altered
functioning that arises from substance use, medical conditions or other causes.
Mania is a “natural” state which is the characteristic of bipolar I disorder. A single
manic phase is sufficient to make the diagnosis of bipolar I disorder, although
most cases of bipolar I also involve hypomanic and depressed episodes.
 Option D: Many families bring their loved ones to the emergency room due to
the excessive behavioral changes they have noticed over a brief period. Patients
amid a manic phase commonly engage in goal-directed activities that may result
in harmful consequences, such as spending excessive money, starting businesses
unprepared, traveling, or promiscuity.

 11. Question 1 point(s)

A client tells a nurse. “Everyone would be better off if I wasn’t alive.” Which nursing
diagnosis would be made based on this statement?

 A. Disturbed thought processes


B. Ineffective coping
C. Risk for self-directed violence
D. Impaired social interaction

Incorrect Correct Answer: C. Risk for self-directed violence
The nurse should take any nurse statements indicating suicidal thoughts seriously and
further assess for other risk factors. The early identification and appropriate treatment of
mental disorders is an important prevention strategy – especially given the relevant
contribution of depression and other psychiatric problems to suicidal behavior. Equally
important is early identification and treatment for people with alcohol and substance
abuse problems.
 Option A: A variety of stressful events or circumstances can put people at
increased risk of harming themselves including the loss of loved ones,
interpersonal conflicts with family or friends, and legal or work-related problems.
To act as precipitating factors for suicide, though, they must happen to someone
who is predisposed or otherwise especially vulnerable to self-harm. The early
identification and appropriate treatment of mental disorders is an important
prevention strategy – especially given the relevant contribution of depression and
other psychiatric problems to suicidal behavior. Equally important is early
identification and treatment for people with alcohol and substance abuse
problems.
 Option B: People who are suicidal generally express difficulty in solving
problems. Behavioral therapy approaches are designed to probe underlying factors
and to help patients develop problem-solving skills. While conclusive answers are
not yet known, there is some evidence to suggest that behavioral therapy
approaches are effective in reducing suicidal thoughts and behavior.
 Option D: Certain social and environmental factors also increase the likelihood
of suicide. Rates of suicide, for instance, are higher during economic recessions
and periods of high unemployment. They are also higher during periods of social
disintegration, political instability, and social collapse.

 12. Question 1 point(s)

Which information is the most essential in the initial teaching session for the family of a
young adult recently diagnosed with schizophrenia?

 A. Symptoms of this disease imbalance in the brain.


B. Genetic history is an important factor related to the development of
schizophrenia.
C. Schizophrenia is a serious disease affecting every aspect of a person’s
functioning.
D. The distressing symptoms of this disorder can respond to treatment with
medications.

Correct Correct Answer: D. The distressing symptoms of this disorder can respond
to treatment with medications.
This statement provides accurate information and an element of hope for the family of a
schizophrenic client. For the initial treatment of acute psychosis, it is recommended to
commence an oral second-generation antipsychotic (SGA) such as aripiprazole,
olanzapine, risperidone, quetiapine, asenapine, lurasidone, sertindole, ziprasidone,
brexpiprazole, molindone, iloperidone, etc. Sometimes, if clinically needed, alongside a
benzodiazepine such as diazepam, clonazepam, or lorazepam to control behavioral
disturbances and non-acute anxiety. First-generation antipsychotics (FGA) like
trifluoperazine, Fluphenazine, haloperidol, pimozide, sulpiride, flupentixol,
chlorpromazine, etc. are not commonly used as the first line but can be used.
 Option A: There are also arguments that schizophrenia is a neurodevelopmental
disorder based on abnormalities present in the cerebral structure, an absence of
gliosis suggesting in utero changes, and the observation that motor and cognitive
impairments in patients precede the illness onset.
 Option B: Several studies postulate that the development of schizophrenia results
from abnormalities in multiple neurotransmitters, such as dopaminergic,
serotonergic, and alpha-adrenergic hyperactivity or glutaminergic and GABA
hypoactivity. Genetics also play a fundamental role – there is a 46% concordance
rate in monozygotic twins and a 40% risk of developing schizophrenia if both
parents are affected. The gene neuregulin (NGR1) which is involved in glutamate
signaling and brain development has been implicated, alongside dysbindin
(DTNBP1) which helps glutamate release, and catecholamine O-
methyltransferase (COMT) polymorphism, which regulates dopamine function.
 Option C: Although the remaining statements are true, they do not provide the
empathic response the family needs after just learning about the diagnosis. These
facts can become part of the ongoing teaching. The first schizophrenic episode
usually occurs during early adulthood or late adolescence. Individuals often lack
insight at this stage; therefore few will present directly to seek help for their
psychotic symptoms. Common presentations include a relative noticing social
withdrawal, personality changes, or uncharacteristic behavior; deliberate self-
harm or suicide attempts; calling the police to report their delusional symptoms,
or referral via the criminal justice system.

 13. Question 1 point(s)

A nurse is working with a client who has schizophrenia, paranoid type. Which of the
following outcomes related to the client’s delusional perceptions would the nurse
establish?

 A. The client will demonstrate realistic interpretation of daily events in the unit.
B. The client will perform daily hygiene and grooming without assistance.
C. The client will take prescribed medications without difficulty.
D. The client will participate in unit activities.

Incorrect Correct Answer: A. The client will demonstrate realistic interpretation of
daily events in the unit.
A client with schizophrenia, paranoid type, has distorted perceptions and views people,
institutions, and aspects of the environment as plotting against him. The desired outcome
for someone with delusional perceptions would be to have a realistic interpretation of
daily events. Unlike DSM-5, ICD-10 further subcategories schizophrenia based on the
key presenting symptoms as either paranoid schizophrenia, hebephrenic schizophrenia,
catatonic schizophrenia, undifferentiated schizophrenia, post-schizophrenic depression,
residual schizophrenia, and simple schizophrenia.
 Option B: The client with a distorted perception of the environment would not
necessarily have impairments affecting hygiene and grooming skills. A thorough
risk assessment must also be undertaken to determine the risk of harm to self and
others. The first schizophrenic episode usually occurs during early adulthood or
late adolescence. Individuals often lack insight at this stage; therefore few will
present directly to seek help for their psychotic symptoms.
 Option C: For the initial treatment of acute psychosis, it is recommended to
commence an oral second-generation antipsychotic (SGA) such as aripiprazole,
olanzapine, risperidone, quetiapine, asenapine, lurasidone, sertindole, ziprasidone,
brexpiprazole, molindone, iloperidone, etc. Sometimes, if clinically needed,
alongside a benzodiazepine such as diazepam, clonazepam or lorazepam to
control behavioral disturbances and non-acute anxiety. First generation
antipsychotic (FGA) like trifluoperazine, Fluphenazine, haloperidol, pimozide,
sulpiride, flupentixol, chlorpromazine, etc. are not commonly used as the first line
but can be used.
 Option D: Although taking medications and participating in unit activities may
be appropriate outcomes for nursing intervention; these responses are not related
to client perceptions. Cognitive-behavioral therapy (CBT) and the use of art and
drama therapies help counteract the negative symptoms of the disease, improve
insight, and assist relapse prevention.

 14. Question 1 point(s)

A client with bipolar disorder, manic type, exhibits extreme excitement, delusional
thinking, and command hallucinations. Which of the following is the priority nursing
diagnosis?

 A. Anxiety
B. Impaired social interaction
C. Disturbed sensory-perceptual alteration (auditory)
D. Risk for other-directed violence

Correct Correct Answer: D. Risk for other-directed violence
A client with these symptoms would have poor impulse control and would therefore be
prone to acting-out behavior that may be harmful to either himself or others. All of the
remaining nursing diagnoses may apply to the client with mania; however, the priority
diagnosis would be risk for violence. Mania, or a manic phase, is a period of 1 week or
more in which a person experiences a change in normal behavior that drastically affects
their functioning.
 Option A: The defining characteristics of mania are increased talkativeness, rapid
speech, decreased the need for sleep (unlike depression or anxiety in which the
need for sleep exists, but there is an inability to sleep), racing thoughts,
distractibility, increase in goal-directed activity, and psychomotor agitation. Some
other hallmarks of mania are an elevated or expansive mood, mood lability,
impulsivity, irritability, and grandiosity. If the individual experiencing these
symptoms requires hospitalization, then this period automatically qualifies as true
mania and not hypomania, even if the symptoms are present for less than one
week.
 Option B: Many families bring their loved ones to the emergency room due to the
excessive behavioral changes they have noticed over a brief period. Patients amid
a manic phase commonly engage in goal-directed activities that may result in
harmful consequences, such as spending excessive money, starting businesses
unprepared, traveling, or promiscuity. Many patients engage in property damage
or even harm themselves or others through verbal or physical assaults. They may
also become highly aggressive, agitated, or irritable.
 Option C: Mania also commonly presents with psychotic features, which include
delusions or hallucinations. Many patients endorse grandiose delusions, believing
they are high-level operatives such as spies, government officials, members of
secret agencies, or that they are knowledgeable professionals (even when they
have no such background). These individuals may also experience auditory or
visual hallucinations, which only present when they are in the manic phases.

 15. Question 1 point(s)

A client who abuses alcohol and cocaine tells a nurse that he only uses substances
because of his stressful marriage and difficult job. Which defense mechanisms is this
client using?

 A. Displacement
B. Projection
C. Rationalization
D. Sublimation

Incorrect Correct Answer: C. Rationalization
Rationalization is the defense mechanism that involves offering excuses for maladaptive
behavior. The client is defending his substance abuse by providing reasons related to life
stressors. This is a common defense mechanism used by clients with substance abuse
problems.
 Option A: Displacement is transferring one’s emotional burden or emotional
reaction from one entity to another. This defense mechanism may be present in
someone who has a stressful day at work and then lashes out against their family
at home.
 Option B: Projection is attributing one’s own maladaptive inner impulses to
someone else. For example, someone who commits an episode of infidelity in
their marriage may then accuse their partner of infidelity or may become more
suspicious of their partner.
 Option D: Sublimation is transforming one’s anxiety or emotions into pursuits
that are considered by societal or cultural norms to be more useful. This defense
mechanism may be present in someone who channels their aggression and energy
into playing sports.

 16. Question 1 point(s)

An 11-year-old child diagnosed with conduct disorder is admitted to the psychiatric unit
for treatment. Which of the following behaviors would the nurse assess?
 A. Restlessness, short attention span, hyperactivity.
B. Physical aggressiveness, low-stress tolerance, disregard for the rights of others.

C. Deterioration in social functioning, excessive anxiety, and worry, bizarre


behavior.
D. Sadness, poor appetite and sleeplessness, loss of interest in activities.

Incorrect Correct Answer: B. Physical aggressiveness, low-stress tolerance
disregard for the rights of others
Physical aggressiveness, low-stress tolerance, and a disregard for the rights of others are
common behaviors in clients with conduct disorders. Conduct disorder (CD) is classified
in the spectrum of disruptive behavior disorders which also includes the diagnosis of
oppositional defiant disorder (ODD). Exhibits a pattern of behavior that violates the
rights of others and disregards social norms.
 Option A: Restlessness, short attention span, and hyperactivity are typical
behaviors in a client with attention deficit hyperactivity disorder. Attention
Deficit-Hyperactivity Disorder (ADHD) is a psychiatric condition that has long
been recognized as affecting children’s ability to function. Individuals suffering
from this disorder show patterns of developmentally inappropriate levels of
inattentiveness, hyperactivity, or impulsivity.
 Option C: Deterioration in social functioning, excessive anxiety and worry and
bizarre behaviors are typical in schizophrenic disorders. Derived from the Greek
‘schizo’ (splitting) and ‘phren’ (mind) with the term first coined by Eugen Bleuler
in 1908, schizophrenia is a functional psychotic disorder characterized by the
presence of delusional beliefs, hallucinations, and disturbances in thought,
perception, and behavior.
 Option D: Sadness, poor appetite, sleeplessness, and loss of interest in activities
are behaviors commonly seen in depressive disorders. Depression is a mood
disorder that causes a persistent feeling of sadness and loss of interest. The
common features of all depressive disorders are sadness, emptiness, or irritable
mood, accompanied by somatic and cognitive changes that significantly affect the
individual’s capacity to function.

 17. Question 1 point(s)

The nurse understands that if a client continues to be dependent on heroin throughout her
pregnancy, her baby will be at high risk for:A. Mental retardation

 A. Mental retardation
B. Heroin dependence
C. Addiction in adulthood
D. Psychological disturbances

Incorrect Correct Answer: B. Heroin dependence
Babies born to heroin-dependent women are also heroin-dependent and need to go
through withdrawal. Heroin use during pregnancy can result in neonatal abstinence
syndrome (NAS). NAS occurs when heroin passes through the placenta to the fetus
during pregnancy, causing the baby to become dependent, along with the mother.
Symptoms include excessive crying, fever, irritability, seizures, slow weight gain,
tremors, diarrhea, vomiting, and possibly death. There is no evidence to support any of
the remaining answer choices.
 Option A: NAS requires hospitalization and treatment with medication (often
morphine) to relieve symptoms; the medication is gradually tapered off until the
baby adjusts to being opioid-free. Methadone maintenance combined with
prenatal care and a comprehensive drug treatment program can improve many of
the outcomes associated with untreated heroin use for both the infant and mother,
although infants exposed to methadone during pregnancy typically require
treatment for NAS as well.
 Option C: A NIDA-supported clinical trial demonstrated that buprenorphine
treatment of opioid-dependent mothers is safe for both the unborn child and the
mother. Once born, these infants require less morphine and shorter hospital stays
compared to infants born of mothers on methadone maintenance treatment.23
Research also indicates that buprenorphine combined with naloxone (compared to
a morphine taper) is equally safe for treating babies born with NAS, further
reducing side effects experienced by infants born to opioid-dependent mothers.
 Option D: Depending on the drug the baby is withdrawing from, common signs
are: excessive crying, tremors, and jitteriness; poor feeding, vomiting, and
swallowing; inability to settle and sleep; or trouble with breathing. If you are
treated with methadone or buprenorphine during pregnancy, your

 18. Question 1 point(s)

The emergency department nurse is assigned to provide care for a victim of a sexual
assault. When following legal and agency guidelines, which intervention is most
important?

 A. Determine the assailant’s identity


B. Preserve the client’s privacy
C. Identify the extent of an injury
D. Ensure an unbroken chain of evidence

Correct Correct Answer: D. Ensure an unbroken chain of evidence
Establishing an unbroken chain of evidence is essential in order to ensure that the
prosecution of the perpetrator can occur. Explain the forensic specimens you plan to
collect; inform the client that they can be used for identification and prosecution of the
rapist, for example blood, combing pubic hairs, semen samples, skin from underneath
nails.
 Option A: Arrange for support follow-up: crisis counseling, group therapy,
individual therapy, rape counselor, or a support group. Many individuals carry
with them constant emotional distress and trauma. Depression and suicidal
ideation are frequent sequelae of rape. As soon as the intervention is carried out,
the less complicated the recovery may be.
 Option B: The nurse will also need to preserve the client’s privacy and identify
the extent of an injury. However, it is essential that the nurse follows legal and
agency guidelines for preserving evidence. Provide strict confidentiality. The
client’s situation is not to be talked over with anyone other than the medical staff
involved unless the client gives consent to it.
 Option C: Identifying the assailant is the job of law enforcement, not the nurse.
Approach the client in a nonjudgmental manner. Nurses’ attitudes can have an
important therapeutic impact. Displays of shock, horror, disgust, or disbelief are
not appropriate. Never use judgmental language.

 19. Question 1 point(s)

Which factor is least important in the decision regarding whether a victim of family
violence can safely remain in the home?

 A. The availability of appropriate community shelters.


B. The non-abusing caretaker’s ability to intervene on the client’s behalf.
C. The client’s possible response to relocation.
D. The family’s socioeconomic status.

Correct Correct Answer: D. The family’s socioeconomic status
Socioeconomic status is not a reliable predictor of abuse in the home so that it would be
the least important consideration in deciding issues of safety for the victim of family
violence. Family and domestic violence (including child abuse, intimate partner abuse,
and elder abuse) is a common problem in the United States. Family and domestic health
violence are estimated to affect 10 million people in the United States every year. It is a
national public health problem, and virtually all healthcare professionals will at some
point evaluate or treat a patient who is a victim of some form of domestic or family
violence.
 Option A: Unfortunately, each form of family violence begets interrelated forms
of violence, and the “cycle of abuse” is often continued from exposed children
into their adult relationships, and finally to the care of the elderly. Domestic and
family violence includes a range of abuse including economic, physical, sexual,
emotional, and psychological toward children, adults, and elders. If the patient
elects to leave their current situation, information for referral to a local domestic
violence shelter to assist the victim should be given.
 Option B: The ability of the non-abusing caretaker to intervene on the client’s
behalf are important factor when making safety decisions. Patients that have
suffered domestic violence may or may not want a referral. Many are fearful of
their lives and financial well-being and hence may be weighing the tradeoff in
leaving the abuser leading to loss of support and perhaps the responsibility of
caring for children alone. The healthcare provider needs to assure the patient that
the decision is voluntary and that the provider will help regardless of the decision.
The goal is to make resources accessible, safe, and to enhance support.
 Option C: The client’s response to possible relocation (if the client is a
competent adult) would be the most important factor to consider; feelings of
empowerment and being treated as a competent person can help a client feel less
like a victim. If the patient does not want to go to a shelter, provide telephone
numbers for domestic violence or crisis hotlines and support services for potential
later use. Provide the patient with instructions but be mindful that written
materials may pose a danger once the patient returns home.

 20. Question 1 point(s)

The nurse would expect a client with early Alzheimer’s disease to have problems with:

 A. Balancing a checkbook
B. Self-care measures
C. Relating to family members
D. Remembering his own name

Incorrect Correct Answer: A. Balancing a checkbook
In the early stage of Alzheimer’s disease, complex tasks (such as balancing a checkbook)
would be the first cognitive deficit to occur. Alzheimer’s disease (AD) is the most
common type of dementia, accounting for at least two-thirds of cases of dementia in
people age 65 and older. Alzheimer’s disease is a neurodegenerative disease with
insidious onset and progressive impairment of behavioral and cognitive functions
including memory, comprehension, language, attention, reasoning, and judgment.
 Option B: Difficulty performing learned motor tasks (dyspraxia), olfactory
dysfunction, sleep disturbances, extrapyramidal motor signs like dystonia,
akathisia, and parkinsonian symptoms occur late in the disease. This is followed
by primitive reflexes, incontinence, and total dependence on caregivers.
 Option C: In the early stages, impairment in executive functioning ranges from
subtle to significant. This is followed by language disorder and impairment of
visuospatial skills. Neuropsychiatric symptoms like apathy, social withdrawal,
disinhibition, agitation, psychosis, and wandering are also common in the mid to
late stages.
 Option D: Symptoms of Alzheimer’s disease depend on the stage of the disease.
Alzheimer’s disease is classified into preclinical or presymptomatic, mild, and
dementia-stage depending on the degree of cognitive impairment. These stages
are different from the DSM-5 classification of Alzheimer’s disease. The initial
and most common presenting symptom is episodic short-term memory loss with
relative sparing of long-term memory and can be elicited in most patients even
when not the presenting symptom.

 21. Question 1 point(s)

Which nursing intervention is most appropriate for a client with Alzheimer’s disease who
has frequent episodes of emotional lability?

 A. Attempt humor to alter the client's mood.


B. Explore reasons for the client’s altered mood.
C. Reduce environmental stimuli to redirect the client’s attention.
D. Use logic to point out reality aspects.

Incorrect Correct Answer: C. Reduce environmental stimuli to redirect the client’s
attention.
The client with Alzheimer’s disease can have frequent episodes of labile mood, which
can best be handled by decreasing a stimulating environment and redirecting the client’s
attention. Maintain a nice quiet neighborhood. Noise, crowds, the crowds are usually the
excessive sensory neurons and can increase interference.
 Option A: The client with Alzheimer’s disease loses the cognitive ability to
respond to either humor or logic. Assess the level of cognitive disorders such as a
change to orientation to people, places and times, range, attention, thinking skills.
It provides the basis for the evaluation or comparison that will come and
influencing the choice of intervention.
 Option B: An over-stimulating environment may cause a labile mood, which will
be difficult for the client to understand. Maintain consistent scheduling with
allowances for patient’s specific needs, and avoid frustrating situations and
overstimulation. It prevents patient agitation, erratic behaviors, and combative
reactions. Scheduling may need revision to show respect for the patient’s sense of
worth and to facilitate the completion of tasks.
 Option D: The client lacks any insight into his or her own behavior and therefore
will be unaware of any causative factors. Assist with establishing cues and
reminders for patient’s assistance. Assists patients with early AD to remember the
location of articles and facilitates some orientation.

 22. Question 1 point(s)

Which neurotransmitter has been implicated in the development of Alzheimer’s disease?

 A. Acetylcholine
B. Dopamine
C. Epinephrine
D. Serotonin

Incorrect Correct Answer: A. Acetylcholine
A relative deficiency of acetylcholine is associated with this disorder. The drugs used in
the early stages of Alzheimer’s disease will act to increase available acetylcholine in the
brain. The remaining neurotransmitters have not been implicated in Alzheimer’s disease.
Cholinergic neurons located in the basal forebrain, including the neurons that form the
nucleus basalis of Meynert, are severely lost in Alzheimer’s disease (AD). AD is the most
ordinary cause of dementia affecting 25 million people worldwide. The hallmarks of the
disease are the accumulation of neurofibrillary tangles and amyloid plaques.
 Option B: Acetylcholine (ACh) was the first neurotransmitter to be identified.
ACh is the neurotransmitter used by all cholinergic neurons, which has a very
important role in the peripheral and central nervous systems. All pre- and
postganglionic parasympathetic neurons and all preganglionic sympathetic
neurons use ACh as a neurotransmitter. In addition, part of the postganglionic
sympathetic neurons also uses ACh as a neurotransmitter.
 Option C: Given its widespread distribution in the brain, it is not surprising that
cholinergic neurotransmission is responsible for modulating important neural
functions. The cholinergic system is involved in critical physiological processes,
such as attention, learning, memory, stress response, wakefulness and sleep, and
sensory information.
 Option D: It has been demonstrated that the cholinergic system plays a role in the
learning process. Moreover, published data indicate that ACh is involved in
memory. Further studies have demonstrated that endogenous acetylcholine is
important for modulation of acquisition, encoding, consolidation, reconsolidation,
extinction, and retrieval of memory.

 23. Question 1 point(s)

Which factors are the most essential for the nurse to assess when providing crisis
intervention for a client?

 A. The client’s communication and coping skills.


B. The client’s anxiety level and ability to express feelings.
C. The client’s perception of the triggering event and availability of situational
supports.
D. The client’s use of reality testing and level of depression.

Incorrect Correct Answer: C. The client’s perception of the triggering event and
availability of situational supports
The most important factors to determine in these situations are the client’s perception of
the crisis event and the availability of support (including family and friends) to provide
basic needs. Crisis intervention is a short-term management technique designed to reduce
potential permanent damage to an individual affected by a crisis. A crisis is defined as an
overwhelming event, which can include divorce, violence, the passing of a loved one, or
the discovery of a serious illness.
 Option A: A successful intervention involves obtaining background information
on the patient, establishing a positive relationship, discussing the events, and
providing emotional support. SAFER-R is a common intervention model used,
which consists of stabilization, acknowledgment, facilitate understanding,
encouragement, recovery, and referral. SAFER-R helps patients return to their
mental baseline following a crisis.
 Option B: In these cases, psychological crisis intervention is necessary to prevent
traumatized victims from developing illnesses. It also alleviates stress upon
healthcare workers so that they can continue helping others. Another major
concern is what coping strategies are most effective. Social support and problem-
solving planning are effective coping mechanisms that are frequently used by
school staff following a crisis.
 Option D: Although the nurse should assess the other factors, they are not as
essential as determining why the client considers this a crisis and whether he can
meet his present needs. The use of humor, emotional support, planning, and
acceptance also correlate with superior mental health outcomes compared to
substance abuse and denial. Positive coping mechanisms, such as the ones listed
above, are reported to be effective in crisis management, and with crisis
intervention services in place, people will be better equipped to handle
unexpected events.

 24. Question 1 point(s)

The nurse considers a client’s response to crisis intervention successful if the client:

 A. Changes coping skills and behavioral patterns.


B. Develops insight into reasons why the crisis occurred.
C. Learns to relate better to others.
D. Returns to his previous level of functioning.

Correct Correct Answer: D. Returns to his previous level of functioning.
Crisis intervention is based on the idea that a crisis is a disturbance in homeostasis
(steady state). The goal is to help the client return to a previous level of equilibrium in
functioning. Based on prior studies, it is evident that crisis intervention plays a significant
role in enhancing outcomes in psychiatric cases. Community Mental Health Centers and
local government agencies often have crisis intervention teams that provide support to the
local community at times of mental health crisis. These teams can also be helpful at times
of natural or man-made emergencies.
 Option A: Another major concern is what coping strategies are most effective.
Social support and problem-solving planning are effective coping mechanisms
that are frequently used by school staff following a crisis. The use of humor,
emotional support, planning, and acceptance also correlate with superior mental
health outcomes compared to substance abuse and denial. Positive coping
mechanisms, such as the ones listed above, are reported to be effective in crisis
management, and with crisis intervention services in place, people will be better
equipped to handle unexpected events.
 Option B: Based on prior studies, it is evident that crisis intervention plays a
significant role in enhancing outcomes in psychiatric cases. Community Mental
Health Centers and local government agencies often have crisis intervention
teams that provide support to the local community at times of mental health crisis.
These teams can also be helpful at times of natural or man-made emergencies.
Crisis intervention teams often assess and triage the situation and can diffuse the
situation and triage for urgent attention of medical or mental health personnel in
emergency or community care settings. They can call upon local police and other
community resources for additional support.
 Option C: There are many approaches to integrating crisis intervention, and a
member of the healthcare team can complete each step. First responders can triage
and assess the situation and administer psychological first aid as needed to victims
of a traumatic event to prevent any long-term mental health problems. This
approach allows immediate access to crisis intervention, which will facilitate care
and lead to improved outcomes. In a hospital setting, the needs of a patient in
crisis should be well communicated throughout the management team.

 25. Question 1 point(s)

Two nurses are co-leading group therapy for seven clients in the psychiatric unit. The
leaders observe that the group members are anxious and look to the leaders for answers.
Which phase of development is this group in?

 A. Conflict resolution phase


B. Initiation phase
C. Working phase
D. Termination phase

Incorrect Correct Answer: B. Initiation phase
Increased anxiety and uncertainty characterize the initiation phase in group therapy.
Group members are more self-reliant during the working and termination phases. During
the beginning phase of group therapy, issues arise around topics such as orientation,
beginners’ anxiety, and the role of the leader. The purpose of the group is articulated,
working conditions of the group are established, members are introduced, a positive tone
is set for the group, and group work begins. This phase may last from 10 minutes to a
number of months. In a revolving group, this orientation will happen each time a new
member joins the group.
 Option A: The group is a forum where clients interact with others. In this give
and take of therapy, clients receive feedback that helps them rethink their
behaviors and move toward productive changes. The leader helps group members
by allocating time to address the issues that arise, by paying attention to relations
among group members, and by modeling a healthy interactional style that
combines honesty with compassion.
 Option C: The group in its middle phase encounters and accomplishes most of
the actual work of therapy. During this phase, the leader balances content, which
is the information and feelings overtly expressed in the group, and process, which
is how members interact in the group. The therapy is in both the content and
process. Both contribute to the connections between and among group members,
and it is those connections that are therapeutic.
 Option D: Termination is a particularly important opportunity for members to
honor the work they have done, to grieve the loss of associations and friendships,
and to look forward to a positive future. Group members should learn and practice
saying “good?bye,” understanding that it is necessary to make room in their lives
for the next “hello.”

 26. Question 1 point(s)

Group members have worked very hard, and the nurse reminds them that termination is
approaching. Termination is considered successful if group members:

 A. Decide to continue
B. Elevate group progress
C. Focus on positive experience
D. Stop attending prior to termination

Incorrect Correct Answer: A. Decide to continue
As the group progresses into the working phase, group members assume more
responsibility for the group. The leader becomes more of a facilitator. Comments about
behavior in a group are indicators that the group is active and involved. In this phase, the
LPN and client evaluate the client’s response to treatment and explore the meaning of the
relationship and what goals have been achieved. Discussing the achievements, how the
client and LPN feel about concluding the relationship, and plans for the future are an
important part of the termination phase.
 Option B: Termination of a meaningful nurse-client relationship should be final
in any setting. To provide the client with even a hint that the relationship will
continue is inappropriate, unprofessional, and unethical; for example, the LPN
informs the client that he/she may contact the client on social media to check on
their condition after discharge.
 Option C: Corresponding to the implementation phase of the nursing process, the
working phase focuses on self-direction and self-management to whatever extent
possible in promoting the client’s health and wellbeing; for example, the LPN
provides information and teaching to a client with diabetes about both the
importance of proper nutrition and how eating healthy will benefit the client long
term with regards to blood glucose levels. Because of teaching, the client decides
not to eat the chocolate bar and chooses to eat the apple instead.
 Option D: When the client stops attending the group, termination is considered
unsuccessful. Every nurse-client relationship, regardless of circumstance, is based
on trust, respect, and professional integrity. It requires the appropriate use of
authority or power. The LPN must work with the client toward achieving the
client’s goals and ensure that the client receives safe competent care. The LPN
utilizes a caring attitude and behaviors to meet the needs of the client.

 27. Question 1 point(s)

The nurse is teaching a group of clients about the mood-stabilizing medications lithium
carbonate. Which medications should she instruct the clients to avoid because of the
increased risk of lithium toxicity?

 A. Antacids
B. Antibiotics
C. Diuretics
D. Hypoglycemic agents

Incorrect Correct Answer: C. Diuretics
The use of diuretics would cause sodium and water excretion, which would increase the
risk of lithium toxicity. Clients taking lithium carbonate should be taught to increase their
fluid intake and to maintain normal intake of sodium. Treatment for lithium toxicity is
primarily hydration and to stop the drug. Give hydration with normal saline, which will
also enhance lithium excretion. Avoid all diuretics. If the patient has severe renal
dysfunction or failure, or severely altered mental status, then start with hemodialysis. 20
to 30 mg of propranolol given 2 to 3 times per day may help reduce tremors.
 Option A: Antacids are a combination of various compounds with various salts of
calcium, magnesium, and aluminum as the active ingredients. The antacids act by
neutralizing the acid in the stomach and by inhibiting pepsin, which is a
proteolytic enzyme. Each of these cationic salts has a characteristic
pharmacological property that determines its clinical use.
 Option B: The pharmacology behind antibiotics includes destroying the bacterial
cell by either preventing cell reproduction or changing a necessary cellular
function or process within the cell. Antimicrobial agents are classically grouped
into 2 main categories based on their in vitro effect on bacteria: bactericidal and
bacteriostatic.
 Option D: FDA approved indications for the use of oral hypoglycemic drugs
include type 2 diabetes mellitus. Non-FDA approved indications of oral
hypoglycemic drugs, such as metformin, are for the prevention of type 2 diabetes
mellitus, treatment of gestational diabetes mellitus, treatment of polycystic ovary
syndrome (PCOS) with menstrual irregularities, and prevention of ovarian
hyperstimulation syndrome in PCOS patients undergoing intracytoplasmic sperm
injection (ICSI) or in vitro fertilization (IVF), and management of antipsychotic-
induced weight gain.

 28. Question 1 point(s)

When providing family therapy, the nurse analyzes the functioning of healthy family
systems. Which situations would not increase stress on a healthy family system?
 A. An adolescent’s going away to college
B. The birth of a child
C. The death of a grandparent
D. Parental disagreement

Correct Correct Answer: D. Parental disagreement
In a functional family, parents typically do not agree on all issues and problems. Open
discussion of thoughts and feelings is healthy, and parental disagreement should not
cause system stress. Families that eat together regularly communicate (as long as the
phones and TVs are turned off). They like to share feelings with each other and cue into
each other’s feelings. Put-downs and sarcasm is rare.
 Option A: A crisis can sometimes be quite obvious, such as a person losing his or
her job, getting divorced, or being involved in some type of accident. In other
cases, a personal crisis might be less apparent but can still lead to dramatic
changes in behavior and mood.
 Option B: Developmental crises occur as part of the process of growing and
developing through various periods of life. Sometimes a crisis is a predictable part
of the life cycle, such as the crisis described in Erikson’s stages of psychosocial
development.
 Option C: If you are coping with a crisis, whether it’s emotional or situational,
there are things that you can do to help ensure your psychological and physical
well-being during this difficult time of your life. It’s important to lean on friends,
family, and loved ones during a crisis, but you should also seek professional help
if you need it. Consider talking to your doctor about what you are dealing with.

 29. Question 1 point(s)

A client taking the monoamine oxidase inhibitor (MAOI) antidepressant isocarboxazid


(Marplan) is instructed by the nurse to avoid which foods and beverages?

 A. Aged cheese and red wine


B. Milk and green, leafy vegetables
C. Carbonated beverages and tomato products
D. Lean red meats and fruit juices

Incorrect Correct Answer: A. Aged cheese and red wine
Aged cheese and red wines contain the substance tyramine which, when taken with an
MAOI, can precipitate a hypertensive crisis. Monoamine oxidase inhibitors (MAOIs)
were first introduced in the 1950s. They are a separate class from other antidepressants,
treating different forms of depression as well as other nervous system disorders such as
panic disorder, social phobia, and depression with atypical features. MAOIs prevent the
breakdown of tyramine found in the body as well as certain foods, drinks, and other
medications. Patients that take MAOIs and consume tyramine-containing foods or drinks
will exhibit high serum tyramine level.
 Option B: These are foods rich in iron. A high level of tyramine can cause a
sudden increase in blood pressure, called the tyramine pressor response. Even
though it is rare, a high tyramine level can trigger a cerebral hemorrhage, which
can even result in death. Eating foods with high tyramine can trigger a reaction
that can have serious consequences. Patients should know that tyramine can
increase with the aging of food; they should be encouraged to have foods that are
fresh instead of leftovers or food prepared hours earlier.
 Option C: Carbonated beverages are unhealthy and tomato products are high in
sodium. Eating foods with high tyramine can trigger a reaction that can have
serious consequences. Patients should know that tyramine can increase with the
aging of food; they should be encouraged to have foods that are fresh instead of
leftovers or food prepared hours earlier. Examples of high levels of tyramine in
food are types of fish, as well as types of meat, including sausage, turkey, liver,
and salami.
 Option D: Lean red meats are rich in protein and fruit juices are rich in fiber.
Also, certain fruits can contain tyramine like overripe fruits, avocados, bananas,
raisins, or figs. Further examples are cheeses, alcohol, and fava beans; all of these
should be avoided even after two weeks of stopping MAOIs. Anyone taking
MAOIs is at risk for an adverse hypertensive reaction, with accompanying
morbidity.

 30. Question 1 point(s)

Prior to administering chlorpromazine (Thorazine) to an agitated client, the nurse should:

 A. Assess skin color and sclera


B. Assess the radial pulse
C. Take the client’s blood pressure
D. Ask the client to void

Incorrect Correct Answer: C. Take the client’s blood pressure
Because chlorpromazine (Thorazine) can cause a significant hypotensive effect (and
possible client injury), the nurse must assess the client’s blood pressure (lying, sitting,
and standing) before administering this drug. When administered as intramuscular or
intravenous injections, it may cause hypotension and headache. Prolonged use of
chlorpromazine may cause corneal deposits and lens opacity. It may prolong the QT
interval.
 Option A: If the client had taken the drug previously, the nurse would also need
to assess the skin color and sclera for signs of jaundice, a possible drug side
effect; however, based on the information given here, there is no evidence that the
client has received chlorpromazine before.
 Option B: The hepatic P450 enzyme CYP2D6 metabolizes the drug, and its half-
life is approximately 30 hours. It gets excreted from the body via urine and in
bile. Studies have shown the correlation between chlorpromazine’s therapeutic
level and the improvement of the psychiatric symptoms. Researchers have noted
that the patients receiving chronic treatment with chlorpromazine have lower
plasma levels as compared to the patients acutely treated on an oral dose of
chlorpromazine.
 Option D: Although the drug can cause urine retention, asking the client to avoid
will not alter this anticholinergic effect. Chlorpromazine is a low-potency
antipsychotic that mainly causes non-neurologic side effects. It is highly lipid-
soluble and stored in body fats, thus very slow to be removed from the body.
Being a low-potency typical antipsychotic, it primarily causes dry mouth,
dizziness, urine retention, blurred vision, and constipation by blocking the
muscarinic receptors. There is a risk of angle-closure glaucoma in the elderly. It
also causes sedation due to the blockade of histamine H1 receptors.

 31. Question 1 point(s)

The nurse understands that electroconvulsive therapy is primarily used in psychiatric care
for the treatment of:

 A. Anxiety disorders
B. Depression
C. Mania
D. Schizophrenia

Incorrect Correct Answer: B. Depression
Electroconvulsive therapy (ECT) can provide relief for patients with severe depression
who have not been able to feel better with other treatments. In some severe cases where
rapid response is necessary or medications cannot be used safely, ECT can even be a
first-line intervention. ECT consists of a series of sessions, typically three times a week,
for two to four weeks. ECT is indicated in patients with treatment-resistant depression or
severe major depression that impairs activities of daily living. The definition of
treatment-resistant depression is depression that is unresponsive to multiple
antidepressant medication trials.
 Option A: Suicidal ideation is rapidly relieved by ECT, and complete resolution
was seen in 38% of patients after one week, 61% of patients after two weeks and
in 81% of patients with the completion of ECT. ECT is also recommended for
patients that have exhibited a favorable response to ECT previously. ECT is a
relatively safe and low-risk procedure that is helpful in the treatment of
depression, suicidality, severe psychosis, food refusal secondary to depression,
and catatonia.
 Option C: There are also suggestions for ECT as a treatment for suicidality,
severe psychosis, food refusal secondary to depression, and catatonia. Bipolar
depressive and manic patients can also receive treatment with ECT. ECT may
have a safer profile than antidepressants or antipsychotics in debilitated, elderly,
pregnant, and breastfeeding patients.
 Option D: In a patient under intravenous sedation or general anesthesia,
electroconvulsive therapy (ECT) uses an electric current to create a generalized
cerebral seizure. Although it is primarily utilized to treat patients with severe
depression, patients with schizophrenia, schizoaffective disorder, catatonia,
neuroleptic malignant syndrome, and bipolar disorder may also benefit.

 32. Question 1 point(s)

A client taking the MAOI phenelzine (Nardil) tells the nurse that he routinely takes all of
the medications listed below. Which medication would cause the nurse to express
concern and therefore initiate further teaching?

 A. Acetaminophen (Tylenol)
B. Diphenhydramine (Benadryl)
C. Furosemide (Lasix)
D. Isosorbide dinitrate (Isordil)

Incorrect Correct Answer: B. Diphenhydramine (Benadryl)
Over-the-counter medications used for allergies and cold symptoms are contraindicated
because they will increase the sympathomimetic effects of MAOIs, possibly causing a
hypertensive crisis. In general, SSRIs, SNRIs, TCAs, bupropion, mirtazapine, St. John’s
Wort and sympathomimetic amines, including stimulants, are contraindicated with
MAOIs. Tramadol, meperidine, dextromethorphan, and methadone are contraindicated in
patients on MAOIs as they are at high risk for causing serotonin syndrome.
 Option A: Acetaminophen (APAP) is considered a non-opioid analgesic and
antipyretic agent used to treat pain and fever. Clinicians can use it for their
patients as a single agent for mild to moderate pain and in combination with an
opioid analgesic for severe pain. Acetaminophen, also called N-acetyl para-
aminophenol or paracetamol, is one of the most widely used over-the-counter
analgesic and antipyretic agents. Although its exact mechanism of action remains
unclear, it is historically categorized along with NSAIDs because it inhibits the
cyclooxygenase (COX) pathways.
 Option C: The Food and Drug Administration (FDA) has approved the use of
furosemide in the treatment of conditions with volume overload and edema
secondary to congestive heart failure exacerbation, liver failure, or renal failure
including the nephrotic syndrome. Furosemide inhibits tubular reabsorption of
sodium and chloride in the proximal and distal tubules, as well as in the thick
ascending loop of Henle by inhibiting sodium-chloride cotransport system
resulting in excessive excretion of water along with sodium, chloride, magnesium,
and calcium.
 Option D: Isosorbide is a nitrate that exerts its pharmacologic effect by releasing
nitric oxide (NO), an endothelium-derived relaxing factor (EDRF).NO is
endogenously produced in the endothelium to dilate the blood vessels. It is for the
prevention or treatment of angina pectoris resulting from coronary artery disease;
however, it is not recommended for use once the anginal episode has started
because the onset of action is not sufficiently rapid enough to abort an acute
anginal event. In the latter case, glyceryl trinitrate is preferable.

 33. Question 1 point(s)

The nurse is administering a psychotropic drug to an elderly client who has a history of
benign prostatic hypertrophy. It is most important for the nurse to teach this client to:

 A. Add fiber to his diet.


B. Exercise on a regular basis.
C. Report incomplete bladder emptying.
D. Take the prescribed dose at bedtime.

Incorrect Correct Answer: C. Report incomplete bladder emptying
Urinary retention is a common anticholinergic side effect of psychotic medications, and
the client with benign prostatic hypertrophy would have increased risk for this problem.
First-generation antipsychotics (FGAs) are associated with significant extrapyramidal
side effects. Anticholinergic adverse effects like dry mouth, constipation, urinary
retention are common with low potency dopamine receptor antagonists like
chlorpromazine, thioridazine.
 Option A: Neuroleptic malignant syndrome is a rare but fatal adverse effect that
can occur at any time during treatment with FGAs. The onset of symptoms is over
24 to 72 hours with increased temperature, severe muscular rigidity, confusion,
agitation, elevation in white blood cell count, elevated creatinine phosphokinase
concentrations, elevated liver enzymes, myoglobinuria, and acute renal failure.
 Option B: Adding fiber to one’s diet and exercising regularly are measures to
counteract another anticholinergic effect, constipation. Second-generation
antipsychotics (SGAs) have a decreased risk of extrapyramidal side effects as
compared to first-generation antipsychotics. SGAs are associated with significant
weight gain and the development of metabolic syndrome.
 Option D: Depending on the specific medication and how it is prescribed, taking
the medication at night may or may not be important. However, it would have
nothing to do with urinary retention in this client. The FDA recommends
monitoring personal and family history of diabetes mellitus, dyslipidemia, weight,
and height, waist circumference, blood pressure, fasting plasma glucose, and
fasting lipid profile for all patients.

 34. Question 1 point(s)

The nurse correctly teaches a client taking the Benzodiazepine Oxazepam (Serax) to
avoid excessive intake of:

 A. Cheese
B. Coffee
C. Sugar
D. Shellfish

Incorrect Correct Answer: B. Coffee
Coffee contains caffeine, which has a stimulating effect on the central nervous system
that will counteract the effect of the antianxiety medication oxazepam. None of the
remaining foods is contraindicated. These drugs may act as depressants to the CNS,
specifically inhibiting respiratory drive. Therefore, careful monitoring of all vitals,
especially blood pressure and respiratory rate, should be performed after the
administration of benzodiazepines. Waveform capnography, if available, should be
seriously considered to monitor respiratory status.
 Option A: The FDA strongly reminds providers that extreme care should be
taken when administering benzodiazepines with other central nervous system
depressants such as alcohol, barbiturates, and opioids. The activated charcoal
administration is contraindicated in benzodiazepine (BZ) ingestion
toxicity/overdose. This is due primarily to altered mental status commonly
associated with BZ overdose, which lends itself to aspiration of the activated
charcoal.
 Option C: Flumazenil is a GABA-A receptor antagonist, acting to reverse the
sedative effects of benzodiazepines. Flumazenil functions through competitive
inhibition of the alpha-gamma subunit of the GABA-A receptor. Administration
of flumazenil should be carried out judiciously, as it may precipitate withdrawal
seizures. Of note, one multi-center trial found that patients with excessive
benzodiazepine ingestion could become “re-sedated” after flumazenil began to
wear off.
 Option D: Contraindications include known hypersensitivity to benzodiazepines
and angle-closure glaucoma. Glaucoma occurs when the intraocular pressure
rises, thereby causing compression of the optic nerve near the posterior surface of
the eye. This compression of the lamina cribrosa can lead to axonal damage with
subsequent disruption of anterograde and retrograde axonal transport. This results
in numerous issues, including ocular pain, nausea/vomiting, blurred vision, an
intraocular pressure greater than 21 mmHg, edema of the corneal epithelium, and
non-reactive pupils.

 35. Question 1 point(s)

The nurse provides a referral to Alcoholics Anonymous to a client who describes a 20-
year history of alcohol abuse. The primary function of this group is to:

 A. Encourage the use of a 12-step program.


B. Help members maintain sobriety.
C. Provide fellowship among members.
D. Teach positive coping mechanisms.

Incorrect Correct Answer: B. Help members maintain sobriety.
The primary purpose of Alcoholics Anonymous is to help members achieve and maintain
sobriety. Alcoholics Anonymous is an international fellowship of men and women who
have had a drinking problem. It is nonprofessional, self-supporting, multiracial,
apolitical, and available almost everywhere. There are no age or education requirements.
Membership is open to anyone who wants to do something about their drinking problem.
 Option A: Alcoholism and drug addiction are often referred to as ” substance
abuse” or “chemical dependency.” Alcoholics and nonalcoholics are, therefore,
sometimes introduced to AA and encouraged to attend AA meetings. AA
members share their experience with anyone seeking help with a drinking
problem; they give person-to-person service or “sponsorship” to the alcoholic
coming to AA from any source. The AA program, set forth in the Twelve Steps,
offers the alcoholic a way to develop a satisfying life without alcohol. This
program is discussed at AA group meetings.
 Option C: Only those with a drinking problem may attend closed meetings or
become AA members. People with problems other than alcoholism are eligible for
AA membership only if they have a drinking problem, too. According to AA
traditions, the only qualification for membership is a desire to stop drinking.
 Option D: Although each of the remaining answer choices may be an outcome of
attendance at Alcoholics Anonymous, the primary purpose is directed toward
sobriety of members. Open AA meetings, which anyone can attend, are usually
“speaker meetings,” at which a member of AA will tell their story—what it was
like, what happened, and what it’s like now. Most AA meetings, however, are
closed meetings for members only.

 36. Question 1 point(s)

Which client outcome is most appropriately achieved in a community approach setting in


psychiatric nursing?

 A. The client performs activities of daily living and learns about crafts.
B. The client is able to prevent aggressive behavior and monitors his use of
medications.
C. The client demonstrates self-reliance and social adaptation.
D. The client experiences anxiety relief and learns about his symptoms.

Incorrect Correct Answer: C. The client demonstrates self-reliance and social
adaptation.
A therapeutic community is designed to help individuals assume responsibility for
themselves, to learn how to respect and communicate with others, and to interact in a
positive manner. The therapeutic community (TC) is an intensive and comprehensive
treatment model developed for use with adults that has been modified successfully to
treat adolescents with substance use disorders.
 Option A: The core goal of TCs has always been to promote a more holistic
lifestyle and to identify areas for change such as negative personal behaviors–
social, psychological, and emotional–that can lead to substance use. Residents
make these changes by learning from fellow residents, staff members, and other
figures of authority.
 Option B: The theoretical framework for the TC model considers substance use a
symptom of much broader problems and, in a residential setting, uses a holistic
treatment approach that has an impact on every aspect of a resident’s life.
Residents are distinguished along dimensions of psychological dysfunction and
social deficits. The community provides habilitation, in which some TC residents
develop socially productive lifestyles for the first time in their lives, and
rehabilitation, in which other residents are helped to return to a previously known
and practiced or rejected healthy lifestyle (De Leon, 1994).
 Option D: The remaining answer choices may be outcomes of psychiatric
treatment, but the use of a therapeutic community approach is concerned with the
promotion of self-reliance and cooperative adaptation to being with others. Part of
the ecological approach to treatment in the TC is the creation of a safe and
nurturing environment, within which adolescents can begin to experience healthy
living. It is important for the staff of the TC to understand what type of home,
neighborhood, and social environment from which each adolescent comes. Many
adolescents enrolled in the TC come from unsafe physical and psychological
environments; the characteristics of the home and neighborhood do not facilitate
healthy living, and many risk factors may be environmental.

 37. Question 1 point(s)

A client with panic disorder experiences an acute attack while the nurse is completing an
admission assessment. List the following interventions according to their level of
priority.

View Answers:
 Teach coping measures Encourage low, deep breathing Remain with the
client Reduce external stimuli Encourage physical activity

Incorrect The correct order is shown above.


Panic disorder and panic attacks are two of the most common problems seen in the world
of psychiatry. Panic disorder is a separate entity than a panic disorder although it is
characterized by recurrent, unexpected panic attacks. Panic attacks are defined by the
Diagnostic and Statistical Manual of Mental Health Disorders (DSM) as “an abrupt surge
of intense fear or discomfort” reaching a peak within minutes. Four or more of a specific
set of physical symptoms accompany a panic attack. These symptoms include;
palpitations, pounding heart, or accelerated heart rate, sweating, trembling or shaking,
sensations of shortness of breath or smothering, feelings of choking, chest pain or
discomfort, nausea or abdominal distress, feeling dizzy, unsteady, light-headedness, or
faint, chills or heat sensations, paresthesias (numbness or tingling sensations),
derealization (feelings of unreality) or depersonalization (being detached from oneself),
fear of losing control or “going crazy”, and fear of dying.
 The nurse should remain with the client to provide support and promote safety.
The main approaches to the treatment of panic disorder include both
psychological and pharmacological interventions. Psychological interventions
consist of cognitive-behavioral therapy.
 Reducing external stimuli, including dimming lights and avoiding crowded areas,
will help decrease anxiety. It is important for a provider to inform the patient
about the symptoms that he may suffer from if he is diagnosed with the disorder.
If a patient is not aware of these symptoms it is probable that he would fear his
condition more and would tend to get frequent attacks. The pharmacotherapy and
cognitive-behavioral therapy should be discussed with the patients so that they
can understand the treatment options for the condition that they have.
 Encouraging the client to use slow, deep breathing will help promote the body’s
relaxation response, thereby interrupting stimulation from the autonomic nervous
system. Breathing training is a method of reducing panic symptomatology by
utilizing capnometry biofeedback to decrease the number of episodes of
hyperventilation. Several of these slow breathing techniques have been shown to
benefit patients with asthma and hypertension. Hyperventilation reduction can
help patients with cardiovascular disease.
 Encouraging physical activity will help him to release energy resulting from the
heightened anxiety state; this should be done only after the client has brought his
breathing under control. The patient needs a thorough education on the disorder
and understands that the symptoms are not life-threatening. The patient needs to
be told about the different treatments available and the need for compliance. Plus,
the pharmacist should caution the patient against the use of alcohol or recreational
drugs. The patient should be taught to recognize the triggers and avoid them.
 Teaching coping measures will help the client learn to handle anxiety; however,
this can only be accomplished when the client’s panic has dissipated and he is
better able to focus. Anxiety and stress-reduction techniques can lower adverse
outcomes in cardiovascular illness by decreasing sympathetic activity.

 38. Question 1 point(s)

The doctor has prescribed haloperidol (Haldol) 2.5 mg. I.M. for an agitated client. The
medication is labeled haloperidol 10 mg/2 ml. The nurse prepares the correct dose by
drawing up how many milliliters in the syringe?

 A. 0.3
B. 0.4
C. 0.5
D. 0.6

Incorrect Correct Answer: C. 0.5
Set up the problem as follows: 2.5mg/10mg = Xml/2ml X=0.5ml. Haloperidol is a first-
generation (typical) antipsychotic medication that is used widely around the world. Food
and Drug Administration (FDA) approved the use of haloperidol is for schizophrenia,
Tourette syndrome (control of tics and vocal utterances in adults and children),
hyperactivity (which may present as impulsivity, difficulty maintaining attention, severe
aggressivity, mood instability, and frustration intolerance), severe childhood behavioral
problems (such as combative, explosive hyperexcitability), intractable hiccups. It is a
typical antipsychotic because it works on positive symptoms of schizophrenia, such as
hallucinations and delusions.
 Option A: Haloperidol is used widely in different countries. It is available in
various forms; the oral route is the most common. For the oral administration, it is
available as a tablet form and oral concentrate form. It is also available in a nasal
spray formulation. Haloperidol lactate is used as a short-acting parenteral solution
available for use intramuscularly and intravenously. Haloperidol decanoate is
available for long-acting intramuscular preparation.
 Option B: Haloperidol in psychosis: In this instance, the oral and intramuscular
forms can be used. For moderate symptomatology: 0.5 to 2 mg 2 to 3 times a day
orally. In some resistant cases, up to 30 mg/day may be necessary. For prompt
control of acute agitation, an intramuscular injection can be given as a 2 to 5 mg
dose every 4 to 8 hours. The maximum intramuscular dose is 20 mg/day.
 Option D: Haloperidol in schizophrenia: In moderately severe patients, dosing is
0.5 to 2 mg haloperidol orally 2 to 3 times a day. It should not exceed 30 mg daily
in case of severe cases. To control acute agitation in a schizophrenic patient,
dosing is 2 to 5 mg haloperidol intramuscularly every 4 to 8 hours.

 39. Question 1 point(s)

The nurse enters the room of a client with a cognitive impairment disorder and asks what
day of the week it is: what the date, month, and year are; and where the client is. The
nurse is attempting to assess:

 A. Confabulation
B. Delirium
C. Orientation
D. Perseveration

Incorrect Correct Answer: C. Orientation
The initial, most basic assessment of a client with cognitive impairment involves
determining his level of orientation (awareness of time, place, and person). Interviews to
assess memory, behavior, mood and functional status (especially complex actions such as
driving and managing money are best conducted with the patient alone, so that family
members or companions cannot prompt the patient. Information can also be gleaned from
the patient’s behavior on arrival in the doctor’s office and interactions with staff.
 Option A: Confabulation is a type of memory error in which gaps in a person’s
memory are unconsciously filled with fabricated, misinterpreted, or distorted
information. When someone confabulates, they are confusing things they have
imagined with real memories. Cognitive impairment in older adults has a variety
of possible causes, including medication side effects, metabolic and/or endocrine
derangements, delirium due to intercurrent illness, depression and dementia, with
Alzheimer’s dementia being most common. Some causes, like medication side
effects and depression, can be reversed with treatment. Others, such as
Alzheimer’s disease, cannot be reversed, but symptoms can be treated for a period
of time and families can be prepared for predictable changes.
 Option B: Delirium is a type of cognitive impairment; however, other symptoms
are necessary to establish this diagnosis. Delirium, also known as the acute
confusional state, is a clinical syndrome that usually develops in the elderly. It is
characterized by an alteration of consciousness and cognition with reduced ability
to focus, sustain, or shift attention. It develops over a short period and fluctuates
during the day. The clinical presentation can vary, but usually, it flourishes with
psychomotor behavioral disturbances such as hyperactivity or hypoactivity with
increased sympathetic activity and impairment in sleep duration and architecture.
 Option D: The nurse may also assess for perseveration in a client with cognitive
impairment, but the questions in this situation would not elicit the symptom
response. Many people who are developing or have dementia do not receive a
diagnosis. One study showed that physicians were unaware of cognitive
impairment in more than 40 percent of their cognitively impaired patients.
Another study found that more than half of patients with dementia had not
received a clinical cognitive evaluation by a physician. The failure to evaluate
memory or cognitive complaints is likely to hinder treatment of underlying
disease and comorbid conditions, and may present safety issues for the patient and
others. In many cases, the cognitive problem will worsen over time.

 40. Question 1 point(s)

Which of the following will the nurse use when communicating with a client who has a
cognitive impairment?

 A. Complete explanations with multiple details


B. Picture or gestures instead of words
C. Stimulating words and phrases to capture the client’s attention
D. Short words and simple sentences

Correct Correct Answer: D. Short words and simple sentences
Short words and simple sentences minimize client confusion and enhance
communication. Frequently orient the client to reality and surroundings. Allow the client
to have familiar objects around him or her; use other items, such as a clock, a calendar,
and daily schedules, to assist in maintaining reality orientation.
 Option A: Use simple explanations and face-to-face interaction when
communicating with the client. Do not shout messages into the client’s ear.
Speaking slowly and in a face-to-face position is most effective when
communicating with an elderly individual experiencing a hearing loss.
 Option B: Although pictures and gestures may be helpful, they would not
substitute for verbal communication. Teach prospective caregivers how to orient
the client to time, person, place, and circumstances, as required. These caregivers
will be responsible for client safety after discharge from the hospital.
 Option C: Complete explanations with multiple details and stimulating words
and phrases would increase confusion in a client with short attention span and
difficulty with comprehension. Give positive feedback when thinking and
behavior are appropriate, or when the client verbalizes that certain ideas expressed
are not based in reality. Positive feedback increases self-esteem and enhances the
desire to repeat appropriate behavior.

 41. Question 1 point(s)

A 75-year-old client has dementia of the Alzheimer’s type and confabulates. The nurse
understands that this client:

 A. Denies confusion by being jovial


B. Pretends to be someone else
C. Rationalizes various behaviors
D. Fills in memory gaps with fantasy

Correct Correct Answer: D. Fills in memory gaps with fantasy
Confabulation is a communication device used by patients with dementia to compensate
for memory gaps. Confabulation is a type of memory error in which gaps in a person’s
memory are unconsciously filled with fabricated, misinterpreted, or distorted information.
When someone confabulates, they are confusing things they have imagined with real
memories. A person who is confabulating is not lying. They are not making a conscious
or intentional attempt to deceive. Rather, they are confident in the truth of their memories
even when confronted with contradictory evidence.The remaining answer choices are
incorrect.
 Option A: Dementia is a syndrome – usually of a chronic or progressive nature –
in which there is deterioration in cognitive function (i.e. the ability to process
thought) beyond what might be expected from normal aging. It affects memory,
thinking, orientation, comprehension, calculation, learning capacity, language,
and judgment. Consciousness is not affected. The impairment in cognitive
function is commonly accompanied and occasionally preceded, by deterioration in
emotional control, social behavior, or motivation.
 Option B: Dementia results from a variety of diseases and injuries that primarily
or secondarily affect the brain, such as Alzheimer’s disease or stroke. Dementia is
one of the major causes of disability and dependency among older people
worldwide. It can be overwhelming, not only for the people who have it, but also
for their carers and families. There is often a lack of awareness and understanding
of dementia, resulting in stigmatization and barriers to diagnosis and care. The
impact of dementia on carers, family, and society at large can be physical,
psychological, social, and economic.
 Option C: Although age is the strongest known risk factor for dementia, it is not
an inevitable consequence of aging. Further, dementia does not exclusively affect
older people – young onset dementia (defined as the onset of symptoms before the
age of 65 years) accounts for up to 9% of cases. Studies show that people can
reduce their risk of dementia by getting regular exercise, not smoking, avoiding
harmful use of alcohol, controlling their weight, eating a healthy diet, and
maintaining healthy blood pressure, cholesterol, and blood sugar levels.
Additional risk factors include depression, low educational attainment, social
isolation, and cognitive inactivity.

 42. Question 1 point(s)

An elderly client with Alzheimer’s disease becomes agitated and combative when a nurse
approaches to help with morning care. The most appropriate nursing intervention in this
situation would be to:

 A. Tell the client family that it is time to get dressed.


B. Obtain assistance to restrain the client for safety.
C. Remain calm and talk quietly to the client.
D. Call the doctor and request an order for sedation.

Incorrect Correct Answer: C. Remain calm and talk quietly to the client.
Maintaining a calm approach when intervening with an agitated client is extremely
important. Divert attention to a client when agitated or dangerous behaviors like getting
out of bed by climbing the fence bed. Eliminate or minimize sources of hazards in the
environment. Maintain security by avoiding a confrontation that could improve the
behavior or increase the risk for injury.
 Option A: Telling the client firmly that it is time to get dressed may increase his
agitation, especially if the nurse touches him. Assess the degree of impaired
ability of competence, emergence of impulsive behavior, and a decrease in visual
perception. Impairment of visual perception increases the risk of falling. Identify
potential risks in the environment and heighten awareness so that caregivers are
more aware of the danger.
 Option B: Restraints are a last resort to ensure client safety and are inappropriate
in this situation. Assess the patient’s surroundings for hazards and remove them.
AD decreases awareness of potential dangers, and disease progression coupled
with a hazardous environments that could lead to accidents. Help the people
closest to identify the risk of hazards that may arise. An impaired cognitive and
perceptual disorder are beginning to experience the trauma as a result of the
inability to take responsibility for basic security capabilities or evaluating a
particular situation.
 Option D: Sedation should be avoided, if possible because it will interfere with
CNS functioning and may contribute to the client’s confusion. During the middle
and later stages of AD, the patient must not be left unattended. Patients with AD
have impaired thinking and cannot rationalize cause and effect. This can result in
wandering outside without clothes on, exposure to extreme cold or heat, and may
cause dehydration in the long run.

 43. Question 1 point(s)

In clients with a cognitive impairment disorder, the phenomenon of increased confusion


in the early evening hours is called:

 A. Aphasia
B. Agnosia
C. Sundowning
D. Confabulation

Incorrect Correct Answer: C. Sundowning
Sundowning is a common phenomenon that occurs after daylight hours in a client with a
cognitive impairment disorder. The term “sundowning” refers to a state of confusion
occurring in the late afternoon and spanning into the night. Sundowning can cause a
variety of behaviors, such as confusion, anxiety, aggression or ignoring directions.
Sundowning can also lead to pacing or wandering. The other options are incorrect
responses, although all may be seen in this client.
 Option A: Aphasia is a condition that robs you of the ability to communicate. It
can affect your ability to speak, write and understand language, both verbal and
written. Aphasia typically occurs suddenly after a stroke or a head injury. But it
can also come on gradually from a slow-growing brain tumor or a disease that
causes progressive, permanent damage (degenerative). The severity of aphasia
depends on a number of conditions, including the cause and the extent of the brain
damage.
 Option B: Agnosia is a rare disorder whereby a patient is unable to recognize and
identify objects, persons, or sounds using one or more of their senses despite
otherwise normally functioning senses. The deficit cannot be explained by
memory, attention, language problems, or unfamiliarity to the stimuli. Usually,
one of the sensory modalities is affected.
 Option D: Confabulation is a type of memory error in which gaps in a person’s
memory are unconsciously filled with fabricated, misinterpreted, or distorted
information. When someone confabulates, they are confusing things they have
imagined with real memories. a person who is confabulating is not lying. They are
not making a conscious or intentional attempt to deceive. Rather, they are
confident in the truth of their memories even when confronted with contradictory
evidence.

 44. Question 1 point(s)

Which of the following outcome criteria is appropriate for the client with dementia?
 A. The client will return to an adequate level of self-functioning.
B. The client will learn new coping mechanisms to handle anxiety.
C. The client will seek out resources in the community for support.
D. The client will follow an establishing schedule for activities of daily living.

Correct Correct Answer: D. The client will follow an establishing schedule for
activities of daily living.
Following established activity schedules is a realistic expectation for clients with
dementia. Frequently orient the client to reality and surroundings. Allow the client to
have familiar objects around him or her; use other items, such as a clock, a calendar, and
daily schedules, to assist in maintaining reality orientation. Teach prospective caregivers
how to orient the client to time, person, place, and circumstances, as required. These
caregivers will be responsible for client safety after discharge from the hospital.
 Option A: Assess and identify the patient’s previous history of grooming and
bathing, and attempt to maintain similar care. This promotes familiarity with
routine bathing time and type of bath or shower and lessens further confusion and
agitation. Instruct the patient in activity with a short step-by-step method; do not
rush the patient. This promotes self-esteem and feelings of accomplishment;
rushing the patient causes frustration.
 Option B: Assist in defining problems and use of techniques to cope and solve
problems. This provides support for problem solving and management of the
family’s fatigue and chronic stress. Provide an opportunity for the family to
express concerns and lack of control of the situation to provide an opportunity for
the family to express concerns and lack of control of the situation.
 Option C: All of the remaining outcome statements require a higher level of
cognitive ability than can be realistically expected of clients with this disorder.
Identify possible support systems and ability to participate in social activities.
Community resources are available for clients and families dealing with stages of
AD that provide information and assistance. Provide diversional activities as
appropriate for functional ability. Provide rest and sleep periods; avoid situations
that cause frustration, agitation, or sensory overload.

 45. Question 1 point(s)

The school guidance counselor refers a family with an 8-year-old child to the mental
health clinic because of the child’s frequent fighting in school and truancy. Which of the
following data would be a priority to the nurse doing the initial family assessment?

 A. The child’s performance in school


B. Family education and work history
C. The family’s perception of the current problem
D. The teacher’s attempt to solve the problem

Incorrect Correct Answer: C. The family’s perception of the current problem
The family’s perception of the problem is essential because change in any one part of a
family system affects all other parts and the system as a whole. Each member of the
family has been affected by the current problems related to the school system and the
nurse would be interested in the data. Research indicates at-risk youth are more likely to
experience emotional and psychological problems. Young people who are often truant
from school represent a group of at-risk youth, but one for which mental health issues are
understudied.
 Option A: The child’s performance in school and the teacher’s attempts to solve
the problem are relevant and may be assessed; however, priority would be given
to the family’s perception of the problem. Truancy is a serious problem that
affects most school districts in the U.S. Research on truancy can be challenging
because there is not a uniform definition of truancy and statistics on truancy rates
are lacking and/or inconsistently reported across school districts. Psychological
research reports a high prevalence of mental health problems among youths
characterized as school refusers. School refusers demonstrate symptoms of mood
disorders such as depression and dysthymia, anxiety disorders such as generalized
anxiety, separation anxiety, and panic disorder, and disruptive behavior disorders
such as oppositional defiant, attention deficit hyperactivity disorder (ADHD) and
conduct disorders.
 Option B: The family education and work history may be relevant, but are not a
priority. Generally, truancy is defined as unauthorized, intentional absence from
compulsory schooling. It is estimated that thousands of youth in the U.S. are
absent from school each day. For example, recent statistics on truancy in Los
Angeles County and Colorado indicate truancy rates greater than 10 percent, with
the highest rates in urban high schools. Comparable statistics corroborating high
rates of truancy can also be found in other jurisdictions.
 Option D: Truancy appears to be a risk factor for a life-course trajectory toward
more negative behaviors. As Garry observed, truancy may be the beginning of a
lifetime of problems among students who routinely skip school, including poor
standardized test performance, high school dropout, a stressed family life,
difficulties in emotional/psychological functioning, drug use, and progression to
both juvenile delinquency and adult criminal offending. Related research has also
documented a link between truancy and later problems with employment, adult
crime and incarceration.

 46. Question 1 point(s)

The parents of a young man with schizophrenia express feelings of responsibility and
guilt for their son’s problems. How can the nurse best educate the family?

 A. Acknowledge the parent’s responsibility.


B. Explain the biological nature of schizophrenia.
C. Refer the family to a support group.
D. Teach the parents various ways they must change.

Incorrect Correct Answer: B. Explain the biological nature of schizophrenia.
The parents are feeling responsible and this inappropriate self-blame can be limited by
supplying them with the facts about the biological basis of schizophrenia. Schizophrenia
is a psychiatric disorder, which is characterized by slow functional deterioration and
episodes of relapse or acute exacerbation of psychotic symptoms. The mean age of onset
in early adulthood, deterioration in patients’ activities of daily living and ability to sustain
employment, and the propensity of the disorder to affect insight leave many patients
requiring assistance and care for an extended period of time.
 Option A: Acknowledging the patient’s responsibility is neither accurate nor
helpful to the parents and would only reinforce their feelings of guilt. Caregivers
of patients with childhood-onset chronic psychiatric disorders such as autism
spectrum disorders, who are usually the parents, realize at an early stage that there
will be a responsibility for them to care for their child for the rest of their lives in
most cases. They, therefore, tend to adapt accordingly as the child grows up and
experience a comparatively slow change to their lives and expectations regarding
their ill child.
 Option C: Support groups are useful; however, the nurse needs to handle the
parents’ self-blame directly instead of making a referral for this problem. Patients
with schizophrenia can often have a normal childhood and adolescence before
suddenly, unexpectedly, and often dramatically becoming ill. Because of the age
of onset, care responsibilities are suddenly thrust upon mostly parents, even
before they have come to terms with the shock of the sudden, dramatic onset of
the illness. It often comes at a time when they would expect their child to gain
independence and when they themselves are at an age when retirement could have
been considered. The lowering of expectation for the future of their child, along
with the new, long-term care responsibilities, tends to weigh heavily on these
parents, requiring a dramatic adjustment to their lives and subjecting them to
unique symptoms and behaviors, which become increasingly difficult to manage,
especially for people of their age.
 Option D: Teaching the parents various ways to change would reinforce the
parental assumption of blame; although parents can learn about schizophrenia and
what is helpful and not helpful, the approach suggested in this option implies the
parents’ behavior is at fault. Caring for family members with schizophrenia
subjects caregivers to mostly negative experiences, which in turn negatively
impact the caregivers themselves. These negative aspects experienced by patients’
relatives as a consequence of their caregiving role are collectively known as
‘burden’. Attempts have been made in the literature to better define ‘burden’ as
the existence of serious psychosocial and emotional problems, difficulties or
negative events, stressful situations or significant life changes that influence the
family member of an ill relative.

 47. Question 1 point(s)

The nurse collecting family assessment data asks. “Who is in your family and where do
they live?” Which of the following is the nurse attempting to identify?
 A. Boundaries
B. Ethnicity
C. Relationships
D. Triangles

Incorrect Correct Answer: A. Boundaries
Family boundaries are parameters that define who is inside and outside the system. The
best method of obtaining this information is asking the family directly who they consider
to be members. Every system has ways of including and excluding elements so that the
line between those within the system and those outside of the system is clear to all. If a
family is permeable and has vague boundaries it is considered “open.” Open boundary
systems allow elements and situations outside the family to influence it. It may even
welcome external influences. Closed boundary systems isolate its members from the
environment and seem isolated and self-contained. No family system is completely
closed or completely open.
 Option B: Ethnicity is a broader term than race. The term is used to categorize
groups of people according to their cultural expression and identification.
Commonalities such as racial, national, tribal, religious, linguistic, or cultural
origin may be used to describe someone’s ethnicity.
 Option C: The relationship between two people or groups is the way in which
they feel and behave towards each other. A relationship is a close connection
between two people, especially one involving romantic or sexual feelings.
 Option D: Triangulation or triangling is defined in the AAMFT Family Therapy
Glossary as the “process that occurs when a third person is introduced into a
dyadic relationship to balance either excessive intimacy, conflict, or distance and
provide stability in the system” (Evert et al. 1984 p. 32). This concept is
associated with Murray Bowen (1978) who saw triangulation as a way to reduce
anxiety in a dyadic relationship.

 48. Question 1 point(s)

According to the family systems theory, which of the following best describes the
process of differentiation?

 A. Cooperative action among members of the family.


B. Development of autonomy within the family.
C. Incongruent messages wherein the recipient is a victim.
D. Maintenance of system continuity or equilibrium.

Incorrect Correct Answer: B. Development of autonomy within the family
Differentiation is the process of becoming an individual developing autonomy while
staying in contact with the family system. “The ability to be in emotional contact with
others yet still autonomous in one’s own emotional functioning is the essence of the
concept of differentiation.” (Kerr & Bowen. 1988) “Differentiation is a product of a way
of thinking that translates into a way of being….Such changes are reflected in the ability
to be in emotional contact with a difficult, emotionally charged problem and not feel
compelled to preach about what others “should” do, not rush in to “fix” the problem and
not pretend to be detached by emotionally insulating oneself.” (Kerr & Bowen 1988).
 Option A: Cooperative action among family members does not refer to
differentiation, although individuals who have a high level of differentiation
would be able to accomplish cooperative action. Bowen’s concept of
‘differentiation of self’ forms the basis of a systems understanding of maturity.
The concept of differentiation can be confusing but, put simply, it refers to the
ability to think as an individual while staying meaningfully connected to others. It
describes the varying capacity each person has to balance their emotions and their
intellect, and to balance their need to be attached with their need to be a separate
self. Bowen proposed that the best way to grow a more solid self was in the
relationships that make up our original families; running away from difficult
family members would only add to the challenges in managing relationship
upsets.
 Option C: Incongruent messages in which the recipient is a victim describe
double-bind communication. In communication, sometimes people say things that
are contradictory to their non-verbal communication cues. When a person’s words
don’t match what he or she is feeling or thinking, the communication is said to be
incongruent.
 Option D: Maintenance of system continuity or equilibrium is homeostasis. It’s
not an easy theory to grasp, as it focuses on the big-picture patterns of a system
rather than the narrower view of what causes difficulties for one individual. These
ideas invite us to see the world through the lens of each family member rather
than just from our own subjective experience; they don’t allow room for simply
seeing victims and villains in our relationship networks. Seeing the system takes
people beyond blame to seeing the relationship forces that set people on their
different paths. This way of seeing our life challenges avoids fault-finding and
provides a unique path to maturing throughout our adult lives.

 49. Question 1 point(s)

The nurse is interacting with a family consisting of a mother, a father, and a hospitalized
adolescent who has a diagnosis of alcohol abuse. The nurse analyzes the situation and
agrees with the adolescent’s view about family rules. Which intervention is most
appropriate?

 A. The nurse should align with the adolescent, who is the family scapegoat.
B. The nurse should encourage the parents to adopt more realistic rules.
C. The nurse should encourage the adolescent to comply with parental rules.
D. The nurse should remain objective and encourage mutual negotiation of
issues.

Correct Correct Answer: D. The nurse should remain objective and encourage
mutual negotiation of issues.
The nurse who wishes to be helpful to the entire family must remain neutral. Taking sides
in a conflict situation in a family will not encourage negotiation, which is important for
problem resolution. Nurses who choose collaboration as their conflict resolution strategy
incorporate others’ ideas into their own; while the result may not be as half-and-half as
with the compromising method, the solution still has aspects of everyone’s opinions and
input, increasing group buy-in and general satisfaction with the final decision.
 Option A: If the nurse aligned with the adolescent, then the nurse would be
blaming the parents for the child’s current problem; this would not help the
family’s situation. Learning to negotiate conflict is a function of a healthy family.
 Option B: Instead of adopting a “me vs. you” mentality, nurses approaching
interpersonal conflict resolution from a compromising mentality aim to reach a
solution that makes both sides at least partially happy. By doing so, both sides
leave with something they want and are able to move forward with implementing
a solution.
 Option C: Encouraging the parents to adopt more realistic rules or the adolescent
to comply with parental rules does not give the family an opportunity to try to
resolve problems on their own. Nurses who choose to use obliging as their main
conflict resolution strategy are people-pleasers. They’re fine accommodating
other ideas even at the expense of shelving or de-prioritizing their own. This can
be helpful when it moves the best solution forward, but it can also be dangerous
because it may lead to a case where an individual withholds valid convictions or
opinions just to “keep the peace.”

 50. Question 1 point(s)

A 16-year-old girl has returned home following hospitalization for treatment of anorexia
nervosa. The parents tell the family nurse performing a home visit that their child has
always done everything to please them and they cannot understand her current
stubbornness about eating. The nurse analyzes the family situation and determines it is
characteristic of which relationship style?

 A. Differentiation
B. Disengagement
C. Enmeshment
D. Scapegoating

Incorrect Correct Answer: C. Enmeshment
Enmeshment is a fusion or over involvement among family members whereby the
expectation exists that all members think and act alike. The child who always acts to
please her parents is an example of how enmeshment affects development in many cases,
a child who develops anorexia nervosa exerts control only in the area of eating behavior.
Enmeshed families are families in which the individual is expected to give up their own
needs and desires. In enmeshed families, there is a total lack of boundaries, which usually
leads to codependent relationships and a dysfunctional family.
 Option A: Differentiation is the process of becoming an individual developing
autonomy while staying in contact with the family system. “The ability to be in
emotional contact with others yet still autonomous in one’s own emotional
functioning is the essence of the concept of differentiation.” (Kerr & Bowen.
1988) “Differentiation is a product of a way of thinking that translates into a way
of being. Such changes are reflected in the ability to be in emotional contact with
a difficult, emotionally charged problem and not feel compelled to preach about
what others “should” do, not rush in to “fix” the problem and not pretend to be
detached by emotionally insulating oneself.” (Kerr & Bowen 1988).
 Option B: The lines of responsibility and authority are strictly enforced and must
be followed; however, they are not necessarily communicated or explained.
Access to all family members, especially parents or those in authority, is limited.
Appropriate communication and expression across subsystems (e.g., children to
parents) is stifled.
 Option D: In Family Systems theory, scapegoating in a dysfunctional family
system is understood to be fueled by unconscious processes whereby the family
displaces their own collective psychological difficulties and complexes onto a
specific family member. ‘The Scapegoat’ is one of the roles ‘assigned’ to a child
growing up in a dysfunctional family system (I say more about this process in my
answer to question 2). The scapegoating typically (but not always) begins in
childhood and often continues into and throughout adulthood, although the role
may be passed around to different family members at times.

 ABOUT
 PRIVACY
 DISCLAIMER
 CONTACT
© 2024 Nurseslabs | Ut in Omnibus Glorificetur Deus!
 1. Question 1 point(s)

Flumazenil (Romazicon) has been ordered for a male client who has overdosed on
oxazepam (Serax). Before administering the medication, nurse Gina should be prepared
for which common adverse effect?

 A. Seizures
B. Shivering
C. Anxiety
D. Chest pain

Correct Correct Answer: A. Seizures
Seizures are the most common serious adverse effect of using flumazenil to reverse
benzodiazepine overdose. The effect is magnified if the client has a combined tricyclic
antidepressant and benzodiazepine overdose. Benzodiazepine reversal has correlations
with seizures. Seizures may happen more frequently in patients who have been on
benzodiazepines for long-term sedation or in patients who are showing signs of severe
tricyclic antidepressant overdose. The required dosage of Flumazenil should be measured
and prepared by the practitioners to manage seizures. Flumazenil use requires caution in
patients relying on a benzodiazepine for seizure control.
 Option B: Shivering is not an adverse effect of flumazenil. Monitor the patient
for the possible return of sedation, mostly in those who are tolerant of
benzodiazepines. Patients should have monitoring for respiratory depression,
benzodiazepine withdrawal, and other residual effects of benzodiazepines for at
least 2 hours.
 Option C: Anxiety is a rare adverse effect for people using flumazenil.
Flumazenil has some associations with precipitation of seizures in patients with
benzodiazepine dependence with a history of seizures. Flumazenil overdose is
extremely rare. There is no precise antidote for flumazenil toxicity. In mild to
severe toxicity, symptomatic and supportive treatment should be a consideration.
 Option D: An overdose of flumazenil in a patient who is not a chronic
benzodiazepine user would not be expected. Chronic benzodiazepines users may
experience withdrawal with abrupt discontinuation of the drug. Administration of
benzodiazepines or barbiturates may be necessary for seizure control.

 2. Question 1 point(s)

Nurse Tamara is caring for a client diagnosed with bulimia. The most appropriate initial
goal for a client diagnosed with bulimia is to:
 A. Avoid shopping for large amounts of food.
B. Control eating impulses.
C. Identify anxiety-causing situations.
D. Eat only three meals per day.

Incorrect Correct Answer: C. Identify anxiety-causing situations
Bulimic behavior is generally a maladaptive coping response to stress and underlying
issues. The client must identify anxiety-causing situations that stimulate the bulimic
behavior and then learn new ways of coping with the anxiety. Bulimia nervosa is a
condition that occurs most commonly in adolescent females, characterized by indulgence
in binge-eating, and inappropriate compensatory behaviors to prevent weight gain.
 Option A: Controlling shopping for large amounts of food isn’t a goal early in
treatment. It is important to educate patients who abuse laxatives that these
medications work in the gastrointestinal tract after the areas where caloric
absorption has occurred primarily. It is crucial to inform patients that a period of
edema and weight gain may follow up to several weeks after discontinuation of
purging behavior.
 Option B: Managing eating impulses and replacing them with adaptive coping
mechanisms can be integrated into the plan of care after initially addressing stress
and underlying issues. The primary objective of treatment is a cessation of the
binging and purging behavior. Selective serotonin reuptake inhibitors such as
fluoxetine, citalopram, and sertraline have shown to reduce symptoms of bulimia
nervosa. Fluoxetine is the only FDA approved medication for bulimia nervosa. It
appears that a higher dose (60 mg) is significantly better than a placebo in
decreasing the frequency of binge and vomiting episodes.
 Option D: Eating three meals per day isn’t a realistic goal early in treatment.
Patients with bulimia nervosa who purge by vomiting often brush their teeth
immediately after purging, which can accelerate dental erosion. The clinician
should instruct the patients who persist in vomiting to rinse their mouths with
water or fluoride rather than brushing their teeth within 30 minutes of each
episode. Consider consulting a dentist to address dental issues associated with
vomiting.

 3. Question 1 point(s)

A female client who’s at high risk for suicide needs close supervision. To best ensure the
client’s safety, Nurse Mary should:

 A. Check the client frequently at irregular intervals throughout the night.


B. Assure the client that the nurse will hold in confidence anything the client says.

C. Repeatedly discuss previous suicide attempts with the client.


D. Disregard decreased communication by the client because this is common with
suicidal clients.

Correct Correct Answer: A. Check the client frequently at irregular intervals
throughout the night
Checking the client frequently but at irregular intervals prevents the client from
predicting when observation will take place and altering behavior in a misleading way at
these times. Once the patient is deemed to be at risk for suicide, then intervention steps
must be initiated right away. The individual must not be left alone. Enlist the help of a
support person while at home. The suicidal individual must be treated in a safe and secure
place. In addition, the place has to be monitored.
 Option B: This may encourage the client to try to manipulate the nurse or seek
attention for having a secret suicide plan. Assessing the individual’s judgment is
critical. One should try and determine how the individual can handle stress. Does
he or she have an impairment in decision making? Does the individual know that
jumping in front of a train is dangerous? Reflect empathy and concern. Offer a
hand to help. Provide the patient with confidence that he or she can overcome the
issues.
 Option C: This may reinforce suicidal ideas. Help develop internal coping
strategies (e.g., exercise, journaling, reading, developing a hobby). Utilize the
help of healthcare professionals to follow up on therapy. Once the individual is
safe as an inpatient or outpatient, a formal treatment plan should be established.
The next step is to refer all patients deemed to be at higher risk for suicide to a
mental health counselor as soon as possible. Every state has laws and procedures
regarding this process which must be incorporated into the clinical practice when
addressing individuals at high suicide risk.
 Option D: Decreased communication is a sign of withdrawal that may indicate
the client has decided to commit suicide; the nurse shouldn’t disregard it. In some
cases, assessment of the mental status may provide a clue to the individual’s
potential for self-harm. Depressed patients will often tend to appear unclean and
unkempt. The clothing may not be ironed or dirty. The risk of suicide is often
high in people who appear very anxious or depressed. The patient may exhibit a
flat affect or no emotions at all. Some depressed patients may develop
hallucinations that may be telling him or her to kill themselves. The majority of
these hallucinations are auditory.

 4. Question 1 point(s)

Which of the following drugs should Nurse Mary prepare to administer to a client with a
toxic acetaminophen (Tylenol) level?

 A. Deferoxamine mesylate (Desferal)


B. Succimer (Chemet)
C. Flumazenil (Romazicon)
D. Acetylcysteine (Mucomyst)

Correct Correct Answer: D. Acetylcysteine (Mucomyst)
The antidote for acetaminophen toxicity is acetylcysteine. It enhances conversion of toxic
metabolites to nontoxic metabolites. Acetaminophen (N-acetyl-para-aminophenol,
paracetamol, APAP) toxicity is common primarily because the medication is so readily
available, and there is a perception that it is very safe. More than 60 million Americans
consume acetaminophen on a weekly basis. All patients with high levels of
acetaminophen need admission and treatment with N-acetyl-cysteine (NAC). This agent
is fully protective against liver toxicity if given within 8 hours after ingestion.
 Option A: Deferoxamine mesylate is the antidote for iron intoxication. Desferal
is indicated for the treatment of acute iron intoxication and chronic iron overload
due to transfusion-dependent anemias. Desferal is an adjunct to, and not a
substitute for, standard measures used in treating acute iron intoxication, which
may include the following: induction of emesis with syrup of ipecac; gastric
lavage; suction and maintenance of a clear airway; control of shock with
intravenous fluids, blood, oxygen, and vasopressors; and correction of acidosis.
 Option B: Succimer is an antidote for lead poisoning. Succimer is an oral heavy
metal chelating agent used to treat lead and heavy metal poisoning. Succimer has
been linked to a low rate of transient serum aminotransferase elevations during
therapy, but its use has not been linked to cases of clinically apparent liver injury
with jaundice. Succimer does not significantly chelate essential metals such as
zinc, copper, or iron, and its specificity, safety and oral availability make it
preferable to other chelating agents for treating lead poisoning such as Ca-EDTA
which must be given intravenously and dimercaprol (British anti-Lewisite [BAL)
which requires intramuscular administration.
 Option C: Flumazenil reverses the sedative effects of benzodiazepines.
Flumazenil is a benzodiazepine antagonist. Flumazenil is also indicated for the
management and treatment of benzodiazepine overdose in adults. It is useful in
reversing coma due to benzodiazepine overdose. Flumazenil is more effective in
reversing sedation or coma in patients with benzodiazepine intoxication rather
than in patients with multiple drug overdoses.

 5. Question 1 point(s)

A male client is admitted to the substance abuse unit for alcohol detoxification. Which of
the following medications is Nurse Alice most likely to administer to reduce the
symptoms of alcohol withdrawal?

 A. Naloxone (Narcan)
B. Haloperidol (Haldol)
C. Magnesium sulfate
D. Chlordiazepoxide (Librium)

Correct Correct Answer: D. Chlordiazepoxide (Librium)
Chlordiazepoxide (Librium) and other tranquilizers help reduce the symptoms of alcohol
withdrawal. Chlordiazepoxide is a long-acting benzodiazepine and is an FDA approved
medication for adults with mild-moderate to severe anxiety disorder, preoperative
apprehension and anxiety, and withdrawal symptoms of acute alcohol use disorder.
Chlordiazepoxide has anti-anxiety, sedative, appetite-stimulating, and weak analgesic
actions. It binds to benzodiazepine receptors at the GABA-A ligand-gated chloride
channel complex and enhances GABA’s inhibitory effects.
 Option A: Naloxone (Narcan) is administered for narcotic overdose. Naloxone is
indicated for the treatment of opioid toxicity, specifically to reverse respiratory
depression from opioid use. It is useful in accidental or intentional overdose and
acute or chronic toxicity. Naloxone is a pure, competitive opioid antagonist with a
high affinity for the mu-opioid receptor, allowing for reversal of the effects of
opioids. The onset of action varies depending on the route of administration but
can be as fast as one minute when delivered intravenously (IV) or intraosseous
(IO).
 Option B: Haloperidol (Haldol) may be given to treat clients with psychosis,
severe agitation, or delirium. Haloperidol is a first-generation (typical
antipsychotic) which exerts its antipsychotic action by blocking dopamine D2
receptors in the brain. When 72% of dopamine receptors are blocked, this drug
achieves its maximal effect. Haloperidol is not selective for the D2 receptor. It
also has noradrenergic, cholinergic, and histaminergic blocking action. The
blocking of these receptors is associated with various side effects.
 Option C: Magnesium sulfate and other anticonvulsant medications are only
administered to treat seizures if they occur during withdrawal. Magnesium sulfate
administration can be oral (PO), intramuscular (IM), intraosseous (IO), or
intravenous (IV). For every 1 gram of magnesium sulfate, it contains 98.6 mg or
8.12Eq of elemental magnesium. Magnesium sulfate can be combined with
dextrose 5% or water to make intravenous solutions.

 6. Question 1 point(s)

During postprandial monitoring, a female client with bulimia nervosa tells the nurse,
“You can sit with me, but you’re just wasting your time. After you had sat with me
yesterday, I was still able to purge. Today, my goal is to do it twice.” What is the nurse’s
best response?

 A. “I trust you not to purge.”


B. “How are you purging and when do you do it?”
C. “Don’t worry. I won’t allow you to purge today.”
D. “I know it’s important for you to feel in control, but I’ll monitor you for
90 minutes after you eat.”

Correct Correct Answer: D. “I know it’s important for you to feel in control, but I’ll
monitor you for 90 minutes after you eat.”
This response acknowledges that the client is testing limits and that the nurse is setting
them by performing postprandial monitoring to prevent self-induced emesis. Clients with
bulimia nervosa need to feel in control of the diet because they feel they lack control over
all other aspects of their lives. Since recovery involves patients having to face their
deepest, most painful, and traumatic thoughts and emotions, supporting them as they go
through treatment can be emotionally challenging for nurses. This emotional challenge
can be exacerbated when the patient has also been diagnosed with Obsessive-Compulsive
Disorder (OCD), depression, or substance abuse, as these may require more intensive
one-to-one support.
 Option A: Because their therapeutic relationships with caregivers are less
important than their need to purge, they don’t fear betraying the nurse’s trust by
engaging in the activity. They commonly plot to purge and rarely share their
secrets about it. As this might take nurses out of their comfort zone or clinical
remit, worksheets are available for nurses to use in efforts to help patients
challenge and overcome their obsessive and ritualistic behaviors and to adopt a
more flexible perspective in day-to-day life.6 These can be supplemented by
nurses familiarising themselves with the detailed guidelines and resources offered
by NICE.
 Option B: Learning motivational interviewing techniques can help facilitate
communication with those who might be resistant to discussing topics related to
food, weight, and recovery. Such techniques can help develop the skills of
empathic understanding, rolling with resistance, and gently assisting patients to
make their own, autonomous decision to work towards recovery. Often, the aim is
to help patients learn new and healthier ways of coping, and nurses can achieve
this through a mix of emotional support, education, and signposting.
 Option C: An authoritarian or challenging response may trigger a power struggle
between the nurse and client. Assisting patients to remain strong and adhere to
treatment requires nurses to develop a relationship that is caring, empathetic and
trusting, and in line with the person-centered approach to care. Patients affected
by eating disorders require individualized support to better understand their
condition, rediscover their identity, learn to accept themselves, enhance a positive
body image and sense of self-worth, and achieve a balance in their lives so that
they can move towards better health and wellbeing.

 7. Question 1 point(s)

A male client admitted to the psychiatric unit for treatment of substance abuse says to the
nurse, “It felt so wonderful to get high.” Which of the following is the most appropriate
response?

 A. “If you continue to talk like that, I’m going to stop speaking to you.”
B. “You told me you got fired from your last job for missing too many days after
taking drugs all night.”
C. “Tell me more about how it felt to get high.”
D. “Don’t you know it’s illegal to use drugs?”

Incorrect Correct Answer: B. “You told me you got fired from your last job for
missing too many days after taking drugs all night.”
Confronting the client with the consequences of substance abuse helps to break through
denial. Present reality by spending time with the client to facilitate reality orientation
because your physical presence is the reality. Be simple, direct, and concise when
speaking to the client. Talk with the client about concrete or familiar things; avoid
ideological or theoretical discussions. The client’’s ability to process abstractions or
complexities is impaired.
 Option A: Making threats isn’t an effective way to promote self-disclosure or
establish a rapport with the client. Motivational counseling works according to the
idea that motivation for change is dynamic rather than static. Professional uses
may influence change by developing a therapeutic relationship to increase
therapeutic alliance, developing insight, and coping skills to resolve ambivalence,
and change health-related behavior.
 Option C: Although the nurse should encourage the client to discuss feelings, the
discussion should focus on how the client felt before, not during, an episode of
substance abuse. Encouraging elaboration about his experience while getting high
may reinforce the abusive behavior. Persons may withdraw from their
environment with regressive behavior, fail to engage with others, or even notice
physical illness and pain. Social exclusion and homelessness may ensue. In the
longer term, psychosis and its potential disruption of the capacity to fulfill social
roles can result in further burdens.
 Option D: The client undoubtedly is aware that drug use is illegal; a reminder to
this effect is unlikely to alter behavior. Drug addiction exacerbates social
alienation and increases potential for violent lashing out and low self-esteem,
along with poor coping skills. Under these circumstances, emotional, social, or
symptom-related cues can provoke recourse to available substances and suicidal
ideation. They may also contribute to psychosocial instability, self-image issues,
and achievement motivation. In some cases, social hostility and rejection may
result.

 8. Question 1 point(s)

For a female client with anorexia nervosa, Nurse Jimmy is aware that which goal takes
the highest priority?

 A. The client will establish adequate daily nutritional intake.


B. The client will make a contract with the nurse that sets a target weight.
C. The client will identify self-perceptions about body size as unrealistic.
D. The client will verbalize the possible physiological consequences of self-
starvation.

Incorrect Correct Answer: A. The client will establish adequate daily nutritional
intake.
According to Maslow’s hierarchy of needs, all humans need to meet basic physiological
needs first. Because a client with anorexia nervosa eats little or nothing, the nurse must
first plan to help the client meet this basic, immediate physiological need. Treatment for
anorexia nervosa is centered on nutrition rehabilitation and psychotherapy. Refeeding
syndrome can occur following prolonged starvation. As the body utilizes glucose to
produce molecules of adenosine triphosphate (ATP), it depletes the remaining stores of
phosphorus. Also, glucose entry into cells is mediated by insulin and occurs rapidly
following long periods without food. Both cause electrolyte abnormalities such as
hypophosphatemia and hypokalemia, triggering cardiac and respiratory compromise.
Patients should be followed carefully for signs of refeeding syndrome and electrolytes
closely monitored.
 Option B: Recovery from an eating disorder can be a long process that requires
not only a qualified team of professionals but also the love and support of family
and friends. It is not uncommon for someone who suffers from an eating disorder
to feel uncertain about their progress or for their loved ones to feel disengaged
from the treatment process. These potential roadblocks may lead to feelings of
ambivalence, limited progress, and treatment dropout.
 Option C: Anorexia nervosa is a psychiatric disease in which patients restrict
their food intake relative to their energy requirements through eating less,
exercising more, and/or purging food through laxatives and vomiting. Despite
being severely underweight, they do not recognize it and have distorted body
images. They can develop complications from being underweight and purging
food. Diagnose by history, physical, and lab work that rules out other conditions
that can make people lose weight. Treatment includes gaining weight (sometimes
in a hospital if severe), therapy to address body image, and management of
complications from malnourishment.
 Option D: The nurse may give lesser priority to goals that address long-term
plans, self-perception, and potential complications. Eating disorders can affect
every organ system in the body, and people struggling with an eating disorder
need to seek professional help. The earlier a person with an eating disorder seeks
treatment, the greater the likelihood of physical and emotional recovery.

 9. Question 1 point(s)

When interviewing the parents of an injured child, which of the following is the strongest
indicator that child abuse may be a problem?

 A. The injury isn’t consistent with the history or the child’s age.
B. The mother and father tell different stories regarding what happened.
C. The family is poor.
D. The parents are argumentative and demanding with emergency department
personnel.

Incorrect Correct Answer: A. The injury isn’t consistent with the history or the
child’s age.
When the child’s injuries are inconsistent with the history given or impossible because of
the child’s age and developmental stage, the emergency department nurse should be
suspicious that child abuse is occurring. Physical indicators may include injuries to a
child that are severe, occur in a pattern or occur frequently. These injuries range from
bruises to broken bones to burns or unusual lacerations. The child may present for care
unrelated to the abuse, and the abuse may be found incidentally.
 Option B: The parents may tell different stories because their perception may be
different regarding what happened. If they change their story when different
health care workers ask the same question, this is a clue that child abuse may be a
problem. Physical abuse should be considered in the evaluation of all injuries of
children. A thorough history of present illness is important to make a correct
diagnosis. Important aspects of the history-taking involve gathering information
about the child’s behavior before, during, and after the injury occurred. History-
taking should include the interview of each caretaker separately and the verbal
child, as well. The parent or caretaker should be able to provide their history
without interruptions in order not to be influenced by the physician’s questions or
interpretations.
 Option C: Child abuse occurs in all socioeconomic groups. All races, ethnicities,
and socioeconomic groups are affected by child abuse with boys and adolescents
more commonly affected. Infants tend to have increased morbidity and mortality
with physical abuse. Multiple factors increase a child’s risk of abuse. These
include risks at an individual level (child’s disability, unmarried mother, maternal
smoking or parent’s depression); risks at a familial level (domestic violence at
home, more than two siblings at home); risks at a community level (lack of
recreational facilities); and societal factors (poverty).
 Option D: Parents may argue and be demanding because of the stress of having
an injured child. To diagnose a patient with child maltreatment is difficult since
the victim may be nonverbal or too frightened or severely injured to talk. Also,
the perpetrator will rarely admit to the injury, and witnesses are uncommon.
Physicians will see children of maltreatment in a range of ways that include the
perpetrators may be concerned that the abuse is severe and bring in the patient for
medical care.

 10. Question 1 point(s)

For a female client with anorexia nervosa, nurse Rose plans to include the parents in
therapy sessions along with the client. What fact should the nurse remember to be typical
of parents of clients with anorexia nervosa?

 A. They tend to overprotect their children.


B. They usually have a history of substance abuse.
C. They maintain emotional distance from their children.
D. They alternate between loving and rejecting their children.

Incorrect Correct Answer: A. They tend to overprotect their children.
Clients with anorexia nervosa typically come from a family with parents who are
controlling and overprotective. These clients use eating to gain control of an aspect of
their lives. Similarly, issues like anxiety, depression, and addiction can also run in
families, and have also been found to increase the chances that a person will develop an
eating disorder. Many people with anorexia report that, as children, they always followed
the rules and felt there was one “right way” to do things.
 Option B: Substance abuse and eating disorders frequently co-occur, with up to
50% of individuals with eating disorders who abuse alcohol or illicit drugs, a rate
five times higher than the general population. Substance abuse problems may
begin before or during an eating disorder, or even after recovery. Those struggling
with co-occurring substance use and disordered eating should speak with a trained
professional who can understand, diagnose, and treat both substance use disorders
and eating disorders.
 Option C: Loneliness and isolation are some of the hallmarks of anorexia; many
with the disorder report having fewer friends and social activities, and less social
support. Whether this is an independent risk factor or linked to other potential
causes (such as social anxiety) isn’t clear.
 Option D: Eating disorders are complex and affect all kinds of people. Risk
factors for all eating disorders involve a range of biological, psychological, and
sociocultural issues. These factors may interact differently in different people, so
two people with the same eating disorder can have very diverse perspectives,
experiences, and symptoms. Still, researchers have found broad similarities in
understanding some of the major risks for developing eating disorders.

 11. Question 1 point(s)

In the emergency department, a client with facial lacerations states that her husband beat
her with a shoe. After the health care team repairs her lacerations, she waits to be seen by
the crisis intake nurse, who will evaluate the continued threat of violence. Suddenly the
client’s husband arrives, shouting that he wants to “finish the job.” What is the first
priority of the health care worker who witnesses this scene?

 A. Remaining with the client and staying calm.


B. Calling a security guard and another staff member for assistance.
C. Telling the client’s husband that he must leave at once.
D. Determining why the husband feels so angry.

Incorrect Correct Answer: B. Calling a security guard and another staff member
for assistance.
The health care worker who witnesses this scene must take precautions to ensure personal
as well as client safety but shouldn’t attempt to manage a physically aggressive person
alone. Therefore, the first priority is to call a security guard and another staff member.
Domestic violence is defined as a pattern of abusive behaviors by one partner against
another in an intimate relationship such as marriage, dating, family, or cohabitation. In
this definition, domestic violence takes many forms, including physical aggression or
assault, sexual abuse, emotional abuse, controlling or domineering behavior, intimidation,
stalking, passive/covert abuse, and economic deprivation.
 Option A: After doing this, the health care worker should inform the husband
what is expected, speaking in concise statements, and maintaining a firm but calm
demeanor. This approach makes it clear that the health care worker is in control
and may diffuse the situation until the security guard arrives. Nurses can play an
important role in working toward the creation of a violence-free community but
they must first become informed. They must then insist the organizations in which
they work to accept this responsibility and work together to create environments
that support people experiencing domestic violence.
 Option C: Telling the husband to leave would probably be ineffective because of
his agitated and irrational state. Although the exact rates are widely disputed,
especially within the United States, there is a large body of cross-cultural
evidence that women are subjected to domestic violence significantly more often
than men. In addition, there is broad consensus that women are more often
subjected to severe forms of abuse and are more likely to be injured by an abusive
partner. According to a report by the United States Department of Justice, a
survey of 16,000 Americans showed 22.1 percent of women and 7.4 percent of
men reported being physically assaulted by a current or former spouse, cohabiting
partner, boyfriend, girlfriend, or date in their lifetime.
 Option D: Exploring his anger doesn’t take precedence over safeguarding the
client and staff. Gender roles and expectations play a role in abusive situations,
and exploring these roles and expectations can be helpful in addressing abusive
situations. Likewise, it can be helpful to explore factors such as race, class,
religion, sexuality, and philosophy. However, studies investigating whether sexist
attitudes are correlated with domestic violence have shown conflicting results.

 12. Question 1 point(s)

Nurse Mary is caring for a client with bulimia. Strict management of dietary intake is
necessary. Which intervention is also important?

 A. Fill out the client’s menu and make sure she eats at least half of what is on her
tray.
B. Let the client eat her meals in private. Then engage her in social activities for
at least 2 hours after each meal.
C. Let the client choose her own food. If she eats everything she orders, then stay
with her for 1 hour after each meal.
D. Let the client eat food brought in by the family if she chooses, but she should
keep a strict calorie count.

Incorrect Correct Answer: C. Let the client choose her own food. If she eats
everything she orders, then stay with her for 1 hour after each meal
Allowing the client to select her own food from the menu will help her feel some sense of
control. Assisting patients to remain strong and adhere to treatment requires nurses to
develop a relationship that is caring, empathetic and trusting, and in line with the person-
centered approach to care. Patients affected by eating disorders require individualized
support to better understand their condition, rediscover their identity, learn to accept
themselves, enhance a positive body image and sense of self-worth, and achieve a
balance in their lives so that they can move towards better health and wellbeing.
 Option A: She must then eat 100% of what she selected. During the early stages
of treatment when patients are still new to recovery, they look to nurses to provide
them with a highly structured environment, which sometimes involves nurses
making food and behavioral decisions on their behalf. While this might not be an
ongoing issue for primary care nurses, they may still be required to offer decisive
advice on these areas. Here, it is imperative that nurses offer such advice with a
clear message that patients have the power to make these decisions themselves.
 Option B: Remaining with the client for at least 1 hour after eating will prevent
purging. As treatment progresses, patients eventually grow to appreciate nurses
who act as role models and educate them on how to normalize their diet and
involvement in social activities. Towards the end of treatment, nurses become
more of a support system, encouraging the patient to move forward
autonomously, while providing them with guidance on where to seek help if it is
needed.
 Option D: Bulimic clients should only be allowed to eat food provided by the
dietary department. From awareness of the eating disorder to recovery
maintenance, the role of the primary care nurse evolves, but what doesn’t change
is the positive influence nurses can have on those with an eating disorder. With
the skills of listening, empathy, adaptability, and communication, primary care
nurses can assist in identifying at-risk individuals and optimizing the delivery of a
multidisciplinary and holistic approach to care.

 13. Question 1 point(s)

Nurse Mary is assigned to care for a suicidal client. Initially, which is the nurse’s highest
care priority?

 A. Assessing the client’s home environment and relationships outside the hospital.

B. Exploring the nurse’s own feelings about suicide.


C. Discussing the future with the client.
D. Referring the client to a clergyperson to discuss the moral implications of
suicide.

Incorrect Correct Answer: B. Exploring the nurse’s own feelings about suicide.
The nurse’s values, beliefs, and attitudes toward self-destructive behavior influence
responses to a suicidal client; such responses set the overall mood for the nurse-client
relationship. Therefore, the nurse initially must explore personal feelings about suicide to
avoid conveying negative feelings to the client.
 Option A: Assessment of the client’s home environment and relationships may
reveal the need for family therapy; however, conducting such an assessment isn’t
a nursing priority. A clear and complete evaluation and clinical interview provide
the information upon which to base a suicide intervention. Although risk factors
offer major indications of the suicide danger, nothing can substitute for a focused
patient inquiry. However, although all the answers a patient gives may be
inclusive, a therapist often develops a visceral sense that his or her patient is
going to commit suicide. The clinician’s reaction counts and should be considered
in the intervention.
 Option C: Discussing the future and providing anticipatory guidance can help the
client prepare for future stress, but this isn’t a priority. If suicidal ideation is
present, the next question must be about any plans for suicidal acts. The general
formula is that more specific plans indicate greater danger. Although vague
threats, such as a threat to commit suicide sometime in the future, are the reason
for concern, responses indicating that the person has purchased a gun, has
ammunition, has made out a will, and plans to use the gun are more dangerous.
The plan demands further questions. If the person envisions a gun-related death,
determine whether he or she has the weapon or access to it.
 Option D: Referring the client to a clergyperson may increase the client’s trust or
alleviate guilt; however, it isn’t the highest priority. The only way to prevent
suicides is to work in an interprofessional team that includes a mental health
nurse, psychiatrist, the primary care provider, social worker, and nurse
practitioner. Practitioners must work with the patient’s family and friends, as well
as with the other patients who knew the client.

 14. Question 1 point(s)

A 24-year old client with anorexia nervosa tells the nurse, “When I look in the mirror, I
hate what I see. I look so fat and ugly.” Which strategy should the nurse use to deal with
the client’s distorted perceptions and feelings?

 A. Avoid discussing the client’s perceptions and feelings.


B. Focus discussions on food and weight.
C. Avoid discussing unrealistic cultural standards regarding weight.
D. Provide objective data and feedback regarding the client’s weight and
attractiveness.

Correct Correct Answer: D. Provide objective data and feedback regarding the
client’s weight and attractiveness
By focusing on reality, this strategy may help the client develop a more realistic body
image and gain self-esteem. Anorexia nervosa is an eating disorder defined by restriction
of energy intake relative to requirements, leading to a significantly low body weight.
Patients will have an intense fear of gaining weight and distorted body image with the
inability to recognize the seriousness of their significantly low body weight. The mental
health nurse should educate the patient on changes in behavior, easing stress, and
overcoming any emotional issues.
 Option A: This is inappropriate because discussing the client’s perceptions and
feelings wouldn’t help her to identify, accept, and work through them. Since
recovery involves patients having to face their deepest, most painful, and
traumatic thoughts and emotions, supporting them as they go through treatment
can be emotionally challenging for nurses. This emotional challenge can be
exacerbated when the patient has also been diagnosed with Obsessive-
Compulsive Disorder (OCD), depression, or substance abuse, as these may
require more intensive one-to-one support.
 Option B: Focusing discussions on food and weight would give the client
attention for not eating. During the early stages of treatment when patients are still
new to recovery, they look to nurses to provide them with a highly structured
environment, which sometimes involves nurses making food and behavioral
decisions on their behalf. While this might not be an ongoing issue for primary
care nurses, they may still be required to offer decisive advice on these areas.
Here, it is imperative that nurses offer such advice with a clear message that
patients have the power to make these decisions themselves.
 Option C: This is inappropriate because recognizing unrealistic cultural standards
wouldn’t help the client establish more realistic weight goals. Furthermore,
learning motivational interviewing techniques can help facilitate communication
with those who might be resistant to discussing topics related to food, weight, and
recovery. Such techniques can help develop the skills of empathic understanding,
rolling with resistance, and gently assisting patients to make their own,
autonomous decision to work towards recovery. Often, the aim is to help patients
learn new and healthier ways of coping, and nurses can achieve this through a mix
of emotional support, education, and signposting.

 15. Question 1 point(s)

Nurse Alice is caring for a client being treated for alcoholism. Before initiating therapy
with disulfiram (Antabuse), the nurse teaches the client that he must read labels carefully
on which of the following products?

 A. Carbonated beverages
B. Aftershave lotion
C. Toothpaste
D. Cheese

Incorrect Correct Answer: B. Aftershave lotion
Disulfiram may be given to clients with chronic alcohol abuse who wish to curb impulse
drinking. Disulfiram works by blocking the oxidation of alcohol, inhibiting the
conversion of acetaldehyde to acetate. As acetaldehyde builds up in the blood, the client
experiences noxious and uncomfortable symptoms. Even alcohol rubbed onto the skin
can produce a reaction. The client receiving disulfiram must be taught to read ingredient
labels carefully to avoid products containing alcohol such as aftershave lotions. Close
monitoring of adverse events is necessary, in particular, in patients with polysubstance
abuse. Patients taking disulfiram require monitoring for signs and symptoms of hepatitis,
including fatigue, weakness, anorexia, nausea, vomiting, jaundice, malaise, and dark
urine.
 Option A: Disulfiram is one of three drugs approved by the FDA for the
treatment of alcohol dependence. It is a second-line option (acamprosate and
naltrexone are first-line treatments) in patients with sufficient physician
supervision. Disulfiram is safe and efficient in supervised short-term and long-
term treatment of individuals dependent on alcohol but who are motivated to
discontinue alcohol use.
 Option C: Disulfiram irreversibly inhibits aldehyde dehydrogenase (ALDH1A1)
by competing with nicotinamide adenine dinucleotide (NAD) at the cysteine
residue in the active site of the enzyme. ALDH1A1 is a hepatic enzyme of the
major oxidative pathway of alcohol metabolism converting ethanol to
acetaldehyde. At therapeutic doses of disulfiram, alcohol consumption results in
increased serum acetaldehyde, causing diaphoresis, palpitations, facial flushing,
nausea, vertigo, hypotension, and tachycardia.
 Option D: Patients receiving metronidazole, paraldehyde, alcohol, or alcohol-
containing preparations (sauces, cough mixtures, vinegar) should not receive
disulfiram and should be educated in advance to avoid a disulfiram-alcohol
reaction. Never administer to a patient if alcohol use is suspected or without the
patient’s consent and understanding of disulfiram-alcohol reaction.

 16. Question 1 point(s)

Nurse Harry is developing a plan of care for a client with anorexia nervosa. Which action
should the nurse include in the plan?

 A. Restrict visits with the family until the client begins to eat.
B. Provide privacy during meals.
C. Set up a strict eating plan for the client.
D. Encourage the client to exercise, which will reduce her anxiety.

Incorrect Correct Answer: C. Set up a strict eating plan for the client.
Establishing a consistent eating plan and monitoring the client’s weight is important for
this disorder. Establish a minimum weight goal and daily nutritional requirements.
Malnutrition is a mood-altering condition, leading to depression and agitation and
affecting cognitive function and decision making. Improved nutritional status enhances
thinking ability, allowing initiation of psychological work. Make a selective menu
available, and allow the patient to control choices as much as possible. Patient who gains
confidence in himself and feels in control of the environment is more likely to eat
preferred foods.
 Option A: The family should be included in the client’s care. Involve patients in
setting up or carrying out a program of behavior modification. Provide a reward
for weight gain as individually determined; ignore the loss. It provides a
structured eating situation while allowing the patient some control in choices.
Behavior modification may be effective in mild cases or for short-term weight
gain.
 Option B: The client should be monitored during meals — not given privacy.
Provide one-to-one supervision and have a patient with bulimia remain in the day
room area with no bathroom privileges for a specified period (2 hr) following
eating, if contracting is unsuccessful. Prevents vomiting during and after eating.
Patients may desire food and use a binge-purge syndrome to maintain weight.
Note: Patients may purge for the first time in response to the establishment of a
weight gain program.
 Option D: Exercise must be limited and supervised. Monitor exercise programs
and set limits on physical activities. Chart activity and level of work (pacing and
so on). Moderate exercise helps in maintaining muscle tone, weight and
combating depression; however, patients may exercise excessively to burn
calories.

 17. Question 1 point(s)

Nurse Taylor is aware that the victims of domestic violence should be assessed for what
important information?

 A. Reasons they stay in the abusive relationship (for example, lack of financial
autonomy and isolation).
B. Readiness to leave the perpetrator and knowledge of resources.
C. Use of drugs or alcohol.
D. History of previous victimization.

Incorrect Correct Answer: B. Readiness to leave the perpetrator and knowledge of
resources.
Victims of domestic violence must be assessed for their readiness to leave the perpetrator
and their knowledge of the resources available to them. Nurses can then provide the
victims with information and options to enable them to leave when they are ready.
Training and support programs for clinicians and administrative staff have been shown to
improve identification of women experiencing domestic violence and referral to
advocacy services. Use of a domestic violence advocate in the ED resulted in a higher
incidence of detection of incidents of acute violence than the data reported in the
literature.
 Option A: The reasons they stay in the relationship are complex and can be
explored at a later time. Reportedly, at least 40% of domestic violence victims
never contact the police. Of female victims of domestic violence homicide, 44%
had visited an ED within 2 years of their murder.
 Option C: The use of drugs or alcohol is irrelevant. Since substance abuse may
develop or worsen as a result of domestic violence, it is appropriate to consider
domestic violence when evaluating a patient for alcohol intoxication, drug
toxicity, or drug overdose. A family history of alcohol and drug abuse or similar
history in the patient’s partner is also an important risk factor.
 Option D: There is no evidence to suggest that previous victimization results in a
person’s seeking or causing abusive relationships. The frequency and severity of
previous attacks indicate the degree of present danger. Threats are as important as
any actual injury. The presence of weapons in the home is a risk factor. In
addition to threats and physical abuse, relationships with high risk for injury or
death commonly feature exaggerated forms of coercion and manipulation to
maintain the partner’s dependence. This may result in the Stockholm syndrome.

 18. Question 1 point(s)

A male client is hospitalized with fractures of the right femur and right humerus sustained
in a motorcycle accident. Police suspect the client was intoxicated at the time of the
accident. Laboratory tests reveal a blood alcohol level of 0.2% (200 mg/dl). The client
later admits to drinking heavily for years. During hospitalization, the client periodically
complains of tingling and numbness in the hands and feet. Nurse Gian realizes that these
symptoms probably result from:

 A. Acetate accumulation
B. Thiamine deficiency
C. Triglyceride buildup.
D. A below-normal serum potassium level

Incorrect Correct Answer: B. Thiamine deficiency
Numbness and tingling in the hands and feet are symptoms of peripheral polyneuritis,
which results from inadequate intake of vitamin B1 (thiamine) secondary to prolonged
and excessive alcohol intake. Treatment includes reducing alcohol intake, correcting
nutritional deficiencies through diet and vitamin supplements, and preventing such
residual disabilities as foot and wrist drop.
 Option A: When thiamine stores are depleted (which takes about 4 weeks after
stopping intake), symptoms start to appear. When evaluating for thiamine
deficiency, the typical history may include poor nutritional intake, excessive
alcohol intake, or the patient belonging to the special populations of individuals
previously mentioned (pregnant women, recipients of bariatric surgery, patients
with prolonged diuretic use, anyone with poor overall nutritional status, etc.).
 Option C: Initial symptoms of B1 deficiency include anorexia, irritability, and
difficulties with short-term memory. With prolonged thiamine deficiency, patients
may endorse loss of sensation in the extremities, symptoms of heart failure
including swelling of the hands or feet and chest pain related to demand ischemia,
or feelings of vertigo, double vision, and memory loss. Additionally, close friends
and family of the patient may describe confusion or symptoms of confabulation.
 Option D: Detection of thiamine deficiency relies on relevant history and
physical exam findings and follow up with laboratory testing for confirmation.
Functional enzymatic assay of transketolase activity is the activity of
transketolase measured before and after the addition of thiamine pyrophosphate;
>25% stimulation response is abnormal. Measurement of thiamine or the
phosphorylated esters of thiamine in serum or blood using high-performance
liquid chromatography is used. Urine studies exist but are not a reliable test for
the evaluation of total body thiamine.

 19. Question 1 point(s)

A parent brings a preschooler to the emergency department for treatment of a dislocated


shoulder, which allegedly happened when the child fell down the stairs. Which action
should make the nurse suspect that the child was abused?

 A. The child cries uncontrollably throughout the examination.


B. The child pulls away from contact with the physician.
C. The child doesn’t cry when the shoulder is examined.
D. The child doesn’t make eye contact with the nurse.

Incorrect Correct Answer: C. The child doesn’t cry when the shoulder is examined.
A characteristic behavior of abused children is the lack of crying when they undergo a
painful procedure or are examined by a healthcare professional. Therefore, the nurse
should suspect child abuse. Physical abuse may include beating, shaking, burning, and
biting. The threshold for defining corporal punishment as abuse is unclear. Rib fractures
are found to be the most common finding associated with physical abuse. Any child
younger than two years old for whom there is a concern of physical abuse should have a
skeletal survey. Additionally, any sibling younger than two years of age of an abused
child should also have a skeletal survey. A skeletal survey consists of 21 dedicated views,
as recommended by the American College of Radiology.
 Option A: The World Health Organization (WHO) defines child maltreatment as
“all forms of physical and emotional ill-treatment, sexual abuse, neglect, and
exploitation that results in actual or potential harm to the child’s health,
development or dignity.” There are four main types of abuse: neglect, physical
abuse, psychological abuse, and sexual abuse. Abuse is defined as an act of
commission and neglect is defined as an act of omission in the care leading to
potential or actual harm.
 Option B: Physical abuse should be considered in the evaluation of all injuries of
children. A thorough history of present illness is important to make a correct
diagnosis. Important aspects of the history-taking involve gathering information
about the child’s behavior before, during, and after the injury occurred. History-
taking should include the interview of each caretaker separately and the verbal
child, as well. The parent or caretaker should be able to provide their history
without interruptions in order not to be influenced by the physician’s questions or
interpretations.
 Option D: The second most common type of child abuse after neglect is physical
abuse. Eighty percent of abusive fractures occur in non-ambulatory children,
particularly in children younger than 18 months of age. The most important risk
factor for abusive skeletal injury is age. There is no fracture pathognomonic for
abuse, but there are some fractures that are more suggestive of abuse. These
include posterior or lateral rib fractures and “corner” or “bucket handle” fractures,
which occur at the ends of long bones and which result from a twisting
mechanism. Other highly suspicious fractures are sternal, spinal and scapular
fractures.

 20. Question 1 point(s)

When planning care for a client who has ingested phencyclidine (PCP), nurse Wayne is
aware that the following is the highest priority?

 A. Client’s physical needs


B. Client’s safety needs
C. Client’s psychosocial needs
D. Client’s medical needs

Incorrect Correct Answer: B. Client’s safety needs
The highest priority for a client who has ingested PCP is meeting safety needs of the
client as well as the staff. Drug effects are unpredictable and prolonged, and the client
may lose control easily. Phencyclidine (PCP) is a dissociative anesthetic that is a
commonly used recreational drug. PCP is a crystalline powder that can be ingested orally,
injected intravenously, inhaled, or smoked. PCP is available as a powder, crystal, liquid,
and tablet. It produces both stimulation and depression of the CNS. PCP is a non-
competitive antagonist to the NMDA receptor, which causes analgesia, anesthesia,
cognitive defects, and psychosis.
 Option A: Depending on the dose and route of administration, PCP can have a
wide range of central nervous system (CNS) manifestations. Emergency
department providers should become familiar with how to manage patients with
PCP toxicity since rhabdomyolysis, hypoglycemia, seizures, hypertensive crisis,
coma, and trauma are several of the complications that can arise with PCP use
 Option C: PCP blocks the uptake of dopamine and norepinephrine, leading to
sympathomimetic effects such as hypertension, tachycardia, bronchodilation, and
agitation. PCP can also cause sedation, muscarinic, and nicotinic signs by binding
to acetylcholine receptors and GABA receptors. Sigma receptor stimulation by
PCP causes lethargy and coma.
 Option D: Most patients survive PCP intoxication with supportive care. Airway,
breathing, circulation, and hemodynamic monitoring are essential to the care of
patients with PCP toxicity. Intubation with ventilatory support may be required
for airway protection. Gastrointestinal decontamination is generally unnecessary
in PCP ingestions; however, activated charcoal may be beneficial with a massive
ingestion of PCP or a dangerous coingestion. Activated charcoal therapy should
only be started within one hour from the time of ingestion. The activated charcoal
dose is 1 g/kg, with a maximum dose of 50 g.

 21. Question 1 point(s)

The nurse is aware that the outcome criteria would be appropriate for a child diagnosed
with oppositional defiant disorder?

 A. Accept responsibility for own behaviors.


B. Be able to verbalize own needs and assert rights.
C. Set firm and consistent limits with the client.
D. Allow the child to establish his own limits and boundaries.

Incorrect Correct Answer: A. Accept responsibility for own behaviors
Children with oppositional defiant disorder frequently violate the rights of others. They
are defiant, disobedient, and blame others for their actions. Accountability for their
actions would demonstrate progress for the oppositional child. Oppositional defiant
disorder (ODD) is a type of childhood disruptive behavior disorder that primarily
involves problems with the self-control of emotions and behaviors. According to the
Diagnostic and Statistical Manual of Mental Disorders, Fifth Edition (DSM-5), the main
feature of ODD is a persistent pattern of angry or irritable mood, argumentative or defiant
behavior, or vindictiveness toward others.
 Option B: This is incorrect as the oppositional child usually, focuses on his own
needs. Temperamental factors such as irritability, impulsivity, poor frustration,
tolerance, and high levels of emotional reactivity are commonly associated with
ODD. While not all children diagnosed with ODD show callous and unemotional
traits, it has been shown that such traits are highly heritable and may be seen more
frequently in a subset of children with more significant disruptive behaviors.
 Option C: Treatment of oppositional defiant disorder is multimodal and should
involve the patient, family, school, and community. Identifying and treating
comorbidities (like ADHD, depression, and anxiety) and modifiable risk factors
(such as bullying and learning difficulties) should be done. Treatment may also
vary based on whether oppositional behavior primarily occurs in specific contexts
or if the behavior is pervasive and thus requires more intensive treatment.
 Option D: This is not an outcome criterion but an intervention. Parent
Management Training or PMT is based on the principles of social learning theory
and is the main treatment for oppositional behaviors. The guiding principle in
PMT is the use of operant conditioning (using the role of positive reinforcement
in changing behaviors) to decrease unwanted behaviors and promote prosocial
behaviors. Methods include teaching parents to identify problem behaviors as
well as positive interactions and to apply punishment or reinforcement as
appropriate.

 22. Question 1 point(s)

A male client is found sitting on the floor of the bathroom in the day treatment clinic with
moderate lacerations on both wrists. Surrounded by broken glass, he sits staring blankly
at his bleeding wrists while staff members call for an ambulance. How should Nurse
Anuktakanuk approach her initially?

 A. Enter the room quietly and move beside him to assess his injuries.
B. Call for staff back-up before entering the room and restraining him.
C. Move as much glass away from him as possible and sit next to him quietly.
D. Approach him slowly while speaking in a calm voice, calling his name, and
telling him that the nurse is here to help him.

Correct Correct Answer: D. Approach her slowly while speaking in a calm voice,
calling her name, and telling her that the nurse is here to help her
Ensuring the safety of the client and the nurse is the priority at this time. Therefore, the
nurse should approach the client cautiously while calling her name and talking to her in a
calm, confident manner. Nursing’s hands-on approach to patient care and our ability to
create therapeutic connections with patients enables us to pick up on key cues.
Identifying these cues starts with understanding that suicidal behaviors are neither
considered an illness nor a condition, but rather a complex set of behaviors that actually
exists on a continuum that ranges from ideas/thoughts to eventual actions.
 Option A: The nurse should keep in mind that the client shouldn’t be startled or
overwhelmed. After explaining that the nurse is there to help, the nurse should
observe the client’s response carefully. The promotion of a care environment that
is safe and conducive to their full recovery is essential in carrying out
comprehensive care in mental health. The first step is qualified listening, but it
cannot be immersed in a bigoted discourse, full of judgment. One must consider
that not always the person is willing to express or externalize what they really
feel, and so a new challenge to the health professional emerges, which is the
careful observation of the reality of the patient and the listening of silence when
the person is not willing to talk.
 Option B: If the client shows signs of agitation or confusion or poses a threat, the
nurse should retreat and request assistance. For the care to surpass the technical
focus, the psychological care and the continuous observation of patients and
family members are also necessary, aiming to prioritize the communication in
accordance with the qualified listening, as these patients are often insecure. It is
important to highlight that all people who attempted suicide should receive
professional care due to the emotional fragility in which they find themselves.
The competence of the emergency team is saving lives, considering not only the
physical aspects but also the psychological aspects involved in the process of
caring
 Option C: The nurse shouldn’t attempt to sit next to the client or examine injuries
without first announcing the nurse’s presence and assessing the dangers of the
situation. There are some essential behaviors that nursing can use to meet a person
who attempted suicide or has suicidal ideation, namely: listen carefully, be
empathetic, convey non-verbal messages of acceptance, express respect for the
opinion of another, talk honestly, show concern, and focus on the feelings of the
person. The mere interaction with the patient has a great potential to calm down,
prevent, or minimize the severity and intensity of the symptoms. Still, the team
should try to establish a bond of trust from the start, whereas, on the other hand,
the idea that the patient attempted suicide to manipulate others should be
abandoned.

 23. Question 1 point(s)

A female client with anorexia nervosa describes herself as “a whale.” However, the
nurse’s assessment reveals that the client is 5′ 8″ (1.7 m) tall and weighs only 90 lb (40.8
kg). Considering the client’s unrealistic body image, which intervention should nurse
Angel be included in the plan of care?

 A. Asking the client to compare her figure with magazine photographs of women
her age.
B. Assigning the client to group therapy in which participants provide realistic
feedback about her weight.
C. Confronting the client about her actual appearance during one-on-one sessions,
scheduled during each shift.
D. Telling the client of the nurse’s concern for her health and desire to help
her make decisions to keep her healthy.

Correct Correct Answer: D. Telling the client of the nurse’s concern for her health
and desire to help her make decisions to keep her healthy
A client with anorexia nervosa has an unrealistic body image that causes consumption of
little or no food. Therefore, the client needs assistance with making decisions about
health. Respond (confront) with reality when a patient makes unrealistic statements. The
patient may be denying the psychological aspects of their own situation and is often
expressing a sense of inadequacy and depression.
 Option A: Instead of protecting the client’s health, option A may serve to make
the client defensive and more entrenched in her unrealistic body image. Allow the
patient to draw a picture of self. It provides an opportunity to discuss the patient’s
perception of self and body image and realities of an individual situation.
 Option B: Encourage personal development program, preferably in a group
setting. Provide information about the proper application of makeup and
grooming. Learning about methods to enhance personal appearance may be
helpful to a long-range sense of self-esteem and image. Feedback from others can
promote feelings of self-worth.
 Option C: Establish a therapeutic nurse-patient relationship. Within a helping
relationship, the patient can begin to trust and try out new thinking and behaviors.
Assist the patient to assume control in areas other than dieting and weight loss
such as management of their own daily activities, work, and leisure choices.
Feelings of personal ineffectiveness, low self-esteem, and perfectionism are often
part of the problem. The patient feels helpless to change and requires assistance to
problem-solve methods of control in life situations.

 24. Question 1 point(s)

Eighteen hours after undergoing an emergency appendectomy, a client with a reported


history of social drinking displays these vital signs: temperature, 101.6° F (38.7° C);
heart rate, 126 beats/minute; respiratory rate, 24 breaths/minute; and blood pressure,
140/96 mm Hg. The client exhibits gross hand tremors and is screaming for someone to
kill the bugs in the bed. Nurse Melinda should suspect:

 A. A postoperative infection
B. Alcohol withdrawal
C. Acute sepsis.
D. Pneumonia.

Incorrect Correct Answer: B. Alcohol withdrawal
The client’s vital signs and hallucinations suggest delirium tremens or alcohol withdrawal
syndrome. Alcohol withdrawal symptoms occur when patients stop drinking or
significantly decrease their alcohol intake after long-term dependence. Withdrawal has a
broad range of symptoms from mild tremors to a condition called delirium tremens,
which results in seizures and could progress to death if not recognized and treated
promptly.
 Option A: GABA (gamma-aminobutyric acid) is the major inhibitory
neurotransmitter in the central nervous center. GABA has particular binding sites
available for ethanol, thus increasing the inhibition of the central nervous system
when present. Chronic ethanol exposure to GABA creates constant inhibition or
depressant effects on the brain. Ethanol also binds to glutamate, which is one of
the excitatory amino acids in the central nervous system. When it binds to
glutamate, it inhibits the excitation of the central nervous system, thus worsening
the depression of the brain.
 Option C: Alcohol withdrawal can range from very mild symptoms to the severe
form, which is named delirium tremens. The hallmark is autonomic dysfunction
resulting from the excitation of the central nervous system. Mild signs/symptoms
can arise within six hours of alcohol cessation. If symptoms do not progress to
more severe symptoms within 24 to 48 hours, the patient will likely recover.
 Option D: Although pneumonia may arise as postoperative complications; it
wouldn’t cause this client’s signs and symptoms and typically would occur later
in the postoperative course. Mild symptoms can be insomnia, tremulousness,
hyperreflexia, anxiety, gastrointestinal upset, headache, palpitations. Moderate
symptoms include alcohol withdrawal seizures (rum fits) that can occur 12 to 24
hours after cessation of alcohol and are typically generalized in nature. There is a
3% incidence of status epilepticus in these patients. About 50% of patients who
have had a withdrawal seizure will progress to delirium tremens.

 25. Question 1 point(s)


Clonidine (Catapres) can be used to treat conditions other than hypertension. Nurse Sally
is aware that the following conditions might the drug be administered?

 A. Phencyclidine (PCP) intoxication


B. Alcohol withdrawal
C. Opiate withdrawal
D. Cocaine withdrawal

Incorrect Correct Answer: C. Opiate withdrawal
Clonidine is used as adjunctive therapy in opiate withdrawal. Symptomatic treatment in
opioid withdrawal includes loperamide for diarrhea, promethazine for nausea/vomiting,
and ibuprofen for myalgia. Clonidine can be given to reduce blood pressure. Opioid
withdrawal syndrome is a life-threatening condition resulting from opioid dependence.
Opioids are a group of drugs used for the management of severe pain. They are also
commonly used as psychoactive substances around the world.
 Option A: Benzodiazepines and neuroleptic agents are typically used to treat PCP
intoxication. Benzodiazepines are the preferred medication for chemical sedation
in patients with PCP toxicity. Lorazepam 2 to 4 mg intravenous (IV) or
intramuscular (IM), or diazepam 5 to 10 mg IV or IM are recommended.
Benzodiazepines are also the first-line treatment for PCP-induced hypertension
and seizures. Hyperthermia from PCP toxicity is due to psychomotor agitation
and can be successfully treated with benzodiazepines as well.
 Option B: Benzodiazepines, such as chlordiazepoxide (Librium), and neuroleptic
agents, such as haloperidol, are used to treat alcohol withdrawal. The hallmark of
management for severe symptoms is the administration of long-acting
benzodiazepines. The most commonly used benzodiazepines are intravenous
diazepam (Valium) or intravenous lorazepam (Ativan) for management. Patients
with severe withdrawal symptoms may require escalating doses and intensive care
level monitoring.
 Option D: Antidepressants and medications with dopaminergic activity in the
brain, such as fluoxetine (Prozac), are used to treat cocaine withdrawal. Central
nervous system (CNS) stimulants like cocaine and amphetamine can also produce
withdrawal symptoms. Like opioids, the withdrawal symptoms are mild and not
life-threatening. Often the individual will develop marked depression, excessive
sleep, hunger, dysphoria, and severe psychomotor retardation but all vital
functions are well preserved. Recovery is usually slow, and depression can last for
several weeks.

 26. Question 1 point(s)

A male client with a history of cocaine addiction is admitted to the coronary care unit for
evaluation of substernal chest pain. The electrocardiogram (ECG) shows a 1-mm ST-
segment elevation of the anteroseptal leads and T-wave inversion in leads V3 to V5.
Considering the client’s history of drug abuse, nurse Greg expects the physician to
prescribe:
 A. Lidocaine (Xylocaine).
B. Procainamide (Pronestyl).
C. Nitroglycerin (Nitro-Bid IV).
D. Epinephrine.

Incorrect Correct Answer: C. Nitroglycerin (Nitro-Bid IV).
The elevated ST segments in this client’s ECG indicate myocardial ischemia. To reverse
this problem, the physician is most likely to prescribe an infusion of nitroglycerin to
dilate the coronary arteries. Nitroglycerin is a vasodilatory drug used primarily to provide
relief from anginal chest pain. Although nitroglycerin has a vasodilatory effect in both
arteries and veins, the profound desired effects caused by nitroglycerin are primarily due
to venodilation. Venodilation causes pooling of blood within the venous system, reducing
preload to the heart, which causes a decrease in cardiac work, reducing anginal symptoms
secondary to demand ischemia.
 Option A: Lidocaine, formerly also referred to as lignocaine, is an amide local
anesthetic agent. The drug is commonly used for local anesthesia, often in
combination with epinephrine (which acts as a vasopressor and extends its
duration of action at a site by opposing the local vasodilatory effects of lidocaine).
 Option B: Procainamide is a cardiac drug that may be indicated for this client at
some point but isn’t used for coronary artery dilation. Procainamide is a
medication used in the management and treatment of ventricular arrhythmias,
supraventricular arrhythmias, atrial flutter, atrial fibrillation, AV nodal reentrant
tachycardia, and Wolf-Parkinson-White syndrome. It is a Class 1A antiarrhythmic
agent.
 Option D: If a cocaine user experiences ventricular fibrillation or asystole, the
physician may prescribe epinephrine. However, this drug must be used with
caution because cocaine may potentiate its adrenergic effects. Epinephrine is one
of the most commonly used agents in a variety of settings as it functions as
medication and hormone. It is currently FDA-approved for various situations,
including emergency treatment of type 1 hypersensitivity reactions including
anaphylaxis, induction, and maintenance of mydriasis during intraocular
surgeries, and hypotension due to septic shock.

 27. Question 1 point(s)

A 14-year-old client was brought to the clinic by her mother. Her mother expresses
concern about her daughter’s weight loss and constant dieting. Nurse Kris conducts a
health history interview. Which of the following comments indicates that the client may
be suffering from anorexia nervosa?

 A. “I like the way I look. I just need to keep my weight down because I’m a
cheerleader.”
B. “I don’t like the food my mother cooks. I eat plenty of fast food when I’m out
with my friends.”
C. “I just can’t seem to get down to the weight I want to be. I’m so fat compared
to other girls.”
D. “I do diet around my periods; otherwise, I just get so bloated.”

Incorrect Correct Answer: C. “I just can’t seem to get down to the weight I want to
be. I’m so fat compared to other girls.”
Low self-esteem is the highest risk factor for anorexia nervosa. Constant dieting to get
down to a “desirable weight” is characteristic of the disorder. Feeling inadequate when
compared to peers indicates poor self-esteem. Anorexia is also more common among
teenagers. Still, people of any age can develop this eating disorder, though it’s rare in
those over 40. Teens may be more at risk because of all the changes their bodies go
through during puberty. They may also face increased peer pressure and be more
sensitive to criticism or even casual comments about weight or body shape.
 Option A: Most clients with anorexia nervosa don’t like the way they look, and
their self-perception may be distorted. A girl with cachexia may perceive herself
to be overweight when she looks in the mirror. Some people with anorexia may
have obsessive-compulsive personality traits that make it easier to stick to strict
diets and forgo food despite being hungry. They may have an extreme drive for
perfectionism, which causes them to think they’re never thin enough. And they
may have high levels of anxiety and engage in restrictive eating to reduce it.
 Option B: Preferring fast food over healthy food is common in this age-group.
Although it’s not yet clear which genes are involved, there may be genetic
changes that make some people at higher risk of developing anorexia. Some
people may have a genetic tendency toward perfectionism, sensitivity, and
perseverance — all traits associated with anorexia.
 Option D: Because of the absence of body fat necessary for proper hormone
production, amenorrhea is common for a client with anorexia nervosa. Patients
will report symptoms such as amenorrhea, cold intolerance, constipation,
extremity edema, fatigue, and irritability. They may describe restrictive behaviors
related to food like calorie counting or portion control, and purging methods, for
example, self-induced vomiting or use of diuretics or laxatives.

 28. Question 1 point(s)

Nurse Fey is aware that the drug of choice for treating Tourette syndrome?

 A. Fluoxetine (Prozac)
B. Fluvoxamine (Luvox)
C. Haloperidol (Haldol)
D. Paroxetine (Paxil)

Incorrect Correct Answer: C. Haloperidol (Haldol)
Haloperidol is the drug of choice for treating Tourette syndrome. Antipsychotic
medications have been the most extensively studied. Haloperidol and pimozide are the
first-generation antipsychotics with the most data showing efficacy in reducing tic
severity. However, their use is limited by potentially severe side effects such as sedation,
acute dystonia, and other drug-induced movement disorders like weight gain, and
prolonged QTc interval (pimozide).
 Option A: Fluoxetine has FDA-approval for major depressive disorder (age eight
and older), obsessive-compulsive disorder (age seven and older), panic disorder,
bulimia, binge eating disorder, premenstrual dysphoric disorder, bipolar
depression (as an adjunct with olanzapine also known as Symbyax), and
treatment-resistant depression when used in combination with olanzapine.
 Option B: Fluvoxamine is used to treat obsessive-compulsive disorder (OCD). It
helps decrease persistent/unwanted thoughts (obsessions) and urges to perform
repeated tasks (compulsions such as hand-washing, counting, checking) that
interfere with daily living. Fluvoxamine is known as a selective serotonin
reuptake inhibitor (SSRI). This medication works by helping to restore the
balance of a certain natural substance (serotonin) in the brain.
 Option D: Paroxetine is a selective serotonin reuptake inhibitor (SSRI), and, as
such, is identified as an antidepressant. It is FDA approved for major depressive
disorder (MDD), obsessive-compulsive disorder (OCD), social anxiety disorder
(SAD), panic disorder, posttraumatic stress disorder (PTSD), generalized anxiety
disorder (GAD), and premenstrual dysphoric disorder (PMDD), vasomotor
symptoms associated with menopause.

 29. Question 1 point(s)

A male client tells the nurse he was involved in a car accident while he was intoxicated.
What would be the most therapeutic response from nurse Julia?

 A. “Why didn’t you get someone else to drive you?”


B. “Tell me how you feel about the accident.”
C. “You should know better than to drink and drive.”
D. “I recommend that you attend an Alcoholics Anonymous meeting.”

Incorrect Correct Answer: B. “Tell me how you feel about the accident.”
An open-ended statement or question is the most therapeutic response. It encourages the
widest range of client responses, makes the client an active participant in the
conversation, and shows the client that the nurse is interested in his feelings. mix open-
ended questions with focus questions. Open-ended questions may allow the patient to
express their thoughts and feelings, and focused questions allow the interviewer to obtain
important details with yes or no answers in a more time-efficient manner.
 Option A: Asking the client why he drove while intoxicated can make him feel
defensive and intimidated. The first question posed in the interview is often open-
ended. For example, “What is the main reason you seek medical assistance
today?” This provides an opportunity for the interviewer to allow the patient to
share their concerns, and the interviewer can show he or she is actively listening.
This includes listening without judgment and displaying concern for the patient
during communication.
 Option C: A judgmental approach isn’t therapeutic. During the interview,
meaningful questions inquired positively will reduce defensiveness from the
patient. Often this can be accomplished by suggesting or sharing a common
behavior associated with the actions of the patient. For example, the interviewer
may convey the commonality for people to consume alcohol when under stress. It
then becomes acceptable to inquire if this is also occurring with the patient. The
patient may feel a sense of trust and therefore share pertinent information.
 Option D: By giving advice, the nurse suggests that the client isn’t capable of
making decisions, thus fostering dependency. At the conclusion of the patient
interview, an appropriate transition statement to begin the physical exam may be,
“Is there anything else that you would like to share with me before I start the
physical examination?” This statement serves 2 purposes. First, it elicits any
additional information the patient deems necessary, and second, it signals a
transition to the physical exam. Lastly, before concluding the interview, it is
important to discuss the probable follow-up plan and further treatment. In the
outpatient setting, this may include admission to the hospital or going home and
returning for a follow-up appointment at a designated time.

 30. Question 1 point(s)

A male adult client voluntarily admits himself to the substance abuse unit. He confesses
that he drinks one (1) qt or more of vodka each day and uses cocaine occasionally. Later
that afternoon, he begins to show signs of alcohol withdrawal. What are some early signs
of this condition?

 A. Vomiting, diarrhea, and bradycardia


B. Dehydration, temperature above 101° F (38.3° C), and pruritus
C. Hypertension, diaphoresis, and seizures
D. Diaphoresis, tremors, and nervousness

Correct Correct Answer: D. Diaphoresis, tremors, and nervousness
Alcohol withdrawal syndrome includes alcohol withdrawal, alcoholic hallucinosis, and
alcohol withdrawal delirium (formerly delirium tremens). Signs of alcohol withdrawal
include diaphoresis, tremors, nervousness, nausea, vomiting, malaise, increased blood
pressure and pulse rate, sleep disturbance, and irritability.
 Option A: Although diarrhea may be an early sign of alcohol withdrawal,
tachycardia — not bradycardia — is associated with alcohol withdrawal. Alcohol
withdrawal symptoms occur when patients stop drinking or significantly decrease
their alcohol intake after long-term dependence. Withdrawal has a broad range of
symptoms from mild tremors to a condition called delirium tremens, which results
in seizures and could progress to death if not recognized and treated promptly.
 Option B: Dehydration and an elevated temperature may be expected, but a
temperature above 101° F indicates an infection rather than alcohol withdrawal.
Pruritus rarely occurs in alcohol withdrawal. Alcohol withdrawal can range from
very mild symptoms to the severe form, which is named delirium tremens. The
hallmark is autonomic dysfunction resulting from the excitation of the central
nervous system. Mild signs/symptoms can arise within six hours of alcohol
cessation. If symptoms do not progress to more severe symptoms within 24 to 48
hours, the patient will likely recover.
 Option C: If withdrawal symptoms remain untreated, seizures may arise later.
Withdrawal seizures can typically be managed with benzodiazepines as well, but
may require adjunct therapy with phenytoin, barbiturates, and may even require
intubation and sedation with propofol (Diprivan), ketamine (Ketalar), or in the
most severe cases dexmedetomidine (Precedex).

 31. Question 1 point(s)

When monitoring a female client recently admitted for treatment of cocaine addiction,
nurse Aaron notes sudden increases in the arterial blood pressure and heart rate. To
correct these problems, the nurse expects the physician to prescribe:

 A. Norepinephrine (Levophed) and Lidocaine (Xylocaine)


B. Nifedipine (Procardia) and Lidocaine.
C. Nitroglycerin (Nitro-Bid IV) and Esmolol (Brevibloc)
D. Nifedipine and Esmolol

Correct Correct Answer: D. Nifedipine and Esmolol
This client requires a vasodilator, such as nifedipine, to treat hypertension, and a beta-
adrenergic blocker, such as esmolol, to reduce the heart rate. Nifedipine is a calcium
channel blocker that belongs to the dihydropyridine subclass. It is primarily used as an
antihypertensive and antianginal medication. Esmolol (esmolol hydrochloride) is an
intravenous cardioselective beta-1 adrenergic antagonist. Esmolol is FDA-approved for
short-term duration use in control of supraventricular tachycardia, such as a rapid
ventricular rate in patients with atrial fibrillation or atrial flutter.
 Option A: Norepinephrine’s predominant use is as a peripheral vasoconstrictor.
Specifically, the FDA has approved its use for blood pressure control in specific
acute hypotensive states, as well as being a potential adjunct in the treatment of
cardiac arrest with profound hypotension.
 Option B: Lidocaine, an antiarrhythmic, isn’t indicated because the client doesn’t
have an arrhythmia. The drug is commonly used for local anesthesia, often in
combination with epinephrine (which acts as a vasopressor and extends its
duration of action at a site by opposing the local vasodilatory effects of lidocaine).
 Option C: Although nitroglycerin may be used to treat coronary vasospasm, it
isn’t the drug of choice in hypertension. Nitroglycerin is a vasodilatory drug used
primarily to provide relief from anginal chest pain. Nitroglycerin has been FDA
approved since 2000 and was first sold by Pfizer under the brand name Nitrostat.
It is currently FDA approved for the acute relief of an attack or acute prophylaxis
of angina pectoris secondary to coronary artery disease.

 32. Question 1 point(s)

A 25 –year old client experiencing alcohol withdrawal is upset about going through
detoxification. Which of the following goals is a priority?

 A. The client will commit to a drug-free lifestyle.


B. The client will work with the nurse to remain safe.
C. The client will drink plenty of fluids daily.
D. The client will make a personal inventory of strength.

Incorrect Correct Answer: B. The client will work with the nurse to remain safe.
The priority goal in alcohol withdrawal is maintaining the client’s safety. Alcohol
withdrawal can range from very mild symptoms to the severe form, which is named
delirium tremens. The hallmark is autonomic dysfunction resulting from the excitation of
the central nervous system. Mild signs/symptoms can arise within six hours of alcohol
cessation. If symptoms do not progress to more severe symptoms within 24 to 48 hours,
the patient will likely recover.
 Option A: Delirium tremens is the most severe form of alcohol withdrawal, and
its hallmark is that of an altered sensorium with significant autonomic dysfunction
and vital sign abnormalities. It includes visual hallucinations, tachycardia,
hypertension, hyperthermia, agitation, and diaphoresis. Symptoms of delirium
tremens can last up to seven days after alcohol cessation and may last even
longer.
 Option C: The diagnosis of alcohol withdrawal can be made by taking an
excellent history and performing a thorough physical examination. It is a clinical
diagnosis based on mild, moderate, or severe symptoms. Patients with suspicion
for alcohol withdrawal should be evaluated for other underlying disease processes
such as dehydration, infection, cardiac issues, electrolyte abnormalities,
gastrointestinal bleeding, and traumatic injury. Laboratory studies (electrolytes,
blood counts) may be drawn, but will likely be nondiagnostic.
 Option D: Patients with prolonged altered sensorium or significant renal
abnormalities should have an evaluation for the potential ingestion of another
toxic alcohol. Patients who become financially strapped due to alcoholism could
ingest other alcohols to become intoxicated. These can include isopropyl alcohol,
commonly known as rubbing alcohol, which can lead to acidemia without ketosis
as well as hemorrhagic gastritis.

 33. Question 1 point(s)

A male client is admitted to a psychiatric facility by court order for evaluation for
antisocial personality disorder. This client has a long history of initiating fights and
abusing animals and recently was arrested for setting a neighbor’s dog on fire. When
evaluating this client for the potential for violence, nurse Perry should assess for which
behavioral clues?
 A. A rigid posture, restlessness, and glaring
B. Depression and physical withdrawal
C. Silence and noncompliance
D. Hypervigilance and talk of past violent acts

Incorrect Correct Answer: A. A rigid posture, restlessness, and glaring
Behavioral clues that suggest the potential for violence include a rigid posture,
restlessness, glaring, a change in usual behavior, clenched hands, overtly aggressive
actions, physical withdrawal, noncompliance, overreaction, hostile threats, recent alcohol
ingestion or drug use, talk of past violent acts, inability to express feelings, repetitive
demands and complaints, argumentativeness, profanity, disorientation, inability to focus
attention, hallucinations or delusions, paranoid ideas or suspicions, and somatic
complaints.
 Option B: Before performing a comprehensive psychiatric assessment of the
patient, a careful history and physical examination is necessary. “The DSM-5
diagnostic criteria for Antisocial Personality Disorder. A pervasive pattern of
disregard for and violation of the rights of others, since age 15 years, as indicated
by three (or more) of the following: failure to conform to social norms concerning
lawful behaviors, such as performing acts that are grounds for arrest;
deceitfulness, repeated lying, use of aliases, or conning others for pleasure or
personal profit; Impulsivity or failure to plan; Irritability and aggressiveness,
often with physical fights or assaults; reckless disregard for the safety of self or
others; consistent irresponsibility, failure to sustain consistent work behavior, or
honor monetary obligations; lack of remorse, being indifferent to or rationalizing
having hurt, mistreated, or stolen from another person.
 Option C: Of those children with conduct disorder, 25% of girls and 40% of boys
will meet the diagnostic criteria for antisocial personality disorder. Boys exhibit
symptoms earlier than girls, who often only elicit these symptoms in puberty.
Children who do not develop conduct disorder and progress to the age of 15
without antisocial behaviors will not develop ASPD.
 Option D: Antisocial personality disorder, although a chronic condition with a
lifelong presentation, has had moderations shown with advancing ages, with the
mean remitted age of 35 years old. Those with less baseline symptomatology
showed better-remitted rates. Studies in the past revealed remission rates of 12 to
27% and 27 to 31% rates of improvement, but not remitted. Crime rates and
severity reflect this relation as well, with peak crime statistics in late teens and
higher severity of crimes at younger ages.

 34. Question 1 point(s)

A male client is brought to the psychiatric clinic by family members, who tell the
admitting nurse that the client repeatedly drives while intoxicated despite their pleas to
stop. During an interview with the nurse Linda, which statement by the client most
strongly supports a diagnosis of psychoactive substance abuse?
 A. “I’m not addicted to alcohol. In fact, I can drink more than I used to without
being affected.”
B. “I only spend half of my paycheck at the bar.”
C. “I just drink to relax after work.”
D. “I know I’ve been arrested three times for drinking and driving, but the
police are just trying to hassle me.”

Correct Correct Answer: D. “I know I’ve been arrested three times for drinking
and driving, but the police are just trying to hassle me.”
According to the Diagnostic and Statistical Manual of Mental Disorders, 4th edition,
diagnostic criteria for psychoactive substance abuse include a maladaptive pattern of such
use, indicated either by continued use despite knowledge of having a persistent or
recurrent social, occupational, psychological, or physical problem caused or exacerbated
by substance abuse or recurrent use in dangerous situations (for example, while driving).
For this client, psychoactive substance dependence must be ruled out; criteria for this
disorder include a need for increasing amounts of the substance to achieve intoxication
(option A), increased time and money spent on the substance (option B), inability to
fulfill role obligations (option C), and typical withdrawal symptoms.
 Option A: A shortened version of the term used in the ICD-10 – Mental and
behavioral disorders due to psychoactive substance use. The term encompasses
acute intoxication, harmful use, dependence syndrome, withdrawal state,
withdrawal state with delirium, psychotic disorder, and amnesic syndrome. For a
particular substance, these conditions may be grouped together as, for example,
alcohol disorders, cannabis use disorders, stimulant use disorders. Psychoactive
substance use disorders are defined as being of clinical relevance; the term
‘psychoactive substance use problems’ is a broader one, which includes
conditions and events not necessarily of clinical relevance.
 Option B: Production, distribution, sale, or non-medical use of many
psychoactive drugs is either controlled or prohibited outside legally sanctioned
channels by law. Psychoactive drugs have different degrees of restriction of
availability, depending on their risks to health and therapeutic usefulness, and
classified according to a hierarchy of schedules at both national and international
levels. At the international level, there are international drug conventions
concerned with the control of production and distribution of psychoactive drugs:
the 1961 Single Convention on Narcotic Drugs, amended by a 1972 Protocol; the
1971 Convention on Psychotropic Substances; the 1988 Convention Against
Illicit Traffic in Narcotic Drugs and Psychotropic Substances.
 Option C: It is an essential characteristic of the dependence syndrome that either
substance taking or a desire to take a particular substance should be present; the
subjective awareness of compulsion to use drugs is most commonly seen during
attempts to stop or control substance use. This diagnostic requirement would
exclude, for instance, surgical patients given opiate drugs for the relief of pain and
who may show signs of an opiate withdrawal state when drugs are not given, but
who have no desire to continue taking drugs.

 35. Question 1 point(s)

A female client with borderline personality disorder is admitted to the psychiatric unit.
Initial nursing assessment reveals that the client’s wrists are scratched from a recent
suicide attempt. Based on this finding, the nurse Lenny should formulate a nursing
diagnosis of:

 A. Ineffective individual coping related to feelings of guilt.


B. Situational low self-esteem related to feelings of loss of control.
C. Risk for violence: Self-directed related to impulsive mutilating acts.
D. Risk for violence: Directed toward others related to verbal threats.

Incorrect Correct Answer: C. Risk for violence: Self-directed related to impulsive
mutilating acts.
The predominant behavioral characteristic of the client with borderline personality
disorder is impulsiveness, especially of a physically self-destructive sort. The observation
that the client has scratched wrists doesn’t substantiate the other options. Borderline
personality disorder (BPD) is 1 of 4 Cluster-B disorders that include borderline,
antisocial, narcissistic, and histrionic. Borderline personality disorder (BPD) is
characterized by hypersensitivity to rejection and resulting instability of interpersonal
relationships, self-image, affect, and behavior
 Option A: Inappropriate, intense anger, or difficulty controlling anger, for
example, frequent displays of temper, constant anger, recurrent physical fights. A
careful history and physical exam should be performed before performing a
comprehensive psychiatric assessment. There are structured diagnostic screening
tools used to assess personality disorders and specifically borderline personality
disorder, for example, the Zanarini Rating Scale for Borderline Personality
Disorder.
 Option B: There is identity disturbance which is a markedly and persistently
unstable self-image or sense of self. Borderline personality disorder is
multifactorial in etiology. There is a genetic predisposition. Twin studies show
over 50% heritability (greater than that for major depression). Twin studies
performed in 2000 and 2008 both demonstrated higher concordance of the rate of
borderline personality disorder for monozygotic versus dizygotic twins.
 Option D: Self-injurious behavior, boundary issues, and frequent suicidal threats
present therapeutic challenges specific to the treatment of patients with borderline
personality disorder. High rates of comorbid substance abuse may also confound
the treatment of borderline personality disorder patients.

 36. Question 1 point(s)

A male client recently admitted to the hospital with sharp, substernal chest pain suddenly
complains of palpitations. Nurse Ryan notes a rise in the client’s arterial blood pressure
and a heart rate of 144 beats/minute. On further questioning, the client admits to having
used cocaine recently after previously denying use of the drug. The nurse concludes that
the client is at high risk for which complication of cocaine use?

 A. Coronary artery spasm


B. Bradyarrhythmias
C. Neurobehavioral deficits
D. Panic disorder

Incorrect Correct Answer: A. Coronary artery spasm
Cocaine use may cause such cardiac complications as coronary artery spasm, myocardial
infarction, dilated cardiomyopathy, acute heart failure, endocarditis, and sudden death.
Cocaine blocks reuptake of norepinephrine, epinephrine, and dopamine, causing an
excess of these neurotransmitters at postsynaptic receptor sites. Cocaine and its
metabolites may cause arterial vasoconstriction hours after use. Epicardial coronary
arteries are especially vulnerable to these effects, leading to a decreased myocardial
oxygen supply.
 Option B: Consequently, the drug is more likely to cause tachyarrhythmias than
bradyarrhythmias. Cocaine-induced central sympathetic stimulation and direct
cardiac effects may lead to tachycardia, hypertension, and coronary or cerebral
artery vasoconstriction leading to myocardial infarction and stroke.
 Option C: Although neurobehavioral deficits are common in neonates born to
cocaine users, they are rare in adults. CNS reactions may be more excitatory than
depressant. In its mild form, the patient may display anxiety, restlessness, and
excitement. Full-body tonic-clonic seizures may result from moderate to severe
CNS stimulation. These seizures are often followed by CNS depression, with
death resulting from respiratory failure and/or asphyxiation if concomitant emesis
is present.
 Option D: As craving for the drug increases, a person who’s addicted to cocaine
typically experiences euphoria followed by depression, not panic disorder.
Cardiovascular toxicity and agitation are best-treated first-line with
benzodiazepines to decrease CNS sympathetic outflow. However, there is a risk
of over-sedation and respiratory depression with escalating and numerous doses
of benzodiazepines, which is often necessary. Non-dihydropyridine calcium
channel blockers such as diltiazem and verapamil have shown the ability to
reduce hypertension reliably, but not tachycardia.

 37. Question 1 point(s)

A male client is being admitted to the substance abuse unit for alcohol detoxification. As
part of the intake interview, the nurse asks him when he had his last alcoholic drink. He
says that he had his last drink six (6) hours before admission. Based on this response,
nurse Lorena should expect early withdrawal symptoms to:

 A. Begin after seven (7) days.


B. Not occur at all because the time period for their occurrence has passed.
C. Begin anytime within the next one (1) to two (2) days.
D. Begin within two (2) to seven (7) days.

Incorrect Correct Answer: C. Begin anytime within the next one (1) to two (2) days
Acute withdrawal symptoms from alcohol may begin 6 hours after the client has stopped
drinking and peak 1 to 2 days later. Delirium tremens may occur 2 to 4 days — even up
to 7 days — after the last drink. Moderate symptoms include alcohol withdrawal seizures
(rum fits) that can occur 12 to 24 hours after cessation of alcohol and are typically
generalized in nature. There is a 3% incidence of status epilepticus in these patients.
About 50% of patients who have had a withdrawal seizure will progress to delirium
tremens.
 Option A: Mild signs/symptoms can arise within six hours of alcohol cessation.
If symptoms do not progress to more severe symptoms within 24 to 48 hours, the
patient will likely recover. However, the time to presentation and range of
symptoms can vary greatly depending on the patient, their duration of alcohol
dependence, and volume typically ingested.
 Option B: Most cases should be described by their severity of symptoms, not the
time since their last drink. Noting the time of their last drink is essential in any
patient with an alcohol dependence history who may be presenting with other
complaints. Mild symptoms can be insomnia, tremulousness, hyperreflexia,
anxiety, gastrointestinal upset, headache, palpitations.
 Option D: Delirium tremens is the most severe form of alcohol withdrawal, and
its hallmark is that of an altered sensorium with significant autonomic dysfunction
and vital sign abnormalities. It includes visual hallucinations, tachycardia,
hypertension, hyperthermia, agitation, and diaphoresis. Symptoms of delirium
tremens can last up to seven days after alcohol cessation and may last even
longer.

 38. Question 1 point(s)

Nurse Helen is assigned to care for a client with anorexia nervosa. Initially, which
nursing intervention is most appropriate for this client?

 A. Providing one-on-one supervision during meals and for one (1) hour afterward.

B. Letting the client eat with other clients to create a normal mealtime
atmosphere.
C. Trying to persuade the client to eat and thus restore nutritional balance.
D. Giving the client as much time to eat as desired.

Incorrect Correct Answer: A. Providing one-on-one supervision during meals and
for one (1) hour afterward.
Because the client with anorexia nervosa may discard food or induce vomiting in the
bathroom, the nurse should provide one-on-one supervision during meals and for 1 hour
afterward. Provide one-to-one supervision and have a patient with bulimia remain in the
day room area with no bathroom privileges for a specified period (1 hr) following eating,
if contracting is unsuccessful. Prevents vomiting during and after eating. The patient may
desire food and use a binge-purge syndrome to maintain weight. Note: The patient may
purge for the first time in response to the establishment of a weight gain program.
 Option B: This wouldn’t be therapeutic because other clients may urge the client
to eat and give attention for not eating. Supervise the patient during mealtimes
and for a specified period after meals (usually one hour). It prevents vomiting
during or after eating.
 Option C: This would reinforce control issues, which are central to this client’s
underlying psychological problem. Establish a minimum weight goal and daily
nutritional requirements. Malnutrition is a mood-altering condition, leading to
depression and agitation and affecting cognitive function and decision making.
Improved nutritional status enhances thinking ability, allowing initiation of
psychological work.
 Option D: Instead of giving the client unlimited time to eat, the nurse should set
limits and let the client know what is expected. Make a selective menu available,
and allow the patient to control choices as much as possible. Patient who gains
confidence in himself and feels in control of the environment is more likely to eat
preferred foods. Be alert to choices of low-calorie foods and beverages; hoarding
food; disposing of food in various places, such as pockets or wastebaskets.
Patients will try to avoid taking in what is viewed as excessive calories and may
go to great lengths to avoid eating.

 39. Question 1 point(s)

A female client begins to experience alcoholic hallucinosis. Nurse Joy is aware that the
best nursing intervention at this time?

 A. Keeping the client restrained in bed.


B. Checking the client’s blood pressure every 15 minutes and offering juices.
C. Providing a quiet environment and administering medication as needed and
prescribed.
D. Restraining the client and measuring blood pressure every 30 minutes.

Incorrect Correct Answer: C. Providing a quiet environment and administering
medication as needed and prescribed.
Manifestations of alcoholic hallucinosis are best treated by providing a quiet environment
for reducing stimulation and administering prescribed central nervous system depressants
in dosages that control symptoms without causing oversedation. Encourage the patient to
rest by controlling minimal interpersonal contact with the patient. Decrease
environmental stimuli with controlled lighting, and provide a calm, quiet private room.
The individualized, symptom-triggered approach to benzodiazepine use satisfies the need
to use medication only when needed and may also reduce inpatient hospital stays.
Benzodiazepines stimulate GABA receptors causing a decrease in neuronal activity
resulting in sedation.
 Option A: Although bed rest is indicated, restraints are unnecessary unless the
client poses a danger to himself or others. Also, restraints may increase agitation
and make the client feel trapped and helpless when hallucinating. Present reality
without challenging or escalating the patient’s anxiety and thought disturbances.
Build a therapeutic rapport with the patient by providing relief from his or her
symptoms and meeting physiologic and safety needs. Meet the patient’s needs
promptly to reduce the risk of violence or aggression. Do not approach the patient
with loose items that the patient could grab if he or she becomes agitated, such as
a clipboard or dangling ID badge or phone.
 Option B: Offering juice is appropriate, but measuring blood pressure every 15
minutes would interrupt the client’s rest. The nurse also documents the patient’s
vital signs, looking for an upward trend indicating increased withdrawal
symptoms. On a scale of 0 (none) to 3 (severe), the nurse then rates key signs and
symptoms such as nausea/vomiting; tremors; diaphoresis; anxiety; agitation;
tactile, auditory, and visual disturbances; headache; and orientation.
 Option D: To avoid overstimulating the client, the nurse should check blood
pressure every 2 hours. As direct caregivers, nurses are ideally positioned to
improve patient outcomes by using the symptom-triggered approach. Based on an
objective withdrawal severity scale, a symptom-triggered approach provokes
faster and more-effective relief of withdrawal symptoms than treatment based on
clinicians’ subjective judgment alone.

 40. Question 1 point(s)

Nurse Bella is aware that assessment finding is most consistent with early alcohol
withdrawal?

 A. Heart rate of 120 to 140 beats/minute


B. Heart rate of 50 to 60 beats/minute
C. Blood pressure of 100/70 mmHg
D. Blood pressure of 140/80 mmHg

Incorrect Correct Answer: A. Heart rate of 120 to 140 beats/minute
Tachycardia, a heart rate of 120 to 140 beats/minute, is a common sign of alcohol
withdrawal. Blood pressure may be labile throughout withdrawal, fluctuating at different
stages. Hypertension typically occurs in early withdrawal. Hypotension, although rare
during the early withdrawal stages, may occur in later stages. Hypotension is associated
with cardiovascular collapse and most commonly occurs in clients who don’t receive
treatment. The nurse should monitor the client’s vital signs carefully throughout the
entire alcohol withdrawal process.
 Option B: Delirium tremens is the most severe form of alcohol withdrawal, and
its hallmark is that of an altered sensorium with significant autonomic dysfunction
and vital sign abnormalities. It includes visual hallucinations, tachycardia,
hypertension, hyperthermia, agitation, and diaphoresis. Symptoms of delirium
tremens can last up to seven days after alcohol cessation and may last even
longer.
 Option C: Alcohol withdrawal can range from very mild symptoms to the severe
form, which is named delirium tremens. The hallmark is autonomic dysfunction
resulting from the excitation of the central nervous system. Mild signs/symptoms
can arise within six hours of alcohol cessation. If symptoms do not progress to
more severe symptoms within 24 to 48 hours, the patient will likely recover.
 Option D: Patients should be kept calm in a controlled environment to try to
reduce the risks of progression from mild symptoms to hallucinations. With mild
to moderate symptoms, patients should receive supportive therapy in the form of
intravenous rehydration, correction of electrolyte abnormalities, and have
comorbid conditions as listed above ruled out.

 41. Question 1 point(s)

Nurse Amy is aware that the client is at highest risk for suicide?

 A. One who appears depressed frequently thinks of dying and gives away all
personal possessions.
B. One who plans a violent death and has the means readily available.
C. One who tells others that he or she might do something if life doesn’t get better
soon.
D. One who talks about wanting to die.

Incorrect Correct Answer: B. One who plans a violent death and has the means
readily available.
The client at highest risk for suicide is one who plans a violent death (for example, by
gunshot, jumping off a bridge, or hanging), has a specific plan (for example, after the
spouse leaves for work), and has the means readily available (for example, a rifle hidden
in the garage). Several suicide-related demographic factors often occur in the same
person. For example, if a male police officer with major depression and a significant
problem with alcohol commits suicide using his service revolver (which, unfortunately,
happens not infrequently), 5 risk factors are involved: sex, occupation, depression,
alcohol, and gun availability.
 Option A: A host of thoughts and behaviors are associated with self-destructive
acts. Although many assume that people who talk about suicide will not follow
through with it, the opposite is true; a threat of suicide can lead to the completed
act, and suicidal ideation is highly correlated with suicidal behaviors.
 Option C: They are without hope and therefore cannot foresee things ever
improving; they also view themselves as helpless in 2 ways: (1) they cannot help
themselves, and all their efforts to liberate themselves from the sea of depression
in which they are drowning are to no avail; and (2) no one else can help them.
 Option D: A client who talks about wanting to die or attempting suicide is
considered at a lower risk for suicide because this behavior typically serves to
alert others that the client is contemplating suicide and wishes to be helped.
Determine whether the person has any thoughts of hurting him or herself. Suicidal
ideation is highly linked to completed suicide. Some inexperienced clinicians
have difficulty asking this question. They fear the inquiry may be too intrusive or
that they may provide the person with an idea of suicide. In reality, patients
appreciate the question as evidence of the clinician’s concern. A positive response
requires further inquiry.

 42. Question 1 point(s)

Nurse Penny is aware that the following medical conditions are commonly found in
clients with bulimia nervosa?

 A. Allergies
B. Cancer
C. Diabetes mellitus
D. Hepatitis A

Incorrect Correct Answer: C. Diabetes mellitus
Bulimia nervosa can lead to many complications, including diabetes, heart disease, and
hypertension. Girls and young women with type 1 diabetes have about twice the risk of
developing eating disorders as their peers without diabetes. This may be because of the
weight changes that can occur with insulin therapy and good metabolic control and the
extra attention people with diabetes must pay to what they eat.
 Option A: The most common features of eating disorders in girls and young
women with type 1 diabetes are dissatisfaction with their body weight and shape
and desire to be thinner; dieting or manipulation of insulin doses to control
weight; and, binge eating. Researchers estimate that 10–20 percent of girls in their
mid-teen years and 30–40 percent of late teenaged girls and young adult women
with diabetes skip or alter insulin doses to control their weight.
 Option B: In people with diabetes, eating disorders can lead to poor metabolic
control and repeated hospitalizations for dangerously high or low blood sugar.
Chronic poor blood sugar control leads to long-term complications, such as eye,
kidney, and nerve damage. Diabulimia is a media-coined term that refers to an
eating disorder in a person with diabetes, typically type I diabetes, wherein the
person purposefully restricts insulin in order to lose weight. Some medical
professionals use the term ED-DMT1, Eating Disorder-Diabetes Mellitus Type 1,
which is used to refer to any type of eating disorder comorbid with type 1
diabetes.
 Option D: The human body is surprisingly resilient and people with diabulimia
often manage to function with much higher blood sugars than should be possible.
Thus, the major consequences of diabulimia or ED-DMT1 are usually related to
prolonged elevated blood sugar. These complications can be severe and
irreversible, so proper treatment and early detection are critical. High blood sugar
causes the body to produce certain enzymes and hormones that negatively affect
the immune system and reduce the body’s defense against infection. This risk of
infection plus slowed healing heightens a person’s chance of developing
gangrene, sepsis, or a bone infection.

 43. Question 1 point(s)

Kellan, a high school student is referred to the school nurse for suspected substance
abuse. Following the nurse’s assessment and interventions, what would be the most
desirable outcome?

 A. The student discusses conflicts over drug use.


B. The student accepts a referral to a substance abuse counselor.
C. The student agrees to inform his parents of the problem.
D. The student reports increased comfort with making choices.

Incorrect Correct Answer: B. The student accepts a referral to a substance abuse
counselor
All of the outcomes stated are desirable; however, the best outcome is that the student
would agree to seek the assistance of a professional substance abuse counselor. The basic
goal for a client in any substance abuse treatment setting is to reduce the risk of harm
from continued use of substances. The greatest degree of harm reduction would
obviously result from abstinence, however, the specific goal for each individual client is
determined by his consumption pattern, the consequences of his use, and the setting in
which the brief intervention is delivered.
 Option A: Primary care providers find many brief intervention techniques
effective in addressing the substance abuse issues of clients who are unable or
unwilling to access specialty care. Examples of brief interventions include asking
clients to try nonuse to see if they can stop on their own, encouraging
interventions directed toward attending a self-help group (e.g., Alcoholics
Anonymous [AA] or Narcotics Anonymous [NA]), and engaging in brief,
structured, time-limited efforts to help pregnant clients stop using.
 Option C: The clinician can use brief interventions to motivate particular
behavioral changes at each stage of this process. For example, in the
contemplation stage, a brief intervention could help the client weigh the costs and
benefits of change. In the preparation stage, a similar brief intervention could
address the costs and benefits of various change strategies (e.g., self-change, brief
treatment, intensive treatment, self-help group attendance). In the action stage,
brief interventions can help maintain motivation to continue on the course of
change by reinforcing personal decisions made at earlier stages.
 Option D: To consider change, clients at the precontemplation stage must have
their awareness raised. To resolve their ambivalence, clients in the contemplation
stage must be helped to choose positive change over their current circumstances.
Clients in the preparation stage need help in identifying potential change
strategies and choosing the most appropriate ones. Clients in the action stage need
help to carry out and comply with the change strategies.

 44. Question 1 point(s)

A male client who reportedly consumes one (1) qt of vodka daily is admitted for alcohol
detoxification. To try to prevent alcohol withdrawal symptoms, Dr. Smith is most likely
to prescribe which drug?

 A. Clozapine (Clozaril)
B. Thiothixene (Navane)
C. Lorazepam (Ativan)
D. Lithium carbonate (Eskalith)

Incorrect Correct Answer: C. Lorazepam (Ativan)
The best choice for preventing or treating alcohol withdrawal symptoms is lorazepam, a
benzodiazepine. Lorazepam is a benzodiazepine medication developed by DJ Richards. It
went on the market in the United States in 1977. Lorazepam has common use as the
sedative and anxiolytic of choice in the inpatient setting owing to its fast (1 to 3 minute)
onset of action when administered intravenously. Lorazepam is also one of the few
sedative-hypnotics with a relatively clean side effect profile. ff-label (non-FDA-
approved) uses for Lorazepam include rapid tranquilization of the agitated patient,
alcohol withdrawal delirium, alcohol withdrawal syndrome, insomnia, panic disorder,
delirium, chemotherapy-associated anticipatory nausea and vomiting (adjunct or
breakthrough), as well as psychogenic catatonia.
 Option A: Clozapine is an FDA-approved atypical antipsychotic drug for
treatment-resistant schizophrenia.[1] The definition of treatment-resistant
schizophrenia is persistent or moderate delusions or hallucinations after failing
two trials of antipsychotic medicines. Clozapine is part of a group of drugs known
as second-generation antipsychotics or atypical antipsychotics.[1] Antipsychotic
drugs are vital in treating the core symptoms of schizophrenia: hallucinations and
delusions.
 Option B: Thiothixene is used to treat the symptoms of schizophrenia (a mental
illness that causes disturbed or unusual thinking, loss of interest in life, and strong
or inappropriate emotions). Thiothixene is in a group of medications called
conventional antipsychotics. It works by decreasing abnormal excitement in the
brain.
 Option D: Lithium was the first mood stabilizer and is still the first-line treatment
option, but is underutilized because it is an older drug. Lithium is a commonly
prescribed drug for a manic episode in bipolar disorder as well as maintenance
therapy of bipolar disorder in a patient with a history of a manic episode. The
primary target symptoms of lithium are mania and unstable mood. Lithium is also
prescribed for major depressive disorder as an adjunct therapy, bipolar disorder
without a history of mania, treatment of vascular headaches, and neutropenia.
These are off-label uses, meaning they are not FDA-approved. Patients with rapid
cycling and mixed state types of bipolar disorder generally do less well on
lithium.

 45. Question 1 point(s)

A male client is being treated for alcoholism. After a family meeting, the client’s spouse
asks the nurse about ways to help the family deal with the effects of alcoholism. Nurse
Lily should suggest that the family join which organization?

 A. Al-Anon
B. Make Today Count
C. Emotions Anonymous
D. Alcoholics Anonymous

Incorrect Correct Answer: A. Al-Anon
Al-Anon is an organization that assists family members to share common experiences
and increase their understanding of alcoholism. Al?Anon members come to understand
problem drinking as a family illness that affects everyone in the family. By listening to
Al?Anon members speak at Al?Anon meetings, they can hear how they came to
understand their own role in this family illness. This insight put them in a better position
to play a positive role in the family’s future.
 Option B: Make Today Count is a support group for people with life-threatening
or chronic illnesses. MTC is a mutual support group for persons with terminal
illnesses. Organized in 1974, it is part of what some have called the “happy death
movement.” This movement seeks to make death more humane and less
technological.
 Option C: Emotions Anonymous is a support group for people experiencing
depression, anxiety, or similar conditions. Emotions Anonymous International
(EAI) is a nonprofit organization that facilitates the ongoing efforts of an
international fellowship of men and women who desire to improve their
emotional well-being. EA members come together in weekly meetings for the
purpose of working toward recovery from any sort of emotional difficulties. EA
members are of diverse ages, races, economic status, social and educational
backgrounds. The only requirement for membership is a desire to become well
emotionally.
 Option D: Alcoholics Anonymous is an organization that helps alcoholics
recover by using a twelve-step program. Alcoholics Anonymous is an
international fellowship of men and women who have had a drinking problem. It
is nonprofessional, self-supporting, multiracial, apolitical, and available almost
everywhere. There are no age or education requirements. Membership is open to
anyone who wants to do something about their drinking problem.

 46. Question 1 point(s)


A female client is admitted to the psychiatric clinic for treatment of anorexia nervosa. To
promote the client’s physical health, nurse Tair should plan to:

 A. Severely restrict the client’s physical activities.


B. Weigh the client daily, after the evening meal.
C. Monitor vital signs, serum electrolyte levels, and acid-base balance.
D. Instruct the client to keep an accurate record of food and fluid intake.

Incorrect Correct Answer: C. Monitor vital signs, serum electrolyte levels, and acid-
base balance
An anorexic client who requires hospitalization is in poor physical condition from
starvation and may die as a result of arrhythmias, hypothermia, malnutrition, infection, or
cardiac abnormalities secondary to electrolyte imbalances. Therefore, monitoring the
client’s vital signs, serum electrolyte level, and acid-base balance is crucial.
 Option A: Restricting the client’s physical activities may worsen anxiety. Clients
with anorexia appear slow, lethargic, and fatigued; they may be emaciated
depending on the amount of weight loss; clients with bulimia may be underweight
or overweight but are generally close to expected body weight for age and size.
 Option B: This is incorrect because a weight obtained after breakfast is more
accurate than one obtained after the evening meal. When clients can eat, a diet of
1200 to 1500 calories per day is ordered, with gradual increases in calories until
clients are ingesting adequate amounts for height, activity level, and growth
needs; the nurse is responsible for monitoring meals and snacks and often initially
will sit with a client during eating at a table away from other clients; after each
meal or snack, clients may be required to remain in view of staff for 1 to 2 hours
to ensure that they do not empty the stomach by vomiting.
 Option D: This would reward the client with attention for not eating and
reinforce the control issues that are central to the underlying psychological
problem; also, the client may record food and fluid intake inaccurately. The nurse
can help clients begin to recognize emotions such as anxiety or guilt by asking
them to describe how they are feeling and allowing adequate time for response.

 47. Question 1 point(s)

Kevin is remanded by the courts for psychiatric treatment. His police record, which dates
to his early teenage years, includes delinquency, running away, auto theft, and vandalism.
He dropped out of school at age 16 and has been living on his own since then. His history
suggests maladaptive coping, which is associated with:

 A. Antisocial personality disorder


B. Borderline personality disorder
C. Obsessive-compulsive personality disorder
D. Narcissistic personality disorder

Incorrect Correct Answer: A. Antisocial personality disorder
The client’s history of delinquency, running away from home, vandalism, and dropping
out of school are characteristic of antisocial personality disorder. This maladaptive
coping pattern is manifested by a disregard for societal norms of behavior and an inability
to relate meaningfully to others. Antisocial personality disorder (ASPD) is a deeply
ingrained and rigid dysfunctional thought process that focuses on social irresponsibility
with exploitive, delinquent, and criminal behavior with no remorse. Disregard for and the
violation of others’ rights are common manifestations of this personality disorder, which
displays symptoms that include failure to conform to the law, inability to sustain
consistent employment, deception, manipulation for personal gain, and incapacity to form
stable relationships.
 Option B: In borderline personality disorder, the client exhibits mood instability,
poor self-image, identity disturbance, and labile affect. Borderline personality
disorder (BPD) is 1 of 4 Cluster-B disorders that include borderline, antisocial,
narcissistic, and histrionic. Borderline personality disorder (BPD) is characterized
by hypersensitivity to rejection and resulting instability of interpersonal
relationships, self-image, affect, and behavior.
 Option C: Obsessive-compulsive personality disorder is characterized by a
preoccupation with impulses and thoughts that the client realizes are senseless but
can’t control. Obsessive-compulsive disorder (OCD) is often a disabling condition
consisting of bothersome intrusive thoughts that elicit a feeling of discomfort. To
reduce the anxiety and distress associated with these thoughts, the patient may
employ compulsions or rituals. These rituals may be personal and private, or they
may involve others to participate; the rituals are to compensate for the ego-
dystonic feelings of the obsessional thoughts and can cause a significant decline
in function.
 Option D: Narcissistic personality disorder is marked by a pattern of self-
involvement, grandiosity, and demand for constant attention. Narcissistic
personality disorder (NPD) is a pattern of grandiosity, need for admiration, and
lack of empathy per the Diagnostic and Statistical Manual of Mental Disorders
(DSM–5). The disorder is classified in the dimensional model of “Personality
Disorders.”NPD is highly comorbid with other disorders in mental health.

 48. Question 1 point(s)

Macoy and Helen seek emergency crisis intervention because he slapped her repeatedly
the night before. The husband indicates that his childhood was marred by an abusive
relationship with his father. When intervening with this couple, nurse Gerry knows they
are at risk for repeated violence because the husband:

 A. Has only moderate impulse control.


B. Denies feelings of jealousy or possessiveness.
C. Has learned violence as an acceptable behavior.
D. Feels secure in his relationship with his wife.

Incorrect Correct Answer: C. Has learned violence as an acceptable behavior
Family violence usually is a learned behavior, and violence typically leads to further
violence, putting this couple at risk. Unfortunately, each form of family violence begets
interrelated forms of violence, and the “cycle of abuse” is often continued from exposed
children into their adult relationships, and finally to the care of the elderly. Domestic
violence is thought to be underreported. Domestic violence affects the victim, families,
co-workers, and community. It causes diminished psychological and physical health,
decreases the quality of life, and results in decreased productivity.
 Option A: Repeated slapping may indicate poor, not moderate, impulse control.
According to the CDC, 1 in 4 women and 1 in 7 men will experience physical
violence by their intimate partner at some point during their lifetimes. About 1 in
3 women and nearly 1 in 6 men experience some form of sexual violence during
their lifetimes. Intimate partner violence, sexual violence, and stalking are high,
with intimate partner violence occurring in over 10 million people each year.
 Option B: At least 5 million acts of domestic violence occur annually to women
aged 18 years and older, with over 3 million involving men. While most events
are minor, for example grabbing, shoving, pushing, slapping, and hitting, serious
and sometimes fatal injuries do occur. Approximately 1.5 million intimate partner
female rapes and physical assaults are perpetrated annually, and approximately
800,000 male assaults occur. About 1 in 5 women have experienced completed or
attempted rape at some point in their lives. About 1% to 2% of men have
experienced completed or attempted rape.
 Option D: Violent people commonly are jealous and possessive and feel insecure
in their relationships. While the research is not definitive, a number of
characteristics are thought to be present in perpetrators of domestic violence.
Abusers tend to be possessive, jealous, suspicious, and paranoid. Approximately
one-third of women and one-fifth of men will be victims of abuse. The most
common sites of injuries are the head, neck, and face. Clothes may cover injuries
to the body, breasts, genitals, rectum, and buttocks. One should be suspicious if
the history is not consistent with the injury.

 49. Question 1 point(s)

A client whose husband just left her has a recurrence of anorexia nervosa. Nurse Vic
caring for her realizes that this exacerbation of anorexia nervosa results from the client’s
effort to:

 A. Manipulate her husband.


B. Gain control of one part of her life.
C. Commit suicide.
D. Live up to her mother’s expectations.

Incorrect Correct Answer: B. Gain control of one part of her life
By refusing to eat, a client with anorexia nervosa is unconsciously attempting to gain
control over the only part of her life she feels she can control. Assist the patient to
confront changes associated with puberty and sexual fears. Provide sex education as
necessary. Encourage personal development program, preferably in a group setting.
Provide information about the proper application of makeup and grooming. Learning
about methods to enhance personal appearance may be helpful to a long-range sense of
self-esteem and image. Feedback from others can promote feelings of self-worth.
 Option A: This eating disorder doesn’t represent an attempt to manipulate others
or live up to their expectations (although anorexia nervosa has a high incidence in
families that emphasize achievement). Assist the patient to assume control in
areas other than dieting and weight loss such as management of their own daily
activities, work, and leisure choices. Feelings of personal ineffectiveness, low
self-esteem, and perfectionism are often part of the problem. The patient feels
helpless to change and requires assistance to problem-solve methods of control in
life situations.
 Option C: The client isn’t attempting to commit suicide through starvation;
rather, by refusing to eat, she is expressing feelings of despair, worthlessness, and
hopelessness. Help the patient formulate goals for self (not related to eating) and
create a manageable plan to reach those goals, one at a time, progressing from
simple to more complex. Patients need to recognize the ability to control other
areas in life and may need to learn problem-solving skills to achieve this control.
Setting realistic goals fosters success.
 Option D: Encourage patients to take charge of their own lives in a more
healthful way by making their own decisions and accepting self as she or he is at
this moment (including inadequacies and strengths). Patient often does not know
what she or he may want for themselves. Parents (mother) often make decisions
for the patient. Patient may also believe she or he has to be the best in everything
and holds self-responsible for being perfect.

 50. Question 1 point(s)

A male client has approached the nurse asking for advice on how to deal with his alcohol
addiction. Nurse Sally should tell the client that the only effective treatment for
alcoholism is:

 A. Psychotherapy
B. Total abstinence
C. Alcoholics Anonymous (AA)
D. Aversion therapy

Incorrect Correct Answer B. Total abstinence
Total abstinence is the only effective treatment for alcoholism. For people who have
severe alcohol use disorder, this is a key step. The goal is to stop drinking and give the
body time to get the alcohol out of the system. That usually takes a few days to a week.
Psychotherapy, attendance at AA meetings, and aversion therapy are all adjunctive
therapies that can support the client in his efforts to abstain.
 Option A: With alcohol use disorder, controlling your drinking is only part of the
answer. You also need to learn new skills and strategies to use in everyday life.
Psychologists, social workers, or alcohol counselors can teach you how to change
the behaviors that make you want to drink; deal with stress and other triggers;
build a strong support system; and set goals and reach them.
 Option C: Group therapy or a support group can help during rehab and help the
client stay on track as life gets back to normal. Group therapy, led by a therapist,
can give the client the benefits of therapy along with the support of other
members. Support groups aren’t led by therapists. Instead, these are groups of
people who have alcohol use disorder. Examples include Alcoholics Anonymous,
SMART Recovery, and other programs. The peers can offer understanding and
advice and help keep the client accountable. Many people stay in groups for
years.
 Option D: Aversion therapy is a type of behavioral therapy that involves
repeatedly pairing an unwanted behavior with discomfort. For example, a person
undergoing aversion therapy to stop smoking might receive an electrical shock
every time they view an image of a cigarette. The goal of the conditioning process
is to make the individual associate the stimulus with unpleasant or uncomfortable
sensations.

 1. Question Nurse Florence assesses Mrs. B with borderline personality disorder. Which
of the following behaviors are common to this diagnosis? Select all that apply.

 A. Intense fear of being alone.


B. Evidence of self-mutilating attempts.
C. Evidence of suspiciousness and mistrust of others.
D. Indifferent attitude toward approval of criticism.
E. Unstable moods with impulsive behaviors.
F. Presence of odd mannerisms, speech, and behaviors.

Incorrect Correct Answers: A, B, E
These are all common characteristics of an individual with borderline personality
disorder. BPD can often interfere with your ability to enjoy life or achieve fulfillment in
relationships, work, or school. It’s associated with specific and significant problems in
interpersonal relationships, self-image, emotions, behaviors, and thinking.
 Option A: BPD is associated with an intense fear of being abandoned by loved
ones and attempts to avoid real or imagined abandonment. This usually leads to
difficulty trusting others, putting a strain on relationships.
 Option B: BPD is associated with a tendency to engage in risky and impulsive
behaviors, such as going on shopping sprees, drinking excessive amounts of
alcohol or abusing drugs, engaging in promiscuous or risky sex, or binge eating.
Also, people with BPD are more prone to engage in self-harming behaviors, such
as cutting or burning and attempting suicide.
 Option C: Suspiciousness and mistrust of others (option C) are characteristic of
paranoid personality disorder. The primary characteristic of this condition is a
chronic and pervasive distrust and suspicion of others.
 Option D: This is a characteristic of someone with schizoid personality disorder,
which is generally aloof in relationships and has unusual speech and mannerisms.
They have a sense of indifference to praise and affirmation, as well as to criticism
or rejection.
 Option E: Emotional instability is a key feature of BPD. Individuals feel like
they’re on an emotional roller coaster with quick mood shifts (i.e., going from
feeling OK to feeling extremely down or blue within a few minutes). Mood
changes can last from minutes to days and are often intense. Anger, anxiety, and
overwhelming emptiness are common as well.
 Option F: People with schizoid personality disorder are often described by others
as aloof, cold, and detached. Those who have the disorder may prefer being alone,
but some may also experience loneliness and social isolation as a result.

 2. Question When a client with a personality disorder begins demonstrating manipulative


behavior, which of the following nursing actions are most appropriate? Select all that
apply.

 A. Ask the client to think about the consequences of behavior.


B. Allow the client time to perform specific rituals.
C. Develop a consistent team approach to handle the client's behaviors.
D. Help the client to express anxiety verbally rather than with specific symptoms.
E. Provide immediate feedback concerning the client's specific behaviors.
F. Set limits in a clear, direct manner.

Incorrect Correct Answers: A, C, E, F
These interventions allow the nurse to immediately confront the client’s manipulative
behavior and provide consistent structure (through limit-setting and team approach).
 Option A: Be very clear about the consequences if policies/limits are not adhered
to. The client needs to understand the consequences of breaking the rules.
 Option B: This is appropriate for the client with obsessive-compulsive behavior.
During the beginning of treatment, allow plenty of time for rituals. Do not be
judgmental or verbalize disapproval of the behavior to deny the client this activity
can precipitate panic level of anxiety.
 Option C: Make a clear and concrete written plan of care so other staff can
follow. Helps minimize manipulations and might help encourage cooperation.
 Option D: For someone with somatization problems. Encourage the client to
explore feelings and concerns (e.g., identify fears, loneliness, self-hate). Client is
used to acting out feelings.
 Option E: Give the client positive attention when behaviors are appropriate and
productive. Avoid giving any attention (when possible and not dangerous to self
or others) when the client’s behaviors are inappropriate.
 Option F: Use assertiveness when setting limits on the client’s unreasonable
demands for attention and time. Firm, clear, nonjudgmental limits give the client
structure.

 3. Question Barbara is a client with a borderline personality disorder. She is defensive


and emotionally labile and often becomes suddenly and explosively angry. When
interacting with her, you as a nurse would:

 A. Point out how angry Barbara is becoming, and confront the behavior.
B. Take a calm, quiet, and non-confrontational approach, and avoid arguing with
Barbara.
C. Tell Barbara to calm down and to avoid becoming explosive or restraints will
be used.
D. Use a gentle touch and a caring approach to calm Barbara.

Incorrect Correct Answer: B. Take a calm, quiet, and non-confrontational
approach, and avoid arguing with Barbara.
The best way to respond to the client with angry behavior is a calm, non-confrontational,
non-argumentative approach. This will avoid further escalating the client’s behavior. Use
a calm and firm approach. Provides structure and control for a client who is out of
control.
 Option A: Confronting the client’s behavior could exacerbate anger and trigger
explosive behavior. Remain neutral as possible; Do not argue with the client. The
client can use inconsistencies and value judgments as justification for arguing and
escalating mania.
 Option C: Telling the client to calm down minimizes the client’s problems, and
the mention of restraints may be perceived as threatening to the client. Redirect
agitation and potentially violent behaviors with physical outlets in an area of low
stimulation (e.g., punching bag).
 Option D: Touch may also be perceived as threatening; it is not recommended for
a client who may become explosive. Maintain a consistent approach, employ
consistent expectations, and provide a structured environment.

 4. Question Nurse Danita is working with clients who have personality disorders. Which
of the following techniques would the nurse use to deal with her own feelings that
interfere with therapeutic performance?

 A. Active listening techniques


B. Challenging the client's assertions
C. Forming social relations
D. Seeking peer or supervisor direction

Correct Correct Answer: D. Seeking peer or supervisor direction
The nurse is likely to have strong reactions to clients with personality disorders,
especially those who display intense emotions and manipulative behaviors. Seeking the
direction of peers and supervisors can help clarify issues and determine the best nursing
responses to difficult behaviors.
 Option A: Be aware of flattery as an attempt to feed into your needs to feel
special. Giving into the client’s thinking that you are “the best” or “the only one”
can pit you against other staff and undermine the client’s need for limits.
 Option B: If the client becomes hostile or projects blame onto you or staff,
project a neutral, calm demeanor, and avoid power struggles. Focus on the client’s
underlying feelings. Defuses tension and opens up productive interaction.
 Option C: Guard against personal feelings of frustration and lack of progress.
Change if often very slow and may seem to take longer than it actually is. Nurture
yourself outside the job. Keep your “bucket” full of laughter and high regard from
family and friends.

 5. Question A client with antisocial personality disorder was admitted in a unit at


Nurseslabs Hospital. The newly admitted client stole money from an elderly in the unit.
Which of the following is the most appropriate for the nurse to say to this client?

 A. "Why did you take the money?"


B. "Let's talk about how you felt when you took the money."
C. "The consequences of stealing are a loss of privileges."
D. "This client is defenseless against you."

Incorrect Correct Answer: C. “The consequences of stealing are loss of privileges.”
The most appropriate response is to reinforce the consequences of behavior that disregard
the rights of others. Be very clear about the consequences if policies/limits are not
adhered to. Client needs to understand the consequences of breaking the rules.
 Option A: This client is likely to rationalize and excuse the behavior. Approach
the client in a consistent manner in all interactions. Enhances feelings of security
and provides structure. Exceptions encourage manipulative behavior.
 Option B: The nurse should not encourage the client to provide excuses or
explanations of behaviors that are clearly against the rules. Be clear with the client
as to the unit/hospital/clinic policies. Give brief concrete reasons for the rules, if
asked, and then move on.
 Option D: A client with antisocial personality disorder is unlikely to have
compassion for others and typically lacks respect for the rights of others. When
limits or policies are not followed, enforce the consequences in a matter-of-fact,
nonjudgmental manner. Helps minimize manipulations and might help encourage
cooperation.

 6. Question Angela has a history of conflict-filled relationships. Despite an expressed


desire for friends, she acts in ways that tend to alienate people. Which nursing
intervention would be important for Angela?
 A. Establish a therapeutic relationship in which the nurse uses role-modeling and
role-playing for appropriate behaviors.
B. Help the client to select friends who are kind and extra caring.
C. Point out that the client acts in ways that alienate others.
D. Recognize that this client is unlikely to change and therefore intervention is
inappropriate.

Incorrect Correct Answer: A. Establish a therapeutic relationship in which the
nurse uses role-modeling and role-playing for appropriate behaviors.
A therapeutic relationship shows acceptance, and using role modeling and role-playing
can help the client to learn appropriate behaviors. Problem solve and role play with client
acceptable social skills that will help obtain needs effectively and appropriately.
 Option B: This is an inappropriate and unrealistic solution to the client’s problem
behaviors. Over time, alternative ways of experiencing interpersonal relationships
might emerge. Take one small skill that the client is willing to work on, break it
down into small parts, and work on it with the client.
 Option C: This is also inappropriate because the client is not likely to accept
direct criticism of her behavior; such individuals do not perceive a problem with
their own behavior. Expand limits by clarifying expectations for clients in a
number of settings. When time is taken in initial meetings to clarify expectations,
confrontations, and power struggles with clients can be minimized and even
avoided.
 Option D: Ignores the client’s potential for growth and improvement. Collaborate
with the client, as well as the multidisciplinary team, to establish a reward system
for compliance with clearly defined expectations. Tangible reinforcement for
meeting expectations can strengthen the client’s positive behaviors.

 7. Question Tyrion describes himself as “very religious, with strong opinions about what
is right and what is wrong.” He is quite judgmental about beliefs and lifestyles that are
“unacceptable.” Which statement supports the nurse’s analysis that this client’s behavior
is typical of someone with a personality disorder?

 A. Inflexible behaviors, along with the use of rigid defense mechanisms, are
characteristic.
B. Judgmental behavior, including self-insight, is common.
C. Religious fanatics often have personality disorders.
D. Strong belief systems are common and can help identify evidence of
instability.

Incorrect Correct Answer: A. Inflexible behaviors, along with use of rigid defense
mechanisms, are characteristic.
Individuals with personality disorder have inflexible behavior patterns and rigid defense
mechanisms. They are unlikely to change over time. Personality disorders tend to appear
in adolescence or early adulthood, continue over many years, and can cause a great deal
of distress. They can potentially cause enormous conflict with other people, impacting
relationships, social situations, and life goals. People with personality disorders often
don’t recognize that they have problems and are often confusing and frustrating to people
around them (including clinicians).
 Option B: Such individuals generally lack self-insight and are more likely to have
external locus of control thinking (blaming others for problems). These
personality disorders are characterized by odd or eccentric behavior. People with
cluster A personality disorders tend to experience major disruptions in
relationships because their behavior may be perceived as peculiar, suspicious, or
detached.
 Option C: Religious fanatics may be motivated by other psychodynamics
(possibly psychotic states). The cluster B personality disorders are characterized
by dramatic or erratic behavior. People who have a personality disorder from this
cluster tend to either experience very intense emotions or engage in extremely
impulsive, theatrical, promiscuous, or law-breaking behaviors.
 Option D: However, strong belief systems do not necessarily mean mental
instability. A mentally healthy person may have belief systems that are strong and
that govern conduct. Cluster C personality disorders are characterized by anxiety.
People with personality disorders in this cluster tend to experience pervasive
anxiety and/or fearfulness.

 8. Question Which statement about an individual with a personality disorder is true?

 A. Psychotic behavior is common during acute episodes.


B. Prognosis for recovery is good with therapeutic intervention.
C. The individual typically remains in the mainstream of society, although he has
problems in social and occupational roles.
D. The individual usually seeks treatment willingly for symptoms that are
personally distressful.

Incorrect Correct Answer: C. The individual typically remains in the mainstream of
society, although he has problems in social and occupational roles.
An individual with a personality disorder usually is not hospitalized unless a coexisting
Axis I psychiatric disorder is present. Generally, these individuals make marginal
adjustments and remain in society, although they typically experience relationship and
occupational problems related to their inflexible behaviors.
 Option A: Personality disorders are chronic, lifelong patterns of behavior; acute
episodes do not occur. Psychotic behavior is usually not common, although it can
occur in either schizotypal personality disorder or borderline personality disorder.
 Option B: Because these disorders are enduring and evasive and the individual is
inflexible, the prognosis for recovery is unfavorable. Compared to mood disorders
such as clinical depression and bipolar disorder, there have historically been
relatively few studies on how to effectively treat personality disorders.
 Option D: Generally, the individual does not seek treatment because he does not
perceive problems with his own behavior. Distress can occur based on other
people’s reaction to the individual’s behavior.

 9. Question Kyle is a client with an anxious, fearful personality who has difficulty
accomplishing work assignments because of his fear of failure. He has been referred to
the employee assistance program because of repeated absences from work and evidence
of an alcohol problem. Which nursing diagnosis would be most appropriate?

 A. Ineffective coping
B. Decisional conflict
C. Disturbed thought process
D. Risk for self-directed violence

Incorrect Correct Answer: A. Ineffective coping
The client is experiencing difficulty in occupational functioning as well as problems with
alcohol; therefore, he meets the criteria for the diagnosis of Ineffective coping. Work
with the client on problem-solving skills using a situation that is bothering the client.
Client might not know how to articulate the problem. Helping identify alternatives gives
the client a sense of control. Evaluating the pros and cons of the alternatives facilitates
choosing potential solutions.
 Option B: Keep goals very realistic and go in small steps. There are no overnight
successes with people with personality disorders. It can take a long time to
positively change ingrained, life-long, maladaptive habits; however, change is
always possible.
 Option C: Identify behavioral limits and behaviors that are expected. Client
needs a clear structure. Expect frequent testing of limits initially. Maintaining
limits can enhance feelings of safety in the client.
 Option D: If the client becomes hostile or projects blame onto you or staff,
project a neutral, calm demeanor, and avoid power struggles. Focus on the client’s
underlying feelings. Defuses tension and opens up productive interaction.

 10. Question Santino is hospitalized at Nurseslabs Medical Center following a suicide


attempt. His history reveals a previous diagnosis of schizoid personality disorder. Which
of the following behaviors would be atypical of a client with this disorder?

 A. Actions designed to please the nurse.


B. Limited expressions of feelings and emotions.
C. Odd ideas and mannerisms.
D. Reluctance to join group activities.

Incorrect Correct Answer: A. Actions designed to please the nurse.
A client with schizoid personality disorder is typically detached, aloof, and socially
isolated. He has no interest in seeking the approval of others and would not behave in
ways to please the nurse. The behaviors included in the remaining options are
characteristic of someone with schizoid personality disorder.
 Option B: People with schizoid personality disorder are often described by others
as aloof, cold, and detached. Those who have the disorder may prefer being alone,
but some may also experience loneliness and social isolation as a result.
 Option C: The DSM-5 defines schizoid personality disorder as a “pervasive
pattern of social and interpersonal deficits marked by acute discomfort with, and
reduced capacity to form close relationships as well as by cognitive or perceptual
distortions and eccentricities of behavior, beginning by early adulthood and
present in a variety of contexts.”
 Option D: Those with this disorder also tend to have few friendships, date rarely,
and often do not marry. The symptoms of the disorder may also make it difficult
to work in positions that require a lot of social interaction or people skills. People
with a schizoid personality disorder may do better in jobs that involve working in
solitude.

 11. Question The community nurse is following up on Mrs. Jenner who was hospitalized
at Nurseslabs Medical Center due to depressive disorder, not otherwise specified,
following the death of her spouse. In reviewing the client’s chart, the nurse notes that
Mrs. Jenner has an Axis II diagnosis of dependent personality disorder. Which behavior
would the nurse anticipate in this client?

 A. Difficulty making decisions, lack of self-confidence.


B. Grandiose thinking, attention-seeking behaviors.
C. Odd mannerisms, speech, and behaviors.
D. Unstable moods and impulsive behaviors.

Incorrect Correct Answer: A. Difficulty making decisions, lack of self-confidence
The client with a dependent personality disorder typically demonstrates anxious and
fearful behavior and is reluctant to make decisions. Lack of self-confidence is reflective
of chronic low self-esteem. It involves fear of being alone and often causes those who
have the disorder to do things to try to get other people to take care of them.
 Option B: Grandiose thinking and attention-seeking behaviors are characteristics
of someone with a dramatic, emotional, erratic personality disorder, such as
narcissistic personality. It is associated with self-centeredness, exaggerated self-
image, and lack of empathy for others and is often driven by an underlying
fragility in the sense of self.
 Option C: Odd mannerisms, speech, and behaviors are characteristics of
schizotypal personality disorder, in which odd, eccentric behavior is displayed.
Schizotypal personality disorder features odd speech, behavior, and appearance,
as well as strange beliefs and difficulty forming relationships.
 Option D: Unstable moods and impulsive behaviors describe a borderline
personality disorder. It is characterized by instability in interpersonal
relationships, emotions, self-image, and impulsive behaviors.

 12. Question Ralph is admitted to Nurseslabs Medical Center with the diagnosis of
bipolar disorder, a single manic episode. Which of the following behaviors would the
nurse expect to assess?

 A. Apathy, poor insight, and poverty of ideas.


B. Anxiety, somatic complaints, and insomnia.
C. Elation, hyperactivity, and impaired judgment.
D. Social isolation, delusional thinking, and clang associations.

Incorrect Correct Answer: C. Elation, hyperactivity, and impaired judgment
A client with bipolar disorder, manic episode, would demonstrate flight of ideas and
hyperactivity as part of the increased psychomotor activity. The mood is one of elation,
and the feeling is that one is invincible; therefore, judgment may be quite impaired. The
defining characteristics of mania are increased talkativeness, rapid speech, decreased the
need for sleep (unlike depression or anxiety in which the need for sleep exists, but there
is an inability to sleep), racing thoughts, distractibility, increase in goal-directed activity,
and psychomotor agitation.
 Option A: These symptoms would be more characteristic of an individual with
long-term schizophrenia. Negative symptoms include anhedonia, poverty of
speech, and lack of motivation. The diagnosis of schizophrenia is clinical; made
exclusively after obtaining a full psychiatric history and excluding other causes of
psychosis.
 Option B: The symptoms are more characteristic of someone with an anxiety
disorder, although a manic individual may also not sleep because of excessive
energy. Generalized anxiety disorder is characterized by persistent, excessive, and
unrealistic worry about everyday things. This worry could be multifocal such as
finance, family, health, and the future. It is excessive, difficult to control, and is
often accompanied by many non-specific psychological and physical symptoms.
Excessive worry is the central feature of generalized anxiety disorder.
 Option D: These symptoms are characteristic of schizophrenia. Traditionally,
symptoms have divided into two main categories: positive symptoms which
include hallucinations, delusions, and formal thought disorders.

 13. Question In a day treatment program, a manic client is creating considerable chaos,
behaving in a dominating and manipulative way. Which nursing intervention is most
appropriate?

 A. Allow the peer group to intervene.


B. Describe acceptable behavior and set realistic limits with the client.
C. Recommend that the client is hospitalized for treatment.
D. Tell the client that his behavior is inappropriate.

Incorrect Correct Answer: B. Describe acceptable behavior and set realistic limits
with the client.
In this situation, it would be appropriate for the nurse to suggest alternative behaviors in
place of unacceptable ones to help the client gain self-control. Maintain a consistent
approach, employ consistent expectations, and provide a structured environment. Clear
and consistent limits and expectations minimize the potential for the client’s
manipulation of staff.
 Option A: The peer group is not responsible for monitoring the client’s behavior.
Decrease environmental stimuli (e.g., by providing a calming environment or
assigning a private room). Helps decrease escalation of anxiety and manic
symptoms.
 Option C: The client’s behavior does not warrant hospitalization. Redirect
agitation and potentially violent behaviors with physical outlets in an area of low
stimulation (e.g., punching bag). Can help to relieve pent-up hostility and relieve
muscle tension.
 Option D: The client is told only what is unacceptable and is not given any
alternatives. Remain neutral as possible; Do not argue with the client. The client
can use inconsistencies and value judgments as justification for arguing and
escalating mania.

 14. Question An individual with depression has a deficiency in which neurotransmitters,


based on the biogenic amine theory?

 A. Dopamine and thyroxine


B. GABA and acetylcholine
C. Cortisone and epinephrine
D. Serotonin and norepinephrine

Correct Correct Answer: D. Serotonin and norepinephrine
The biogenic amine theory of depression describes deficiencies in the neurotransmitters
serotonin and norepinephrine. Antidepressants medications increase the levels of these
neurotransmitters and therefore help to relieve depressive symptoms.
 Option A: Clinical and preclinical trials suggest a disturbance in central nervous
system serotonin (5-HT) activity as an important factor. Other neurotransmitters
implicated include norepinephrine (NE), dopamine (DA), glutamate, and brain-
derived neurotrophic factor (BDNF).
 Option B: The role of CNS 5-HT activity in the pathophysiology of major
depressive disorder is suggested by the therapeutic efficacy of selective serotonin
reuptake inhibitors (SSRIs). Research findings imply a role for neuronal receptor
regulation, intracellular signaling, and gene expression over time, in addition to
enhanced neurotransmitter availability.
 Option C: According to current research, dopamine, thyroxin, GABA,
acetylcholine, cortisone, and epinephrine are not directly related to depression.
The underlying pathophysiology of major depressive disorder has not been clearly
defined. Current evidence points to a complex interaction between
neurotransmitter availability and receptor regulation and sensitivity underlying the
affective symptoms.

 15. Question Nurse Rica is teaching a client and her family about the causes of
depression. Which of the following causative factors should the nurse emphasize as the
most significant?

 A. Brain structure abnormalities


B. Chemical imbalance
C. Social environment
D. Recessive gene transmission

Incorrect Correct Answer: B. Chemical imbalance
Chemical imbalance of neurotransmitters in the brain is the most significant factor in
depression. However, the exact cause has not been established, so other factors may also
be involved. The underlying pathophysiology of major depressive disorder has not been
clearly defined. Current evidence points to a complex interaction between
neurotransmitter availability and receptor regulation and sensitivity underlying the
affective symptoms.
 Option A: Vascular lesions may contribute to depression by disrupting the neural
networks involved in emotion regulation—in particular, frontostriatal pathways
that link the dorsolateral prefrontal cortex, orbitofrontal cortex, anterior cingulate,
and dorsal cingulate. Other components of limbic circuitry, in particular, the
hippocampus and amygdala, have been implicated in depression.
 Option C: A person’s social environment, including lack of support systems, may
also increase the risk of depression. The etiology of major depressive disorder is
multifactorial with both genetic and environmental factors playing a role. First-
degree relatives of depressed individuals are about 3 times as likely to develop
depression as the general population; however, depression can occur in people
without family histories of depression.
 Option D: Although genetic transmission certainly may be a factor, no definite
pattern of transmission has been identified. Some evidence suggests that genetic
factors play a lesser role in late-onset depression than in early-onset depression.
There are potential biological risk factors that have been identified for depression
in the elderly.

 16. Question Clara is under evaluation for imminent suicide risk, which information
given by her would be most significant?

 A. At least a 2-year history of feeling depressed more days than not.


B. Divorced from spouse six (6) months ago.
C. Feeling loss of energy and appetite.
D. Reference to suicide as best solution to identified problems.

Correct Correct Answer: D. Reference to suicide as best solution to identified
problems.
An individual who talks about suicide as a solution to a problem is at high risk. This
client’s suicidal threats need to be taken seriously because he does not see any other
variable solutions to problems in living. Determine whether the person has any thoughts
of hurting him or herself. Suicidal ideation is highly linked to completed suicide.
 Option A: A clear and complete evaluation and clinical interview provide the
information upon which to base a suicide intervention. Although risk factors offer
major indications of the suicide danger, nothing can substitute for a focused
patient inquiry. However, although all the answers a patient gives may be
inclusive, a therapist often develops a visceral sense that his or her patient is
going to commit suicide. The clinician’s reaction counts and should be considered
in the intervention.
 Option B: A host of thoughts and behaviors are associated with self-destructive
acts. Although many assume that people who talk about suicide will not follow
through with it, the opposite is true; a threat of suicide can lead to the completed
act, and suicidal ideation is highly correlated with suicidal behaviors.
 Option C: All of the factors included in the other options would increase the
client’s risk for depression; however, actual statements about suicidal intent are
red flags indicating imminent danger.

 17. Question Rendell is admitted in an acute psychiatric unit at Nurseslabs Medical


Center. He suddenly tells Nurse Matt about his plans for suicide. The nurse’s priority is
to:

 A. Allow the client time alone for reflection.


B. Encourage the client to use problem solving.
C. Follow agency protocol for suicide precautions.
D. Stimulate the client's interest in activities.

Incorrect Correct Answer: C. Follow agency protocol for suicide precautions.
The nurse must act to safeguard the client from danger, including self-harm
implementing the specific agency protocol for suicidal precautions would best protect the
client. Follow unit protocol for suicide regarding creating a safe environment (taking
away potential weapons– belts, sharp objects, items, and so on).
 Option A: A client with suicidal intent should not be left alone. One-to-one
observations are generally part of suicide precautions. Put on either suicide
precaution (one-on-one monitoring at one arm’s length away) or suicide
observation (15-minute visual check of mood, behavior, and verbatim statements),
depending on level of suicide potential.
 Option B: Encourage the client to talk about their feelings and problem solve
alternatives. Talking about feelings and looking at alternatives can minimize
suicidal acting out.
 Option D: Encouraging the client to use problem-solving and stimulating his
interest in activities would be helpful for someone with depression; however, the
nurse’s priority is to protect the client by initiating suicide precautions.

 18. Question Which mood disorder is characterized by the client feeling depressed most
of the day for a 2-year period?

 A. Cyclothymia
B. Dysthymia
C. Melancholic depressive disorder
D. Seasonal affective disorder

Incorrect Correct Answer: B. Dysthymia
Dysthymia is characterized by at least a 2-year history of depression, occurring most of
the day for more days than not. Persistent depressive disorder (PDD), formerly known as
dysthymia, is a fairly new diagnosis that is characterized by chronic depression. The
latest edition of the Diagnostic and Statistical Manual of Mental Disorders (DSM-5)
consolidated dysthymia and chronic major depressive disorder under the umbrella of
persistent depressive disorder, which includes any chronic depression running on a
spectrum from mild to severe.
 Option A: Cyclothymia is characterized by at least two (2) years of several
periods of hypomanic symptoms. Criterion A from the Diagnostic and Statistical
Handbook of Mental Disorders, Fifth Edition (DSM-5) defines cyclomania as:
“For at least two years (at least one year in children and adolescents) there have
been numerous periods with hypomanic symptoms that do not meet criteria for a
hypomanic episode and numerous periods with depressive symptoms that do not
meet criteria for a major depressive episode.”
 Option C: Melancholic depressive disorder is characterized by either anhedonia
in relation to all activities or lack of mood reactivity to usually pleasurable
stimuli. Melancholic depression is a form of major depressive disorder (MDD)
that is characterized by a profound presentation of severe depression. With this
form of depression, there is a complete loss of pleasure in all or almost
everything. In the DSM-5, melancholia is a specifier for MDD, so a person would
be diagnosed as having major depressive disorder (the broader illness) with
melancholic features (the specific symptoms).
 Option D: Seasonal affective disorder is characterized by depressed feelings in
fall and winter, associated with loss of sunlight. People with SAD experience
mood changes and symptoms similar to depression. The symptoms usually occur
during the fall and winter months when there is less sunlight and usually improve
with the arrival of spring. The most difficult months for people with SAD in the
United States tend to be January and February. While it is much less common,
some people experience SAD in the summer.

 19. Question Using cognitive-behavioral therapy, which treatment would be appropriate


for a client with depression?
 A. Challenging negative thinking
B. Encouraging analysis of dreams
C. Prescribing antidepressant medications
D. Using ultraviolet light therapy

Incorrect Correct Answer: A. Challenging negative thinking
Cognitive-behavioral therapy includes identifying and challenging a client’s negative
cognitions. The belief is that these negative thoughts influence the feelings and behaviors
of depression. Cognitive behavioral therapy (CBT) is a type of psychotherapeutic
treatment that helps people learn how to identify and change destructive or disturbing
thought patterns that have a negative influence on behavior and emotions.
 Option B: Dream analysis would be used in psychoanalytic psychotherapy.
Dream analysis is a therapeutic technique best known for its use in
psychoanalysis. Sigmund Freud viewed dreams as “the royal road” to the
unconscious and developed dream analysis, or dream interpretation, as a way of
tapping into this unconscious material.
 Option C: Antidepressant medication could be part of a treatment program for an
individual with depression; however, this would not be considered cognitive-
behavioral therapy. The main aim of treatment with antidepressants is to relieve
the symptoms of severe depression, such as feeling very down and exhausted, and
prevent them from coming back.
 Option D: Ultraviolet light therapy would be a somatic approach to treatment for
the seasonal affective disorder. Although light therapy is a recognized effective
treatment for seasonal affective disorder (SAD), there has been little research into
the critical wavelengths of light that produce the antidepressant effect. Previous
studies found conflicting results for the importance of the ultraviolet (UV)
spectrum in the therapeutic effect of light therapy.

 20. Question Nurse Nadine is assessing James who is diagnosed with bipolar disorder.
The nurse would expect to find a history of:

 A. A depressive episode followed by prolonged sadness.


B. A series of depressive episodes that recur periodically.
C. Symptoms of mania that may or may not be followed by depression.
D. Symptoms of mania that include delusional thoughts.

Incorrect Correct Answer: C. Symptoms of mania that may or may not be followed
by depression.
The definition of bipolar disorder is a mood disturbance in which the symptoms of mania
have occurred at least one time. None of the other options indicate a correct
understanding of bipolar disorder. The bipolar affective disorder is a chronic and
complex disorder of mood that is characterized by a combination of manic (bipolar
mania), hypomanic and depressive (bipolar depression) episodes, with substantial
subsyndromal symptoms that commonly present between major mood episodes.
 Option A: Depression may or may not occur as a separate episode in bipolar
disorder. Bipolar 2 disorder is also known as recurrent major depression with
hypomania. The DSM V criteria for the diagnosis of bipolar 2 disorder is one or
more major depressive episodes and at least one episode of hypomania. If mania
is present, then the diagnosis is bipolar 1 disorder.
 Option B: Bipolar 1 disorder has been frequently associated with serious medical
and psychiatric comorbidity, early mortality, high levels of functional disability
and compromised quality of life. The necessary feature of bipolar 1 disorder
involves the occurrence of at least one lifetime manic episode, although
depressive episodes are common.
 Option D: According to the DSM V, the only requirement for the diagnosis of
bipolar 1 disorder is at least one lifetime episode of mania. Depression may or
may not be present. Elevated mood with unusually bright affect in mania. The
patient may be euphoric or irritable and may have a labile mood during mania.
Depressed mood, limited or flat affect in depression.

 21. Question A client completing requirements for student teaching reports to the nurse
an incident in which a student was rude and disrespectful. The client states, “None of the
students respects my teaching ability.” The nurse identifies this as an example of which
common negative cognition?

 A. Labeling
B. Fortune telling
C. Overgeneralization
D. "Should" statement

Incorrect Correct Answer: C. Overgeneralization
The client in this situation is overgeneralizing the response of one particular student,
inferring that the entire class has this attitude and blowing the incident but of proportion.
Overgeneralization frequently affects people with depression or anxiety disorders. It is a
way of thinking where you apply one experience to all experiences, including those in the
future.
 Option A: Labeling is the application of negative labels to oneself or others. This
label may be a reasonable reflection of who they are right now, but it also carries
a belief that the behavior reflects a person’s essence.
 Option B: Fortune-telling is the conviction that things will not turn out right,
despite evidence to the contrary. Fortune telling is a cognitive distortion in which
you predict a negative outcome without realistically considering the actual odds
of that outcome. It is linked to anxiety and depression, and is one of the most
common cognitive distortions that arise during the course of cognitive
restructuring.
 Option D: “Should” statements refer to statements establishing standards for self
and others. Should statements are a common negative thinking pattern, or
cognitive distortion, that can contribute to feelings of fear and worry. They also
put unreasonable demands and pressure on ourselves, which can make us feel
guilty or like we’ve failed.

 22. Question The community nurse is speaking to a group of new mothers as part of a
primary prevention program. Which self-measures would be most helpful as a strategy to
decrease the occurrence of mood disorders?

 A. Keeping busy, so as not to confront problem areas.


B. Medication with antidepressants.
C. Use of crisis intervention services.
D. Verbalizing rather than internalizing feelings.

Correct Correct Answer: D. Verbalizing rather than internalizing feelings.
Individuals who develop mood disorders often have difficulty expressing feelings,
especially feelings of anger toward significant others. Internalizing those feelings can
contribute to loss of self-esteem and guilt, and therefore negative cognitions and
depression.
 Option A: Ignoring problems is not a helpful strategy. Recognizing problems and
using problem-solving methods will contribute to mental health. The nurse can
direct their need for movement into socially acceptable, large motor activities
such as arranging chairs for a community meeting or walking.
 Option B: Antidepressants are certainly necessary in the treatment of the mood
disorder of depression; however, they are not used in primary prevention.
Decreasing environmental stimulation may assist the client to relax; the nurse
must provide a quiet environment without noise, television, and other distractions;
finger foods or things the client can eat while moving around are the best options
to improve nutrition.
 Option C: Crisis intervention would be a useful strategy in handling the
immediate needs of someone experiencing a crisis; it is not a tool of primary
prevention. A primary nursing responsibility is to provide a safe environment for
the client and others; for clients who feel out of control, the nurse must establish
external controls emphatically and nonjudgmentally.

 23. Question Nurse Marge teaches the family of a client with major depression disorder.
Which of the following information should be included in the teaching? Select all that
apply.

 A. Depression is characterized by sadness, feelings of hopelessness, and


decreased self-worth
B. It is common for a pressed individual to have thoughts of suicide.
C. Attempts to cheer up a person with depression are often helpful.
D. Talk therapy, along with antidepressant medications, is usually the treatment.
E. Someone with depression may be preoccupied with spending money and too
busy to sleep.
F. Encourage a person with depression to keep a regular routine of activity
and rest.

Incorrect Correct Answer: A, B, D, F
These statements about major depressive disorders provide correct information and will
be helpful to the client’s family. Depression exists on a continuum of severity, ranging
from relatively mild, transient states of low mood to severe, long term symptoms that
have a major impact on a person’s quality of life. When a person’s symptoms have
reached the chronic end of the spectrum and require professional treatment, it’s typically
referred to as clinical depression.
 Option A: Also known as major depressive disorder or unipolar depression, this
form is what most people think of when they hear “depression.” Major depression
is typically characterized by sadness, feelings of emptiness, and feelings of
worthlessness or guilt.
 Option B: Thinking about death or dying or planning or attempting suicide is a
symptom of major depression. Encourage clients to express feelings (anger,
sadness, guilt) and come up with alternative ways to handle feelings of anger and
frustration.
 Option C: It is better to acknowledge the client’s sad mood and offer reassurance
that his mood will improve. Initially, provide activities that require minimal
concentration (e.g., drawing, playing simple board games). Depressed people lack
concentration and memory. Activities that have no “right or wrong” or “winner or
loser” minimizes opportunities for the client to put himself/herself down.
 Option D: Psychotherapy is another popular choice for treating depression, both
on its own and combined with antidepressants. Psychotherapy involves working
with a therapist, either by yourself or with a group, to talk through how you feel,
your experiences, and how you view yourself and the world.
 Option E: This is more characteristic of someone in a manic phase of bipolar
disorder. Have valuables, credit cards, and large sums of money sent home with
family or put in hospital safe until the client is discharged. During manic
episodes, people give away valuables and money indiscriminately to strangers,
often leaving themselves broke and in debt.
 Option F: When the client is in the most depressed state, Involve the client in a
one-to-one activity. Maximizes the potential for interactions while minimizing
anxiety levels. Involve the client in gross motor activities that call for very little
concentration (e.g., walking). Such activities will aid in relieving tensions and
might help in elevating the mood.

 24. Question Bianca is teaching a group of students regarding obsessive-compulsive


disorders (OCD) and obsessive-compulsive personality disorder (OCPD). Which of the
following statements differentiates OCPD from OCD?

 A. OPCD is characterized by obsessions and compulsions while OCD is marked


by perfectionisms and fixation for control.
B. People with OCPD often seek medical help while those with OCD do not.
C. Individuals with OCPD are more self-aware while those with OCD usually are
not.
D. None of the above

Correct Correct Answer: D. None of the above
 Option A: OCPD is a personality disorder that is characterized by perfectionism,
fixation for control and order. While OCD is an anxiety disorder marked by
obsessions and compulsions.
 Options B & C: Individuals with OCPD will usually avoid treatment since they
do not believe that their behaviors are normal. While people with OCD will seek
medical help since they are fully aware that their thoughts and compulsions are
irrational.

 25. Question Latrice was invited by her friend Sharon to her birthday party at a disco
ball. Sharon noticed that Latrice is avoiding interaction with the other guests and is seen
leaving the party early. She asked her what seems to be the problem. She replied that “I
might say something stupid”. Based on these observations, Latrice is experiencing:

 A. Social phobia
B. Situational phobia
C. Claustrophobia
D. Agoraphobia

Incorrect Correct Answer: A. Social phobia
Social phobia is characterized by avoidance of social situations in which there is a fear of
getting scrutinized by others that include talking in groups, eating in public, starting
conversations, or public speaking.
 Option B: Situational phobia is a fear of specific situations such as riding in a car,
flying, going over a bridge, or driving.
 Option C: Claustrophobia is the fear of enclosed spaces. It causes an
uncomfortable feeling or fear of losing control.
 Option D: Agoraphobia is a fear of open spaces and avoidance of specific
situations from which escape will be difficult or help is not possible if a panic
attack occurs.

 ABOUT
 PRIVACY
 DISCLAIMER
 CONTACT
© 2024 Nurseslabs | Ut in Omnibus Glorificetur Deus!

Gandang Swak sa Budget


Gandang swak sa budget on Shopee! Shop
exclusive deals on your favorite beauty
products Shopee

Shop Now

 1. Question The nurse closely observes the client who has been displaying aggressive
behavior. The nurse observes that the client’s anger is escalating. Which approach is least
helpful for the client at this time?

 A. Acknowledge the client’s behavior.


B. Maintain a safe distance from the client.
C. Assist the client to an area that is quiet.
D. Initiate confinement measures.

Correct Correct Answer: D. Initiate confinement measures
The proper procedure for dealing with harmful behavior is to first try to calm the patient
verbally. When verbal and psychopharmacologic interventions are not adequate to handle
aggressiveness, seclusion or restraints may be applicable. Alert staff if a potential for
seclusion appears imminent. Usual priority of interventions would be firmly setting
limits; chemical restraints (tranquilizers); and seclusions.
 Option A: Frequently assess client’s behavior for signs of increased agitation and
hyperactivity. Early detection and intervention of escalating mania will prevent
the possibility of harm to self or others, and decrease the need for seclusions.
 Option B: Redirect agitation and potentially violent behaviors with physical
outlets in an area of low stimulation (e.g., punching bag). Can help to relieve
pent-up hostility and relieve muscle tension.
 Option C: Assisting the client in a quiet place an appropriate approach during the
escalation phase of aggression. Decrease environmental stimuli (e.g., by
providing a calming environment or assigning a private room). Helps decrease
escalation of anxiety and manic symptoms.

 2. Question The charge nurse of a psychiatric unit is planning the client assignment for
the day. The most appropriate staff to be assigned to a client with a potential for violence
is which of the following:
 A. A timid nurse
B. A mature, experienced nurse
C. An inexperienced nurse
D. A soft-spoken nurse

Incorrect Correct Answer: B. A mature, experienced nurse
The unstable, aggressive client should be assigned to the most experienced nurse.
Maintain a consistent approach, employ consistent expectations, and provide a structured
environment. Clear and consistent limits and expectations minimize the potential for the
client’s manipulation of staff.
 Option A: The timid nurse may not be able to lay out consistent limits to an
angry client. The client can use inconsistencies and value judgments as
justification for arguing and escalating mania.
 Option C: An inexperienced nurse may not know how to handle the situation
appropriately. Using a calm and firm approach may provide structure and control
for a client who is out of control.
 Option D: A soft-spoken nurse may feel intimidated by the angry patient. Using
short, simple, and brief explanations or statements is appropriate for the client. A
short attention span limits understanding of small pieces of information.

 3. Question The client jumps up and throws a chair out of the window. He was restrained
after his behavior can no longer be controlled by the staff. Which of these
documentations indicates the safeguarding of the patient’s rights?

 A. There was a doctor’s order for restraints/seclusion.


B. The patient’s rights were explained to him.
C. The staff observed confidentiality.
D. The staff carried out less restrictive measures but were unsuccessful.

Correct Correct Answer: D. The staff carried out less restrictive measures but were
unsuccessful.
This documentation indicates that the client has been placed in restraints after the least
restrictive measures failed in containing the client’s violent behavior. Chart, in nurse’s
notes, behaviors; interventions; what seemed to escalate agitation; what helped to calm
agitation; when as-needed (PRN) medications were given and their effect; and what
proved most helpful.
 Option A: If nursing interventions (quiet environment and firm limit setting) and
chemical restraints (tranquilizers–e.g., haloperidol [Haldol]) have not helped
dampen escalating manic behaviors, then seclusion might be warranted.
 Option B: Use a calm and firm approach. Provides structure and control for a
client who is out of control. Use short, simple, and brief explanations or
statements. A short attention span limits understanding to small pieces of
information.
 Option C: Frequently assess client’s behavior for signs of increased agitation and
hyperactivity. Staff will begin to recognize potential signals for escalating manic
behaviors and have a guideline for what might work best for the individual client.

 4. Question Situation: Clients with personality disorders have difficulties in their social
and occupational functions.
Clients with a personality disorder will most likely:

 A. Recover with therapeutic intervention.


B. Respond to anti anxiety medication.
C. Manifest enduring patterns of inflexible behaviors.
D. Seek treatment willingly from some personally distressing symptoms.

Incorrect Correct Answer: C. Manifest enduring patterns of inflexible behaviors.
Personality disorders are characterized by inflexible traits and characteristics that are
lifelong. Each is a distinct mental illness defined by personality traits that can be
troubling enough to create problems with relating to other people in healthy ways, and
can lead to significant distress or impairment in important areas of functioning.
 Option A: Compared to mood disorders such as clinical depression and bipolar
disorder, there have historically been relatively few studies on how to effectively
treat personality disorders. Many experts believe that personality disorders are
difficult to treat because they are, by definition, long-standing patterns of
personality. However, there are an increasing number of evidence-based
treatments that are being found effective for personality disorders.
 Option B: Medications are generally not recommended for personality disorders.
The National Alliance on Mental Illness (NAMI) lists several types of
psychotherapy that may be useful in the treatment of personality disorders:
Cognitive behavior therapy (CBT), the goal of which as stated by NAMI is “to
recognize negative thoughts and learn effective coping strategies.” Mentalization-
based therapy (MBT), which teaches people to notice and reflect on their internal
states of mind and those of others. Psychodynamic therapy, which places a large
emphasis on the unconscious mind, where upsetting feelings, urges, and thoughts
that are too painful for us to directly look at are housed.
 Option D: This disorder is manifested by life-long patterns of behavior. The
client with this disorder will not likely present himself for treatment unless
something has gone wrong in his life so he may not recover from therapeutic
intervention.

 5. Question A client tends to be insensitive to others, engages in abusive behaviors and


does not have a sense of remorse. Which personality disorder is he likely to have?

 A. Narcissistic
B. Paranoid
C. Histrionic
D. Antisocial

Correct Correct Answer: D. Antisocial
These are the characteristics of an individual with an antisocial personality. Antisocial
personality disorder includes a pattern of disregarding or violating the rights of others. A
person with antisocial personality disorder may not conform to social norms, may
repeatedly lie or deceive others, or may act impulsively.
 Option A: Narcissistic personality disorder is characterized by grandiosity and a
need for constant admiration from others. A pattern of need for admiration and
lack of empathy for others. A person with a narcissistic personality disorder may
have a grandiose sense of self-importance, a sense of entitlement, take advantage
of others, or lack empathy.
 Option B: Individuals with paranoid personality demonstrate a pattern of distrust
and suspiciousness and interpret others’ motives as threatening. A pattern of being
suspicious of others and seeing them as mean or spiteful. People with a paranoid
personality disorder often assume people will harm or deceive them and don’t
confide in others or become close to them.
 Option C: Individuals with histrionic have excessive emotionality and attention-
seeking behaviors. A pattern of excessive emotion and attention-seeking. People
with a histrionic personality disorder may be uncomfortable when they are not the
center of attention, may use physical appearance to draw attention to themselves,
or have rapidly shifting or exaggerated emotions.

 6. Question The client joins a support group and frequently preaches against abuse, is
demonstrating the use of:

 A. Denial
B. Reaction formation
C. Rationalization
D. Projection

Incorrect Correct Answer: B. Reaction formation
Reaction formation is the adoption of behavior or feelings that are exactly opposite of
one’s true emotions. Reaction formation reduces anxiety by taking up the opposite
feeling, impulse, or behavior. An example of reaction formation would be treating
someone you strongly dislike in an excessively friendly manner in order to hide your true
feelings.
 Option A: Denial is a refusal to accept a painful reality. Denial is an outright
refusal to admit or recognize that something has occurred or is currently
occurring. People living with drug or alcohol addiction often deny that they have
a problem, while victims of traumatic events may deny that the event ever
occurred.
 Option C: Rationalization is attempting to justify one’s behavior by presenting
reasons that sound logical. Rationalization is a defense mechanism that involves
explaining an unacceptable behavior or feeling in a rational or logical manner,
avoiding the true reasons for the behavior.
 Option D: Projection is attributing one’s behaviors and feelings to another
person. Projection is a defense mechanism that involves taking our own
unacceptable qualities or feelings and ascribing them to other people. For
example, if you have a strong dislike for someone, you might instead believe that
they do not like you. Projection works by allowing the expression of the desire or
impulse, but in a way that the ego cannot recognize, therefore reducing anxiety.

 7. Question A teenage girl is diagnosed to have borderline personality disorder. Which


manifestations support the diagnosis?

 A. Lack of self-esteem, strong dependency needs, and impulsive behavior.


B. Social withdrawal, inadequacy, sensitivity to rejection and criticism.
C. Suspicious, hypervigilance and coldness.
D. Preoccupation with perfectionism, orderliness, and need for control.

Incorrect Correct Answer: A. Lack of self-esteem, strong dependency needs, and
impulsive behavior.
These are the characteristics of a client with borderline personality. A pattern of
instability in personal relationships, intense emotions, poor self-image and impulsivity. A
person with borderline personality disorder may go to great lengths to avoid being
abandoned, have repeated suicide attempts, display inappropriate intense anger or have
ongoing feelings of emptiness.
 Option B: This describes the avoidant personality. There is a pattern of extreme
shyness, feelings of inadequacy and extreme sensitivity to criticism. People with
avoidant personality disorder may be unwilling to get involved with people unless
they are certain of being liked, be preoccupied with being criticized or rejected, or
may view themselves as not being good enough or socially inept.
 Option C: These are the characteristics of a client with a paranoid personality. It
has a pattern of being suspicious of others and seeing them as mean or spiteful.
People with a paranoid personality disorder often assume people will harm or
deceive them and don’t confide in others or become close to them.
 Option D: This describes the obsessive-compulsive personality. There is a pattern
of preoccupation with orderliness, perfection, and control. A person with an
obsessive-compulsive personality disorder may be overly focused on details or
schedules, may work excessively not allowing time for leisure or friends, or may
be inflexible in their morality and values.

 8. Question The plan of care for clients with borderline personality should include:

 A. Limit setting and flexibility in schedule.


B. Giving medications to prevent acting out.
C. Restricting her from other clients.
D. Ensuring she adheres to certain restrictions.

Correct Correct Answer: D. Ensuring she adheres to certain restrictions.
The client is manipulative. The client must be informed about the policies, expectations,
rules, and regulations upon admission. The nurse must be quite clear about establishing
the boundaries of the therapeutic relationship to ensure that neither the client’s nor the
nurse’s boundaries are violated.
 Option A: Limits should be firmly and consistently implemented. Flexibility and
bargaining are not therapeutic in dealing with a manipulative client. Regardless of
the clinical setting, the nurse must provide structure and limit setting in the
therapeutic relationship; in a clinic setting, this may mean seeing the client for
scheduled appointments of a predetermined length rather than whenever the client
appears and demands the nurse’s immediate attention.
 Option B: There is no specific medication prescribed for this condition.
Medications are in no way curative for any personality disorder; they should be
viewed as an adjunct to psychotherapy so that the patient may productively
engage in psychotherapy.
 Option C: This is not part of the care plan. Interaction with other clients are
allowed, but the client should be observed and given limits in her attempt to
manipulate and dominate others. It is important to teach basic communication
skills such as eye contact, active listening, taking turns talking, validating the
meaning of another’s communication, and using “I” statements.

 9. Question Situation: A 42-year-old male client, is admitted to the ward because of


bizarre behaviors. He was given a diagnosis of schizophrenia paranoid type. The client
should have achieved the developmental task of:

 A. Trust vs. Mistrust


B. Industry vs. Inferiority
C. Generativity vs. Stagnation
D. Ego integrity vs. Despair

Incorrect Correct Answer: C. Generativity vs. Stagnation
The client belongs to the middle adulthood stage (30 to 65 yrs.) The developmental task
generativity is characterized by concern and care for others. It is a productive and
creative stage. Adults need to create or nurture things that will outlast them, often by
having children or creating a positive change that benefits other people. Success leads to
feelings of usefulness and accomplishment, while failure results in shallow involvement
in the world.
 Option A: The infancy stage (0 – 18 mos.) is concerned with the gratification of
oral needs. If a child successfully develops trust, the child will feel safe and
secure in the world. Caregivers who are inconsistent, emotionally unavailable, or
rejecting contribute to feelings of mistrust in the children under their care. Failure
to develop trust will result in fear and a belief that the world is inconsistent and
unpredictable.
 Option B: School Age Child (6 – 12 yrs.) is characterized by the acquisition of
school competencies and social skills. The fourth psychosocial stage takes place
during the early school years from approximately ages 5 to 11. Through social
interactions, children begin to develop a sense of pride in their accomplishments
and abilities. Children need to cope with new social and academic demands.
Success leads to a sense of competence, while failure results in feelings of
inferiority.
 Option D: Late adulthood ( 60 and above) Concerned with a reflection on the
past and his contributions to others and face the future. Erikson’s theory differed
from many others because it addressed development throughout the entire
lifespan, including old age. Older adults need to look back on life and feel a sense
of fulfillment. Success at this stage leads to feelings of wisdom, while failure
results in regret, bitterness, and despair.

 10. Question Clients who are suspicious primarily use projection for which purpose:

 A. Deny reality.
B. To deal with feelings and thoughts that are not acceptable.
C. To show resentment towards others.
D. Manipulate others.

Incorrect Correct Answer: B. To deal with feelings and thoughts that are not
acceptable.
Projection is a defense mechanism where one attributes one’s feelings and inadequacies
to others to reduce anxiety. Projection is a defense mechanism that involves taking our
own unacceptable qualities or feelings and ascribing them to other people.
 Option A: This is not true in all instances of projection. Denial is an outright
refusal to admit or recognize that something has occurred or is currently
occurring. People living with drug or alcohol addiction often deny that they have
a problem, while victims of traumatic events may deny that the event ever
occurred.
 Option C: For example, if you have a strong dislike for someone, you might
instead believe that they do not like you. Projection works by allowing the
expression of the desire or impulse, but in a way that the ego cannot recognize,
therefore reducing anxiety.
 Option D: This focuses on the self rather than others. Manipulation is using
others for one’s own advantage as a self defense mechanism. Manipulating others
to try and get people to do what is desired for personal gain usually backfires
eventually.

 11. Question The client says “ the FBI is out to get me.” The nurse’s best response is:
 A. “The FBI is not out to catch you.”
B. “I don’t believe that.”
C. “I don’t know anything about that. You are afraid of being harmed.”
D. “ What made you think of that.”

Incorrect Correct Answer: C. “I don’t know anything about that. You are afraid of
being harmed.”
This presents reality and acknowledges the client’s feelings. Interact with clients on the
basis of things in the environment. Try to distract the client from their delusions by
engaging in reality-based activities (e.g., card games, simple arts and crafts projects etc).
When thinking is focused on reality-based activities, the client is free of delusional
thinking during that time. Helps focus attention externally.
 Option A: Initially do not argue with the client’s beliefs or try to convince the
client that the delusions are false and unreal. Arguing will only increase a client’s
defensive position, thereby reinforcing false beliefs. This will result in the client
feeling even more isolated and misunderstood.
 Option B: This statement is not a therapeutic response because these disagree
with the client’s false belief and makes the client feel challenged. Attempt to
understand the significance of these beliefs to the client at the time of their
presentation. Important clues to underlying fears and issues can be found in the
client’s seemingly illogical fantasies.
 Option D: This statement is an unnecessary exploration of the false. Identify
feelings related to delusions. If a client believes someone is going to harm
him/her, the client is experiencing fear. When people believe that they are
understood, anxiety might lessen.

 12. Question The client on Haldol has pill rolling tremors and muscle rigidity. He is
likely manifesting:

 A. Tardive dyskinesia
B. Pseudoparkinsonism
C. Akinesia
D. Dystonia

Incorrect Correct Answer: B. Pseudoparkinsonism
Pseudoparkinsonism is a side effect of antipsychotic drugs characterized by mask-like
faces, pill-rolling tremors, muscle rigidity. Patients with this disorder have apraxic
slowness, paratonic rigidity, frontal gait disorder, and elements of akinesia that, taken
together, may be mistaken for true parkinsonism. Pseudoparkinsonism appears to be
common and is most often due to Alzheimer’s disease or vascular dementia.
 Option A: Tardive dyskinesia is manifested by lip-smacking, wormlike
movement of the tongue. Tardive dyskinesia (TD) is a syndrome that includes a
group of iatrogenic movement disorders caused due to a blockade of dopamine
receptors. The movement disorders include akathisia, dystonia, buccolingual
stereotypy, myoclonus, chorea, tics, and other abnormal involuntary movements
which are commonly caused by the long-term use of typical antipsychotics.
 Option C: Akinesia is characterized by a feeling of weakness and muscle fatigue.
The term akinesia refers to the inability to perform a clinically perceivable
movement. It can present as a delayed response, freezing mid-action, or even total
abolition of movement. Akinesia occurs when movement is not perceived either
because the amplitude of the movement is small or because the time taken to
initiate the reaction is significantly increased.
 Option D: Dystonia is manifested by torticollis and rolling back of the eyes.
Dystonia is defined by involuntary maintained contraction of agonist and
antagonist muscles yielding abnormal posturing, twisting, and repetitive
movements or tremulous and can be initiated or worsened by attempted
movement.

 13. Question The client is very hostile toward one of the staff for no apparent reason.
The client is manifesting:

 A. Splitting
B. Transference
C. Countertransference
D. Resistance

Incorrect Correct Answer: B. Transference
Transference is a positive or negative feeling associated with a significant person in the
client’s past that is unconsciously assigned to another. Transference describes a situation
where the feelings, desires, and expectations of one person are redirected and applied to
another person. Most commonly, transference refers to a therapeutic setting, where a
person in therapy may apply certain feelings or emotions toward the therapist.
 Option A: Splitting is a defense mechanism commonly seen in a client with a
personality disorder in which the world is perceived as all good or all bad.
Splitting is a term used in psychiatry to describe the inability to hold opposing
thoughts, feelings, or beliefs. Some might say that a person who splits sees the
world in terms of black or white—all or nothing. It’s a distorted way of thinking
in which the positive or negative attributes of a person or event are neither
weighed nor cohesive.
 Option C: Countertransference is a phenomenon where the nurse shifts feelings
assigned to someone in her past to the patient. Countertransference, which occurs
when a therapist transfers emotions to a person in therapy, is often a reaction to
transference, a phenomenon in which the person in treatment redirects feelings for
others onto the therapist.
 Option D: Resistance is the client’s refusal to submit himself to the care of the
nurse. Resistance in psychology refers to any opposition to the therapeutic
process. Resistance is a way of pushing back against suggestions, even those that
could help you solve mental or emotional health concerns.

 14. Question Situation: An 18-year-old female is sexually attacked while on her way
home from work. She was brought to the hospital by her mother. Rape is an example of
which type of crisis:

 A. Situational
B. Adventitious
C. Developmental
D. Internal

Incorrect Correct Answer: B. Adventitious
Adventitious crisis is a crisis involving a traumatic event. It is not part of everyday life.
An adventitious crisis is a crisis of disaster that is not a part of everyday life. It is
unplanned or accidental. Adventitious crisis include natural disasters, national disasters,
and crimes of violence. Sexual molestation falls within this classification.
 Option A: Situational crisis is from an external source that upsets one’s
psychological equilibrium. Situational crisis arises from an external source such
as a job loss, divorce, or other loss affecting self-concept or self-esteem. These
occur as part of the process of growing and developing through various periods of
life. Sometimes a crisis is a predictable part of the life cycle, such as the crisis
described in Erikson’s stages of psychosocial development.
 Options C & D: These are the same. They are transitional or developmental
periods in life. A life crisis in which usual coping mechanisms are inadequate in
dealing with stress common to a particular stage in the life cycle or with stress
caused by a transition from one stage to another.

 15. Question During the initial care of rape victims, the following are to be considered
except:

 A. Assure privacy.
B. Touch the client to show acceptance and empathy.
C. Accompany the client to the examination room.
D. Maintain a non-judgmental approach.

Incorrect Correct Answer: B. Touch the client to show acceptance and empathy.
The client finds touch intrusive and therefore should be avoided. Establish trust and
rapport. Since the victim may misinterpret any statements unrelated to her immediate
situation as blaming or rejecting, the nurse should delay asking questions until the
therapeutic nature is established.
 Option A: Privacy is one of the rights of a victim of rape. Provide strict
confidentiality. The client’s situation is not to be talked over with anyone other
than medical staff involved unless the client gives consent to it.
 Option C: The client is anxious. Accompanying the client in a quiet room ensures
the safety and offers emotional support. Have someone stay with the client
(friend, neighbor, or staff member) while he or she waits to be treated. People
with high levels of anxiety need to feel physical safety by providing someone by
his/her side until the anxiety level is down to moderate.
 Option D: Guilt feeling is common among rape victims. They should not be
blamed. Stress that they did the right thing to save their life. Rape victims might
feel guilt and shame. Reinforcing that they did what they had to do to stay alive
can reduce guilt and maintain self-esteem.

 16. Question The nurse acts as a patient advocate when she does one of the following:

 A. She encourages the client to express her feelings regarding her experience.
B. She assesses the client for injuries.
C. She postpones the physical assessment until the client is calm.
D. Explains to the client that her reactions are normal.

Incorrect Correct Answer: C. She postpones the physical assessment until the client
is calm
The nurse acts as a patient advocate as she protects the client from psychological harm.
Nurse advocates support the patient’s best interests while respecting the family’s
important role. Advocates become facilitators when patients and family members need to
discuss uncomfortable information or explore its implications. Nurses may sometimes
need to advocate for patients against their families.
 Option A: The nurse acts as a counselor. A need for counseling is a clinical
judgment made by the nurse, and his/her response will be immediate situational
counseling, continuing counseling sessions, or referral. Types of counseling
situations that the nursing professional may encounter are outlined, including four
escapes utilized by students, such as substance abuse, suicide, fear of AIDS, and
anorexia/bulimia. Since students do utilize health services, the nurse counselor
with a holistic view of nursing care, which includes health education and
counseling, has an opportunity and a responsibility to act upon this knowledge.
 Option B: The nurse acts as a technician. Nurse technicians are medical care
providers who give basic medical care to patients. A nurse tech generally works
under the supervision of a Registered Nurse. Also known as nursing attendants or
nursing aides, they provide important services to help the registered nurses
complete their tasks.
 Option D: This exemplifies the role of a teacher. Nurse educators are registered
nurses with advanced education who are also teachers. Most work for several, if
not many, years before deciding to turn to a career teaching future nurses. Most
nurse educators have extensive clinical experience, and many continue caring for
patients after becoming educators.

 17. Question Crisis intervention carried out to the client has this primary goal:

 A. Assist the client to express her feelings.


B. Help her identify her resources.
C. Support her adaptive coping skills.
D. Help her return to her pre-rape level of function.

Correct Correct Answer: D. Help her return to her pre-rape level of function.
The goal of crisis intervention is to help the client return to her level of function prior to
the crisis. Crisis intervention is a short-term management technique designed to reduce
potential permanent damage to an individual affected by a crisis. A crisis is defined as an
overwhelming event, which can include divorce, violence, the passing of a loved one, or
the discovery of a serious illness.
 Option A: A successful intervention involves obtaining background information
on the patient, establishing a positive relationship, discussing the events, and
providing emotional support. SAFER-R is a common intervention model used,
which consists of stabilization, acknowledgment, facilitate understanding,
encouragement, recovery, and referral. SAFER-R helps patients return to their
mental baseline following a crisis.
 Option B: Based on prior studies, it is evident that crisis intervention plays a
significant role in enhancing outcomes in psychiatric cases. Community Mental
Health Centers and local government agencies often have crisis intervention
teams that provide support to the local community at times of mental health crisis.
 Option C: Another major concern is what coping strategies are most effective.
Social support and problem-solving planning are effective coping mechanisms
that are frequently used by school staff following a crisis. The use of humor,
emotional support, planning, and acceptance also correlate with superior mental
health outcomes compared to substance abuse and denial.

 18. Question Five months after the incident the client complains of difficulty to
concentrate, poor appetite, inability to sleep and guilt. She is likely suffering from:

 A. Adjustment disorder
B. Somatoform Disorder
C. Generalized Anxiety Disorder
D. Post-traumatic disorder

Correct Correct Answer: D. Post-traumatic disorder
Post-traumatic stress disorder is characterized by flashbacks, irritability, difficulty falling
asleep, and concentrating following an extremely traumatic event. This lasts for more
than one month. Posttraumatic stress disorder (PTSD) is a syndrome that results from
exposure to real or threatened death, serious injury, or sexual assault. Following the
traumatic event, PTSD is common and is one of the serious health concerns that is
associated with comorbidity, functional impairment, and increased mortality with suicidal
ideations and attempts.
 Option A: Adjustment disorder is the maladaptive reaction to stressful events
characterized by anxiety, depression, and work or social impairments. This occurs
within three (3) months after the event. Adjustment disorders involve markedly
distressing and impairing emotional and/or behavioral symptoms caused by an
identifiable stressor.
 Option B: Somatoform disorders are anxiety-related disorders characterized by
the presence of physical symptoms without a demonstrable organic basis. The
somatoform disorders are a group of psychiatric disorders in which patients
present with a myriad of clinically significant but unexplained physical
symptoms. They include somatization disorder, undifferentiated somatoform
disorder, hypochondriasis, conversion disorder, pain disorder, body dysmorphic
disorder, and somatoform disorder not otherwise specified.
 Option C: Generalized anxiety disorder is characterized by chronic, excessive
anxiety for at least 6 months. Generalized anxiety disorder is one of the most
common mental disorders. Up to 20% of adults are affected by anxiety disorders
each year. Generalized anxiety disorder produces fear, worry, and a constant
feeling of being overwhelmed.

 19. Question Situation: A 29-year-old client newly diagnosed with breast cancer is
pacing, with rapid speech headache and inability to focus on what the doctor was saying.
The nurse assesses the level of anxiety as:

 A. Mild
B. Moderate
C. Severe
D. Panic

Incorrect Correct Answer: C. Severe
The client’s manifestations indicate severe anxiety. Severe anxiety is intensely
debilitating, and symptoms of severe anxiety meet key diagnostic criteria for clinically-
significant anxiety disorders. People with severe anxiety typically score higher on scales
of distress and lower on functioning. Severe anxiety symptoms also frequently co-occur
with major depression, which can contribute to greater disability.
 Option A: Mild anxiety is manifested by slight muscle tension, slight fidgeting,
alertness, the ability to concentrate, and capable of problem-solving. Although
often described as sub-clinical or clinically non-significant, mild anxiety can
impact emotional, social, and professional functioning. Mild anxiety symptoms
may present as social anxiety or shyness and can be experienced in early
childhood through to adulthood. If left unaddressed, mild anxiety can lead to
maladaptive coping strategies or more severe mental conditions.
 Option B: Moderate muscle tension, increased vital signs, periodic slow pacing,
increased rate of speech, and difficulty in concentrating are noted in moderate
anxiety. People with moderate levels of anxiety have more frequent or persistent
symptoms than those with mild anxiety, but still have better daily functioning
than someone with severe anxiety or panic disorder. For example, people with
moderate anxiety may report experiencing symptoms such as feeling on edge,
being unable to control their worrying, or being unable to relax several days or the
majority of days in a week, but not every day.
 Option D: Panic anxiety is characterized by immobilization, incoherence, a
feeling of being overwhelmed, and disorganization. Panic level anxiety, or panic
disorder, is characterized by frequent, recurring, and unexpected panic attacks.
Panic attacks usually last around 10 minutes. The triggers for panic attacks vary
from person to person, and the cause of an attack may be familiar to a person or
unknown.

 20. Question Anxiety is caused by:

 A. An objective threat.
B. A subjectively perceived threat.
C. Hostility turned to the self.
D. Masked depression.

Incorrect Correct Answer: B. A subjectively perceived threat.
Anxiety is caused by a subjectively perceived threat. Anxiety is one of the most common
mental disorders, with 19.1% of adults in the U.S. being affected in the past year. Anxiety
can begin early in life, with an average age of onset of 11 years old, and it may range
from mildly uncomfortable symptoms to severe and debilitating panic that can interfere
with a person’s ability to live normally.
 Option A: Fear is caused by an objective threat. Sometimes fear stems from real
threats, but it can also originate from imagined dangers. Fear can also be a
symptom of some mental health conditions including panic disorder, social
anxiety disorder, phobias, and post-traumatic stress disorder (PTSD).
 Option C: A depressed client internalizes hostility. The common features of all
the depressive disorders are sadness, emptiness, or irritable mood, accompanied
by somatic and cognitive changes that significantly affect the individual’s
capacity to function.
 Option D: Mania is due to masked depression. Some other hallmarks of mania
are an elevated or expansive mood, mood lability, impulsivity, irritability, and
grandiosity. If the individual experiencing these symptoms requires
hospitalization, then this period automatically qualifies as true mania and not
hypomania, even if the symptoms are present for less than one week.

 21. Question It would be most helpful for the nurse to deal with a client with severe
anxiety by:

 A. Give specific instructions using speak in concise statements.


B. Ask the client to identify the cause of her anxiety.
C. Explain in detail the plan of care developed.
D. Urge the client to focus on what the nurse is saying.

Incorrect Correct Answer: A. Give specific instructions using speak in concise
statements.
The client has narrowed the perceptual field. Lengthy explanations cannot be followed by
the client. Maintain a calm, non-threatening manner while working with the client.
Anxiety is contagious and may be transferred from health care provider to client or vice
versa. The client develops a feeling of security in presence of a calm staff person.
 Option B: The client will not be able to identify the cause of anxiety. Establish
and maintain a trusting relationship by listening to the client; displaying warmth,
answering questions directly, offering unconditional acceptance; being available
and respecting the client’s use of personal space.
 Option C: Move the client to a quiet area with minimal stimuli such as a small
room or seclusion area (dim lighting, few people, and so on.) Anxious behavior
escalates by external stimuli. A smaller or secluded area enhances a sense of
security as compared to a large area which can make the client feel lost and
panicked.
 Option D: The client has difficulty concentrating and will not be able to focus.
Remain with the client at all times when levels of anxiety are high (severe or
panic); reassure the client of his or her safety and security. The client’s safety is
utmost priority. A highly anxious client should not be left alone as his anxiety will
escalate.

 22. Question Which of the following medications will likely be ordered for the client?”

 A. Prozac
B. Valium
C. Risperdal
D. Lithium

Incorrect Correct Answer: B. Valium
Diazepam is an anxiolytic benzodiazepine, first patented and marketed in the United
States in 1963. It is a fast-acting, long-lasting benzodiazepine commonly used in the
treatment of anxiety disorders, as well as alcohol detoxification, acute recurrent seizures,
severe muscle spasm, and spasticity associated with neurologic disorders. In the setting of
acute alcohol withdrawal, diazepam is useful for symptomatic relief of agitation, tremor,
alcoholic hallucinosis, and acute delirium tremens.
 Option A: Fluoxetine has FDA-approval for major depressive disorder (age eight
and older), obsessive-compulsive disorder (age seven and older), panic disorder,
bulimia, binge eating disorder, premenstrual dysphoric disorder, bipolar
depression (as an adjunct with olanzapine also known as Symbyax), and
treatment-resistant depression when used in combination with olanzapine. Non-
FDA-approved uses for fluoxetine include social anxiety disorder (social phobia),
post-traumatic stress disorder in adults, borderline personality disorder, Raynaud
phenomenon, and selective mutism.
 Option C: In addition to psychotic symptoms, risperidone can be used for
aggression and agitation in patients with dementia. Risperidone has also been
used for augmentation of antidepressant therapy in the treatment of non-psychotic
unipolar depression. In addition to irritability associated with autism, risperidone
has also been used for social impairment, stereotypical behaviors, cognitive
problems, and hyperactivity in autism.
 Option D: Lithium was the first mood stabilizer and is still the first-line treatment
option, but is underutilized because it is an older drug. Lithium is a commonly
prescribed drug for a manic episode in bipolar disorder as well as maintenance
therapy of bipolar disorder in a patient with a history of a manic episode. The
primary target symptoms of lithium are mania and unstable mood.

 23. Question Which of the following is included in the health teachings among clients
receiving Valium?

 A. Avoid foods rich in tyramine.


B. Take the medication after meals.
C. It is safe to stop it anytime after long term use.
D. Double up the dose if the client forgets her medication.

Incorrect Correct Answer: B. Take the medication after meals.
Anti Anxiety medications cause G.I. upset so it should be taken after meals.
Benzodiazepines are a class of medicines approved to treat generalized anxiety disorder,
insomnia, seizures, social phobia, and panic disorder.
 Option A: This is specific for antidepressant MAOI. Taking tyramine-rich food
can cause a hypertensive crisis. MAOIs prevent the breakdown of tyramine found
in the body as well as certain foods, drinks, and other medications. Patients that
take MAOIs and consume tyramine-containing foods or drinks will exhibit high
serum tyramine level. A high level of tyramine can cause a sudden increase in
blood pressure, called the tyramine pressor response.
 Option C: Valium causes dependency. In which case, the medication should be
gradually withdrawn to prevent the occurrence of convulsion. However, the
potential of overdose from diverted diazepam always exists when combined with
opioids, alcohol, or other centrally acting agents. Overdose in adults frequently
involves the co-ingestion of other CNS depressants, which work synergistically to
increase toxicity. In the case of single-agent diazepam overdose, symptoms
manifest as CNS depression and are very rarely fatal. In mild cases, lethargy,
drowsiness, and confusion are common symptoms.
 Option D: The dose of Valium should not be doubled if the previous dose was
not taken. It can intensify the CNS depressant effects. In cases of severe overdose,
symptoms manifest as ataxia, diminished reflexes, hypotonia, hypotension,
respiratory depression, coma (rarely), and death (very rarely).

 24. Question Mrs. B is diagnosed with borderline personality disorder and has a nursing
diagnosis of Risk for self-directed violence, which is related to the client’s self-mutilation
behavior (burning arms with cigarettes). Which client behavior would indicate a positive
outcome of intervention?

 A. Mrs. B denies feelings of wanting to harm anyone.


B. Mrs. B expresses feelings of anger towards others.
C. Mrs. B requests cigarettes at appropriate times.
D. Mrs. B tells the nurse about wanting to burn herself.

Correct Correct Answer: D. Mrs. B tells the nurse about wanting to burn herself.
The fact that Mrs. B directly tells the nurse about wanting to self-mutilate, rather than
acting on these feelings, is evidence of her responding to nursing intervention. Use a
matter-of-fact approach when self-mutilation occurs. Avoid criticizing or giving
sympathy. A neutral approach prevents blaming, which increases anxiety, giving special
attention that encourages acting out.
 Option A: Identify feelings experienced before and around the act of self-
mutilation. Feelings are a guideline for future intervention (e.g., rage at feeling
left out or abandoned). Explore with the client what these feelings might mean.
 Option B: Set and maintain limits on acceptable behavior and make clear client’s
responsibilities. If the client is hospitalized at the time, be clear regarding the unit
rules. Clear and non-punitive limit setting is essential for decreasing negative
behaviors.
 Option C: Be consistent in maintaining and enforcing the limits, using a non-
punitive approach. Consistency can establish a sense of security.

 25. Question Nurse Aldrich is working with the family of Mary Ann, a client with a
personality disorder. Which of the following should Nurse Aldrich encourage the family
members to work on?

 A. Avoiding direct expressions of problems with family.


B. Changing Mary Ann's problem behaviors.
C. Improving self-functioning.
D. Supporting Mary Ann's defenses.

Incorrect Correct Answer: C. Improving self-functioning.
Family members typically benefit from working on ways to improve self-functioning.
This facilitates ownership of problems among individuals involved in ongoing
relationship difficulties. Keep goals very realistic and go in small steps. There are no
overnight successes with people with personality disorders.
 Option A: The direct expression of problems is helpful and therefore should not
be avoided. When the client is ready and interested, teach client coping skills to
help defuse tension and trouble feelings (e.g., anxiety reduction, assertiveness
skills).
 Option B: It would be impossible to change the client’s behavior; encouraging
family members to do so would frustrate them. Give the client positive attention
when behaviors are appropriate and productive. Avoid giving any attention (when
possible and not dangerous to self or others) when client’s behaviors are
inappropriate.
 Option D: The client’s defenses are likely to blame others for problems;
consequently, supporting his blaming others is not helpful. Use assertiveness
when setting limits on client’s unreasonable demands for attention and time.

 ABOUT
 PRIVACY
 DISCLAIMER
 CONTACT
© 2024 Nurseslabs | Ut in Omnibus Glorificetur Deus!

Gandang Swak sa Budget


Shopee

Face Brush Mask Double ended Blackhead remover

 1. Question The nurse closely observes the client who has been displaying aggressive
behavior. The nurse observes that the client’s anger is escalating. Which approach is least
helpful for the client at this time?

 A. Acknowledge the client’s behavior.


B. Maintain a safe distance from the client.
C. Assist the client to an area that is quiet.
D. Initiate confinement measures.

Correct Correct Answer: D. Initiate confinement measures
The proper procedure for dealing with harmful behavior is to first try to calm the patient
verbally. When verbal and psychopharmacologic interventions are not adequate to handle
aggressiveness, seclusion or restraints may be applicable. Alert staff if a potential for
seclusion appears imminent. Usual priority of interventions would be firmly setting
limits; chemical restraints (tranquilizers); and seclusions.
 Option A: Frequently assess client’s behavior for signs of increased agitation and
hyperactivity. Early detection and intervention of escalating mania will prevent
the possibility of harm to self or others, and decrease the need for seclusions.
 Option B: Redirect agitation and potentially violent behaviors with physical
outlets in an area of low stimulation (e.g., punching bag). Can help to relieve
pent-up hostility and relieve muscle tension.
 Option C: Assisting the client in a quiet place an appropriate approach during the
escalation phase of aggression. Decrease environmental stimuli (e.g., by
providing a calming environment or assigning a private room). Helps decrease
escalation of anxiety and manic symptoms.

 2. Question The charge nurse of a psychiatric unit is planning the client assignment for
the day. The most appropriate staff to be assigned to a client with a potential for violence
is which of the following:

 A. A timid nurse
B. A mature, experienced nurse
C. An inexperienced nurse
D. A soft-spoken nurse

Incorrect Correct Answer: B. A mature, experienced nurse
The unstable, aggressive client should be assigned to the most experienced nurse.
Maintain a consistent approach, employ consistent expectations, and provide a structured
environment. Clear and consistent limits and expectations minimize the potential for the
client’s manipulation of staff.
 Option A: The timid nurse may not be able to lay out consistent limits to an
angry client. The client can use inconsistencies and value judgments as
justification for arguing and escalating mania.
 Option C: An inexperienced nurse may not know how to handle the situation
appropriately. Using a calm and firm approach may provide structure and control
for a client who is out of control.
 Option D: A soft-spoken nurse may feel intimidated by the angry patient. Using
short, simple, and brief explanations or statements is appropriate for the client. A
short attention span limits understanding of small pieces of information.

 3. Question The client jumps up and throws a chair out of the window. He was restrained
after his behavior can no longer be controlled by the staff. Which of these
documentations indicates the safeguarding of the patient’s rights?

 A. There was a doctor’s order for restraints/seclusion.


B. The patient’s rights were explained to him.
C. The staff observed confidentiality.
D. The staff carried out less restrictive measures but were unsuccessful.

Correct Correct Answer: D. The staff carried out less restrictive measures but were
unsuccessful.
This documentation indicates that the client has been placed in restraints after the least
restrictive measures failed in containing the client’s violent behavior. Chart, in nurse’s
notes, behaviors; interventions; what seemed to escalate agitation; what helped to calm
agitation; when as-needed (PRN) medications were given and their effect; and what
proved most helpful.
 Option A: If nursing interventions (quiet environment and firm limit setting) and
chemical restraints (tranquilizers–e.g., haloperidol [Haldol]) have not helped
dampen escalating manic behaviors, then seclusion might be warranted.
 Option B: Use a calm and firm approach. Provides structure and control for a
client who is out of control. Use short, simple, and brief explanations or
statements. A short attention span limits understanding to small pieces of
information.
 Option C: Frequently assess client’s behavior for signs of increased agitation and
hyperactivity. Staff will begin to recognize potential signals for escalating manic
behaviors and have a guideline for what might work best for the individual client.

 4. Question Situation: Clients with personality disorders have difficulties in their social
and occupational functions.
Clients with a personality disorder will most likely:

 A. Recover with therapeutic intervention.


B. Respond to anti anxiety medication.
C. Manifest enduring patterns of inflexible behaviors.
D. Seek treatment willingly from some personally distressing symptoms.

Incorrect Correct Answer: C. Manifest enduring patterns of inflexible behaviors.
Personality disorders are characterized by inflexible traits and characteristics that are
lifelong. Each is a distinct mental illness defined by personality traits that can be
troubling enough to create problems with relating to other people in healthy ways, and
can lead to significant distress or impairment in important areas of functioning.
 Option A: Compared to mood disorders such as clinical depression and bipolar
disorder, there have historically been relatively few studies on how to effectively
treat personality disorders. Many experts believe that personality disorders are
difficult to treat because they are, by definition, long-standing patterns of
personality. However, there are an increasing number of evidence-based
treatments that are being found effective for personality disorders.
 Option B: Medications are generally not recommended for personality disorders.
The National Alliance on Mental Illness (NAMI) lists several types of
psychotherapy that may be useful in the treatment of personality disorders:
Cognitive behavior therapy (CBT), the goal of which as stated by NAMI is “to
recognize negative thoughts and learn effective coping strategies.” Mentalization-
based therapy (MBT), which teaches people to notice and reflect on their internal
states of mind and those of others. Psychodynamic therapy, which places a large
emphasis on the unconscious mind, where upsetting feelings, urges, and thoughts
that are too painful for us to directly look at are housed.
 Option D: This disorder is manifested by life-long patterns of behavior. The
client with this disorder will not likely present himself for treatment unless
something has gone wrong in his life so he may not recover from therapeutic
intervention.

 5. Question A client tends to be insensitive to others, engages in abusive behaviors and


does not have a sense of remorse. Which personality disorder is he likely to have?

 A. Narcissistic
B. Paranoid
C. Histrionic
D. Antisocial

Correct Correct Answer: D. Antisocial
These are the characteristics of an individual with an antisocial personality. Antisocial
personality disorder includes a pattern of disregarding or violating the rights of others. A
person with antisocial personality disorder may not conform to social norms, may
repeatedly lie or deceive others, or may act impulsively.
 Option A: Narcissistic personality disorder is characterized by grandiosity and a
need for constant admiration from others. A pattern of need for admiration and
lack of empathy for others. A person with a narcissistic personality disorder may
have a grandiose sense of self-importance, a sense of entitlement, take advantage
of others, or lack empathy.
 Option B: Individuals with paranoid personality demonstrate a pattern of distrust
and suspiciousness and interpret others’ motives as threatening. A pattern of being
suspicious of others and seeing them as mean or spiteful. People with a paranoid
personality disorder often assume people will harm or deceive them and don’t
confide in others or become close to them.
 Option C: Individuals with histrionic have excessive emotionality and attention-
seeking behaviors. A pattern of excessive emotion and attention-seeking. People
with a histrionic personality disorder may be uncomfortable when they are not the
center of attention, may use physical appearance to draw attention to themselves,
or have rapidly shifting or exaggerated emotions.

 6. Question The client joins a support group and frequently preaches against abuse, is
demonstrating the use of:

 A. Denial
B. Reaction formation
C. Rationalization
D. Projection

Incorrect Correct Answer: B. Reaction formation
Reaction formation is the adoption of behavior or feelings that are exactly opposite of
one’s true emotions. Reaction formation reduces anxiety by taking up the opposite
feeling, impulse, or behavior. An example of reaction formation would be treating
someone you strongly dislike in an excessively friendly manner in order to hide your true
feelings.
 Option A: Denial is a refusal to accept a painful reality. Denial is an outright
refusal to admit or recognize that something has occurred or is currently
occurring. People living with drug or alcohol addiction often deny that they have
a problem, while victims of traumatic events may deny that the event ever
occurred.
 Option C: Rationalization is attempting to justify one’s behavior by presenting
reasons that sound logical. Rationalization is a defense mechanism that involves
explaining an unacceptable behavior or feeling in a rational or logical manner,
avoiding the true reasons for the behavior.
 Option D: Projection is attributing one’s behaviors and feelings to another
person. Projection is a defense mechanism that involves taking our own
unacceptable qualities or feelings and ascribing them to other people. For
example, if you have a strong dislike for someone, you might instead believe that
they do not like you. Projection works by allowing the expression of the desire or
impulse, but in a way that the ego cannot recognize, therefore reducing anxiety.

 7. Question A teenage girl is diagnosed to have borderline personality disorder. Which


manifestations support the diagnosis?

 A. Lack of self-esteem, strong dependency needs, and impulsive behavior.


B. Social withdrawal, inadequacy, sensitivity to rejection and criticism.
C. Suspicious, hypervigilance and coldness.
D. Preoccupation with perfectionism, orderliness, and need for control.

Incorrect Correct Answer: A. Lack of self-esteem, strong dependency needs, and
impulsive behavior.
These are the characteristics of a client with borderline personality. A pattern of
instability in personal relationships, intense emotions, poor self-image and impulsivity. A
person with borderline personality disorder may go to great lengths to avoid being
abandoned, have repeated suicide attempts, display inappropriate intense anger or have
ongoing feelings of emptiness.
 Option B: This describes the avoidant personality. There is a pattern of extreme
shyness, feelings of inadequacy and extreme sensitivity to criticism. People with
avoidant personality disorder may be unwilling to get involved with people unless
they are certain of being liked, be preoccupied with being criticized or rejected, or
may view themselves as not being good enough or socially inept.
 Option C: These are the characteristics of a client with a paranoid personality. It
has a pattern of being suspicious of others and seeing them as mean or spiteful.
People with a paranoid personality disorder often assume people will harm or
deceive them and don’t confide in others or become close to them.
 Option D: This describes the obsessive-compulsive personality. There is a pattern
of preoccupation with orderliness, perfection, and control. A person with an
obsessive-compulsive personality disorder may be overly focused on details or
schedules, may work excessively not allowing time for leisure or friends, or may
be inflexible in their morality and values.

 8. Question The plan of care for clients with borderline personality should include:

 A. Limit setting and flexibility in schedule.


B. Giving medications to prevent acting out.
C. Restricting her from other clients.
D. Ensuring she adheres to certain restrictions.

Correct Correct Answer: D. Ensuring she adheres to certain restrictions.
The client is manipulative. The client must be informed about the policies, expectations,
rules, and regulations upon admission. The nurse must be quite clear about establishing
the boundaries of the therapeutic relationship to ensure that neither the client’s nor the
nurse’s boundaries are violated.
 Option A: Limits should be firmly and consistently implemented. Flexibility and
bargaining are not therapeutic in dealing with a manipulative client. Regardless of
the clinical setting, the nurse must provide structure and limit setting in the
therapeutic relationship; in a clinic setting, this may mean seeing the client for
scheduled appointments of a predetermined length rather than whenever the client
appears and demands the nurse’s immediate attention.
 Option B: There is no specific medication prescribed for this condition.
Medications are in no way curative for any personality disorder; they should be
viewed as an adjunct to psychotherapy so that the patient may productively
engage in psychotherapy.
 Option C: This is not part of the care plan. Interaction with other clients are
allowed, but the client should be observed and given limits in her attempt to
manipulate and dominate others. It is important to teach basic communication
skills such as eye contact, active listening, taking turns talking, validating the
meaning of another’s communication, and using “I” statements.

 9. Question Situation: A 42-year-old male client, is admitted to the ward because of


bizarre behaviors. He was given a diagnosis of schizophrenia paranoid type. The client
should have achieved the developmental task of:
 A. Trust vs. Mistrust
B. Industry vs. Inferiority
C. Generativity vs. Stagnation
D. Ego integrity vs. Despair

Incorrect Correct Answer: C. Generativity vs. Stagnation
The client belongs to the middle adulthood stage (30 to 65 yrs.) The developmental task
generativity is characterized by concern and care for others. It is a productive and
creative stage. Adults need to create or nurture things that will outlast them, often by
having children or creating a positive change that benefits other people. Success leads to
feelings of usefulness and accomplishment, while failure results in shallow involvement
in the world.
 Option A: The infancy stage (0 – 18 mos.) is concerned with the gratification of
oral needs. If a child successfully develops trust, the child will feel safe and
secure in the world. Caregivers who are inconsistent, emotionally unavailable, or
rejecting contribute to feelings of mistrust in the children under their care. Failure
to develop trust will result in fear and a belief that the world is inconsistent and
unpredictable.
 Option B: School Age Child (6 – 12 yrs.) is characterized by the acquisition of
school competencies and social skills. The fourth psychosocial stage takes place
during the early school years from approximately ages 5 to 11. Through social
interactions, children begin to develop a sense of pride in their accomplishments
and abilities. Children need to cope with new social and academic demands.
Success leads to a sense of competence, while failure results in feelings of
inferiority.
 Option D: Late adulthood ( 60 and above) Concerned with a reflection on the
past and his contributions to others and face the future. Erikson’s theory differed
from many others because it addressed development throughout the entire
lifespan, including old age. Older adults need to look back on life and feel a sense
of fulfillment. Success at this stage leads to feelings of wisdom, while failure
results in regret, bitterness, and despair.

 10. Question Clients who are suspicious primarily use projection for which purpose:

 A. Deny reality.
B. To deal with feelings and thoughts that are not acceptable.
C. To show resentment towards others.
D. Manipulate others.

Incorrect Correct Answer: B. To deal with feelings and thoughts that are not
acceptable.
Projection is a defense mechanism where one attributes one’s feelings and inadequacies
to others to reduce anxiety. Projection is a defense mechanism that involves taking our
own unacceptable qualities or feelings and ascribing them to other people.
 Option A: This is not true in all instances of projection. Denial is an outright
refusal to admit or recognize that something has occurred or is currently
occurring. People living with drug or alcohol addiction often deny that they have
a problem, while victims of traumatic events may deny that the event ever
occurred.
 Option C: For example, if you have a strong dislike for someone, you might
instead believe that they do not like you. Projection works by allowing the
expression of the desire or impulse, but in a way that the ego cannot recognize,
therefore reducing anxiety.
 Option D: This focuses on the self rather than others. Manipulation is using
others for one’s own advantage as a self defense mechanism. Manipulating others
to try and get people to do what is desired for personal gain usually backfires
eventually.

 11. Question The client says “ the FBI is out to get me.” The nurse’s best response is:

 A. “The FBI is not out to catch you.”


B. “I don’t believe that.”
C. “I don’t know anything about that. You are afraid of being harmed.”
D. “ What made you think of that.”

Incorrect Correct Answer: C. “I don’t know anything about that. You are afraid of
being harmed.”
This presents reality and acknowledges the client’s feelings. Interact with clients on the
basis of things in the environment. Try to distract the client from their delusions by
engaging in reality-based activities (e.g., card games, simple arts and crafts projects etc).
When thinking is focused on reality-based activities, the client is free of delusional
thinking during that time. Helps focus attention externally.
 Option A: Initially do not argue with the client’s beliefs or try to convince the
client that the delusions are false and unreal. Arguing will only increase a client’s
defensive position, thereby reinforcing false beliefs. This will result in the client
feeling even more isolated and misunderstood.
 Option B: This statement is not a therapeutic response because these disagree
with the client’s false belief and makes the client feel challenged. Attempt to
understand the significance of these beliefs to the client at the time of their
presentation. Important clues to underlying fears and issues can be found in the
client’s seemingly illogical fantasies.
 Option D: This statement is an unnecessary exploration of the false. Identify
feelings related to delusions. If a client believes someone is going to harm
him/her, the client is experiencing fear. When people believe that they are
understood, anxiety might lessen.

 12. Question The client on Haldol has pill rolling tremors and muscle rigidity. He is
likely manifesting:

 A. Tardive dyskinesia
B. Pseudoparkinsonism
C. Akinesia
D. Dystonia

Incorrect Correct Answer: B. Pseudoparkinsonism
Pseudoparkinsonism is a side effect of antipsychotic drugs characterized by mask-like
faces, pill-rolling tremors, muscle rigidity. Patients with this disorder have apraxic
slowness, paratonic rigidity, frontal gait disorder, and elements of akinesia that, taken
together, may be mistaken for true parkinsonism. Pseudoparkinsonism appears to be
common and is most often due to Alzheimer’s disease or vascular dementia.
 Option A: Tardive dyskinesia is manifested by lip-smacking, wormlike
movement of the tongue. Tardive dyskinesia (TD) is a syndrome that includes a
group of iatrogenic movement disorders caused due to a blockade of dopamine
receptors. The movement disorders include akathisia, dystonia, buccolingual
stereotypy, myoclonus, chorea, tics, and other abnormal involuntary movements
which are commonly caused by the long-term use of typical antipsychotics.
 Option C: Akinesia is characterized by a feeling of weakness and muscle fatigue.
The term akinesia refers to the inability to perform a clinically perceivable
movement. It can present as a delayed response, freezing mid-action, or even total
abolition of movement. Akinesia occurs when movement is not perceived either
because the amplitude of the movement is small or because the time taken to
initiate the reaction is significantly increased.
 Option D: Dystonia is manifested by torticollis and rolling back of the eyes.
Dystonia is defined by involuntary maintained contraction of agonist and
antagonist muscles yielding abnormal posturing, twisting, and repetitive
movements or tremulous and can be initiated or worsened by attempted
movement.

 13. Question The client is very hostile toward one of the staff for no apparent reason.
The client is manifesting:

 A. Splitting
B. Transference
C. Countertransference
D. Resistance

Incorrect Correct Answer: B. Transference
Transference is a positive or negative feeling associated with a significant person in the
client’s past that is unconsciously assigned to another. Transference describes a situation
where the feelings, desires, and expectations of one person are redirected and applied to
another person. Most commonly, transference refers to a therapeutic setting, where a
person in therapy may apply certain feelings or emotions toward the therapist.
 Option A: Splitting is a defense mechanism commonly seen in a client with a
personality disorder in which the world is perceived as all good or all bad.
Splitting is a term used in psychiatry to describe the inability to hold opposing
thoughts, feelings, or beliefs. Some might say that a person who splits sees the
world in terms of black or white—all or nothing. It’s a distorted way of thinking
in which the positive or negative attributes of a person or event are neither
weighed nor cohesive.
 Option C: Countertransference is a phenomenon where the nurse shifts feelings
assigned to someone in her past to the patient. Countertransference, which occurs
when a therapist transfers emotions to a person in therapy, is often a reaction to
transference, a phenomenon in which the person in treatment redirects feelings for
others onto the therapist.
 Option D: Resistance is the client’s refusal to submit himself to the care of the
nurse. Resistance in psychology refers to any opposition to the therapeutic
process. Resistance is a way of pushing back against suggestions, even those that
could help you solve mental or emotional health concerns.

 14. Question Situation: An 18-year-old female is sexually attacked while on her way
home from work. She was brought to the hospital by her mother. Rape is an example of
which type of crisis:

 A. Situational
B. Adventitious
C. Developmental
D. Internal

Incorrect Correct Answer: B. Adventitious
Adventitious crisis is a crisis involving a traumatic event. It is not part of everyday life.
An adventitious crisis is a crisis of disaster that is not a part of everyday life. It is
unplanned or accidental. Adventitious crisis include natural disasters, national disasters,
and crimes of violence. Sexual molestation falls within this classification.
 Option A: Situational crisis is from an external source that upsets one’s
psychological equilibrium. Situational crisis arises from an external source such
as a job loss, divorce, or other loss affecting self-concept or self-esteem. These
occur as part of the process of growing and developing through various periods of
life. Sometimes a crisis is a predictable part of the life cycle, such as the crisis
described in Erikson’s stages of psychosocial development.
 Options C & D: These are the same. They are transitional or developmental
periods in life. A life crisis in which usual coping mechanisms are inadequate in
dealing with stress common to a particular stage in the life cycle or with stress
caused by a transition from one stage to another.

 15. Question During the initial care of rape victims, the following are to be considered
except:

 A. Assure privacy.
B. Touch the client to show acceptance and empathy.
C. Accompany the client to the examination room.
D. Maintain a non-judgmental approach.

Incorrect Correct Answer: B. Touch the client to show acceptance and empathy.
The client finds touch intrusive and therefore should be avoided. Establish trust and
rapport. Since the victim may misinterpret any statements unrelated to her immediate
situation as blaming or rejecting, the nurse should delay asking questions until the
therapeutic nature is established.
 Option A: Privacy is one of the rights of a victim of rape. Provide strict
confidentiality. The client’s situation is not to be talked over with anyone other
than medical staff involved unless the client gives consent to it.
 Option C: The client is anxious. Accompanying the client in a quiet room ensures
the safety and offers emotional support. Have someone stay with the client
(friend, neighbor, or staff member) while he or she waits to be treated. People
with high levels of anxiety need to feel physical safety by providing someone by
his/her side until the anxiety level is down to moderate.
 Option D: Guilt feeling is common among rape victims. They should not be
blamed. Stress that they did the right thing to save their life. Rape victims might
feel guilt and shame. Reinforcing that they did what they had to do to stay alive
can reduce guilt and maintain self-esteem.

 16. Question The nurse acts as a patient advocate when she does one of the following:

 A. She encourages the client to express her feelings regarding her experience.
B. She assesses the client for injuries.
C. She postpones the physical assessment until the client is calm.
D. Explains to the client that her reactions are normal.

Incorrect Correct Answer: C. She postpones the physical assessment until the client
is calm
The nurse acts as a patient advocate as she protects the client from psychological harm.
Nurse advocates support the patient’s best interests while respecting the family’s
important role. Advocates become facilitators when patients and family members need to
discuss uncomfortable information or explore its implications. Nurses may sometimes
need to advocate for patients against their families.
 Option A: The nurse acts as a counselor. A need for counseling is a clinical
judgment made by the nurse, and his/her response will be immediate situational
counseling, continuing counseling sessions, or referral. Types of counseling
situations that the nursing professional may encounter are outlined, including four
escapes utilized by students, such as substance abuse, suicide, fear of AIDS, and
anorexia/bulimia. Since students do utilize health services, the nurse counselor
with a holistic view of nursing care, which includes health education and
counseling, has an opportunity and a responsibility to act upon this knowledge.
 Option B: The nurse acts as a technician. Nurse technicians are medical care
providers who give basic medical care to patients. A nurse tech generally works
under the supervision of a Registered Nurse. Also known as nursing attendants or
nursing aides, they provide important services to help the registered nurses
complete their tasks.
 Option D: This exemplifies the role of a teacher. Nurse educators are registered
nurses with advanced education who are also teachers. Most work for several, if
not many, years before deciding to turn to a career teaching future nurses. Most
nurse educators have extensive clinical experience, and many continue caring for
patients after becoming educators.

 17. Question Crisis intervention carried out to the client has this primary goal:

 A. Assist the client to express her feelings.


B. Help her identify her resources.
C. Support her adaptive coping skills.
D. Help her return to her pre-rape level of function.

Correct Correct Answer: D. Help her return to her pre-rape level of function.
The goal of crisis intervention is to help the client return to her level of function prior to
the crisis. Crisis intervention is a short-term management technique designed to reduce
potential permanent damage to an individual affected by a crisis. A crisis is defined as an
overwhelming event, which can include divorce, violence, the passing of a loved one, or
the discovery of a serious illness.
 Option A: A successful intervention involves obtaining background information
on the patient, establishing a positive relationship, discussing the events, and
providing emotional support. SAFER-R is a common intervention model used,
which consists of stabilization, acknowledgment, facilitate understanding,
encouragement, recovery, and referral. SAFER-R helps patients return to their
mental baseline following a crisis.
 Option B: Based on prior studies, it is evident that crisis intervention plays a
significant role in enhancing outcomes in psychiatric cases. Community Mental
Health Centers and local government agencies often have crisis intervention
teams that provide support to the local community at times of mental health crisis.
 Option C: Another major concern is what coping strategies are most effective.
Social support and problem-solving planning are effective coping mechanisms
that are frequently used by school staff following a crisis. The use of humor,
emotional support, planning, and acceptance also correlate with superior mental
health outcomes compared to substance abuse and denial.

 18. Question Five months after the incident the client complains of difficulty to
concentrate, poor appetite, inability to sleep and guilt. She is likely suffering from:

 A. Adjustment disorder
B. Somatoform Disorder
C. Generalized Anxiety Disorder
D. Post-traumatic disorder

Correct Correct Answer: D. Post-traumatic disorder
Post-traumatic stress disorder is characterized by flashbacks, irritability, difficulty falling
asleep, and concentrating following an extremely traumatic event. This lasts for more
than one month. Posttraumatic stress disorder (PTSD) is a syndrome that results from
exposure to real or threatened death, serious injury, or sexual assault. Following the
traumatic event, PTSD is common and is one of the serious health concerns that is
associated with comorbidity, functional impairment, and increased mortality with suicidal
ideations and attempts.
 Option A: Adjustment disorder is the maladaptive reaction to stressful events
characterized by anxiety, depression, and work or social impairments. This occurs
within three (3) months after the event. Adjustment disorders involve markedly
distressing and impairing emotional and/or behavioral symptoms caused by an
identifiable stressor.
 Option B: Somatoform disorders are anxiety-related disorders characterized by
the presence of physical symptoms without a demonstrable organic basis. The
somatoform disorders are a group of psychiatric disorders in which patients
present with a myriad of clinically significant but unexplained physical
symptoms. They include somatization disorder, undifferentiated somatoform
disorder, hypochondriasis, conversion disorder, pain disorder, body dysmorphic
disorder, and somatoform disorder not otherwise specified.
 Option C: Generalized anxiety disorder is characterized by chronic, excessive
anxiety for at least 6 months. Generalized anxiety disorder is one of the most
common mental disorders. Up to 20% of adults are affected by anxiety disorders
each year. Generalized anxiety disorder produces fear, worry, and a constant
feeling of being overwhelmed.

 19. Question Situation: A 29-year-old client newly diagnosed with breast cancer is
pacing, with rapid speech headache and inability to focus on what the doctor was saying.
The nurse assesses the level of anxiety as:

 A. Mild
B. Moderate
C. Severe
D. Panic

Incorrect Correct Answer: C. Severe
The client’s manifestations indicate severe anxiety. Severe anxiety is intensely
debilitating, and symptoms of severe anxiety meet key diagnostic criteria for clinically-
significant anxiety disorders. People with severe anxiety typically score higher on scales
of distress and lower on functioning. Severe anxiety symptoms also frequently co-occur
with major depression, which can contribute to greater disability.
 Option A: Mild anxiety is manifested by slight muscle tension, slight fidgeting,
alertness, the ability to concentrate, and capable of problem-solving. Although
often described as sub-clinical or clinically non-significant, mild anxiety can
impact emotional, social, and professional functioning. Mild anxiety symptoms
may present as social anxiety or shyness and can be experienced in early
childhood through to adulthood. If left unaddressed, mild anxiety can lead to
maladaptive coping strategies or more severe mental conditions.
 Option B: Moderate muscle tension, increased vital signs, periodic slow pacing,
increased rate of speech, and difficulty in concentrating are noted in moderate
anxiety. People with moderate levels of anxiety have more frequent or persistent
symptoms than those with mild anxiety, but still have better daily functioning
than someone with severe anxiety or panic disorder. For example, people with
moderate anxiety may report experiencing symptoms such as feeling on edge,
being unable to control their worrying, or being unable to relax several days or the
majority of days in a week, but not every day.
 Option D: Panic anxiety is characterized by immobilization, incoherence, a
feeling of being overwhelmed, and disorganization. Panic level anxiety, or panic
disorder, is characterized by frequent, recurring, and unexpected panic attacks.
Panic attacks usually last around 10 minutes. The triggers for panic attacks vary
from person to person, and the cause of an attack may be familiar to a person or
unknown.

 20. Question Anxiety is caused by:

 A. An objective threat.
B. A subjectively perceived threat.
C. Hostility turned to the self.
D. Masked depression.

Incorrect Correct Answer: B. A subjectively perceived threat.
Anxiety is caused by a subjectively perceived threat. Anxiety is one of the most common
mental disorders, with 19.1% of adults in the U.S. being affected in the past year. Anxiety
can begin early in life, with an average age of onset of 11 years old, and it may range
from mildly uncomfortable symptoms to severe and debilitating panic that can interfere
with a person’s ability to live normally.
 Option A: Fear is caused by an objective threat. Sometimes fear stems from real
threats, but it can also originate from imagined dangers. Fear can also be a
symptom of some mental health conditions including panic disorder, social
anxiety disorder, phobias, and post-traumatic stress disorder (PTSD).
 Option C: A depressed client internalizes hostility. The common features of all
the depressive disorders are sadness, emptiness, or irritable mood, accompanied
by somatic and cognitive changes that significantly affect the individual’s
capacity to function.
 Option D: Mania is due to masked depression. Some other hallmarks of mania
are an elevated or expansive mood, mood lability, impulsivity, irritability, and
grandiosity. If the individual experiencing these symptoms requires
hospitalization, then this period automatically qualifies as true mania and not
hypomania, even if the symptoms are present for less than one week.

 21. Question It would be most helpful for the nurse to deal with a client with severe
anxiety by:

 A. Give specific instructions using speak in concise statements.


B. Ask the client to identify the cause of her anxiety.
C. Explain in detail the plan of care developed.
D. Urge the client to focus on what the nurse is saying.

Incorrect Correct Answer: A. Give specific instructions using speak in concise
statements.
The client has narrowed the perceptual field. Lengthy explanations cannot be followed by
the client. Maintain a calm, non-threatening manner while working with the client.
Anxiety is contagious and may be transferred from health care provider to client or vice
versa. The client develops a feeling of security in presence of a calm staff person.
 Option B: The client will not be able to identify the cause of anxiety. Establish
and maintain a trusting relationship by listening to the client; displaying warmth,
answering questions directly, offering unconditional acceptance; being available
and respecting the client’s use of personal space.
 Option C: Move the client to a quiet area with minimal stimuli such as a small
room or seclusion area (dim lighting, few people, and so on.) Anxious behavior
escalates by external stimuli. A smaller or secluded area enhances a sense of
security as compared to a large area which can make the client feel lost and
panicked.
 Option D: The client has difficulty concentrating and will not be able to focus.
Remain with the client at all times when levels of anxiety are high (severe or
panic); reassure the client of his or her safety and security. The client’s safety is
utmost priority. A highly anxious client should not be left alone as his anxiety will
escalate.

 22. Question Which of the following medications will likely be ordered for the client?”

 A. Prozac
B. Valium
C. Risperdal
D. Lithium

Incorrect Correct Answer: B. Valium
Diazepam is an anxiolytic benzodiazepine, first patented and marketed in the United
States in 1963. It is a fast-acting, long-lasting benzodiazepine commonly used in the
treatment of anxiety disorders, as well as alcohol detoxification, acute recurrent seizures,
severe muscle spasm, and spasticity associated with neurologic disorders. In the setting of
acute alcohol withdrawal, diazepam is useful for symptomatic relief of agitation, tremor,
alcoholic hallucinosis, and acute delirium tremens.
 Option A: Fluoxetine has FDA-approval for major depressive disorder (age eight
and older), obsessive-compulsive disorder (age seven and older), panic disorder,
bulimia, binge eating disorder, premenstrual dysphoric disorder, bipolar
depression (as an adjunct with olanzapine also known as Symbyax), and
treatment-resistant depression when used in combination with olanzapine. Non-
FDA-approved uses for fluoxetine include social anxiety disorder (social phobia),
post-traumatic stress disorder in adults, borderline personality disorder, Raynaud
phenomenon, and selective mutism.
 Option C: In addition to psychotic symptoms, risperidone can be used for
aggression and agitation in patients with dementia. Risperidone has also been
used for augmentation of antidepressant therapy in the treatment of non-psychotic
unipolar depression. In addition to irritability associated with autism, risperidone
has also been used for social impairment, stereotypical behaviors, cognitive
problems, and hyperactivity in autism.
 Option D: Lithium was the first mood stabilizer and is still the first-line treatment
option, but is underutilized because it is an older drug. Lithium is a commonly
prescribed drug for a manic episode in bipolar disorder as well as maintenance
therapy of bipolar disorder in a patient with a history of a manic episode. The
primary target symptoms of lithium are mania and unstable mood.

 23. Question Which of the following is included in the health teachings among clients
receiving Valium?

 A. Avoid foods rich in tyramine.


B. Take the medication after meals.
C. It is safe to stop it anytime after long term use.
D. Double up the dose if the client forgets her medication.

Incorrect Correct Answer: B. Take the medication after meals.
Anti Anxiety medications cause G.I. upset so it should be taken after meals.
Benzodiazepines are a class of medicines approved to treat generalized anxiety disorder,
insomnia, seizures, social phobia, and panic disorder.
 Option A: This is specific for antidepressant MAOI. Taking tyramine-rich food
can cause a hypertensive crisis. MAOIs prevent the breakdown of tyramine found
in the body as well as certain foods, drinks, and other medications. Patients that
take MAOIs and consume tyramine-containing foods or drinks will exhibit high
serum tyramine level. A high level of tyramine can cause a sudden increase in
blood pressure, called the tyramine pressor response.
 Option C: Valium causes dependency. In which case, the medication should be
gradually withdrawn to prevent the occurrence of convulsion. However, the
potential of overdose from diverted diazepam always exists when combined with
opioids, alcohol, or other centrally acting agents. Overdose in adults frequently
involves the co-ingestion of other CNS depressants, which work synergistically to
increase toxicity. In the case of single-agent diazepam overdose, symptoms
manifest as CNS depression and are very rarely fatal. In mild cases, lethargy,
drowsiness, and confusion are common symptoms.
 Option D: The dose of Valium should not be doubled if the previous dose was
not taken. It can intensify the CNS depressant effects. In cases of severe overdose,
symptoms manifest as ataxia, diminished reflexes, hypotonia, hypotension,
respiratory depression, coma (rarely), and death (very rarely).

 24. Question Mrs. B is diagnosed with borderline personality disorder and has a nursing
diagnosis of Risk for self-directed violence, which is related to the client’s self-mutilation
behavior (burning arms with cigarettes). Which client behavior would indicate a positive
outcome of intervention?

 A. Mrs. B denies feelings of wanting to harm anyone.


B. Mrs. B expresses feelings of anger towards others.
C. Mrs. B requests cigarettes at appropriate times.
D. Mrs. B tells the nurse about wanting to burn herself.

Correct Correct Answer: D. Mrs. B tells the nurse about wanting to burn herself.
The fact that Mrs. B directly tells the nurse about wanting to self-mutilate, rather than
acting on these feelings, is evidence of her responding to nursing intervention. Use a
matter-of-fact approach when self-mutilation occurs. Avoid criticizing or giving
sympathy. A neutral approach prevents blaming, which increases anxiety, giving special
attention that encourages acting out.
 Option A: Identify feelings experienced before and around the act of self-
mutilation. Feelings are a guideline for future intervention (e.g., rage at feeling
left out or abandoned). Explore with the client what these feelings might mean.
 Option B: Set and maintain limits on acceptable behavior and make clear client’s
responsibilities. If the client is hospitalized at the time, be clear regarding the unit
rules. Clear and non-punitive limit setting is essential for decreasing negative
behaviors.
 Option C: Be consistent in maintaining and enforcing the limits, using a non-
punitive approach. Consistency can establish a sense of security.

 25. Question Nurse Aldrich is working with the family of Mary Ann, a client with a
personality disorder. Which of the following should Nurse Aldrich encourage the family
members to work on?

 A. Avoiding direct expressions of problems with family.


B. Changing Mary Ann's problem behaviors.
C. Improving self-functioning.
D. Supporting Mary Ann's defenses.

Incorrect Correct Answer: C. Improving self-functioning.
Family members typically benefit from working on ways to improve self-functioning.
This facilitates ownership of problems among individuals involved in ongoing
relationship difficulties. Keep goals very realistic and go in small steps. There are no
overnight successes with people with personality disorders.
 Option A: The direct expression of problems is helpful and therefore should not
be avoided. When the client is ready and interested, teach client coping skills to
help defuse tension and trouble feelings (e.g., anxiety reduction, assertiveness
skills).
 Option B: It would be impossible to change the client’s behavior; encouraging
family members to do so would frustrate them. Give the client positive attention
when behaviors are appropriate and productive. Avoid giving any attention (when
possible and not dangerous to self or others) when client’s behaviors are
inappropriate.
 Option D: The client’s defenses are likely to blame others for problems;
consequently, supporting his blaming others is not helpful. Use assertiveness
when setting limits on client’s unreasonable demands for attention and time.

 ABOUT
 PRIVACY
 DISCLAIMER
 CONTACT
© 2024 Nurseslabs | Ut in Omnibus Glorificetur Deus!

Gandang Swak sa Budget


Shopee

Face Brush Mask Double ended Blackhead remover


Exfoliator Mask tool Hypoallergenic Gentle Silicone
₱49
Shop now

 1. Question 1 point(s)

Which of the following behaviors by a client with dependent personality disorder shows
the client has made progress toward the goal of increasing problem solving skills?

 A. The client is courteous.


B. The client asks questions.
C. The client stops acting out.
D. The client controls emotions.

Incorrect Correct Answer: B. The client asks questions.
The client with a dependent personality disorder is passive and tries to please others. By
asking questions, the client is beginning to gather information, the first step of decision
making. Keep goals very realistic and go in small steps. There are no overnight successes
with people with personality disorders. It can take a long time to positively change
ingrained, life-long, maladaptive habits; however, change is always possible.
 Option A: Give the client positive attention when behaviors are appropriate and
productive. Avoid giving any attention (when possible and not dangerous to self
or others) when client’s behaviors are inappropriate. Reinforcing positive
behaviors might increase the likelihood of repetition. Ignoring negative behaviors
(when feasible) robs the client of even negative attention.
 Option C: The client with DPD does not act out. The nurse must be quite clear
about establishing the boundaries of the therapeutic relationship to ensure that
neither the client’s nor the nurse’s boundaries are violated.
 Option D: The nurse can help the clients to identify their feelings and learn to
tolerate them without exaggerated responses such as destruction of property or
self-harm; keeping a journal often helps clients gain awareness of feelings.

 2. Question 1 point(s)

A client with schizotypal personality disorder is sitting in a puddle of urine. She’s playing
in it, smiling, and softly singing a child’s song. Which action would be best?

 A. Admonish the client for not using the bathroom.


B. Firmly tell the client that her behavior is unacceptable.
C. Ask the client if she’s ready to get cleaned up now.
D. Help the client to the shower, and change the bedclothes.

Correct Correct Answer: D. Help the client to the shower, and change the
bedclothes.
A client with schizotypal personality disorder can experience high levels of anxiety and
regress to childlike behaviors. This client may require help needing self-care needs. The
client may not respond to the other options or those options may generate more anxiety.
 Option A: Approach the client in a consistent manner in all interactions.
Enhances feelings of security and provides structure. Exceptions encourage
manipulative behavior. When the client is ready and interested, teach client
coping skills to help defuse tension and trouble feelings (e.g., anxiety reduction,
assertiveness skills).
 Option B: Be clear with the client as to the unit/hospital/clinic policies. Give
brief concrete reasons for the rules, if asked, and then move on. Institutional
policies provide structure and safety.
 Option C: Give the client positive attention when behaviors are appropriate and
productive. Avoid giving any attention (when possible and not dangerous to self
or others) when client’s behaviors are inappropriate. Reinforcing positive
behaviors might increase the likelihood of repetition. Ignoring negative behaviors
(when feasible) robs the client of even negative attention.

 3. Question 1 point(s)

A client with avoidant personality disorder says occupational therapy is boring and
doesn’t want to go. Which action would be best?

 A. State firmly that you’ll escort him to OT.


B. Arrange with OT for the client to do a project on the unit.
C. Ask the client to talk about why OT is boring.
D. Arrange for the client not to attend OT until he is feeling better.

Incorrect Correct Answer: A. State firmly that you’ll escort him to OT.
If given the chance, a client with avoidant personality disorder typically elects to remain
immobilized. The nurse should insist that the client participates in OT. Expand limits by
clarifying expectations for clients in a number of settings. When time is taken in initial
meetings to clarify expectations, confrontations, and power struggles with clients can be
minimized and even avoided.
 Option B: In a respectful, neutral manner, explain expected client behaviors,
limits, and responsibilities during sessions with nurse clinician. Clearly state the
rules and regulations of the institution, and the consequences when these rules are
not adhered to. From the beginning, clients need to have explicit guidelines and
boundaries for expected behaviors on their part, as well as what the client can
expect from the nurse. Clients need to be fully aware that they will be held
responsible for their behaviors.
 Option C: Addressing an invalid issue such as the client’s perceived boredom
avoids the real issue: the client’s need for therapy. Understand that PD clients, in
particular, will be resistant to change and that this is symptomatic of PDs. This is
particularly true in the beginning phases of therapy.
 Option D: Arranging for the client to do a project on the unit validates and
reinforces the client’s desire to avoid getting to OT. Responding to client’s
resistance and seeming lack of change in a neutral manner is part of the
foundation for trust. In other words, the nurse does not have a vested interest in
the client “getting better.”. The nurse remains focused on the client’s needs and
issues in any event.

 4. Question 1 point(s)

A nurse discusses job possibilities with a client with schizoid personality disorder. Which
suggestion by the nurse would be helpful?

 A. “You can work in a family restaurant part-time on the weekend and holidays.”
B. “Maybe your friend could get you that customer service job where you work
only on the weekends.”
C. “Your idea of applying for the position of filing and organizing records is
worth pursuing.”
D. “Being an introvert limits the employment opportunities you can pursue.”

Incorrect Correct Answer: C. “Your idea of applying for the position of filing and
organizing records is worth pursuing.”
Clients with schizoid personality disorder prefer solitary activities, such as filing, to
working with others. Working as a cashier or in customer service would involve
interacting with many people. They’re often described as eccentric or bizarre. They may
be suspicious and paranoid of others. They come across as “stiff” and don’t seem to fit in
anywhere they go.
 Option A: Individuals with schizotypal personality disorders experience extreme
discomfort during interpersonal interactions. Unlike in social anxiety disorder,
where an individual is likely to grow more comfortable with time, individuals
with schizotypal personality disorder remain uncomfortable even when they’re
interacting with the same people in the same environment over and over again.
 Option B: The disorder also involves distorted thinking and eccentric behavior—
which tends to push people away and create even more isolation. Sometimes,
individuals with schizotypal personality disorder are superstitious or preoccupied
with paranormal phenomena that are outside what would be expected in their
culture.
 Option D: They might also appear constricted and show little emotion during
their interactions. They may have unusual mannerisms, such as an unkempt
manner of dress. They may occasionally express sadness over their lack of close
relationships but their behavior suggests they have little desire for close
connections. They often interact with people when they have to but prefer to keep
to themselves.

 5. Question 1 point(s)

When assessing a client diagnosed with impulse control disorder, the nurse observes
violent, aggressive, and assaultive behavior. Which of the following assessment data is
the nurse also likely to find? Select all that apply.

 A. The client functions well in other areas of his life.


B. The degree of aggressiveness is out of proportion to the stressor.
C. The violent behavior is most often justified by the stressor.
D. The client has a history of parental alcoholism and chaotic, abusive family
life.

Incorrect Correct Answers: A, B, & D
A client with an impulse control disorder who displays violent, aggressive, and assaultive
behavior generally functions well in other areas of his life. An impulse control disorder is
a condition in which a person has trouble controlling emotions or behaviors. Often, the
behaviors violate the rights of others or conflict with societal norms and the law.
 Option A: This disorder, also known as IED, is characterized by persistent
impulsive and angry outbursts. The person may be violent or aggressive toward
people, animals, or property. These tirades may only last about a half hour and are
usually triggered by a minor issue with someone the person knows. The outbursts
may result in legal or financial issues, disrupt a person’s interpersonal
relationships, cause significant distress, and potentially result in problems at work
or school.
 Option B: A person with oppositional defiant disorder (ODD) frequently loses
their temper, is easily annoyed, and often becomes angry or resentful. They
challenge authority figures, flout rules, bother other people on purpose, and blame
other people for their problems. As a result of these behaviors, the person suffers
problems at work and school, and socially. Symptoms may appear as early as
preschool. The prevalence rate is estimated to be around 3.3%.
 Option C: The degree of aggressiveness is typically out of proportion with the
stressor. Conduct disorder consists of persistent behavior that violates social rules.
The person may be aggressive toward people or animals, destroy other people’s
property, lie or steal, or violate rules, such as run away or skip school at an early
age. The behavior causes serious problems at school or socially. People are
usually not diagnosed with this disorder over the age of 18. Adults with these
symptoms are diagnosed with antisocial personality disorder. The prevalence rate
is estimated to be about 4%.
 Option D: About 19.7 million people in the United States aged 12 and older
battled a substance use disorder in 2017, the National Survey on Drug Use and
Health (NSDUH) reports, and about 8.5 million Americans age 18 and older
suffered from co-occurring substance abuse and mental health disorders in 2017.
Impulse control disorders and substance use disorders frequently co-occur.
 Option E: Such a client commonly has a history of parental alcoholism and a
chaotic family life, and often verbalizes sincere remorse and guilt for the
aggressive behavior. Feelings such as guilt or shame may follow, and repeated
impulsive acts may lead to a number of negative consequences, such as greater
emotional distress or regret, in the long term.

 6. Question 1 point(s)

Which is the best indicator of success in the long-term management of the client with a
somatic disorder?

 A. His symptoms are replaced by indifference to his feelings.


B. He participates in diversionary activities.
C. He learns to verbalize his feelings and concerns.
D. He states that his behavior is irrational.

Incorrect Correct Answer: C. He learns to verbalize his feelings and concerns.
The client is encouraged to talk about his feelings and concerns instead of using body
symptoms to manage his stressors. Teach the client to reframe and dispute cognitive
distortions. Disputes need to be strong, specific, and nonjudgmental. Practice and belief
in the disputes over time help clients gain a more realistic appraisal of events, the world,
and themselves.
 Option A: The client is encouraged to acknowledge feelings rather than being
indifferent to her feelings. Work with the client to recognize cognitive distortions.
Encourage the client to keep a log. Cognitive distortions are automatic. Keeping a
log helps make automatic, unconscious thinking clear.
 Option B: Participation in activities diverts the client’s attention away from his
bodily concerns but this is not the best indicator of success. Problem solve and
role play with client acceptable social skills that will help obtain needs effectively
and appropriately.
 Option D: Help the client recognize that his physical symptoms occur because of
or are exacerbated by specific stressor, not as irrational. Give the client honest and
genuine feedback regarding your observations as to his or her strengths, and areas
that could use additional skills. Feedback helps give clients a more accurate view
of self, strengths, areas to work on, as well as a sense that someone is trying to
understand them.

 7. Question 1 point(s)

Situation: A young woman is brought to the emergency room appearing depressed. The
nurse learned that her child died a year ago due to an accident. The initial nursing
diagnosis is dysfunctional grieving. The statement of the woman that supports this
diagnosis is:

 A. “I feel envious of mothers who have toddlers”


B. “I haven’t been able to open the door and go into my baby’s room “
C. “I watch other toddlers and think about their play activities and I cry.”
D. “I often find myself thinking of how I could have prevented the death."

Incorrect Correct Answer: B. “I haven’t been able to open the door and go into my
baby’s room.”
This indicates denial. This defense is adaptive as an initial reaction to loss but extended,
unsuccessful use of denial is dysfunctional. Allow the patient to recognize and express
feelings and determine the connection between the feelings and the event. Expressing
feelings in a non-threatening environment can aid patients in handling unresolved issues
that may be partly responsible for the depression. It can also aid the patient to relate the
feeling to the event.
 Option A: This indicates acknowledgment of the loss. Expressing feelings openly
is acceptable. Suggest alternative methods to determine and cope with underlying
feelings of anger, hurt, and rejection. Allows individuals to explore more ways of
handling such situations.
 Option C: This indicates the stage of depression in the grieving process. Discuss
and educate the patient on the normal stages of grief and accept the reality of
related feelings such as guilt, anger, and powerlessness. This allows the patient to
acknowledge these normal feelings and remove some of the guilt caused by these
feelings.
 Option D: Remembering both positive and negative aspects of the deceased love
one signals successful mourning. Assist the patient in recognizing early signs of
depression and identify methods to mitigate these signs. If the symptoms persist
or worsen, suggest other professional support.

 8. Question 1 point(s)

The client said “I can’t even take care of my baby. I’m good for nothing.” Which is the
appropriate nursing diagnosis?

 A. Ineffective individual coping related to loss.


B. Impaired verbal communication related to inadequate social skills.
C. Low esteem related to failure in role performance.
D. Impaired social interaction related to repressed anger.

Incorrect Correct Answer: C. Low esteem related to failure to role performance.
This indicates the client’s negative self-evaluation. A sense of worthlessness may
accompany depression. Allow the patient to engage in simple recreational activities,
advancing to more complex activities in a group environment. The patient may feel
overwhelmed at the start when participating in a group setting.
 Option A: Ask the patient about the losses that happen in his or her life. Discuss
how the patient views them. Individuals tend to not recognize the significance of a
loss. They fail to accept or talk about their pain and seem that all is well. Denial
necessitates physical and psychic energy. When people become depressed, they
likely do so in a physically and emotionally depleted state.
 Option B: Assist the patient to determine the problem, recognize the need to
address the problem differently, and thoroughly describe all facets of the problem.
Before individuals can admit changing, they need clearness about what the
problem is.
 Option D: The cues do not indicate the inability to use coping resources,
decreased ability to transmit/process symbols, nor the insufficient quality of social
exchange. Assist the patient in recognizing early signs of depression and identify
methods to mitigate these signs. If the symptoms persist or worsen, suggest other
professional support.

 9. Question 1 point(s)

The following medications will likely be prescribed for the client except:

 A. Prozac
B. Tofranil
C. Parnate
D. Zyprexa

Correct Correct Answer: D. Zyprexa
This is an antipsychotic. Olanzapine is a second-generation (atypical) antipsychotic
medication. Olanzapine also has approval for use with fluoxetine, a selective serotonin
reuptake inhibitor (SSRI), in patients with episodes of depression associated with bipolar
disorder type 1 and treatment-resistant depression. It is important to note that olanzapine
is not FDA approved for patients under the age of 13, and the combination of olanzapine
with fluoxetine does not have approval for patients under the age of 10.
 Option A: Prozac is a SSRI antidepressant. Fluoxetine has FDA-approval for
major depressive disorder (age eight and older), obsessive-compulsive disorder
(age seven and older), panic disorder, bulimia, binge eating disorder, premenstrual
dysphoric disorder, bipolar depression (as an adjunct with olanzapine also known
as Symbyax), and treatment-resistant depression when used in combination with
olanzapine.
 Option B: Tofranil antidepressant belongs to the Tricyclic group. Imipramine is a
tertiary amine tricyclic antidepressant. Tricyclic antidepressants (TCAs) had been
approved by the Food and Drug Administration (FDA) as antidepressants in the
1950s. Although it is FDA approved for the treatment of depression, it is a
second-line treatment notably in severe depression with melancholic and atypical
features, due to its undesirable side effects and due to its toxicity in overdose.
 Option C: Parnate is a MAOI antidepressant. The main FDA-approved indication
of tranylcypromine is for major depressive disorder without melancholia. The
non-FDA-approved indications for this medication include treatment-resistant
depression, treatment-resistant social anxiety disorder, treatment-resistant panic
disorder, and atypical depression. Atypical depression consists of hyperphagia,
hypersomnia, rejection sensitivity, and leaden paralysis accompanying the
depression.

 10. Question 1 point(s)

Which is the highest priority in the post-ECT care?

 A. Observe for confusion.


B. Monitor respiratory status.
C. Reorient to time, place and person.
D. Document the client’s response to the treatment.

Incorrect Correct Answer: B. Monitor respiratory status.
A side effect of ECT which is life-threatening is a respiratory arrest. The anesthesiologist
who manages the airway during this period is careful to protect the patient from
aspiration while maintaining ventilation and oxygenation. Sometimes a soft preshaped
airway device (oral airway) is placed to facilitate this. In most patients, the return of
spontaneous respiration occurs within 3 to 5 minutes after induction, before the effects of
the anesthetic have worn off and the patient is awake.
 Option A: Clinically, patients may exhibit confusion, delirium, disorientation,
and memory loss. ECT is classified as a low-risk procedure by the AHA-ACC
guidelines because it is well-tolerated, and demonstrates only transient
hemodynamic lability and low mortality rate.
 Option C: Confusion and disorientation are side effects of ECT but these are not
the highest priority. The stigma associated with ECT is largely due to the lack of
anesthesia with early treatments resulting in significant injury and severe memory
loss. The antidepressant effect is seen relatively quickly and may last up to a few
years. Overall, the mortality rate is very low with ECT administered in a
controlled setting, but it continues to cause mild memory loss in the long term.
 Option D: The procedure is relatively safe, and does work. However, the delivery
of ECT requires an interprofessional team that includes a nurse, anesthesiologist,
psychiatrist, and neurologist. The benefits of ECT are seen after several sessions
and the results are durable. The key is to educate the patient and family about
ECT because the procedure has been associated with many false and illogical
beliefs.

 11. Question 1 point(s)

Situation: A 27-year-old writer is admitted for the second time accompanied by his wife.
He is demanding, arrogant, talks fast, and hyperactive. Initially, the nurse should plan this
for a manic client:

 A. Set realistic limits to the client’s behavior.


B. Repeat verbal instructions as often as needed.
C. Allow the client to get out feelings to relieve tension.
D. Assign a staff to be with the client at all times to help maintain control.

Incorrect Correct Answer: A. Set realistic limits to the client’s behavior
The manic client is hyperactive and may engage in injurious activities. A quiet
environment and consistent and firm limits should be set to ensure safety. Maintain a
consistent approach, employ consistent expectations, and provide a structured
environment. Clear and consistent limits and expectations minimize the potential for the
client’s manipulation of staff.
 Option B: Clear, concise directions are given because of the distractibility of the
client but this is not the priority. Use short, simple, and brief explanations or
statements. A short attention span limits understanding to small pieces of
information.
 Option C: The manic client tends to externalize hostile feelings, however only
non-destructive methods of expression should be allowed. Redirect agitation and
potentially violent behaviors with physical outlets in an area of low stimulation
(e.g., punching bag). Can help to relieve pent-up hostility and relieve muscle
tension.
 Option D: Nurses set limits as needed. Assigning staff to be with the client at all
times is not realistic. Decrease environmental stimuli (e.g., by providing a
calming environment or assigning a private room). Helps decrease the escalation
of anxiety and manic symptoms.

 12. Question 1 point(s)

An activity appropriate for the client with mania is:

 A. Table tennis
B. Painting
C. Chess
D. Cleaning

Correct Correct Answer: D. Cleaning
The client’s excess energy can be rechanneled through physical activities that are not
competitive like cleaning. This is also a way to dissipate tension. Maintain a low level of
stimuli in the client’s environment (e.g., loud noises, bright light, low-temperature
ventilation). Redirect violent behavior. Physical exercise can decrease tension and
provide focus.
 Option A: Tennis is a competitive activity which can stimulate the client. When
possible, provide an environment with minimum stimuli (e.g., quiet, soft music,
dim lighting). Reduction in stimuli lessens distractibility.
 Option B: When less manic, the client might join one or two other clients in
quiet, non-stimulating activities (e.g., drawing, board games, cards). As mania
subsides, involvement in activities that provide a focus and social contact
becomes more appropriate. Competitive games can stimulate aggression and can
increase psychomotor activity.
 Option C: Solitary activities requiring short attention spans with mild physical
exertion are best initially (e.g., writing, taking photos, painting, or walks with
staff). Solitary activities minimize stimuli; mild physical activities release tension
constructively.

 13. Question 1 point(s)

The client is arrogant and manipulative. In ensuring a therapeutic milieu, the nurse does
one of the following:

 A. Agree on a consistent approach among the staff assigned to the client.


B. Suggest that the client take a leading role in the social activities.
C. Provide the client with extra time for one on one sessions.
D. Allow the client to negotiate the plan of care.

Incorrect Correct Answer: A. Agree on a consistent approach among the staff
assigned to the client.
A consistent firm approach is appropriate. This is a therapeutic way to handle attempts of
exploiting the weakness in others or create conflicts among the staff. Bargaining should
not be allowed. Maintain a consistent approach, employ consistent expectations, and
provide a structured environment. Clear and consistent limits and expectations minimize
the potential for the client’s manipulation of staff.
 Option B: This is not therapeutic because the client tends to control and dominate
others. Use a calm and firm approach. Provides structure and control for a client
who is out of control.
 Option C: Limits are set for interaction time. Alert staff if a potential for
seclusion appears imminent. Usual priority of interventions would be firmly
setting limits. If nursing interventions (quiet environment and firm limit setting)
and chemical restraints (tranquilizers–e.g., haloperidol [Haldol]) have not helped
dampen escalating manic behaviors, then seclusion might be warranted.
 Option D: Allowing the client to negotiate, may reinforce manipulative behavior.
Remain neutral as possible; Do not argue with the client. The client can use
inconsistencies and value judgments as justification for arguing and escalating
mania.

 14. Question 1 point(s)

A client on Lithium has diarrhea and vomiting. What should the nurse do first:

 A. Recognize this as a drug interaction.


B. Give the client Cogentin.
C. Reassure the client that these are common side effects of lithium therapy.
D. Hold the next dose and obtain an order for a stat serum lithium level.

Correct Correct Answer: D. Hold the next dose and obtain an order for a stat serum
lithium level.
Diarrhea and vomiting are manifestations of Lithium toxicity. The next dose of lithium
should be withheld, and a test is done to validate the observation. All patients with
toxicity signs and symptoms, even those with normal serum lithium levels, should be
admitted for monitoring in the hospital. In case of moderate or severe symptoms, the
patient has to be admitted to an intensive care unit.
 Option A: The manifestations are not due to drug interaction. Lithium toxicity
signs are obvious and can be identified and managed easily; however, ignoring it
can be fatal. Indeed, in some cases, lithium toxicity can lead to coma, brain
damage, or even death. Moreover, lithium can induce serotonin syndrome, a
potentially fatal and life-threatening condition.
 Option B: Cogentin is used to manage the extrapyramidal symptom side effects
of antipsychotics. It is useful for drug-induced extrapyramidal symptoms and the
prevention of dystonic reactions and acute treatment of dystonic reactions.
Furthermore, benztropine has further off-label use as it can treat chronic
sialorrhea occurring in developmentally-disabled patients.
 Option C: The common side effects of Lithium are fine hand tremors, nausea,
polyuria, and polydipsia. Confusion, memory problems, new or worsening tremor,
hyperreflexia, clonus, slurred speech, ataxia, stupor, delirium, coma, and seizures
(rarely). These effects are theoretically due to excess action on the same sites that
mediate therapeutic action.

 15. Question 1 point(s)

Situation: A widow age 28, whose husband died one (1) year ago due to AIDS, has just
been told that she has AIDS. Panky says to the nurse, “Why me? How could God do this
to me?” This reaction is one of:

 A. Depression
B. Denial
C. Anger
D. Bargaining

Incorrect Correct Answer: C. Anger
Anger is experienced as reality sets in. This may either be directed to God, the deceased,
or displaced on others. It is common to experience anger after the loss of a loved one. We
are trying to adjust to a new reality and we are likely experiencing extreme emotional
discomfort. There is so much to process that anger may feel like it allows us an emotional
outlet.
 Option A: Depression is a painful stage where the individual mourns for what
was lost. We start to feel the loss of our loved one more abundantly. As our panic
begins to subside, the emotional fog begins to clear and the loss feels more
present and unavoidable.
 Option B: Denial is the first stage of the grieving process evidenced by the
statement “No, it can’t be true.” The individual does not acknowledge that the
loss has occurred to protect self from the psychological pain of the loss. The first
stage in this theory, denial helps us minimize the overwhelming pain of loss. As
we process the reality of our loss, we are also trying to survive emotional pain. It
can be hard to believe we have lost an important person in our lives, especially
when we may have just spoken with this person the previous week or even the
previous day.
 Option D: In bargaining the individual holds out hope for additional alternatives
to forestall the loss, evidenced by the statement “If only…” When coping with
loss, it isn’t unusual to feel so desperate that you are willing to do almost anything
to alleviate or minimize the pain. Losing a loved one can cause us to consider any
way we can avoid the current pain or the pain we are anticipating from loss. There
are many ways we may try to bargain.

 16. Question 1 point(s)

Situation: A widow age 28, whose husband died one (1) year ago due to AIDS, has just
been told that she has AIDS. Panky says to the nurse, “Why me? How could God do this
to me?” The nurse’s therapeutic response is:

 A. “I will refer you to a clergy who can help you understand what is happening to
you.”
B. “ It isn’t fair that an innocent like you will suffer from AIDS.”
C. “That is a negative attitude.”
D. ”It must really be frustrating for you. How can I best help you?”

Correct Correct Answer: D. ”It must really be frustrating for you. How can I best
help you?”
This response reflects the pain due to loss. A helping relationship can be forged by
showing empathy and concern. Communicate therapeutically with patient and family
members and allow them to verbalize feelings. Sharing feelings with a healthcare
provider may help the patient find significance in the experience of loss.
 Option A: This is not therapeutic since it passes the buck or responsibility to the
clergy. Support the patient and significant others share mutual fears, concerns,
plans, and hopes for each other. Keeping secrets won’t do any help during this
time. These times of stress can be used as an opportunity for growth and family
development.
 Option B: This response is not therapeutic because it gives the client the
impression that she is right which prevents the client from reconsidering her
thoughts. Help the client accept that the loss is real by providing sensitive, factual
information concerning the loss.
 Option C: This statement passes judgment on the client. Acknowledge the
patient’s need to review the loss experience. In this way, the patient and family
members integrate the event into their experience.

 17. Question 1 point(s)

One morning the nurse sees the client in a depressed mood. The nurse asks her “What are
you thinking about?” This communication technique is:

 A. Focusing
B. Validating
C. Reflecting
D. Giving broad opening

Correct Correct Answer: D. Giving broad opening
Broad opening technique allows the client to take the initiative in introducing the topic.
Therapeutic communication is often most effective when patients direct the flow of
conversation and decide what to talk about. To that end, giving patients a broad opening
such as “What’s on your mind today?” or “What would you like to talk about?” can be a
good way to allow patients an opportunity to discuss what’s on their mind.
 Option A: Sometimes during a conversation, patients mention something
particularly important. When this happens, nurses can focus on their statement,
prompting patients to discuss it further. Patients don’t always have an objective
perspective on what is relevant to their case; as impartial observers, nurses can
more easily pick out the topics to focus on.
 Option B: Validation is an empathetic method of communicating and helping
disoriented adults. Validation techniques can: Help disoriented elderly to express
what they wish to express whether it is verbal or nonverbal communication. Help
the person express suppressed feelings.
 Option C: Patients often ask nurses for advice about what they should do about
particular problems or in specific situations. Nurses can ask patients what they
think they should do, which encourages patients to be accountable for their own
actions and helps them come up with solutions themselves.

 18. Question 1 point(s)

The client says to the nurse “Pray for me” and entrusts her wedding ring to the nurse. The
nurse knows that this may signal which of the following:

 A. Anxiety
B. Suicidal ideation
C. Major depression
D. Hopelessness

Incorrect Correct Answer: B. Suicidal ideation
Determine whether the person has any thoughts of hurting him or herself. Suicidal
ideation is highly linked to completed suicide. Some inexperienced clinicians have
difficulty asking this question. They fear the inquiry may be too intrusive or that they
may provide the person with an idea of suicide. In reality, patients appreciate the question
as evidence of the clinician’s concern. A positive response requires further inquiry.
 Option A: The client’s statement is a verbal cue of suicidal ideation, not anxiety.
If suicidal ideation is present, the next question must be about any plans for
suicidal acts. The general formula is that more specific plans indicate greater
danger. Although vague threats, such as a threat to commit suicide sometime in
the future, are the reason for concern, responses indicating that the person has
purchased a gun, has ammunition, has made out a will, and plans to use the gun
are more dangerous.
 Option C: While suicide is common among clients with major depression, this
occurs when their depression starts to lift. Determine what the patient believes his
or her suicide would achieve. This suggests how seriously the person has been
considering suicide and the reason for death. For example, some believe that their
suicide would provide a way for family or friends to realize their emotional
distress. Others see their death as a relief from their own psychic pain. Still others
believe that their death would provide a heavenly reunion with a departed loved
one. In any scenario, the clinician has another gauge of the seriousness of the
planning.
 Option D: Hopelessness indicates no alternatives available and may lead to
suicide, the statement and nonverbal cue of the client indicate suicide. Haunted
and dominated by hopelessness and helplessness. They are without hope and
therefore cannot foresee things ever improving; they also view themselves as
helpless in 2 ways: (1) they cannot help themselves, and all their efforts to liberate
themselves from the sea of depression in which they are drowning are to no avail;
and (2) no one else can help them.

 19. Question 1 point(s)

Which of the following interventions should be prioritized in the care of the suicidal
client?

 A. Remove all potentially harmful items from the client’s room.


B. Allow the client to express feelings of hopelessness.
C. Note the client’s capabilities to increase self-esteem.
D. Set a “no suicide” contract with the client.
20. Question 1 point(s)

For questions #50-55:


Situation: Ryan, a 14-year-old male was admitted to a medical ward due to bronchial
asthma after learning that his mother was leaving soon for U.K. to work as a nurse. The
client has which of the following developmental focus:

 A. Establishing a relationship with the opposite sex and career planning.


B. Parental and societal responsibilities.
C. Establishing one's sense of competence in school.
D. Developing initial commitments and collaboration in work.

 21. Question 1 point(s)

The personality type of Ryan is:

 A. Conforming
B. Dependent
C. Perfectionist
D. Masochistic
 L
 22. Question 1 point(s)

The nurse ensures a therapeutic environment for the client. Which of the following best
describes a therapeutic milieu?

 A. A therapy that rewards adaptive behavior.


B. A cognitive approach to change behavior.
C. A living, learning, or working environment.
D. A permissive and congenial environment.
L.

 23. Question 1 point(s)

Included as a priority of care for the client will be:

 A. Encourage verbalization of concerns instead of demonstrating them through


the body.
B. Divert attention toward activities.
C. Place in Semi-fowler's position and render O2 inhalation as ordered.
D. Help her recognize that her physical condition has an emotional component.

 24. Question 1 point(s)

The client is concerned about his coming discharge, manifested by being unusually sad.
Which is the most therapeutic approach by the nurse?
 A. “You are much better than when you were admitted so there’s no reason to
worry.”
B. “What would you like to do now that you’re about to go home?”
C. “You seem to have concerns about going home.”
D. “Aren’t you glad that you’re going home soon?”

 25. Question 1 point(s)

Situation: The nurse may encounter clients with concerns on sexuality. The most basic
factor in the intervention with clients in the area of sexuality is:

 A. Knowledge about sexuality.


B. Experience in dealing with clients with sexual problems.
C. Comfort with one’s sexuality.
D. Ability to communicate effectively.

 26. Question 1 point(s)

Which of the following statements is true for gender identity disorder?

 A. It is a sexual pleasure derived from inanimate objects.


B. It is a pleasure derived from being humiliated and made to suffer.
C. It is a pleasure to shock the victim with exposure to the genitalia.
D. It is the desire to live or involve in reactions of the opposite sex.

 27. Question 1 point(s)

The sexual response cycle in which the sexual interest continues to build:

 A. Sexual Desire
B. Sexual arousal
C. Orgasm
D. Resolution

 28. Question 1 point(s)

The inability to maintain the physiologic requirements in sexual intercourse is:

 A. Sexual Desire Disorder


B. Sexual Arousal Disorder
C. Orgasm Disorder
D. Sexual Pain Disorder

29. Question 1 point(s)

The nurse asks a client to roll up his sleeves so she can take his blood pressure. The client
replies “If you want I can go naked for you.” The most therapeutic response by the nurse
is:

 A. “You’re attractive, but I’m not interested.”


B. “You wouldn’t be the first that I will see naked.”
C. “I will report you to the guard if you don’t control yourself.”
D. “I only need access to your arm. Putting up your sleeve is fine.”

 30. Question 1 point(s)

Situation: Knowledge and skills in the care of violent clients is vital in the psychiatric
unit. A nurse observes that a client with a potential for violence is agitated, pacing up and
down the hallway and making aggressive remarks. Which of the following statements is
most appropriate to make to this patient?

 A. What is causing you to become agitated?


B. You need to stop that behavior now.
C. You will need to be restrained if you do not change your behavior.
D. You will need to be placed in seclusion

 1. Question 1 point(s)

The nursing diagnosis that would be most appropriate for a 22-year old client who uses
ritualistic behavior would be:

 A. Ineffective coping
B. Impaired adjustment
C. Personal identity disturbance
D. Sensory/perceptual alterations

 2. Question 1 point(s)

A psychiatrist prescribes an anti-obsessional agent for a client who is using ritualistic


behavior. A common anti-anxiety medication used for this type of client would be:

 A. fluvoxamine (Luvox)
B. benztropine (Cogentin)
C. amantadine (Symmetrel)
D. diphenhydramine (Benadryl) .

 3. Question 1 point(s)

A 20-year old college student has been brought to the psychiatric hospital by her parents.
Her admitting diagnosis is borderline personality disorder. When talking with the parents,
which information would the nurse expect to be included in the client’s history? Select
all that apply.

 A. Impulsiveness
B. Lability of mood
C. Ritualistic behavior
D. Psychomotor retardation
E. Self-destructive behavior

 .

 4. Question 1 point(s)

A hospitalized client, diagnosed with a borderline personality disorder, consistently


breaks the unit’s rules. This behavior should be confronted because it will help the client:

 A. Control anger
B. Reduce anxiety
C. Set realistic goals
D. Become more self-aware ”.

 5. Question 1 point(s)

When working with the nurse during the orientation phase of the relationship, a client
with a borderline personality disorder would probably have the most difficulty in:

 A. Controlling anxiety.
B. Terminating the session on time.
C. Accepting the psychiatric diagnosis.
D. Setting mutual goals for the relationship.

 6. Question 1 point(s)

A client with a diagnosis of borderline personality disorder has negative feelings toward
the other clients on the unit and considers them all to be “bad.” The nurse understands
this defense is known as:

 A. Splitting
B. Ambivalence
C. Passive aggression
D. Reaction formation

 7. Question 1 point(s)

The client with antisocial personality disorder:

 A. Suffers from a great deal of anxiety.


B. Is generally unable to postpone gratification.
C. Rapidly learns by experience and punishment.
D. Has a great sense of responsibility toward others.

 .

 8. Question 1 point(s)

A person with antisocial personality disorder has difficulty relating to others because of
never having learned to:

 A. Count on others.
B. Empathize with others.
C. Be dependent on others.
D. Communicate with others socially. .

 9. Question 1 point(s)

A young, handsome man with a diagnosis of antisocial personality disorder is being


discharged from the hospital next week. He asks the nurse for her phone number so that
he can call her for a date. The nurse’s best response would be:

 A. “We are not permitted to date clients.”


B. “No, you are a client and I am a nurse.”
C. “I like you, but our relationship is professional.”
D. “It’s against my professional ethics to date clients.”

 10. Question 1 point(s)

When caring for a client with a diagnosis of schizotypal personality disorder, the nurse
should:

 A. Set limits on manipulative behavior.


B. Encourage participation in group therapy.
C. Respect the client’s needs for social isolation.
D. Understand that seductive behavior is expected.

 11. Question 1 point(s)

A nurse is orienting a new client to the unit when another client rushes down the hallway
and asks the nurse to sit down and talk. The client requesting the nurse’s attention is
extremely manipulative and uses socially acting-out behaviors when demands are unmet.
The nurse should:

 A. Suggest that the client requesting attention speak with another staff member.
B. Leave the new client and talk with the other client to avoid precipitating acting
out behavior.
C. Tell the interrupting client to sit down and be patient, stating, “I’ll be
back as soon as possible.”
D. Introduce the two clients and suggest that the client join the new client and the
nurse on the tour.

 12. Question 1 point(s)

A client with a diagnosis of narcissistic personality disorder has been given a day pass
from the psychiatric hospital. The client is due to return at 6 pm. At 5 pm the client
telephones the nurse in charge of the unit and says “6 o’clock is too early. I feel like
coming back at 7:30.” The nurse would be most therapeutic by telling the client to:

 A. Return immediately, to demonstrate control.


B. Return on time or restrictions will be imposed.
C. Come back at 6:45, as a compromise to set limits.
D. Come back as soon as possible or the police will be sent.

 13. Question 1 point(s)

An adult client with a borderline personality disorder become nauseated and vomits
immediately after drinking 2 ounces of shampoo as a suicide gesture. The most
appropriate initial response by the nurse would be to:

 A. Promptly notify the attending physician.


B. Immediately initiate suicide precautions.
C. Sit quietly with the client until nausea and vomiting subsides.
D. Assess the client’s vital signs and administer syrup of ipecac.
 14. Question 1 point(s)

A nurse notices that a client is mistrustful and shows hostile behavior. Which of the
following types of personality disorder is associated with these characteristics?

 A. Antisocial
B. Avoidant
C. Borderline
D. Paranoid

 15. Question 1 point(s)

Which of the following statements is typical for a client diagnosed with a paranoid
personality disorder?

 A. “I understand you’re the one to blame.”


B. “I must be seen first; it’s not negotiable.”
C. “I see nothing humorous in this situation.”
D. “I wish someone would select the outfit for me.”

16. Question 1 point(s)

Which of the following characteristics is expected for a client with paranoid personality
disorder who receives bad news?

 A. The client is overly dramatic after hearing the facts.


B. The client focuses on self to not become over-anxious.
C. The client responds from a rational, objective point of view.
D. The client doesn’t spend time thinking about the information.

 17. Question 1 point(s)

Which of the following types of behavior is expected from a client diagnosed with a
paranoid personality disorder?

 A. Eccentric
B. Exploitative
C. Hypersensitive
D. Seductive

 18. Question 1 point(s)


Which of the following interventions is important for a client with paranoid personality
disorder taking olanzapine (Zyprexa)?

 A. Explain effects of serotonin syndrome.


B. Teach the client to watch for extrapyramidal adverse reactions.
C. Explain that the drug is less effective if the client smokes.
D. Discuss the need to report paradoxical effects such as euphoria.

 19. Question 1 point(s)

A client with antisocial personality is trying to convince a nurse that he deserves special
privileges and that an exception to the rules should be made for him. Which of the
following responses is the most appropriate?

 A. “I believe we need to sit down and talk about this.”


B. “Don’t you know better than to try to bend the rules?”
C. “What you’re asking me to do is unacceptable.”
D. “Why don’t you bring this request to the community meeting?”

20. Question 1 point(s)

A nurse notices other clients on the unit avoiding a client diagnosed with antisocial
personality disorder. When discussing appropriate behavior in group therapy, which of
the following comments is expected about this client by his peers?

A. Lack of honesty
B. Belief in superstitions
C. Show of temper tantrums
D. Constant need for attention

 21. Question 1 point(s)

Which of the following characteristics or client histories substantiates a diagnosis of


antisocial personality disorder?

 A. Delusional thinking
B. Feelings of inferiority
C. Disorganized thinking
D. Multiple criminal charges
 22. Question 1 point(s)

A client with borderline personality disorder is admitted to the unit after slashing his
wrist. Which of the following goals is most important after promoting safety?
 A. Establish a therapeutic relationship with the client.
B. Identify whether splitting is present in the client’s thoughts.
C. Talk about the client’s acting out and self-destructive tendencies.
D. Encourage the client to understand why he blames others.

 23. Question 1 point(s)

Which of the following characteristics or situations is indicated when a client with


borderline personality disorder has a crisis?

 A. Antisocial behavior
B. Suspicious behavior
C. Relationship problems
D. Auditory hallucinations

 24. Question 1 point(s)

Which of the following assessment findings is seen in a client diagnosed with borderline
personality disorder?

 A. Abrasions in various healing stages.


B. Intermittent episodes of hypertension.
C. Alternating tachycardia and bradycardia.
D. Mild state of euphoria with disorientation.

 25. Question 1 point(s)

In planning care for a client with borderline personality disorder, a nurse must be aware
that this client is prone to develop which of the following conditions?

 A. Binge eating
B. Memory loss
C. Cult membership
D. Delusional thinking

 26. Question 1 point(s)

Which of the following statements is expected from a client with borderline personality
disorder with a history of dysfunctional relationships?

 A. “I won’t get involved in another relationship.”


B. “I’m determined to look for the perfect partner.”
C. “I’ve decided to use better communication skills.”
D. “I’m going to be an equal partner in a relationship.”

 27. Question 1 point(s)

Which of the following conditions is likely to coexist in clients with a diagnosis of


borderline personality disorder?

 A. Depression
B. Delirium
C. Avoidance
D. Disorientation

 28. Question 1 point(s)

Which of the following nursing interventions has priority for a client with borderline
personality disorder?

 A. Maintain consistent and realistic limits.


B. Give instructions for meeting basic self-care needs.
C. Engage in daytime activities to stimulate wakefulness.
D. Have the client attend group therapy on a daily basis.

 A nurse is assessing a client diagnosed with a dependent personality disorder. Which of


the following characteristics is a major component of this disorder?

 A. Abrasive to others
B. Indifferent to others
C. Manipulative of others
D. Over-reliance on others

 30. Question 1 point(s)

Which of the following information must be included for the family of a client diagnosed
with a dependent personality disorder?

 A. Promote exercise programs


B. Explore panic attacks
C. Address coping skills
D. Decrease aggressive outbursts

You might also like